You are on page 1of 168

The

Economics
Compendium
• Head Office : B-32, Shivalik Main Road, Malviya Nagar, New Delhi-110017

• Sales Office : B-48, Shivalik Main Road, Malviya Nagar, New Delhi-110017
Tel. : 011-26691021 / 26691713

Typeset by Disha DTP Team

DISHA PUBLICATION
ALL RIGHTS RESERVED

© Copyright Publisher

No part of this publication may be reproduced in any form without prior permission of the publisher. The author and the
publisher do not take any legal responsibility for any errors or misrepresentations that might have crept in. We have
tried and made our best efforts to provide accurate up-to-date information in this book.

For further information about the books from DISHA,


Log on to www.dishapublication.com or email to info@dishapublication.com
CONTENTS
1. Structure of Economy and Human Resource in India E-1-E-18
Introduction; Salient Features; National Income Of India; Population

2. Planning, Unemployment and Poverty in India E-19-E-44


Introduction; Planning Commission; NITI Aayog; Five Year Plans; Unemployment; Poverty;
Millennium Development Goals; Sustainable Development Goals

3. Agriculture E-45-E-70
Introduction; Features of Indian Agriculture; Growth and Productivity; Agriculture in Five
Year Plans; National Agricultural Policy; Major Agriculture Programmes & Revolutions; Land
Reforms; Co-operative Farming; Rural Credit and Extension Services; Agricultural Insurance,
Marketing & Price Policy; Public Distribution System

4. Fiscal and Monetary Policy E-71-E-90


Introduction; Fiscal policy; Expenditure; Deficit & Taxes; Indian Tax Structure; Black Money;
Finance Commission; Monetary Policy

5. Money Supply and Indian Financial System E-91-E-120


Introduction; Money Market; Capital Market; Money Supply; Inflation; Indian Banking System;
Insurance Sector; Micro Finance

6. Foreign Trade and Investment in India E-121-E-140


Introduction; Foreign Exchange Reserves; India’s Foreign Trade; Balance Of Payments; India’s
Balance of Payment

7. Industry and Infrastructure E-141-E-164


Introduction; Status of Indian Industries; Acts and Regulations; Micro, Small and Medium
Enterprise; Major Industries; Public Sector Units
ECONOMY

STRUCTURE OF ECONOMY AND


1
HUMAN RESOURCE IN INDIA

Introduction
The word ‘Economics’ originates from the Greek word ‘Oikonomikos’; Oikos (means ‘Home’) + Nomos (means ‘Management’)’
means ‘Home management. Economics is the study of how people and society end up choosing, with or without the use of
money, to employ scarce productive resources that could have alternative uses to produce various commodities over time and
distributing them for consumption, now or in the future, among various persons or groups in society. It analyses costs and
benefits of improving patterns of resource allocation. The term Economy means the state of country or region in terms of the
production and consumption of goods and services and the supply of money.

Types of Economy Indian Economy


Salient Features of Indian Economy
Socialist Mixed Capitalist The salient features of Indian Economy include the followings:

Social ownership (1) Mixed Economy


Private ownership
of means of means of production India is a mixed economy. In a mixed economy,
of Production public sector (government-owned) business enterprises
exist alongside the private sector to achieve a welfare
USSR, China USA, UK, France, etc
state with socialistic pattern of society. Ever since
independence, India’s economic development has been
Public- Private Market-based Free-markets guided by the twin objectives of developing:
ownership + allocation + with state (a) a rapidly and technologically progressive economy
of Industry with Economic interventionism by democratic means; and
Planning
(b) a social order based on justice, offering equal
India opportunity to every citizen of the country.
(2) Low per Capita Income
/Sells
Buys ebentures According to IMF GDP per capital of India in 2014 at
s d
, to
Share ds from / s current prices is $1,627. India is the 9th largest economy
n e Borrows
& bo n countri of the world. But, due to its huge population of more
fo r e i g & Lends
Free than 1.26 billion, India is at 145th position in term of
Closed Economy Open movement GDP Per capita. Per capita income of India is 6.69 times
of people lower than world’s average around of $10,880. This
figure is 68.66 times lower than richest country of world
ow
e fl
No economic Maint Fre oods & and 6.5 times greater than poorest country of the world.
relation with rest bilater ai
al/mul n of g rvices India is at 34th position in the list of Asian countries.
of the world relatio t i l ate se On the basis of PPP, GDP, per capita of India stands
nships ral
at 5,855 International Dollar in 2014. GDP (PPP) per
capita of world is 15,189 Int. $.
E-2 Structure of Economy and Human Resource in India

2014-15 India suffers from inadequacy of financial institutions


in rural areas. Similarly, India being a country of a
Per capita Net national Income at current prices - `88533 large number of small farmers, the development of
Per capita Income constant prices ( 2011-12 prices) - `74193 certain agencies of credit for granting loans to farmers
GDP Per Capita at current - `98,983 on easy terms is needed. Likewise, to provide medium
GDP Per Capita at 2011-12 price - `84, 009. and long-term loans to industries for the development of
(3) Dominance of Agriculture and Heavy Population industrial finance corporation is quite necessary. There
Pressure on Agriculture is a great scarcity of skilled and efficient administrators and
managers.
In India, almost 60-70% of the total population still
resides in rural areas and hence they depend on State of the Economy 2014-15
agriculture for their livelihood. An Overview
(4) Over-Population 1. GDP Growth Rate - 7.4 %
India is over populated. In every decade Indian 2. Export growth - 2.4 % (US$265 billion)
population gets increased by about 20%. During 2001- (April-Jan.)
11, population increased by 17.6%. With this high 3. Import growth - 2.2% (US$ 383.4 billion)
growth rate of population about 1.7 crore new persons (April-Jan.)
are added to Indian population every year. According 4. Fiscal Deficit - 4.0%
to 2011 census, the total Indian population stands at a
5. Inflation (WPI) - 3.4% (April-Dec.)
high level of 121.02 crore which is 17.5% of the world’s
total population which is second largest population of the 6. Foreign Exchange - US $ 330.2 billion
world. To maintain this 17.5% of world population India Reserves (6 Feb. 2015)
holds only 2.42% of total land area of the world. * New Base year of National Accounts is 2011-12
(5) Unbalanced Economic Development Structure Composition of Indian Economy
India has not yet achieved the goal of balanced economic There are 3 major sectors of Indian economy- the primary sector,
development. According to latest data available about the secondary and the tertiary sector.
64% of total labour force is dependent on agriculture, Manufactured goods used as Inputs
16% on industries and the rest about 20% on trade,
transport and other services.
Primary Provide Secondary Provide Tertiary
(6) Low rate of capital formation Sector Natural Products Sector Infrastructure Sector
Another basic characteristic of the Indian economy is the
existence of capital deficiency which is reflected in two Services that help in production
ways - firstly, the amount of capital per head available is
low; and secondly, the current rate of capital formation Sector Activity
is also low. Gross Fixed Capital Formation in India
increased to 4975.22 INR billion in the third quarter of Agriculture and Allied Sector
2014 from 4957.25 INR billion in the second quarter of Primary Forestry
2014. Gross Fixed Capital Formation in India averaged
Sector Fishing
3595.81 INR billion from 2001 until 2014, reaching an
all time high of 5356.22 INR billion in the first quarter Mining &Quarrying
of 2014 and a record low of 2021.90 INR billion in the Manufacturing
first quarter of 2002. Secondary
Sector Electricity, Gas
(7) Lack of Infrastructure Facility or and Water Supply
There is a lack of physical infrastructure (i.e. Industrial sector Construction
road,electricity, banking, transportation, insurance, energy)
and social infrastructure(i.e. education, health, housing, Trade, Hotels and Restaurants
drinking water, sanitation) that hinders the development Transport
process of a country.
Storage
(8) Poor Economic Organisation Tertiary
Communication
Another important feature of the Indian economy Sector or Service
is poor economic organisation. Certain institutions Sector Financing, (Banking Insurance)
necessary for economic development are not adequately Real Estate and Business Services
developed. For instance, to mobilise savings and more
Community, Social, Personal and
especially the savings of the rural sector, the creation
other Services
and development of financial institutions is essential.
Structure of Economy and Human Resource in India E-3

Percentage Share of Sectors in GVA at factor cost at current Prices -

Sector 2004-05 Series 2011-12 Series


2011-12 2012-13 2013-14 2011-12 2012-13 2013-14 2014-15
1. Agriculture & Allied Activities 17.9 17.5 18.2 18.9 18.7 18.6 17.6
2. Industry 27.2 26.2 24.8 32.9 31.7 30.5 29.7
3. Service 54.9 56.3 57.0 48.2 49.6 50.9 52.7
Source : CSO’s press Releases of 30 january 2015 & 9 February 2015 on New Series Estimates of National Income.

The Central Statistics Office (CSO), Ministry of Statistics In equation form:


and Programme Implementation have released the advance GNP = GDP + X – M,
estimates of GDP growth rates for 2015-16 and Q1, Q2, Q3 where,
of 2015-16 at constant (2011-12) and current prices are given X = Income earned and received by nationals within
below: the boundaries of foreign countries.
Growth Rates of GDP M = Income received by foreign nationals within the
country.
Constant Current
If X = M, then GNP = GDP.
Prices (2011-12) Prices Similarly, in a closed economy
Annual 2015-16 (Advance) 7.6 8.6 X=M=0
Q1 2015-16 (April-June) 7.6 8.7 then also GNP = GDP
In equation form :
Q2 2015-16 (July-Sep) 7.7 6.4
GNP = GDP + NFIA
Q3 2015-16 (Oct-Dec) 7.3 9.2
Where NFIA = Net Factor Income from abroad
also NFIA = Factor incomes received from abroad —
National Income Of India Factor income paid to abroad.
National income measures the net value of goods and services It is to be noted here that in a closed economy which
does not deal with outside world, has no NFIA, i.e. its
produced in a country during a year and it also includes net NFIA is equal to Zero. Hence, for such countries, GDP
earned foreign income. In other words, a total of national = GNP
income measures the flow of goods and services in an Gross Domestic Product (GDP)
economy. National income is a flow not a stock. As contrasted It is the total money value of all final goods and services
with national wealth which measures the stock of commodities produced within the geographical boundaries of the
held by the nationals of a country at a point of time, national country during a given period of time. So, domestic
income measures the productive power of an economy in a given product emphasis the total output which is raised within
period to turn out goods and services for final consumption. the geographical boundaries of the country, national
product focuses not only on the domestic product but also
Concepts of National Income on goods and services produced outside the boundaries of
a nation.
The various concepts of national income are as follows: 3. Net National Product (NNP)
1. Per Capita Income NNP is obtained by subtracting depreciation value (i.e.
It is a measure of the amount of money that is being capital stock consumption) from GNP.
earned per person in a certain area. In equation form :

National Income NNP = GNP – Depreciation.


PCI =
Population 4. National Income
2. Gross National Product (GNP) GNP is based on market prices of produced goods
Gross National Product refers to the money value of total which includes indirect taxes and subsidies. NNP can be
calculated in two ways-
output or production of final goods and services produced (i) at market prices of goods and services
by the nationals of a country during a given period of (ii) at factor cost
time, generally a year. When NNP is obtained at factor cost, it is known
In the calculation of GNP, we include the money value as National Income. National Income is calculated by
of goods and services produced by nationals outside subtracting net indirect taxes (i.e. total indirect tax-
the country. Hence, income produced and received by subsidy) from NNP at market prices. The obtained value
nationals of a country within the boundaries of foreign is known as NNP at factor cost or National income. So,
countries should be added in Gross Domestic Product NNP at factor cost or National Income
(GDP) of the country. Similarly, income received by = NNP at market price – (Indirect Taxes – Subsidy)
foreign nationals within the boundary of the country
= NNP(mp) – Indirect Tax + Subsidy.
should be excluded from GDP.
E-4 Structure of Economy and Human Resource in India

5. Personal Income In India a combination of production method and income


Personal income is that income which is actually obtained method is used for estimating national income.
by nationals. Symbolically : N.I = C + I + G + (X – M)
Personal income is obtained by subtracting corporate Where,
taxes and payments made for social securities provision C= Total consumption expenditure
from national income and adding to it government I = Total Investment Expenditure
transfer payments, business transfer payments and net G = Total Govt. Expentiture.
interest paid by the government. So, X = Export
Personal Income = National income – undistributed M = Import
profits of corporation – payments for social security
provisions – corporate tax + government transfer Estimates of National Income in India
payments + Business transfer payments + Net interest In 1868, the first attempt was made by Dada Bhai
paid by government. Naoroji. He, in his book ‘Poverty and Un-British Rule in
It should always be kept in mind that personal income India’. estimated Indian per capita annual income at a level
is a flow concept. of Rs. 20.
6. Personal Disposable Income Some Other economists followed it and gave various estimates
of Indian national income, some of these estimates are as
When personal direct taxes are subtracted from personal follows :
income, the obtained value is called disposable personal
Findlay Shirras ( 1911) -   `49
income (DPI). So,
Wadia & Joshi ( 1913-14) -  `44.30
Disposable personal income
Dr. V.K.R.V. Rao (1925-29) -  `76
DPI = [Personal income] – [Direct Taxes]
After independence, the Government of India appointed
A Graphical Representation of relationship between the National Income Committee in August 1949 under the
various measures. chairmanship of Prof. P.C. Mahalanobis, to compile
GNPMP authoritative estimates of national income. For further
estimation of national income, the government established
× × × × × ×
NNPMP = GNPMP – Depreciation Central Statistical Organisation (CSO) which now regularly
× ×× × × × × × ×
× ×
× × × × ×× × × ××× ××
× × × × ×× × × × NNPFC = NNPMP – Indirect taxes publishes national income data.
× × ×× × × × × ×× × ×
× × ××××× × × ××
CSO & NSSO to be Merged :
× ×
× ×
× ×× × × ×× × × ×××
× ×× ×
+ Subsidier
× × × × × ×× × × ×
×× ×
×
× ×
× × × × × × ×
××
×
× ×× × × × ×× × ×
× The government is planning to merge Central Statistical
Organisation (CSO) and National Sample Survey
× × × × × × ×
× × × × ×× × ×
× × × × × ×× ×
Personal Income = NNPFc
×× ×
× ×
× × ××××
× ×
+Net transfer payment Organisation (NSSO) for promoting statistical network
Disposable income in the country. The newly merged unit will be named as
= Personal Income – Direct Taxes National Statistical Organisation (NSO). The head of the
organisation will be designated as ‘Chief of Statistician of
Methods of Measuring National Income
India’ and will be having the rank of Chief Secretary.
National Income of a country is calculated by following three
methods :
Indicators of Economic Development
1. Product Method
In this method net value of final goods and services The major indicators to increase the levels of development are :
produced in a country during a year is obtained and the (i) Net National Product (NNP)
total obtained value is called total final product. This It is defined as the total output produced by a country
represents Gross Domestic Product (GDP). Net income in one financial year. It can be computed by subtracting
earned in foreign boundaries by nationals is added and depreciation from GNP. NNP is also called as National
depreciation is subtracted from GDP. Income.
2. Income Method (ii) Per Capita Income
In this method, a total of net incomes earned by working A high per capital income indicates a better standard of
people in different sectors and commercial enterprises is living and thus, economic development on the whole.
obtained.
Further, a rise in per capita income will always mean a
Symbolically : National Income = total Rent + Total rise in aggregate real output.
Wages + total Interest + Total Profit. (iii) Quality of Life Index (QLI) : The Index of Quality of
3. Consumption Method life depends upon mainly three factors, i.e. life expected,
It is also called expenditure method. Income is either Basic Literacy ad Infant Morality Rate. Most of the
spent on consumption or saved. Hence national income countries with low per capita GNP tends to have to QLI
is the addition of total consumption and total savings. and vice-a-versa.
Structure of Economy and Human Resource in India E-5

(iv) Human Development Index (HDI) : It is one of the actual decline in population in that Census, brought about by an
most recent and significant indicator of economic influenza epidemic. The most spectacular growth in population
development of a country. It is a composite of three was witnessed after Independence, as is clear from the table
indicators, i.e. Life Expectancy Index (LEI); Education below.
Attainment Index (EAI) and Standard of Living Index. Population Growth
(HDI) ranks countries in relations to each other. It can
be computed by using following formula : Year Population Growth Average
(in millions) rate during annual growth

HDI : 3
LEI× EAI×SLI decade (%) rate (%)
1921 251.3 – 0.31 – 0.03
Population 1931 278.9 11 1.04
Theory of Demographic Transitions 1941 318.6 14.22 1.33
Demography is the scientific study of human population. 1951 361 13.31 1.25
The relationship between population growth and economic 1961 439.2 21.51 1.96
development can be explained by the Theory of Demographic 1971 548.1 24.8 2.2
Transition for contemporary developed nations. There are 1981 683.3 24.66 2.22
three stages of population growth.
1991 846.3 23.85 2.14
(i) First stage of stable or slow population growth
2001 1027 21.34 1.9
Due to the high death rate which nullified the high birth
rate. In this stage, these economies were primitive 2011 1.21 –17.64 1.41
and primarily agrarian, with widespread illiteracy, Due to the high rate population growth, it has not been possible
poor sanitation and health care conditions, negligible to make a significant change in the poverty levels in India. The
knowledge of family planning and large family sizes higher population growth, the lower is the rate of growth of per
which contributed to factors such as high fertility rate capita income. The rapid growth in population, which is one
and high death rate. In the period between the first and of the highest in the world and making up one-fifth of the total
the second stages, the death rates start reducing and world population, is eroding India’s considerable development
birth rates remain stable which brings an imbalance in gains and steadily diminishing its ability to provide a decent
the economy. Measures like diseases control, improving quality of life for a large number of its people. However,
nutrition levels, and sanitation improvement are several economists and demographers are of the opinion that
implemented to reduce death rates, but the measures India’s growing population is not hindering rapid development.
for controlling birth rates are not implemented, which According to them a rise in income will automatically bring
results in population explosion. This required a period of down the rate of population growth as in developed nations.
transition for adjustment, thus bringing the second stage However, rapid population growth is a development problem.
of transition. Continuing rapid population growth in the long run is likely
(ii) Second stage of population explosions to mean a lower quality of life for millions of people. In the
In this stage rise in income levels contributed to developing countries the main economic class that suffers are
improvement in health-care, education, disease control the poor. It is also the poor, with little education, low and
and so on which in turn contributed to reducing the insecure income, poor health and large families, who lose
death rates. This accelerated the growth population. The out as rapid population growth hampers development. This
family size reduced and improved trade and economic points to the need for a comprehensive public policy aimed at
conditions resulted in more food and better food habits reducing population growth.
which further helped the population to grow at a much India : Demographics (2013 Estimates)
higher rate than in the first stage. The demographic
1. Population growth rate - 1.25%
changes brought imbalances in the economy, creating
conditions for further transition of society. 2. Birth rate - 20.22 (Per 1000)
3. Death Rate - 7.4 (Per 1000)
(iii) Third stage of low birth rates and low death rates
4. Infant Mortality rate - 40 ( Per 1000)
Modernization and industrilization changed the living
5. Total fertility rate - 2.3
pattern in such nations, the rural population shifted
to cities and family sizes reduced to become nuclear
Birth Rate and Death Rate in India
families. The standard of living increased which further
brought down mortality rates and birth rates. As a result, (Per Thousand Population)
the growth of population declined. Year Birth Rate Death Rate
1950-51 39.9 27.4
Population and development in Indian
1960-61 41.7 22.8
perspective
1970-71 36.9 14.9
India’s population grew slowly till the 1920s. The year 1921
is known as the year of the Great Divide, since there was an 1980-81 33.9 12.5
E-6 Structure of Economy and Human Resource in India

1990-91 29.5 9.8 (i) The immediate objective is to address the unmet needs
for contraception, health care infrastructure and basic
2000-01 25.4 8.4 reproductive and child health care.
2007-08 23.5 7.4 (ii) The medium-term objective is to bring the Total Fertility
2008-09 22.8 7.4 Rate (TFR) to replacement levels of 2.1 by 2010.
2009-10 22.5 7.3 (iii) The long-term objective is to achieve a stable population
by 2045, at a level consistent with the requirement of
2010-11 22.1 7.2 sustainable economic growth, social development and
India is the first country in the world which has officially environmental protection.
sponsored family planning programmes to control population A National Commission on Population, with the Prime Minister
in order to accelerate the overall development of the country. as the President and state chief ministers, Union ministers,
The programme was officially launched in 1962. The first two NGOs, demographers and public health professionals as
family planning clinics were opened in Karnataka in 1930 members has been set up. This Commission of more than 100
which were supposed to be the first such in the world. members will oversee and review the implementation of the
policy.
National Commission on Population The policy emphasises the pivotal role of urban and rural
The National Commission on Population was constituted on local bodies in implementing it. Promotional and motivational
May 11, 2000 under the Chairmanship of the Prime Minister measures in the policy include rewarding the local bodies
to provide overall guidance for population stabilisation by for exemplary performance, cash incentives to mothers and
promoting synergy between demographic, educational, couples who undergo sterilisation, etc.
environmental and development programmes. The NPP has accepted the recommendation of the Swaminathan
Committee regarding extension of the freezing of seats in
On May 19, 2005 the National Commission on Population
legislatures and Parliament on the basis of 1971 Census. The
was reconstituted. This Commission has now been transferred
Parliament has passed the necessary legislation approving the
from Planning Commission to Ministry of Health. The Prime freezing of seats up to 2026, instead of 2011 as suggested by
Minister will remain the chairman of NCP while Deputy- the Committee.
Chairman of Planning Commission and Union Minister of The present approach National Commission on Population
Health and Family Welfare will work as Deputy Chairman of is based on evolving appropriate strategies to improve the
NCP. The membership of NCP has also been reduced from performance of demographically weak states, drawing lessons
131 to 44. from the southern states. The weaker states are : Bihar,
The National Commission on Population has undertaken Jharkhand, Madhya Pradesh, Chhattisgarh, Rajasthan, Uttar
various initiatives for implementing the National Population Pradesh and Odisha.
Policy such as review of the implementation of National
Family Welfare Programme especially in the high fertility Human Development Index (HDI)
States, identification of high fertility districts and preparation HDI is a broad-based index that assesses the development of
of District Action Plans, selection of Social Economic and an economy, on the basis of the performance of the economy
Demographic Indicators for monitoring purpose, promotion of in various indicators. The need to create such an index was
policy-oriented relevant research for population stabilisation felt as the concept of GDP and per capita income could not
and promotion of public private partnership in meeting the demonstrate the various aspects of human development. The
unmet needs of family planning services. concept of HDI was evolved by economist Mahbub-ul-Haq.
The HDI calcualtes the development of an economy in terms
National Population Policy 2000 of three major indicators.
The problem of over population can be solved by two ways
Indicators of HDI
• An effective employment policy, which can absorb the
growing number of workers and promote economic The HDI is an equi-weighted average of-
growth. • Life Expectancy Index (LEI): Infant mortality is not
• An imaginative family planning programme to encourage considered as a separate indicator in this index. Thus, life
families to adopt the small family norm. expectancy refers to life expectancy at birth, not at age one.
Unfortunately, the official policy has relied heavily on the latter • Educational Attainment Index (EAI) It is a combination
without any linkage with the employment policy. The official of adult literacy rate and combined enrolment ratio.
family planning programme started in 1952 was the first by 2
The weight assigned to Adult Literacy Rate (ALR) is
any country. Initially, it was a purely birth control programme. 3
It became evident during the course of time that such a narrow 1
approach will not pay. False reporting of the achievements of while that for combined Enrolment Ratio (CER) is .
3
targets by the executing agency and wasteful expenditure were Therefore,
some of the serious drawbacks of the target approach. Educational Attainment Index may be given as
The Government in February 2000, announced the National 2 1
EAI = ALR + CER
Population Policy (NPP) with the following objectives: 3 3
Structure of Economy and Human Resource in India E-7

• Standard of Living Index (SLI): It is represented here by Human Development Report 2015
the concept of Purchasing Power Parity (PPP). Per capita
The HDR is an annual milestone published by the United Nations
income is converted into Purchasing Power Parity in terms
Development Programme (UNDP). The title of 2015 edition of
of US dollar.
HDR is ‘ Rethinking work for Human Development’. The
Thus, HDI = 1 (FLET + EAI + SLI) top three nations in the 2015 HDI are Norway (1st) Australia
3 ( 2nd) & Suitzerland (3rd). In 2015, India has been placed
at 130th position with 0.609 score in the medium human
• Categories of Countries based on HDI development category.
HDI has categorised countries into Gender Inequality Index 2015
– very high human development (USA, UK, etc.) India’s rank – 130th (Out 155 countries)
– high human development
(Sri lanka, China and India) Global Hunger Index 2015
– low human development India’s Score – 29 (Serious hunger situation)
(Pakistan, Bangladesh and Nepal)
Occupational Distribution of Population
Meaning of HDI Value Occupational structure refers to distribution of work force in
Different levels of development in terms of HDI score the different occupations of the country. As economy grows,
• Very high human development = 0.791 and labour force shifts from primary sector to secondary and
                    above tertiary sector.
• High human development = 0.699 to Occupation 1951 1971 1991 2001
                  0.790 Primary sector 72.1 72.1 62.7 89.3
• Medium human development = 0.541 to
Secondary sector 10.6 11.2 14.9 18.2
                  0.698
• Low human development = 0.337 to Tertiary sector 17.3 16.7 22.14 22.5
                       0.540 Total 100 100 100 100

Growth of Population and Workforce – All India Category


Category Person Male Female
1981-91 1991-01 2001-11 1981-91 1991-01 2001-11 1981-91 1991-01 2001-11
Total Population 2.1 2.0 1.6 2.2 1.9 1.6 2.1 2.0 1.7
Total Workers 2.5 2.5 1.8 2.2 2.1 1.9 3.5 3.5 1.7
Main workers 2.5 0.9 1.5 2.2 0.8 1.3 3.6 1.2 2.1
Marginal workers 2.5 12.3 2.8 2.6 29.5 5.1 3.2 7.9 1.1
Sex Ratio in Population and Workforce – All India
Year Population Total Workers Main Workers Marginal Workers
1981 933 351 253 5245
1991 927 400 290 9425
2001 933 461 303 1520
2011 940 452 327 1032
Note: 1. Rural and urban combined.
Source: Census of India.

Work Participation Rate:


Percentage of workers (main + marginal) to total population.
Main Workers :
A person who has worked for major part of the reference period (i.e.6 months or more during the last one year preceding the
date of enumeration) in any economically productive activity is termed as ‘main Worker.’
Marginal Workers :
A person who has worked for 3 months or less but for less than 6 months of the reference period in any economic activity is
termed as marginal worker.
E-8 Structure of Economy and Human Resource in India

1. Population experts refer to the possible ‘demographic 8. If it will be true to classify India as
bonus’ that may accrue to India around 2016 A.D. They (a) a food-deficit economy
are referring to the phenomenon of (b) a labour-surplus economy
(a) a sharp drop in the total population (c) a trade-surplus economy
(b) a surge in the population in the productive age (d) a capital-surplus economy
group;
9. The Indian Economy is characterised by
(c) a decline in both birth and death rates;
(a) pre-dominance of agriculture
(d) a well-balanced sex ratio
(b) low per capita income
2. The most significant economic consequence of the
(c) massive unemployment
‘greying’ of the Indian population in the decades to
(d) all of the above
come, will be:
10. In India, planned economy is based on
(a) a fall in the growth of population
(a) Gandhian system (b) Socialist system
(b) a reverse flow of wealth from children to parents
(c) Capitalist system (d) Mixed economy system
(c) a decline in the demand for maternity facilities
11. Who coined the term ‘Hindu rate of growth’ for Indian
(d) the need for redesigning apartment buildings
Economy?
3. According to 2011 census urban-rural population ratio
(a) A.K. Sen
was about-
(b) Kirit S. Parich
(a) 35 : 65 (b) 32 : 68
(c) Raj Krishna
(c) 28 : 72 (d) 31 : 69
(d) Montek Singh Ahluwalia
4. One of the reasons for India’s occupational structure
12. Which one of the following is not a method of
remaining more or less the same over the years has been
measurement of National Income?
that
(a) investment pattern has been directed towards capital (a) Value Added Method
intensive industries (b) Income Method
(b) productivity in agriculture has been high enough to (c) Expenditure Method
induce people to stay with agriculture (d) Investment Method
(c) Ceiling on land holdings have enabled more people 13. Which of the following is definitely a major indication of
to own land and hence their preference to stay with the state of the economy of a country?
agriculture (a) Rate of GDP growth
(d) People are largely unaware of the significance of (b) Rate of inflation
transition from agriculture to industry for economic (c) Number of Banks in a country
development (d) None of these
5. Which indicators are used to calculate the Human 14. In terms of economics, the total value of the output
Development Index (HDI)? (goods and services) produced and income received in a
(a) Population growth, per capita GDP, life expectancy year by a domestic resident of a country put together is
(b) Population growth, infant rate, malnutrition called
(c) Life expectancy, educational attainment, per capita (a) Net National Product
GDP
(b) Gross National Product
(d) Current account balance, infant mortality rate,
(c) Gross National Income
educational attainment.
(d) National Income
6. Which of the following could assist economic growth in
15. Which sector of the Indian Economy contributes largest
developing countries?
to the GNP?
(a) An increase in the rate of taxation
(a) Primary sector (b) Secondary sector
(b) Decrease in foreign investment
(c) Tertiary sector (d) Public sector
(c) Decrease in the terms of trade
16. Per capita Income of a country derived from
(d) An increase in the level of education
7. Which sector of Indian Economy has shown remarkable (a) National Income
expansion during the last decade? (b) Population
(a) Primary sector (b) Secondary sector (c) National Income and Population both
(c) Tertiary sector (d) Mining sector (d) None of these
Structure of Economy and Human Resource in India E-9

17. Who wrote a book describing the theory of economic (b) signs of economic recovery during an economy
drain of India during British rule? downturn.
(a) Lala Lajpat Rai (b) Mahatma Gandhi (c) signs of growth of agriculture sector in a declining
(c) J.L. Nehru (d) Dadabhai Naoroji economy
18. The most appropriate measure of a country’s economic (d) signs of economic decline in a developed economy.
growth is its 24. GDP deflator is used to :
(a) Gross Domestic Product (a) measure the relative reduction in GDP growth rate
(b) Net Domestic Product of a country.
(c) Net National Product (b) measure the inflation in a country.
(d) Per capita Real Income (c) compare the GDP of a country vis a vis other
19. Which one among the following countries has the lowest countries of the world.
GDP per capita? (d) estimate the purchasing power of the citizen of a
(a) China (b) India country.
(c) Indonesia (d) Sri Lanka 25. Human Development Index comprises literacy rates, life
20. Which among the following sector of Indian Economy expectancy at birth and
is maximum dependent on economic developments in (a) GDP per head in the US dollars.
advanced rations? (b) GDP per head at real purchasing power.
(a) Manufacturing sector (c) GNP in US dollars
(b) Agricultural sector (d) National Income per head in US dollars.
(c) Mining sector 26. National income ignores
(d) Services sector (a) sales of a firm
21. Which among the following is a most suitable example (b) salary of employees
of double counting in national income ? (c) exports of the IT sector
(a) Wages of bus and train drivers. (d) sale of land
(b) Cotton output and cotton cloth output. 27. The value of all final goods and services produced by
(c) Electricity output and water output. the normal residents of a country and their property,
whether operating within the domestic territory of the
(d)  Tax receipts and earnings of inland revenue
country or outside in a year is termed as
officials.

22. Which among the following is considered to be the
(a) Gross National Income (b) Net National
best measure of an increase in a country’s economic
Income
efficiency?
(c) Gross Domestic Product (d) Net Domestic
(a) Increase in annual private investment.
Product
(b) Increase in real national income.
28. National product at factor cost is equal to
(c) Increase in real per capita income.
(a) Domestic product + Net factor income from abroad
(d) Increase in net annual investment.
(b) National product at market prices - indirect taxes +
23. What does term ‘Green shoots’ represent in an Economy?
subsidies
(a) signs of growth of agriculture sector in a growing (c) Gross domestic-product - depreciation
economy
(d) National product at market prices + Indirect taxes
+ subsidies
E-10 Structure of Economy and Human Resource in India

6. Consider the following specific stages of demographic


Statement Based MCQ transition associated with economic development :
1. Low birth rate with low death rate
1. Consider the following statements: 2. High birth rate with high death rate
1. The Central Statistical Organisation (CSO) is 3. High birth rate with low death rate
responsible for calculating the consumer price indices
Select the correct order of the above stages using the codes
for urban non-manual employees.
given below :
2. National Sample Survey Organisation (NSSO) (a) 1, 2, 3 (b) 2, 1, 3
collects data for estimating national income. (c) 2, 3, 1 (d) 3, 2, 1
3. NSSO collects rural retail price on monthly basis for
7. Indian economy is a developing economy as there is?
calculating Consumer Price Index.
1. Dominance of agriculture.
4. CSO conducts periodic economic census to collect 2. Huge unemployment
data on unorganised enterprises.
3. Low quality of human capital
Which of the following statement/s is/are correct? 4. Low per capita consumption.
(a) 1, 2, 3 and 4 (b) 2 and 4 (a) 1, 2 and 3 (b) 1 and 4
(c) 1, 3 and 4 (d) 3 and 4 (c) 2 and 3 (d) 1, 2, 3 and 4
2. Among the socio-economic factors responsible for the high 8. Estimation of national income in India are not accurate as
birth rate in India we may include there is:
1. large scale poverty 1. illiteracy
2. high mortality rate of children of poor parents 2. non-monetised consumption
3. prevalence of child marriage 3. inflation
4. adverse sex ratio 4. people holding multiple jobs
Choose your answer from the following: (a) 1, 2, 3 and 4 (b) 2 and 3
Codes: (c) 1 and 4 (d) 1, 2 and 4
(a) 1 and 2 (b) 1, 2 and 4 9. Which of the following gives a correct picture of the share
(c) 2, 3 and 4 (d) 1, 2 and 3 of various sectors in economic growth in India over the
3. Which of the following are the main causes of slow rate years?
of growth of per capita income in India? 1. Primary sector-stagnant
1. High capital output ratio 2. Secondary sector-increasing
2. High rate of growth of population 3. Tertiary sector-decreasing
3. High rate of capital formation (a) 1 and 2 (b) 2 and 3
4. High level of fiscal, deficits (c) 2 only (d) 1 only
codes: 10. Consider the following statements:
(a) 1, 2, 3 and 4 (b) 2, 3 and 4 1. Human Development Report is an authoritative
annual publication brought out by the World Bank.
(c) 1, 3 and 4 (d) 1 and 2
2. It measures longevity by life expectancy at birth.
4. Consider the following statements:
3. Knowledge is measured by adult literacy rate.
1. Infant mortality rate takes into account the death of
4. Standard of living is measured by GDP per capita
infants within a month after birth.
(US $ PPP)
2. Infant mortality rate is the number of infant deaths in
Which of the following statements is not correct?
a particular year per 100 live births during that year.
(a) 1 (b) 2
Which of the above statements is/are correct?
(c) 3 (d) 4
(a) 1 only (b) 2 only
11. In the context of Indian economy, consider the following
(c) Both 1 and 2 (d) Neither 1 nor 2
statements
5. Consider the following statements about the megacities of 1. The growth rate of GDP has steadily increased in the
India: last five years.
1. Population of each megacity is more than 5 million. 2. The growth rate in per capita income has steadily
2. All the megacities are important sea ports. increased in the last five years.
3. Megacities are either national or state capitals. 3. The growth rate of GDP has steadily increased.
Which of these statements are correct ? Which of the statements given above is/are correct ?
(a) 1, 2 and 3 (b) 1 and 2 (a) 1 only (b) 2 only
(c) 2 and 3 (d) 1 and 3 (c) Both 1 and 3 (d) None of these
Structure of Economy and Human Resource in India E-11

12. Which is the correct definition of National Income ? 18. With reference to ‘Economics’ which statement is correct?
1. National Income means the total amount of money 1. Economics is an area where people, society and
Govt. earns through direct tax collection. government prioritize their choices in the process of
2. It is the total value of stocks held by a country at a using the infrequent reserves to satisfy the various
particular point of time. needs and wants of life.
3. This is the measure of goods and services produced 2. Economics is considered an exercise in the space-
during an accounting year without double counting. time continuum.
(a) 1 only (b) 2 only 3. Economics is considered an exercise in the time
continuum.
(c) 3 only (d) All 1, 2 and 3
(a) 1 only (b) 1 and 2
13. A dual economy means: (c) 3 only (d) 1, 2 and 3
1. The existence of black money and white money 19. Which of the following is/are economic activity?
2. The existence of agriculture and industry 1. Profit and Loss 2. Salary
3. The existence of commercial agriculture with 3. Employment 4. occupation
subsistence farming (a) 1 and 2 (b) 1, 2 and 3
4. Modern industry and commercial agriculture co- (c) 1, 2, 3 and 4 (d) 4 only
existing with subsistence farming and traditional 20. Among the following, which is the reason for different
handicrafts types of economic system?
(a) 1 only (b) 1, 2 and 4 1. Time requirement
(c) 2, 3 and 4 (d) All of these 2. Money requirement
14. Consider the following statements : (a) 1 Only (b) 2 Only
(c) 1 and 2 (d) None of the Above
(1) Higher growth in GDP and population can occur
21. National Income is defined as:
together.
1. It is the total amount of money government earns
(2) Per capita income always decreases with high
through direct tax collection.
population growth.
2. It measures the flow of goods and services in an
Which of the statements given above is/are correct? economy
(a) 1 only (b) 2 only 3. It is the total value of stocks held by a country at a
(c) Both 1 and 2 (d) Neither 1 nor 2 particular point of time
15. In India, rural incomes are generally lower than the urban (a) 1 only (b) 2 only
incomes. Which of the following reasons account for this? (c) 1 and 2 (d) 3 only
(1) A large number of farmers are illiterate and know 22. Consider the following statements in regard to Gross
little about scientific-agriculture. National product (GNP):
(2) Prices of primary products are lower than those of 1. Gross National Product refers to the money value of
manufactured products. total output or production of final goods and services
(3) Investment in agriculture has been lower when produced by the nationals of a country during a given
compared to investment in industry : period of time.
2. In the calculation of GNP, we include the money
(a) 1, 2 and 3 (b) 1 and 2
value of goods and services produced by nationals
(c) 1 and 3 (d) 2 and 3
outside the country
16. Consider the following statements (a) 1 only (b) 2 only
1. The current Economic Census is the seventh in the (c) Neither 1 nor 2 (d) Both1 and 2
series so far conducted in India. 23. Which among the following is the full form of GDP?
2. The Economic Census in India is undertaken by the 1. Gross Data Product
Department of Economic Affairs (Union Ministry of 2. Gross Domestic Product
Finance). 3. Group Domestic Product
Which of the statements given above is/are correct? 4. Group Data Product
(a) Only 1 (b) Only 2 (a) 1 only (b) 2 only
(c) Both l and 2 (d) Neither 1 nor 2 (c) 3 only (d) 4 only
24. Consider the following statements
17. Which of the following occupations are included under
1. National income measures the net value of goods and
secondary sector as per the national income accounts ?
services produced in a country during a year.
1. Manufacturing
2. India is the first country in the world which has
2. Construction officially sponsored family planning programmes
3. Gas and water supply 3. The National Commission on Population was
4. Mining and quarrying constituted on May 11, 2010
Select the correct answer using the codes given below Which of the statements given above is/are correct?
(a) 1, 2, 3 and 4 (b) 1, 2 and 4 (a) 1 only (b) 2 only
(c) 1, 2 and 3 (d) 3 and 4 (c) 3 only (d) 1 and 2
E-12 Structure of Economy and Human Resource in India

25. Consider the following statements about the population of 30. Consider the following:
India: 1. Mehbub ul Haq
1. Due to the high rate population growth, it has not 2. Inge Kaul
been possible to make a significant change in the 3. David Hilton
poverty levels in India. The team which developed HDI was led which of the above
2. The higher population growth, the higher is the rate (a) 1 and 2 (b) 2 and 3
of growth of per capita income. (c) 1 and 3 (d) 1, 2, 3
3. The rapid growth in population is eroding India’s 31. Match the following in regards to labour force vs sector
considerable development gains and steadily where
diminishing its ability to provide a decent quality of Column A Column B
life for a large number of its people. I. Primary Sector 17.3
4. Continuing rapid population growth in the long run II. Secondary Sector 72.1
is likely to mean a lower quality of life for millions III. Tertiary Sector 10.6
of people. (a) 2, 3, 1 (b) 1, 2, 3
(a) 1 and 2 (b) 1 and 3 (c) 3, 2, 1 (d) 3, 1, 2
(c) 1 and 4 (d) 3 and 4 32. Which among the following is the full form of CAGR.
26. The problem of overpopulation can be solved by 1. Compound Annual Growth Rate
1. An effective employment policy, which can absorb 2. Country Annual Growth Rate
the growing number of workers and promote 3. Country Annual Growth Region
economic growth. 4. Compound Annual Growth Region
2. An imaginative family planning programme to Choose the correct answer using the codes given below:
encourage families to adopt the small family norm. (a) 1 only (b) 2 only
(a) 1 only (b) 2 only (c) 3 only (d) 4 only
(c) 1 and 2 (d) Neither 1 nor 2 33. Consider the following statements:
27. The objective of the National Population Policy (NPP) is 1. GDP is the total money value of all final goods and
1. The immediate objective is to address the unmet services produced within the geographical boundaries
needs for contraception, health care infrastructure of the country during a given period of time.
and basic reproductive and child health care. 2. GNP refers to the money value of total output or
2. The medium-term objective is to bring the Total production of final goods and services produced by
Fertility Rate (TFR) to replacement levels of 2.1 by the nationals of a country during a given period of
2010. time
3. The short-term objective is to achieve a stable Which among the above statements is / are correct?
population by 2045, at a level consistent with the (a) 1 only (b) 2 only
requirement of sustainable economic growth, social (c) 1 and 2 (d) Neither 1 nor 2
development and environmental protection. 34. Arrange the following stages of population growth in
Which of the following statements is not correct? chronological order
(a) 1 (b) 2 1. population explosions
(c) 3 (d) All are correct 2. low birth rates and low death rates
28. The following objective helps in the development of India’s 3. stable or slow population growth
economic development since independence. Select the correct answer using the codes given below.
1. A rapidly and technologically progressive economy (a) 1, 2, 3 (b) 3, 1, 2
by democratic means (c) 3, 2, 1 (d) 1, 3, 2
2. A social order based on justice, offering equal 35. Which among the following equation is correct
opportunity to every citizen of the country. 1. GDP=GNP+X-M
3. Increasing population of the country 2. GNP = GDP + X – M
Which among the following is/are the objective? 3. GNP = GDP + NFIA
(a) 1 only (b) 1 and 2 4. NNP = GNP – Depreciation.
(c) 1 and 3 (d) 1, 2 and 3 Select the correct answer using the codes given below.
29. Consider the following statement in regards to Low per (a) 1 and 2 (b) 1, 2 and 3
capita Income: (c) 2, 3 and 4 (e) 1, 3 and 4
1. India is developing economy 36. Consider the following statements about personal Income:
1. Personal income is that income which is actually
2. India’s per capita income level is higher as compared
obtained by nationals.
with other developed countries
2. Personal Income = National income – undistributed
3. According to World Development Report (2011)
profits of corporation – payments for social security
India’s per capita income was US $ 1499 in 2014 provisions – corporate tax + government transfer
4. India’s per capita income falls in category of Lower payments + Business transfer payments + Net
Middle Income Countries(LMC) interest paid by government.
Which of the statements given above is/are correct? Select the correct answer using the codes given below:
(a) 1 and 2 (b) 1, 2 and 3 (a) 1 only (b) 2 only
(c) 1, 2 and 4 (d) 1, 3 and 4 (c) 1 and 2 (d) Neither 1 nor 2
Structure of Economy and Human Resource in India E-13

37. Which of the following are the roles of government in the 2. To review the working of the national economic plan
economy: from time to time.
1. Provision of public goods 3. To consider important questions of social and
2. Provision of merit goods economic policy affecting national development
3. Poverty reduction Select the correct answer using the codes given below?
Select the correct answer using the codes given below: (a) 1 only (b) 2 only
(a) 1 only (b) 2 only (c) 3 only (d) 1, 2 and 3
(c) 3 only (d) 1, 2 and 3 44. Consider the following systems was/were provided by the
38. Consider the following statement related to population Government of India, Act 1935:
1. An effective employment policy, which can absorb 1. Separation of provincial budgets from the central
the growing number of workers and promote budget for the first time.
economic growth. 2. Introduction of portfolio system in the Executive.
2. An imaginative family planning programme to 3. Establishment of a Federal public service commission.
encourage families to adopt the small family norm.
Which of the systems given above is correct:
Which among the above statements is / are not correct?
(a) 3 only (b) 2 only
(a) 1 only (b) 2 only
(c) 1 only (d) 1, 2 and 3
(c) 1 and 2 (d) Neither 1 nor 2
45. Which of the following are correct
39. Which among the following statement characterize Indian
Economy: 1. These measures include a reduction in spending.
1. pre-dominance of agriculture 2. These measures include an increase in tax
2. low per capita income 3. These measures include reduction in budget deficit.
3. massive unemployment Select the correct answer using the codes given below in
Select the correct answer using the codes given below regards to the austerity measures taken by a country
(a) 1 only (b) 1, 2 and 3 going through adverse economy conditions:
(c) 2 only (d) 1 and 2 (a) 1 only (b) 2 only
40. Which among the following states have the first family (c) 3 only (d) 1, 2 and 3
planning clinic? 46. Consider the following statement
1. New Delhi 2. Uttar Pradesh 1. The immediate objective is to address the unmet
3. Karnataka 4. Maharashtra needs for contraception, health care infrastructure
Select the correct answer using the codes given below and basic reproductive and child health care.
(a) 1 only (b) 2 only 2. The medium-term objective is to bring the Total
(c) 3 only (d) 4 only Fertility Rate (TFR) to replacement levels of 2.1 by
41. Consider the following statement: 2010.
1. High degree of disparity in income/wealth distribution 3. The long-term objective is to achieve a stable
is prevailed in India which is a characteristics feature population by 2045, at a level consistent with the
of developing economy. requirement of sustainable economic growth, social
2. In India, almost 60-70% of the total population still development and environmental protection.
resides in rural areas and hence they depend on Which of the following statement is true for NPP objective?
agriculture for their livelihood. (a) 1 only (b) 2 only
3. India’s per capita income level is much high as (c) 3 only (d) 1, 2 and 3
compared with other developed countries 47. Consider the following Statement
Which among the following statement is not correct:
1. The relationship between population growth and
(a) 3 only (b) 1 and 2
economic development can be explained by the
(c) 1 and 3 (d) 1 only
Theory of Demographic Transition for contemporary
42. Consider the following statement in regards to Unbalanced
developed nations.
Economic Development
1. India has not yet achieved the goal of balanced 2. The three stages of population growth are stable or
economic development. slow population growth, population explosions, low
2. Balanced Growth mentions to a definite type of birth rates and low death rates.
economic growth that is maintainable in the long Select the correct statement using the code given below:
term. (a) 1 only (b) 2 only
3. Balanced growth is contrasting to the high and low (c) 1 and 2 (d) Neither 1 nor 2
nature of economic cycles. 48. Consider the following statements:
Which among the following statement is correct: 1. The basic objective of the Viability Gap Funding
(a) 1 only (b) 2 only Scheme is to promote PPP investment in Infrastructure
(c) 3 only (d) 1, 2 and 3 2. The Infrastructure Projects are funded up to 20% by
43. Which of the following is/are the functions of the National the Central Government
Development Council of India? Which among the above code is / are correct?
1. To prescribe guidelines for preparation of the national (a) 1 only (b) 2 only
economic plan. (c) 1 and 2 (d) Neither 1 nor 2
E-14 Structure of Economy and Human Resource in India

49. Which among the following sector of Indian Economy 54. With reference to Indian economy, consider the following
is maximum dependent on economic developments in statements:
advanced nations? 1. The rate of growth of Real Gross Domestic Product
1. Manufacturing Sector has steadily increased in the last decade.
2. Agricultural Sector 2. The Gross Domestic Product at market prices (in
3. Mining Sector rupees) has steadily increased in the last decade.
4. Services Sector Which of the statements given above is/are correct?
Which statement is/are correct (a) 1 only (b) 2 only
(a) 1 only (b) 2 only (c) Both 1 and 2 (d) None of these
(c) 3 only (d) 4 only
50. Consider the following statement: (d) Neither 1 nor 2
Matching Based MCQ
1. Increase in annual private investment.
2. Increase in real national income.
55. Matching Based MCQ
3. Increase in real per capita income.
Match columns A and B
4. Increase in net annual investment.
Column A Column B
Which among the following is considered to be the best
I. Mixed economy a. India has strong powers
measure of an increase in a country’s economic efficiency?
invested in the Centre with
(a) 1 only (b) 2 only
financial independence
(c) 3 only (d) 4 only
and adequacy
51. Role of the government in the economy is II. Fedral economy b. Public sector business
1. Provision of public goods enterprise exist with
2. Provision of merit goods Private sector
3. Poverty reduction III. Over population c. In each decade Indian
Which among the following code is/are correct population is increased By
(a) 1 only (b) 2 only 20%
(c) 3 only (d) 1, 2 and 3 IV. Disparities in d. India’s per capita income
52. Human Development Index comprises literacy rates, life Income Distribution is about 1/40 of US level
expectancy at birth and Of per capita
1. GDP per head in the US dollars. (a) I-b, II-a, III-d, IV-c
2. GDP per head at real purchasing power. (b) I-a, II-b, III-c, IV-d
3. GNP in US dollars (c) I-a, II-d, III-b, IV-c
4. National Income per head in US dollars. (d) I-c, II-a, III-b, IV-d
Which of the statement is correct 56. Match the following
(a) 1 only (b) 2 only Column A Column B
(c) 3 only (d) 4 only I. 1962 a. The objective of NPP
53. The national income of a country for a given period is equal was announced by govn
to II. 1990 b. First HDR report was
1. total value of goods and services produced by the published
nationals III. 2000 c. Family planning programme
2. sum of total consumption and investment expenditure was launched
3. sum of personal income of all individuals (a) I-c, II-b, III-a
4. money value of final goods and services produced (b) I-a, II-b, III-c
Which of the following code is/are correct (c) I-b, II-a, III-c
(a) 1 only (b) 2 only (d) I-c, II-a, III-b
(c) 3 only (d) 4 only
Structure of Economy and Human Resource in India E-15

EXERCISE-1 residents located in that country. GNP measures


1. (b) 2. (b) 3. (d) 4. (a) 5. (a) 6. (d) the value of goods and services that the country’s
7. (c) During last decade tertiary sector has shown citizens produced regardless of their location.
remarkable expansion. The economy is divided 15. (c) Tertiary sector of the Indian Economy contributes
into three sector on the basis of activities–primary, largest to the GNP. During last decade tertiary
secondary and tertiary. Primary sector is involved sector has shown remarkable expansion. The
into agriculture, Secondary sector is involved economy is divided into three sectors on the basis of
into manufacturing, mining, construction while activities-Primary, Secondary and tertiary. Primary
tertiary sector is involved into trade, transport, sector is involved into agriculture, Secondary sector
communication, banking & other services. In the last is involved into manufacturing, mining, construction
decade India has expanded maximum in providing while tertiary sector is involved into trade, transport,
services like IT, Telecommunication, Healthcare, communication, banking & other services. In the last
Tourism which is contributing around 60% to GDP. decade, India has expanded maximum in providing
8. (b) India is a labour-surplus economy because in India services like IT, Telecommunication, Healthcare,
there is disguised unemployment along with under- Tourism which is contributing around 60% to GDP.
employment which means that qualified, skilled 16. (c) Per capita income of a country derived from
workforce willing to work is available but there are National Income and population both. Per Capita
not enough employment opportunities. Income is obtained by dividing National Income by
9. (d) total population of the country. Per capita income,
10. (b) In India, planned economy is based on socialist also known as income per person, is the mean
system in which all have equal opportunities to income of the people in a country . It is calculated
education, healthcare, non exploitation, equality of by taking a measure of all sources of income in the
wealth etc. The concept was borrowed from Russia aggregate (such as GDP or Gross national income)
and is based on achieving directive principles and dividing it by the total population.
mentioned in our constitution. 17. (d) Dadabhai, known as the Grand Old Man, wrote
11. (c) The term was coined by Indian economist Raj the book ‘Poverty and Un-British Rule in India’
Krishna. The Hindu rate of growth is a derogatory describing the theory of economic drain of India
term referring to the low annual growth rate of during British rule.
the socialist economy of India before 1991, which 18. (d) The most appropriate measure of a country’s
stagnated around 3.5% from 1950s to 1980s. The economic growth is its per capita real income. Per
word “Hindu” implies that the Hindu outlook of capita income is average income, a measure of the
fatalism and contentedness was responsible for the wealth of the population of a nation. It is used to
slow growth. measure a country’s standard of living thus a better
12. (d) Investment method is not a method of measurement indicator of economic growth.
of National Income. There are three methods of 19. (b) India, among the countries has the lowest GDP
measurement; income method, product or value per capita. GDP per capita is as follows- India
added method and the expenditure method. In the 1489 USD,China 6091USD, Sri Lanka 2923 USD,
initial phase, production of goods and services takes Indonesia 3556 USD.
place. During the course of production payment 20. (d) 21. (b) 22. (c)
is made to all factors of production like wages to 23. (b) Green shoots is a term used colloquially and
labour etc. Once the production completes the output propagandistically to indicate signs of economic
is distributed for different uses like consumption etc. recovery during an economic downturn. It was first
13. (a) Rate of GDP growth is a major indication of the state used in this sense by Norman Lamont, the then
of the economy of a country. Economic growth is Chancellor of the Exchequer of the United Kingdom,
the increase in the market value of the goods and during the 1991 recession.
services produced by an economy over time. It 24. (b) GDP deflator is an economic metric that accounts for
is conventionally measured as the percent rate of inflation by converting output measured at current
increase in gross domestic product. Gross domestic prices into constant-collar GDP. The GDP deflator
product (GDP) is the market value of all officially shows how much a change in the base year’s GDP
recognized final goods and services produced within relies upon changes in the price level.
a country in a year, or other given period of time. 25. (d) Human Development Report the HDI combines
14. (b) GNP is the total value of all final goods and services three dimensions: A long and healthy life: Life
produced within a country in a particular year, plus expectancy at birth, Education index: Mean years of
income earned by its citizens (including income schooling and Expected years of schooling and A
of those located abroad), minus income of non- decent standard of living: GNI per capita (PPP US$).
E-16 Structure of Economy and Human Resource in India

26. (d) National Income ignores sale of land. 19. (c)


27. (a) The sum of a nation’s gross domestic product 20. (c) Different types of economy are dependent on time
(GDP) plus net income received from overseas. which helps in providing several ways to structure
Gross national income (GNI) is defined as the sum an economy.
of value added by all producers who are residents in 21. (b) National income measures the net value of goods
a nation, plus any product taxes (minus subsidies) and services produced in a country during a year and
not included in output, plus income received from it also includes net earned foreign income.
abroad such as employee compensation and property 22. (d) Gross National Product refers to the money value of
income. total output or production of final goods and services
28. (a) National product at factor cost is equal to net produced by the nationals of a country during a given
domestic product at factor cost + Net factor Income period of time, generally a year. In the calculation
from Abroad. of GNP, we include the money value of goods and
EXERCISE-2 services produced by nationals outside the country.
1. (a) 2. (a) 3. (d) 23. (b)
4. (d) Infant mortality rate indicates the number of infant 24. (d) The total of national income measures the flow of
death under one years of age per 1000 live birth goods and services in an economy.
under one year of age. The increasing population of India hinders the
5. (d) Megacities of India have more than 5 million growth and progress of country.
population. But not all the major megacities are The National Commission on Population was
located on the sea coast or are sea ports. Delhi, constituted on May 11, 2000 under the Chairmanship
which is the national capital is not a sea port. of the Prime Minister Shri Atal Bihari Vajpayee to
6. (c) 2, 3, 1, i.e. High birth rate with high death rate, provide overall guidance for population stabilisation
High birth rate with low death rate, Low birth rate by promoting synergy between demographic,
with low death rate. educational, environmental and development
7. (d) 8. (d) 9. (c) 10. (a) programmes.
11. (d) 12. (c) 13. (c) 25. (b) The increasing population of India hinders the
14. (c) Higher growth in GDP and population can occur economic growth of the country. It is important to
together. Per capita income always decreases with control the population growth.
high population growth as income per person 26. (c) Over population of India can be controlled by
decreases with rise in population. providing maximum employment and giving proper
15. (a) In India a large number of farmers are illiterate education how to control population.
and know little about scientific-agriculture, prices 27. (c) The long-term objective of NPP is to achieve a stable
of primary products are lower than those of
population by 2045, at a level consistent with the
manufactured products and Investment in agriculture
requirement of sustainable economic growth, social
has been lower when compared to investment in
development and environmental protection.
industry .All these reasons are responsible for lower
28. (b) Since independence, India’s economic development
rural income.
has been guided by the twin objectives (1) a rapidly
16. (d) In 1976, Government of India launched a plan
and technologically progressive economy by
scheme called “Economic Census and Surveys”.
democratic means; and (2) a social order based on
In 1977 Central Statistical Organisation conducted
First economic census in collaboration with the justice, offering equal opportunity to every citizen of
Directorate of Economics & Statistics (DES) in the the country.
States/Union Territories. Current Economic Census 29. (d) India is developing economy. Its per capita income
is Sixth Economic Census held in 2013. level is much low as compared with other developed
The Central Statistics Office (CSO) in the Ministry of countries. According to World Development Report
Statistics and Programme Implementation (MOSPI) (2011) India’s per capita income was US $ 1499
conducted the Sixth Economic Census during in 2014. The per capita income in United States is
January, 2013 to April, 2014 in collaboration with US $ 47240 and hence India’s per capita income
Directorates of Economics and Statistics in all the is about 1/40 of US level of per capita. The per
States and Union Territories. capita income of 2015 has increased by 10.1%, (as
17. (c) Primary sector-Agriculture, fishing, mining and compared to 2014) as stated by Minister of State for
quarrying Statistics and Programme Implementation Gen. V K
Secondary sector- Manufacturing and construction, Singh. The national income in 2014-15 stood at Rs.
Gas and water supply 11,217,079 crore. With the mentioned per capita
Tertiary sector- trade, transport etc, finance & real income India falls in category of Lower Middle
estate, community and personnel services Income Countries(LMC)
18. (b) Economics takes care of various needs and wants of 30. (b) The team which developed HDI was led by Mehbub
life. ul Haq and Inge Kaul.
Structure of Economy and Human Resource in India E-17

31. (a) Occupational structure refers to distribution of work business transfer payments and net interest paid by
force in the different occupations of the country. As the government.
economy grows, labour force shifts from primary 37. (d) Provision of public goods like defence which no
sector to secondary and tertiary sector. private agency can be entrusted with. Provision of
32. (a) merit goods - goods like education and housing -
33. (c) In the calculation of GNP, we include the money value through policies which encourage the consumption
of goods and services produced by nationals outside of such goods and discourage the use of non-merit
the country. Hence, income produced and received goods like cigarettes. Poverty reduction is considered
by nationals of a country within the boundaries of a prime responsibility of all modern governments, as
foreign countries should be added in Gross Domestic the market caters to the needs of only those who can
Product (GDP) of the country. Similarly, income afford to pay, not of the poor.
received by foreign nationals within the boundary of 38. (d) The problem of over population can be solved by
the country should be excluded from GDP. two ways
34. (b) First stage of stable or slow population growth: • An effective employment policy, which can absorb
The growth of population was slow due to the high the growing number of workers and promote
death rate which nullified the high birth rate. In this economic growth.
stage, these economies were primitive and primarily • An imaginative family planning programme to
agrarian, with widespread illiteracy, poor sanitation encourage families to adopt the small family norm.
and health care conditions, negligible knowledge 39. (b) The Indian economy constitute of predominance
of family planning and large family sizes which of agriculture, low per capita income, massive
contributed to factors such as high fertility rate and unemployment.
high death rate. In the period between the first and 40. (c) The family planning programme was officially
the second stages, the death rates start reducing launched in 1962. The first two family planning
and birth rates remain stable which brings an clinics were opened in Karnataka in 1930 which
imbalance in the economy. Measures like diseases were supposed to be the first such in the world
control, improving nutrition levels, and sanitation 41. (a) India is a developing country. Its per capita income
improvement are implemented to reduce death rates, level is much low as compared with other developed
but the measures for controlling birth rates are not countries. Predominance of Agriculture : In India,
implemented, which results in population explosion. almost 60-70% of the total population still resides in
This required a period of transition for adjustment, rural areas and hence they depend on agriculture for
thus bringing the second stage of transition. their livelihood.
Second stage of population explosions: In this stage 42. (d) India has not yet achieved the goal of balanced
rise in income levels contributed to improvement economic development. Balanced Growth mentions
in health-care, education, disease control and so to a definite type of economic growth that is
on which in turn contributed to reducing the death maintainable in the long term. Balanced growth is
rates. This accelerated the growth of population. contrasting to the high and low nature of economic
The family size reduced and improved trade and cycles. According to latest data available about 64%
economic conditions resulted in more food and better of total labour force is dependent on agriculture,
food habits which further helped the population to 16% on industries and the rest about 20% on trade,
grow at a much higher rate than in the first stage. transport and other services.
The demographic changes brought imbalances in the 43. (d) The National Development Council (NDC) or the
economy, creating conditions for further transition Rashtriya Vikas Parishad is the apex body for decision
of society. making and deliberations on development matters
Third stage of low birth rates and low death rates: in India, presided over by the Prime Minister. The
Modernization and industrialization changed the functions of the Council are to prescribe guidelines
living pattern in such nations, the rural population for the formulation of the National Plan; to consider
shifted to cities and family sizes reduced to become the National Plan as formulated by the Planning
nuclear families. The standard of living increased Commission; to consider important questions
which further brought down mortality rates and birth of social and economic policy affecting national
rates. As a result, the growth of population declined. development; and to review the working of the
35. ( c) X =Income from Abroad (income earned and Plan from time to time and to recommend such
received by nationals within the boundaries of measures as are necessary for achieving the aims
foreign countries.)M = Income received by and targets set out in the National Plan.
foreign nationals within the country. 44. (a) The limited advisory function accorded to the Public
36. (c) Personal income is a flow concept. It is the income that Service Commission and the continued stress on this
is obtained by nationals. Personal income is obtained aspect by the leaders of our freedom movement
by subtracting corporate taxes and payments made resulted in the setting up of a Federal Public Service
for social securities provision from national income Commission under the Government of India Act,
and adding to it government transfer payments, 1935. The Federal Public Service Commission
E-18 Structure of Economy and Human Resource in India

became the Union Public Service Commission after economic conditions resulted in more food and better
Independence. The portfolio system in the Executive food habits which further helped the population to
was introduced by the Indian constitution council act, grow at a much higher rate than in the first stage.
1861. The separation of provincial budgets from the The demographic changes brought imbalances in the
central budget was introduced by the Indian councils economy, creating conditions for further transition
Act, 1919. of society.
45. (d) Austerity describes policies used by governments Third stage of low birth rates and low death rates:
to reduce budget deficits during adverse economic Modernization and industrialization changed the
conditions. These policies may include spending living pattern in such nations, the rural population
cuts, tax increases. This is done in economic crisis shifted to cities and family sizes reduced to become
situation to improve the credit rating of the countries nuclear families. The standard of living increased
going through adverse economic condition. which further brought down mortality rates and birth
46. (d) The Government of India in February 2000, rates. As a result, the growth of population declined.
announced the National Population Policy (NPP) 48. (c)
with the objectives The immediate objective is to 49. (b) Indian Economy is highly dependent on agriculture
address the unmet needs for contraception, health because still the maximum population stays in rural
care infrastructure and basic reproductive and area.
child health care. (ii) The medium-term objective 50. (c) Indian economy is increased by real per capita
is to bring the Total Fertility Rate (TFR) to income
replacement levels of 2.1 by 2010. (iii) The long- 51. (d) Provision of public goods like defence which no
term objective is to achieve a stable population by private agency can be entrusted with.
2045, at a level consistent with the requirement of Provision of merit goods - goods like education
sustainable economic growth, social development and housing - through policies which encourage the
and environmental protection. consumption of such goods and discourage the use
47. (c) Population growth directly impacts on Indian of non-merit goods like cigarettes.
economy. The three stages of population growth are: Poverty reduction is considered a prime responsibility
First stage of stable or slow population growth: of all modern governments, as the market caters to
The growth of population was slow due to the high the needs of only those who can afford to pay, not
death rate which nullified the high birth rate. In this of the poor.
stage, these economies were primitive and primarily 52. (d) Human Development Report the HDI combines
agrarian, with widespread illiteracy, poor sanitation three dimensions: A long and healthy life: Life
and health care conditions, negligible knowledge expectancy at birth, Education index: Mean years
of family planning and large family sizes which of schooling and Expected years of schooling and
contributed to factors such as high fertility rate and A decent standard of living: GNI per capita (PPP
high death rate. In the period between the first and US$).
the second stages, the death rates start reducing 53. (d) National Income is the money value of all the final
and birth rates remain stable which brings an goods and services produced by a country during a
imbalance in the economy. Measures like diseases period of one year. National Income consists of a
control, improving nutrition levels, and sanitation collection of different types of goods and services of
improvement are implemented to reduce death rates, different types.
but the measures for controlling birth rates are not 54. (c) Both statements are correct.
implemented, which results in population explosion. 55. (a) There are different types of economic types. The
This required a period of transition for adjustment, benefit of mixed economy is to achieve a welfare
thus bringing the second stage of transition. state with socialistic pattern of society. India is a
Second stage of population explosions: In this stage federal economy. India is developing economy.
rise in income levels contributed to improvement India’s population is getting increased in each
in health-care, education, disease control and so decade.
on which in turn contributed to reducing the death 56. (a) Family planning programme was launched in 1962.
rates. This accelerated the growth of population. First HDR report was published in 1990. The
The family size reduced and improved trade and objective of NPP was announced by govn in 2000.
PLANNING, UNEMPLOYMENT
2
& POVERTY IN INDIA

Introduction
Economic planning is the making of major economic decisions what and how much is to be produced, how, when and
where it is to be produced, and to whom it is to be allocated by the comprehensive survey of the economic system as whole.
(H.D.Dickinsom)

Composition Functions

Planning in India Four decades of planning show that India’s economy, a mix
of public and private enterprise, is too large and diverse to be
Planning in India starts in 1930s. Even before independence, wholly predictable or responsive to directions of the planning
the colonial government had established a planning board authorities.
that lasted from 1944 to 1946. Before independence private
industrialists and economists published three development Important Dates
plans in 1944. India’s leaders adopted the principle of formal • 1934: M. Visvesvaryya, in his book ‘Planned Economy of
India’, advocates the necessity of planning in the country
economic planning soon after independence as an effective
much before Independence.
way to intervene in the economy of faster growth and social • 1944: Bombay Plan, published in January 1944, prepared
justice. by eight leading industrialist of Bombay.
E-20 Planning, Unemployment and Poverty in India

• Gandhian Plan put forward by S.N. Agrawal (1944). Composition: NITI Aayog will have Prime Minister as its
• 1944: Planning Development Council was set up under chairman, one Vice-Chairman cum chief-executive officer,
the chairmanship of A. Dalal. 5 fulltime members and 2 part time members, apart from 4
• Peoples Plan drafted by M.N. Roy (1945). central government ministers.
• 1946: Interim Government sets up the Planning Advisory
Board. Five Year Plans
• 1947: Economic Programme Committee was set up under
the chairmanship of Jawaharlal Nehru. The development plans drawn up by the Planning Commission
• 1950: Planning Commission was set up. to establish India’s economy in five-year phases are called
• 2015: Formation of NITI Aayog. Five-Year Plans. A five-year plan is an indicative plan of
action reflecting largely the intent of the government for that
Planning Commission period at the national, regional, and sectorial level.
The Planning Commission was established in 1950, in accordance First Plan (1951-56)
with Article 39 of the Directive Principles of the Constitution of
Major Objective: Agriculture, Price Stability, Power &
India headed by Prime Minister. The Commission is independent
Transport.
of the Cabinet. A staff drafts plans under the guidance of the
Commission; the draft plans are presented for approval to the The first five year plan focused on to stimulate balanced
National Development Council, which consists of members of economic development while correcting imbalances caused by
the Planning Commission, the Chief Ministers of the States and World War II and partition various objectives were.
Administrators of UTs and All Union Ministers. The Council • It was based on Harrod Domar Model.
can make changes in the draft plan. After Council approval, the • Its Highest priority on agriculture, irrigation and power
projects.
draft is presented to the Cabinet and subsequently to Parliament,
• Rate of investment was targetted at 7% of national income.
whose approval makes the plan an operating document for
• National income grew by 18% and per capita income by
Central and State governments.
11%.
Jawaharlal Nehru was the first chairman of the Planning • Food production increased by 20%. Under the first five
Commission by virtue of his being the Prime Minister of India. year plan provision was made to spend a total of ` 2,378
crore during the plan period. But the actual expenditure
Functions
outcome to ` 1960 crore only.
(i) Assessment of the material, capital and human resources of
the country, including technical personnel and formulation Second Plan (1956-61)
of proposals for the augmentation of such resources;
Major objective: Rapid Industralisation.
(ii) Formulation of plans for effective and balanced
utilization of resources; • It was based on Mahalnobis Model.
• It targetted a 25% increase in national income through
(iii) Defining stages in which the plan should be carried out;
rapid industrialisation. Actual achievement was only 20%.
(iv) Determination of the resources necessary for implemen- • Goal of establishing the socialistic pattern of society.
tation of the plans; • Rate of investment planned to be raised from seven per
(v) Appraisal from time to time of the progress achieved; cent to 11% of national income.
(vi) Public co-operation in national development; • Per capita income rose by eight per cent.
(vii) Perspective planning; • Rapid industrialization with particular emphasis on
National Planning Council: Is an advisory body attached development of basic and heavy industries. Large
to the Planning Commission It was established in 1965. It industries including steel plants (Durgapur, Bhilai
includes experts representing a cross-section of the Indian and Rourkela) were set up. The locomotive factory at
economy. Chittaranjan and Coach factory at Perambur were other
major projects of this period.
NITI Aayog Third Plan (1961-66)
The government of India has replaced Planning Commission Major Objective : Self sustaining growth.
with a new institution named NITI Aayog (National Institution • Emphasis on basic industries continued but agriculture
for Transforming India). and allied sectors (irrigation and power) were allocated
The institution will serve as ‘Think Tank’ of the Government 35% of the outlay.
- a directional and policy dynamo. • A series of crises - China war (1962), Nehru’s death
NITI Aayog will provide Governments at the Central and State (1964), Pakistan war (1965) and Shastri’s death
Levels with relevant strategic and technical advice across the (1966), major drought (1965-66) - marred the smooth
spectrum of key elements of policy, this includes matters of implementation of the plan.
national and international importance on the economic front, • Growth rate of only 2.2% achieved as against a target of
dissemination of best practices from within the country as well 5% per annum.
as from other nations, the infusion of new policy ideas and • Inflation (36%) ate up much of the achievement; Rupee
specific issue-based support. devaluation (1966).
Planning, Unemployment and Poverty in India E-21

Holiday Plan (1966-69) • Actual growth of national income was higher at 5.3%
After the disastrous experience of the Third Plan, a plan (against a target of 5.2%).
holiday was declared for three years. All available resources • Increase of 16% per annum in real investment in fixed
were mobilised for building a buffer stock and for stepping up asset by private sector.
food production learning from the experience of near-famine • Poverty declined from 48.3% in 1977-78 to 37.4% in
years (1965-66). 1983-84.
Favourable monsoons and technological break-through in
Seventh Plan (1985-90)
wheat popularly known as ‘green revolution’ reduced the
inflationary pressure. Nationalisation of banks was another Major objective: Growth, modernisation, Self-reliance
major step during this period. and Social justice.
• Strong emphasis on creation of productive employment
Fourth Plan (1969-74) on farm as well as rural subsidiary occupations.
• Stress on increasing the production of food grains,
Major objective: Growth with stability and progress towards
oilseeds, sugar, textiles, domestic fuel and housing.
self-reliance.
• Outward-looking strategy with exports receiving high
• Emphasis on growth with distributive justice. priority.
• A substantial increase in the outlay for family planning
• Tempo of domestic and external liberalisation hastened.
(` 278 crores from ` 25 crores in third plan).
• The Plan also had a 15-year perspective (1985-2000) for
• Goal of attaining self-sufficiency in agriculture and indus-
trial production. (In agriculture, growth rate of 5% per removal of poverty, providing for basic needs, achieving
annum and in industrial production growth rate of 8% to universal elementary education and total access to health
10% per annum were targeted.) facilities.
• Poor achievement of targets – national income grew by • Average annual growth rate during the plan period was
3.3% per annum; per capita income by 1.2% per annum; 5.6% (target 5%).
agricultural production by 2.8%; industrial production • Agriculture grew at 4.1% against a target of 4%.
by 3.9%. • Manufacturing industries achieved a growth rate of 8.8%
(target 8%).
Fifth Plan (1974-79) • There was a severe short fall in mining sector (5.6%
Major objective: Poverty eradication and attainment of self against a target of 13%).
reliance. • Social sector performance fell far short of targets–
especially in housing for the landless, elementary
• Twin objectives of poverty eradication and attainment of
education and general poverty alleviation.
self-reliance.
• A National Programme for Minimum Needs including Eighth Plan (1992-97)
elementary education, safe drinking water, health care,
and shelter for landless.
Major objective: Human development.
• Adequate collection and distribution system in order to • The plan was launched in 1992 after the plan holiday
provide the commodities of necessary consumption to the during the economically and politically difficult days of
poor people on reasonable and stable prices. 1990-91 and 91-92.
• Stress on Export Promotion and Import Substitution. • It was Manmohan-Rao (F.M- P.M.) Era of economic
• A growth rate of 5.2% (against a target of 4.4%) liberalization.
achieved. • Modernisation of industries was focussed.
• Agricultural production increased by 4.2% – the highest • India became member of WTO to pace with world
so far. economics.
• Moderate inflation of 2.1% per annum during the The economic paradigm had also considerably shifted with
Emergency years (1975-1977). the acceptance of liberalisation and the need for market
• The Janata Government terminated the Plan in 1978. forces to play a greater role.
• Employment generation to be speeded up to achieve full
Sixth Plan (1980-85) employment by 2000.
• Total literacy to be achieved in the 15-35 age group by
Major objective : Poverty Alleviation. covering an additional 110 million people.
• Janata Government had adopted a Sixth Plan (1978-83), • Restructuring the systems of economic management
which was conceived as a rolling plan. Mrs. Gandhi’s through public sector reforms, including selective
Government in 1980 abandoned this and a new sixth plan disinvestment.
was drafted. • The Eighth Plan was to walk on ‘two legs’ - one leg
• Poverty alleviation gives the top priority. of alleviating poverty and removing unemployment; and
• Qualitative improvement in the living standards of people the other ‘leg’ providing a ‘safety net’ for those who
by means of Minimum Need Programme (MNP). will be affected by the structural adjustment programme.
• Schemes for transferring skills (TRYSEM) and assets The plan had thus built in the ‘human face’ element of
(IRDP) and providing slack season employment (NREP). adjustment.
E-22 Planning, Unemployment and Poverty in India

• To strengthen the basic infrastructure (energy, transport, the per capita income in the next ten years, to reduce the
communication, irrigation) in order to support the decadal population growth from 21.3% (1991-2001) to
development process on a sustainable basis. 16.2% by 2010-11 and to ensure that the growth in gainful
• The plan was explicitly indicative with a long-term employment kept pace with the addition to the labour force.
policy approach replacing the target approach. • Against the ambitious target of 8%, the economy grew at
• Per capita national output grew by 3.9% per annum. the rate of 7.7% on an average during the 10th Plan period.
But, this growth masked considerable distortion in the However, an evaluation by the Planning Commission
distribution front. From data regarding inflation and noticed that while the rate of growth was impressive, it was
price indices, there is evidence that the poor became lop-sided and did not benefit all people alike. For too
poorer despite ‘the safety net’. many people still lacked the basic requirements for a
• Actual employment growth was only two percent against decent living in terms of nutritional standards, access to
a target of 2.6%. education and basic health, and also many other public
• Annual growth rate achieved in the Plan period is 6.8% services such as water supply and sewage. The benefits
against the target of 5.6 %. did not reach fully some disadvantages sections like the
Scheduled Castes and Tribes and minorities. Regional
Ninth Plan (1997-2002) imbalances - both across states and even within states -
Major objective: Growth with Equity and Distributive were also noticed.
Justice. Eleventh Plan (2007-12)
• This objective was sought to be achieved through a policy
of concentrating on agriculture and rural development to Major objective: Faster and more Inclusive Growth
provide more employment; ensuring food and nutritional • The Eleventh Plan targets to resolve the regional
security to all, especially the vulnerable; providing imbalance still prevailing in the country. The Plan
basic minimum needs in a time-bound manner; curbing document, sub-titled Inclusive Growth, outlines a
population growth; environmental sustainability of strategy for making growth both faster and more
development; empowerment of women; promotion of inclusive. Encouraged by the achievement of a rate of
Panchayati Raj institutions and strengthening efforts to 7.7% on an average during the 10th Plan, itself a target
build self reliance. of 9% growth during the Plan period, with acceleration
during the period to reach 10% by the end of the Plan.
• The development strategy emphasised the role of markets
and the need for government to intervene to promote • The target of 9% growth requires the average rate of
a degree of competition through suitable legislation. investment to rise from 32% (during 10th Plan) to 37%
in the current plan, reaching 39% at the end of the plan
Licence Raj was to be ended. The Plan emphasised co-
period. Private investment which has contributed 78%
operative federalism. It also stressed the importance of
of the investment during the 10th Plan is expected to
infrastructural development.
maintain its share. Public investment is expected to be
• The Plan was indicative in nature, focusing on policies. It
maintained at the same level of 22% as in the 10th Plan.
also provided a 15-year perspective. It aimed to achieve a
Planning Commission has framed a plan for achieving
growth rate of 8% per annum in the medium term and a
faster growth with greater inclusiveness which involves
rate of 6.5% during the plan period (’97-’02).
the following interrelated components:
• The plan envisaged the creation of 52 million jobs as
(i) a continuation of the policy of economic reform
against the demand for job opportunities for 60.5 million
which has created a competitive private sector
persons. The backlog of unemployment, which was 7.5
capable of benefiting from the opportunities
million at the close of the eighth Plan, was expected to be
provided by greater integration with the world;
6.6 million at the end of the Ninth Plan.
(ii) more emphasis on agriculture,
• The GDP grew only by 5.35% per annum during the plan
(iii) improved access to essential services in health and
period against the target of 6.5%. The shortfall was due
education (including skill development);
to poor performance by agricultural and industrial sectors,
(iv) special thrust on infrastructural development;
as explained in the table below.
(v) special attention to the needs of disadvantaged
Performance of Agricultural and Industrial Sectors
groups, and
Sector 8th Plan 9th Plan (vi) good governance at all level, central, state and
Agriculture 4.69 2.06 local.
Manufacturing 7.58 4.51 • The broad targets fixed by the 11th Plan include a 4%
per cent growth in Agriculture sector, 10% growth in
Services 7.54 7.78 Industries and Minerals, and investment in infrastructure
Total 6.68 5.35 to grow from 5.43% of GDP in 06-07 to 9.43% by the
Source: Planning Commission end of the 11th Plan.
• The total public sector outlay in the Eleventh Plan (both
Tenth Plan (2002-07) Central and States and including the PSEs) is estimated at
Major objective: 8% annual growth rate. ` 36,44,718 crore. Of this total, the share of the Centre
• The Tenth Plan laid down an ambitious target of 8% (including the plans of Public Sector Enterprises (PSEs)
annual growth rate for the economy, against the will amount to ` 21,56,571 crore, while that of the States
prevailing rate of 5.5%. Its long term vision was to double and union territories (UTs) will be ` 14,88,147 crore.
Planning, Unemployment and Poverty in India E-23

The 27 National Targets under 11th Plan • Achieving Growth rate of 9.0 to 9.5% needs strong
The Plan has adopted 27 targets at the national level to policy action. Major sectoral challenges are Energy,
ensure inclusive growth. These are related to: (i) income and Water and Environment, which need to be addressed
poverty, (ii) education, (iii) health, (iv) women and children without sacrificing growth. To find resources to create a
(v) infrastructure and (vi) environment. world class infrastructure in the country.
(i) Targeted growth of GDP at 9% per year. • Require better performance in agriculture, for growth to
be more inclusive. Faster creation of jobs, especially in
(ii) To raise industrial growth rate from 9.2% in 10th Plan to
manufacturing, and stronger effort at health, education
10% in 11th Plan.
and skill development are given importance.
(iii) To reduce unemployment among educated youth to less • Also, need to improve effectiveness of programmes
than 5%.
directly aimed at the poor, special programmes for
(iv) To reduce Infant Moraling Rate (IMR) to 28 and Material
socially vulnerable groups, and special plans for
Morality Rate (MMR) to 1 per 1000 on live births by the
disadvantaged/backward regions.
end of plan.
(v) To increase sex-ratio to 935 by 2011-12 and 950 by Sectorial Growth Rate in Different Five Years Plans
2016-17.
(vi) To ensure that all children enjoy a safe childhood, Plan Growth Agri- Industry Services Actual
without any compulsion to work. Rate culture Growth
(vii) To ensure electricity connection to all villages and BPL Rate
household by 2009 and 24-hour power supply by the end Frist Plan 2.1 2.71 5.54 4.17 3.6
of this plan.
Second Plan 4.5 3.15 5.59 4.94 4.21
(viii) To achive standards of air quality in all cities.
(ix) To treat all urban waste water by 2011-12. Third Plan 5.6 –0.73 6.28 5.26 2.72
(x) To increase forest and tree cover by 5%. Plan Holiday 4.16 1.42 4.1 3.69
Twelfth Five Year Plan (2012-2017) Fourth Plan 5.7 2.57 4.91 3.22 2.01
Major objective: Faster, Sustainable and More Indusive Fifth Plan 4.4 3.28 6.55 5.66 4.83
Growth. Sixth Plan 5.2 2.25 5.32 5.41 5.4
Planning Commission in its meeting held on April 2011, the Seventh Plan 5 3.47 6.77 7.19 6
Prime Minister, Dr. Manmohan Singh, addressed the Planning
Eighth Plan 5.6 4.68 7.58 7.54 6.68
Commission concerning the twelth Five Year Plan of India.
The main point of the Twelfth Plan are: Ninth Plan 6.5 2.06 4.51 7.78 5.4
Tenth Plan 8 2.34 8.9 9.4 7.5
Resource Allocation Priorities in 12th plan
Eleventh Plan 9 (8.1) 4 10.5 9.9 8
• Health and Education received less than projected in
Eleventh Plan. Allocations for these sectors will have to Twelth Plan 8 04 10.9 10 –
be increased in 12th plan.
• Health, Education and Skill Development together in the Unemployment
Centre’s Plan will have to be increased by at least 1.2%
point of GDP. Nature of Unemployment in India
• Infrastructure, including irrigation and watershed Employment refers to a situation when a labour does not
management and urban infrastructure, will need obtain employment opportunity despite his willingness to work
additional 0.7 percentage point of GDP over the next 5 on existing wage rate. India is a developing economy where
years.
the nature of unemployment is entirely different from that of
• Since Centre’s GBS will rise by only 1.3 percentage
developed nations. In India, the unemployment rate measures
points over 5 years, all other sectors will have a slower
the number of people actively looking for a job as a percentage
growth in allocations.
• Decrease the number of Centrally Sponsored Schemes of the Labour force. As per the ministry of Labour and
(CSS) to a few major schemes. For the rest, create new Employment, the unemployment rate for the year 2014 is
flexi-fund which allow Ministries to experiment in other 4.90%.
CSS areas.
Different types of Unemployment in India
• PPP model must be encouraged, including in the social
sector, i.e. health and education. Efforts on this front (1) Structural Unemployment
need to be intensified. In this type of unemployment demand for labour falls
• Distinction between plan and non-plan being reviewed short to the supply of labour due to rapidly growing
by Rangarajan Committee. population and their immobility.
(2) Disguised Unemployment
Objectives from the initial white paper It refers to a state of unemployment in which more
presented on April 21, 2011 people are engeged in work than are really needed. In
• Basic objective is faster, more inclusive, and sustainable the late 1950 s, about one-third of workers in India were
growth. disguisedly unemployed.
E-24 Planning, Unemployment and Poverty in India

(3) Seasonal Unemployment 2011-12 (Planning Commission), the percentage of persons


It refers to an unemployment that occurs at certain below the poverty line in India for the year 2011-12 has
seasons of the years. The period of such unemployment been figured out as 25.7% in rural areas, 13.7% is urban
varies from state to state, depending upon the methods of areas and 21.9% for the country as a whole.
farming, the condition of soil, the type and numbers of Poverty, is defined in human development report as denial
crops grown, etc. of opportunities “to lead a long, healthy, creative life and
(4) Open Unemployment to enjoy a decent standard of living, freedom, dignity, self-
It refers to that economic phenomenon in which persons respect and the respect of others”.
are able and willing to work at the prevailing wage rate, (i) Absolute Poverty
but fail to get work. It is called open unemployment Absolute Poverty is when a person cannot obtain certain
because such unemployment can be seen and corrected absolute standards of minimum requirements, usually
in terms of the number of unemployed people. measured in terms of income. It refers to the total
(5) Industrial Unemployment number of people living below Poverty line. As per this
It refers to the unemployment among the illiterates, who measure, around 30% of India’s population is below
wish to work in industrial establishments. The slow
poverty line.
pace of industralisation is unable to generate sufficient
(ii) Relative Poverty
employment opportunities. As a result, there is a huge
industrial unemployment in the country. Relative Poverty is when a person falls behind others.
It is thus a measure of inequality of income. It refers
(6) Frictional Unemployment
to Poverty of people, in comparison to other people,
It refers to temporary unemployment which exists during
regions or nations.
the period, wherein workers leave one job and join some
other. There are many approaches to measure poverty . Income is
(7) Cyclical Unemployment one approach. Another approach is the basic needs approach,
It is associated with the down-swing and depression according to which poverty should not be looked at merely
phases of business cycle. It is the most common type of in terms of private income but should include access to basic
unemployment in the developed capitalist economies. needs like health, education, employment and other essential
services which the community usually provides to its citizens.
Magnitude of Unemployment There are important limitations in measuring poverty in terms
There is great diversity in the forms of unemployment. of either income or consumption levels. These approaches
It has been found that no single measure can adequately do not pay attention to the assets on which most poor people
capture the magnitude of unemployment in India. In addition rely for their livelihood. These assets can include their access
to the decadal census figures, the National Sample Survey to natural and financial resources, their health and capacity
Organisation (NSSO) conducts more detailed sample surveys to work. Another drawback of the traditional approaches
every five years on employment and unemployment. It uses is that these do not focus on the social relations which lead
three different tests for measuring: to the process of impoverishment. For instance, the lack of
• The Usual Status (US) concept which has a reference political influence of the poor and their being subjected to
period of one year and classifies a person as unemployed discrimination are issues which are relevant while studying the
if she was not working but was available for work for nature and causes of their poverty.
most or all of the period of one year. The ‘Sustainable livelihoods’ is the new approach. This
• The Current Weekly Status (CWS) includes a person if addresses issues such as the capabilities, assets and labour
she has not worked even for one hour during the week, required for a means of living. A livelihood is stated to be
though available for work. sustainable if it can cope with and recover from stresses and
• The Current Daily Status (CDS) measures the shocks and maintain or enhance its assets both now and in the
employment status during the seven days preceding the future.
survey and adds up all the hours of work undertaken
during this reference period to decide the un/ Measurement of Poverty
underemployment status of the person. The overall estimation of poverty is based on the data available
Chronic unemployment can be measured by the US and from NSSO, all-India sample survey of household consumption
CWS data. However, the policy formulation regarding expenditure, where poverty is defined with reference to a poverty
supplementary employment all the three sets of data are used line which is the level of monthly per capita consumption
in conjuction. The Planning Commission and NSSO have expenditure considered to be a minimum necessary for living.
been using the CDS data recently as this gives a more realistic
estimate. Committees constituted for measurement of Poverty
• Dr. Y.K. Alagh (1977)
Poverty • D.T. Lakdawala (1989)
Poverty refers to a state in which an individual is unable to • Suresh D.Tendulkar (2005)
fulfil even the basic necessities of life. The minimum basic • N.C. Saxena (2008)
requirement include food, clothing, housing, education and • S.R. Hashim (2010)
health facilities. According to the survey conducted in • Dr. C. Rangarajan (2012)
Planning, Unemployment and Poverty in India E-25

Number & Percentage of Poor*


Poverty line (in `) Number or Poor (million) Poverty Ratio (%)
Year
Rural Urban Rural Urban Total Rural Urban Total
2004-05 446.8 578.80 326.3 80.8 407.1 41.8 25.7 37.2
2011-12 816.00 1000.00 216.5 52.8 269.3 25.7 13.7 21.9
Source : Economic Survey 2014-15,* Estimated by Tendulkar Method.

According to the World Bank (2014), the national poverty 23. Swarna Jayanti Gram Swarojgar Yojana.
line for 2011-12, after adjusting it to Indian currency using 24. Sampurna Gramin Rojgar Yojana.
PPP (Purchasing Power Parity) Conversion factor is $1.94 25. Indira Awaas Yojana.
a day. India with 17.5 % of total world’s population, had 26. Samagra Awaas Yojana.
20.6% share of world’s poorest in 2011 (World Bank, 2014). Government Efforts for Eliminating Urban Poverty
1. Emphasis on vocational education.
Poverty : (Rural And Urban) 2. Nehru Rozgar Yojana (NRY).
Main Reasons for Rural Poverty 3. Self-Employment Programme for the Urban Poor
1. Rapid population growth. (SEPUP).
2. Lack of capital. 4. Financial assistance for constricting houses.
3. Lack of alternate employment opportunities other than 5. Self-Employment to the Educated Urban Youth (SEEUY)
agriculture. programme.
4. Excessive employment pressure on agriculture. 6. Prime Minister’s Rozgar Yojana (PMRY).
5. Illiteracy. 7. National social Assistance Programme.
6. Regional disparities. 8. Urban Basic Services for the Poor programme (UBSP).
7. Orthodox society. 9. Prime Ministers Integrated Urban Poverty Eradication
8. Child marriage tradition. Programme (PMIUPEP).
9. Indifferent attitude towards investment. 10. Swarna Jayanti Shahri Rozgar Yojana.
10. Lack of proper implementation of public distribution
system. Important Programmes in Brief
11. Lack of vocational education/training. 1. Swarnajayanti Gram Swarojgar Yojana (SGSY)
Government Efforts for Eliminating Rural Poverty Swarnajayanti Gram Swarojgar Yojana launched by the
1. Antyodaya plan. Union Ministry of Rural Development was a poverty alleviation
2. Small Farmer Development Programme (SFDP). programme, came into effect from April 1, 1999 which has
3. Drought Area Development Programme (DADP). replaced IRDP and its allied schemes, viz. Training of Rural
4. Minimum Needs Programme (MNP). Youth for Self Employment (TRYSEM), Development of
5. Food for work programme. Women and Children in Rural Areas (DWCRA), Supply of
6. National Rural Employment Programme (NREP). Improved Toolkits to Rural Artisans (SITRA), Ganga Kalyan
7. Integrated Rural Development Programme (IRDP). Yojana (GKY) and Million Wells Scheme (MWS).
8. Jawahar Gram Samriddhi Yojana (JGSY) (Formerly The main objectives of SGSY are :
known as Jawahar Rojgar Yojana). (i) focussed approach to poverty;
9. Rural Labour Employment Guarantee Programme (ii) capitalising advantages of group lending; and
(RLEGP). (iii) overcoming the problems associated with multiplicity of
10. TRYSEM scheme. programmes.
The SGSY is conceived as a holistic programme of micro
11. Family planning/welfare programme for population
enterprises covering all aspects of self-employment which
control.
includes organising rural poor into Self-Help Groups (SHGs).
12. Scheme for Rural artisans / craftsmen. It integrates various agencies – District Rural Development
13. Mahila Samriddhi Yojana. Agencies, banks, departments, Panchayati Raj Institutions,
14. Rural Housing Programme. Non-government organisations (NGO) and other semi-
15. DWCRA programme. government organisations. It is only self-employment
16. National Social Assistance Programme (NSAP). programme currently being implemented for the rural poor.
17. Employment Assurance Scheme. The scheme aims of establishing a large number of micro
18. Group Life Insurance Scheme for Rural Areas. enterprises in rural areas of the country. The objective of
SGSY is to bring the assisted poor family above the poverty
19. Pradhan Mantri Rojgar Yojana.
line in three years by providing them income generating
20. Agriculture Income Insurance Scheme. assets through a mix of bank credit and government
21. Mahatma Gandhi National Rural Employment Guarantee subsidy. The monthly income from the activity to be
Scheme (MGNREGS). undertaken should not be less than ` 2000, net of repayment
22. Pradhan Mantri Gramodaya Yojana (PMGY). to the Bank, at least in the third year.
E-26 Planning, Unemployment and Poverty in India

Subsidy under SGSY is uniform at 30% of the project cost The programme was implemented by all the three-tiers of
subject to a maximum of ` 7500. In respect of Scheduled Panchayats. Each level of Panchayat was an independent unit
Castes and Scheduled Tribes, it is 50% subject to a maximum for formulation of Action Plan and executing the scheme.
of ` 10,000. For groups, the subsidy is 50% subject to a The resources were distributed among District Panchayat,
ceiling of ` 1.25 lakh. The scheme is being implemented on a Intermediate Panchayats and the Gram Panchayats in the ratio
cost-sharing basis between the centre and states of 75 : 25 for of 20 : 30 : 50.
non-north eastern states and 90 : 10 for north-eastern states. 4. Mahatma Gandhi National Rural Employment
Below the poverty line families in rural areas constitute the Guarantee Scheme (MGNREGS)
target group of the SGSY. Within the target group social After the notification of ambitious step of UPA
safeguards have been provided to vulnerable sections, by way government of the National Rural Employment Guarantee
of reserving 50% benefits for SC/ST, 40% for women and 3% Act on September 7, 2005, a new scheme named “National
Rural Employment Guarantee Scheme (NREGS)” (It was
for disabled persons.
later changed with the name Mahatma Gandhi National
The scheme of SGSY covers all aspects of self-employment Rural Employment Guarantee Act) has been launched on
such as organisation of the poor into self-help groups, training, February 2, 2006. The ongoing programmes of Sampoorna
credit, technology, infrastructure and marketing. Grameen Rozgar Yojana (SGRY) and National Food for Work
2. National Food for Work Programme Programme (NFFWP) were integrated within the NREGS in
National food for work programme was Launched on the 200 districts identified in the initial stage. Implemented
November 14, 2004. by the Ministry of Rural Development, National Rural
It is started in 150 most backward districts of the country Employment Guarantee Act (NREGA) is a flagship programme
identified by the Planning Commission in consultation with the of the government that directly touches lives of the poor and
Ministry of Rural Development and State Government. promotes inclusive growth. NREGS has been launched with
following guidelines:
The objective of the programme was to provide additional
• To enhance livelihood security of households in rural
resources apart from the resources available under Sampoorna areas of the country by providing at least 100 days of
Grameen Rozgar Yojana (SGRY) to 150 most backward guaranteed wage employment in every financial year to
districts of the country so that the generation of supplementary every household whose adult members volunteer to do
employment and providing of food security through creation unskilled manual work.
of need based economic, social and community assets in these • Until such time as a wage rate is fixed by the Central
districts is further intensified. The scheme was 100% central Government, the minimum wage for agricultural
sponsored. The programme has been subsumed in National labourers shall be applicable for the scheme.
Rural Employment Guarantee Act initiated since February • An applicant not provided employment within fifteen
2, 2006, which has now come in force in all districts of the days, to be entitled to a daily unemployment allowance
country. as specified by the State Government.
3. Sampoorna Grameen Rojgar Yojana (SGRY) • Central Employment Guarantee Council to be constituted
This Scheme has been subsumed under ‘National to discharge various functions and duties assigned to the
Council. Every State Government is to constitute a State
Rural Employment Guarantee Programme (NREGP)’
Council for this purpose.
which has been started since February 2, 2006.
• Panchayat at the district level is to constitute a Standing
The ongoing schemes - the Employment Assurance Scheme Committee of its members to supervise, monitor and
(EAS) and the Jawahar Gram Samridhi Yojana (JGSY) were oversee the implementation of the Scheme within the
merged into the Sampoorna Grameen Rojgar Yojana (SGRY) district.
on September 25, 2001. The thrust area of the programme • For every Block, State Governments is to appoint a
was to provide additional wage employment in rural areas and Programme Officer for implementing the Scheme.
also to provide food security, alongside creation of durable • Gram Panchayat is to be responsible for identification
community, social and economic assets and infrastructure of the projects as per the recommendations of the Gram
development in these areas. Preference was given to BPL Sabha and for executing and supervising such works.
families for providing wage employment under SGRY, poor • Central Government is to establish a National
families above the poverty line were also offered employment Employment Guarantee Fund. State Governments
under SGRY. The annual outlay was ` 10,000 crore which to establish State Employment Guarantee Funds for
includes 50 lakh tonnes of foodgrains. The fund was shared implementation to the Scheme.
between the Centre and the States in the ratio of 75 : 25. • The Scheme is to be self-selecting in the sense that those
Foodgrains are provided free of cost to the States/UTs. The among the poor who need work at the minimum wage
payment of foodgrains was made directly to FCI at economic would report for work under the scheme.
cost by the Centre. However, the cost of the transportation of The Act was implemented in a phased manner. In phase one
foodgrains from the FCI godown to the work-site/ PDS and it was introduced in 200 of the most backward districts of the
its distribution are the responsibility of the State Government. country. It was expanded to 330 districts during 2007-08. The
Minimum wages are paid to the workers through a mix of Act has been notified throughout the country with effect from
minimum five kg. of foodgrains and at least 25% of wages in April 1, 2008. During the year 2010-11, 5.48 crore households
cash. were provided employment under the scheme.
Planning, Unemployment and Poverty in India E-27

MGNREGA is the first ever law of the world, that guarantees poverty like drought, deforestation and soil erosion and so
wage employment at an unprecedented scale. The primary encourage sustainable development. The process outcomes
objective of the Act is augmenting wage employment. include strengthening grassroot processes of democracy and
Its auxiliary objective is strengthening natural resource infusing transparency and account ability in governance.
management through works that address causes of chronic
Various Poverty and Unemployment Elimation Programmes initiated by Govt. In Five-Year Plans.
S. No. Name of the Programme Year of Main Objectives
Starting
1. Crash Scheme for Rural Employment 1971-72 Generation of new employment rural development.

2. Pilot Intensive Rural Employment 72-73 Construction work in Villages.

3. Drought Prone Areas Programme 73-74 To develop natural resources in drought prove rural areas.

4. Food for work Programme 77-78 To provide food for work in development process.

5. Antyodaya Yojana 77-78 To give economic assistance to families BPL.

6. National Rural Employment Pro- 80-81 Helping that segment of Population which largely depends
gramme (NREP) on wage employment by providing gainful employment.

7. Rural landless Employment Gurantee 1983-84 To expand Employment.


Programme (RLEGP)

8. Million Wells Scheme (MWS) 1988-89 To provide open irrigation wells, free of cost, to poor small
and Marginal farmers belonging to Sc/ST.

9. Jawahar Rojgar Yojana (JRY) 1989-90 Generation of gainful employment to unemployed in rural
areas.

10. Employment Assurance Scheme 1993-94 Providing assured employment of days unskilled manual
(EAS) work to rural.

11. Prime Minister Rojgar Yojana 1994-95 To provide self-employment to educated unemployed by
(PMRY) setting up of sever lakh micro-enterprises.

12. Jawahar Gram Samridhi Yojana 1999-2000 To create employment and durable assets in rural areas.
(JGSY)

13. Swarnjayanti Gram Swarozgar Yo- 1999-2000 To promote micro-enterprises and helping the rural poor
gana (SG SY) into self-help group.

14. Pradhan Mantri Gramodaya Yojana 2000-01 Improving the quality of life of people in rural areas.

15. Jai Prakash Rojgar Guarantee Yojana 2002-03 To provide employment guarantee to the unemployed in the
(JPRGY) most distressed districts of the country.

16. National food for work Programme 2004-05 To intensify the generation of supplementary wage
(NFWP) employment.

17. National Rural Employment Guaran- 2006 To entitle the rural poor to guaranted employment for 100
tee Act (NREGA) days. w.e.f April 2008 expanded from 200 to all 614 rural
districts of India.

18. Swavalamban Scheme 2010 New Pension Scheme for poorer section in which Govt. will
contribute a sum of ` 1000 along with the subscriber upto `
12000. This scheme will cover 10 lakh subscribers each in
four years beginning 2010-11 bringing the total number to 40
lakh by March, 2014.

19 Pradhan Mantri Jan Dhan Yojana 2014 Start by current PM Narendra Modi to eradicate poverty at
grass root level.
E-28 Planning, Unemployment and Poverty in India

5. Indira Awas Yojana (IAY) rural road network. It is estimated that an investment of about
The Indira Awas Yojana was launched in 1985-86 to ` 48,000 crore would be required for achieving the targets
provide dwellings. In 1989-90, after merging the RLEGP under Bharat Nirman.
with Jawahar Rozgar Yojana, this plan also became a part During the first two years of 11th Five Year Plan, an
of Jawahar Rozgar Yojana (JRY). Since January 1, 1996, expenditure of ` 25780.7 crore has been incurred on rural
it was again separated from JRY. At present, IAY is one roads under PMGSY in which 93636 km long road work has
of the six components of the Bharat Nirman Programme. been completed in rural areas.
The objective of IAY is to provide financial assistance for
7. Swarna Jayanti Shahari Rozgar Yojana (SJSRY)
construction upgradation of houses to BPL rural households
Swarna Jayanti Shahari Rozgar Yojana (SJSRY) became
belonging to Scheduled Castes and Scheduled Tribes, freed
bonded labourers, non -SC/ST rural households, widows and operational Since December 1, 1997. The Government has
physically handicapped persons living in the rural areas. At recently revamped the SJSRY with effect from April 1, 2009.
present, it is the flagship programme for rural housing. Its objective is community impowerment through promoting
The main points of this scheme are as follows: community organisation like Neighbourhood Groups (NHGs)
1. Since 1993-94, the benefit of Indira Awas Yojana is Neighbourhood Committees (NHCs) and Community
being provided even to those rural poor of non-schedule Development Societies (CDSs). CDS will be the nodal agency
caste/schedule tribe who are living below the poverty for project identification and co-ordination. It is founded on a
line. 75:25 proportion between the Centre and states. The scheme
2. A minimum of 60% of funds are to be utilized for provides gainful employment to the urban unemployed and
construction of houses for the SC/ST people. underemployed poor, by encouraging the setting up of self-
3. From 1995-96, IAY benefits have been extended to employment ventures by the urban poor and also by providing
widows or next to kin of defence personnel killed in wage employment and utilizing their labour for construction of
action. socially and economically useful public assets. This is urban
4. Benefits have also been extended to ex-servicemen and counterpart of SGSY. The SJSRY has five targets:
retired members of para military forces as long as they (a) The Urban Self-Employment Programme (USEP) which
fulfil the normal eligibility conditions of IAY. targets individual urban poor for setting up of micro
5. 3% of funds are reserved for the disabled persons living enterprises;
below the poverty line in rural areas. (b) The Urban Women Self-help Programme (UWSP) which
6. Under the plan, the allocation of the house is done in the targets urban poor women self-help groups for setting
name of the female member of the benefited family or in up of group enterprises and providing them assistance
the joint names of husband and wife. through a fund for credit activities;
7. The ceiling on assistance for construction of new houses (c) Skill Training for Employment Promotion amongst
has been increased from ` 25,000 to ` 35000 per unit Urban Poor (STEP-UP) which targets the urban poor
for the plain areas and from ` 27,500 to ` 38,500 per for imparting quality training so as to enhance their
unit for the hilly/difficult areas with effect from April 1, employability for self-employment or better salaried
2008. employment;
8. Under the scheme, financial resources are shared (d) The Urban Wage Employment Programme (UWEP)
between the Centre and State on a 75 : 25 basis. In which seeks to assist the urban poor by utilizing their
case of NE states, the funding pattern has recently been labour for the construction of socially and economically
revised to 90 : 10. useful public assets, in towns having population less than
6. Pradhan Mantri Gram Sadak Yojana (PMGSY) 5 lakhs as per the 1991 census; and
Pradhan Mantri Gram Sadak Yojana (PMGSY), a 100% (e) The Urban Community Development Network (UCDN)
centrally sponsored scheme was launched on December 25, which seeks to assist the urban poor in organizing
2000. The prime objective of the PMGSY was, to provide themselves into self-managed community structures so as
connectivity to all unconnected habitations in the rural areas to gain collective strength to address the issues of poverty
having population of more than 500 persons, by the end of facing them and participate in effective implementation
Tenth Plan Period (2007). In respect of the Hill States (North- of urban poverty-alleviation programmes.
East, Sikkim, Himachal Pradesh, Jammu and Kashmir and
8. Antyodaya Anna Yojana (AAY)
Uttarakhand), Desert Areas and Tribal (Schedule - V) Areas,
In order to make TPDS more focused and targeted
the objective is to connect habitations with a population of 250
towards the poorest section of population, the ‘Antyodaya
persons and above.
Anna Yojana’ (AAY) was launched in December 2000 for one
Under Bharat Nirman, goal has been set to provide connectivity
crore poor families. Initially AAY contemplated identification
to all the habitations with population of more than 1000 in the
of one crore poorest of the poor families from amongst the
plain areas and habitations with a population of 500 or more
BPL families covered under TPDS within the States and
in hilly and tribal areas in a time bound manner by 2009. The
providing them foodgrains at a highly subsidised rate of ` 2
systematic upgradation of the existing rural road networks is
per kg for wheat and ` 3 per kg for rice. The States/UTs
also an integral component of the scheme. Accordingly, an
are required to bear the distribution, cost, including margin to
Action Plan has been prepared for connecting 66,802 habitations
dealers and retailers as well as the transportation cost. Thus
with 1,46,185 km of all-weather roads. This action plan also
the entire food subsidy is being passed on to the consumers
envisages upgradation/renewal of 1,94,130 km of the existing under the scheme.
Planning, Unemployment and Poverty in India E-29

9. Rashtriya Mahila Kosh (National Women Fund) 12. Innovative and Special Employment Scheme (The
The objective of this institution set-up on March 30, 1993 Third Stream of JRY)
is to facilitate credit support to poor women for their socio- The third phase of Jawahar Rozgar Yojana which is
economic upliftment. The support is extended through NGOs, known as the Innovative and Special Employment Scheme is
Women Development Corporations. Suitable state government being implemented since the year 1993-94. In this scheme,
agencies like DRDAs, Dairy Federations, Municipal Councils, special and modern projects are included whose objective
is to stop the migration of labour, to encourage the female
etc. RMK extends loan to the above organisations at 8% and employment and to develop the productivity in the desert
the above. Organisation can lend to the women Self Help areas. Following are some of the main points of this scheme-
Groups/ ultimate women beneficiaries at an interest ranging 1. Under this plan, the schemes like Operation Black Board
from 8% to 18%. The financial assistance given by RMK is were also given place which fulfil the main objectives of
totally security free and RMK does not insist for any sort of Jawahar Rozgar Yojana. The construction of the school
collateral from the organisations availing loan from it. building and of class rooms has been given priority.
10. National Rural Health Mission (NRHM) Operation Black Board scheme was started in 1987.
The National Rural Health Mission (NRHM) was 2. All the projects coming under the third stream are
launched on 12th April, 2005, to provide accessible, affordable given acceptance by a Screening Committee which is
constituted under the chairmanship of the Secretaries of
and accountable quality health services to rural areas. The
Rural Employment and Poverty Eradicating Departments
difficult areas with unsatisfactory health indicators were of the Central Government.
classified as special focus States to ensure greatest attention
13. CAPART
where needed. The thrust of the Mission was on establishing
To promote rural development the Council for
a fully functional, community owned, decentralized health Advancement of People’s Action and Rural Technology
delivery system with inter-sectoral convergence at all levels, (CAPART) was constituted on September 1, 1986. CAPART
to ensure simultaneous action on a wide range of determinants is a registered body under the Ministry of Rural Development.
of health like water, sanitation, education, nutrition, social and The head office of CAPART is at New Delhi. CAPART has
gender equality. Institutional integration within the fragmented nine Regional Committees / Centres at Jaipur, Lucknow,
health sector was expected to provide a focus on outcomes, Ahmedabad, Bhubaneswar, Patna, Chandigarh, Hyderabad,
measured against Indian Public Health Standards for all health Guwahati and Dhanbad. The Regional Committees are
facilities. From narrowly defined schemes, the NRHM shifted empowered to sanction projects proposals to voluntary agencies
upto an outlay of ` 25 lakhs in their respective regions. Its
focus to a functional health system at all levels, from the
main objective is to encourage and assist the voluntary
village to the district. activities for implementing projects for rural prosperity. Some
NRHM is being operationalized throughout the country with important features related to it are as under-
special focus on 18 states which includes 8 Empowered (i) CAPART extends assistance to Jawahar Rojgar Yojana,
Action Group States (Bihar, Jharkhand, Madhya Pradesh, Organisation of the Beneficiaries of Poverty Eradicating
Chhattisgarh, Uttar Pradesh, Uttarakhand, Odisha and Programme, Integrated Rural Development Programme,
Rajasthan), 8 NE states, Himachal Pradesh, Jammu & Development of Women and Children in Rural Areas
Kashmir. and other related organisations.
Among major innovations of the NRHM are creation of a cadre (ii) The Rural Development Department of the Indian
of Accredited Social Health Activists (ASHA) and improved Government provides the required funds to CAPART.
hospital care, decentralisation at district level to improve intra 14. Bharat Nirman
and inter-sectoral convergence and effective utilisation of This programme, launched in 2005-06, for building
resources through NGOs and community in general. infrastructure and basic amenities in rural areas, has six
components, namely rural housing, irrigation potential,
11. Intensified Jawahar Rozgar Yojana (The Second drinking water, rural roads, electrification and rural telephony.
Stream of JRY) A goal has been set to provide connectivity to all villages
Second phase of Jawahar Rozgar Yojana is being with a population of 1000 (500 in hilly or tribal areas) with
implemented in 120 backward districts of 12 states of the all-weather roads. New connectivity is proposed to a total of
country which are badly affected with unemployment and 63940 habitations under Bharat Nirman. As per Economic
under-unemployment problems. These states are Andhra Survey 2011-12, under the rural roads component of bharat
Pradesh, Bihar, Gujarat, Jammu & Kashmir, Karnataka, Nirman, 42249 habitations have been provided all weather road
Madhya Pradesh, Maharashtra, Odisha, Rajasthan, Tamil connectivity up to December, 2011 and projects for connecting
Nadu, Uttar Pradesh and West Bengal. Since January 1, 1996 16126 habitations are at different stages of implementation.
this plan was merged with Employment Assurance Scheme 15. Pradhan Mantri Jan Dhan Yojna (PMJDY)
(EAS). The main points of this plan were as under - PMJDY was introduced on the 15th August, 2014 by
1. Under this programme, those works are given priority Prime Minister Narendra Modi. It is a part of the drive of
which provide ample employment opportunities, like national inclusion. The scheme entails a ‘no-frills account’
creating small irrigation facility on barren lands, foresty, with an overdraft facility of `5000 and a Rupay Debit Card.
etc. Other attractions of the scheme include accidental insurance
2. This plan also includes various activities which create cover of `1 lakh and a life insurance coverage of `30000.
rural infrastructure including primary education An unprecedented number of 1.5 crore bank accounts were
institutions. opened under the scheme on the first day of its launch.
E-30 Planning, Unemployment and Poverty in India

16. Rural Electrification in India Millennium Development Goals (MDGs)


Rural electrification involves supply of energy for two
MDGs were eventuated at the UNs Millennium Summit, 2000,
types of programmes:
where the world leaders agreed on a set of quantifiable and
(a) production-oriented activities like minor irrigation, rural
monitorable goals for development and poverty eradication to
industries, etc. and,
be achieved by 2015. These MDGs were based on the assump-
(b) electrification of villages. Rural Electrification tion that, “progress that world has made over the past 30 years
Corporation (REC) shows that these goals are attainable”.
It was established in July 1969 to finance various projects of
MDGs
rural electrification. REC is a Public Financial Institution under
The main MDGs, as per UN are –
section 4A of the Companies Act 1956. REC is also registered
• Eradicate extreme hunger and poverty.
as a Non-Banking Financial Company (NBFC) under section
• Achieve universal primary education.
451A of RBI Act, 1934. At present REC is a ‘Navratna’
• Promote gender equality and empower women.
Company. The current mission of REC is to facilitate
• Reduce child mortality.
availability of electricity for accelerated growth and for
• Improve maternal health.
enrichment of quality of life of rural and semi-urban population
• Combat HIV/AIDS, malaria and other diseases.
and to act as a competitive, client-friendly and development
• Ensure environmental sustain ability.
oriented organisation for financing and promoting projects • Develop a global partnership for development.
covering power generation, power conservation, power
transmission and power distribution network in the country.
To give impetus to rural electrification, the Government has
Sustainable Development Goals (SDGs)
given special attention for creation and augmentation of Rural At the United Nations Sustainable Development Summit on 25
Electricity Distribution Backbone and Village Electricity September 2015, World Leaders adopted the 2030 Agenda for
Infrastructure so as to cover all unelectrified villages and rural sustainable Development, which includes a set of 17 sustain-
households within a span of five years. able Development Goals to end poverty, fight inequality and
injustice and tackle climate change by 2030.
Rural Electricity Supply Technological Mission (REST) has
17 SDGs are–
been set up to oversee the implementation of schemes under
1. End poverty in all its forms everywhere.
Accelerated Rural Electrification Programme. 2. End hunger, achieve food security and imporve nutrition
Rural Electrification Policy and promote sustainable agriculture.
3. Ensure healthy lives and promote well-being for all at all
On August 23, 2006, Government notified Rural Electrification ages.
Policy under section 4 & 5 of the Electricity Act, 2003. 4. Ensure inclusive and equitable Quality Education and por-
The policy aims at provision of access to electricity to all mote life long learning opportunities for all.
households by year 2009, quality and reliable power supply at 5. Achieve gender equality and empower all women and girls.
reasonable rates and minimum lifeline consumption of 1 unit 6. Ensure availability and sustainable management of water
per household per day as a merit by year 2012. and sanitation for all.
7. Ensure access to affordable, reliable, sustainable and mod-
Important Programmes introduced ern energy for all.
8. Promote sustained, inclusive and sustainable economic
in Union Budget growth, full and productive employment and decent work
for all.
2014-15 9. Build resilient infrastructure, promote inclusive and Sustain-
• Housing for all - 2 crore houses in urban areas & 4 crore able industrialization and foster innovation.
houses in Rural areas 10. Reduce inequality within and among countries.
• Micro units Development Refinance Agency (MUDRA) 11. Make cities and human settlements inclusive, safe, resilient
Bank, with a corpus of ` 20,000 crores, and credit and sustainable.
guarantee corpus of ` 3,000 crores to be created. 12. Ensure sustainable consumption and production patterns.
• A new sheme for providing physical Aids and Assisted 13. Take urgent action To combat climate change and its im-
Living Devices for Senior Citizens, living below the pacts.
poverty line. 14. Conserve and sustainably use the oceans, seas and marine
resources for sustainable development.
• SETU (Self-Employment and Talent Utilization) to
15. Protect, restore and promote sustainable use of terrestrial
establish as Techno-financial, incubation and facilitation ecosystems, sustainable manage forests, combat deserti-
programme to support all aspects of start-up business. fication, and halt and reverse land degradation and halt
• A national skill mission to consolidate skill initiatives biodiversity loss.
spread across several ministeries to be Launched. 16. Promote peaceful and inclusive societies for sustainable
• Deen Dayal Upadhyay Gramin Kaushal Yojana to enhance development, provide access to justice for all and build
the employability of rural youth. effective, accountable and inclusive institutions at all levels.
• Job creation through revival of growth and investment and 17. Strengthen the means of implementation and revitalize the
promotion of domestic manufacturing - “Make in India” global partnership for Sustainable development.
Planning, Unemployment & Poverty in India E-31

1. The rationale given in the first plan for state intervention 11. The phenomenon known as green revolution began to
in the industrial sector was that: manifest during:
(a) private sector was inefficient (a) 2nd Plan
(b) state alone can ensure decentralization of wealth (b) 3rd Plan
(c) private sector was neither willing nor capable of (c) Annual Plans (1966-69)
investing in certain sectors (d) 4th Plan
(d) the commanding heights of the economy should be 12. Relative poverty refers to a situation where:
in the public sector (a) a person falls behind others
2. Which of the following is not one of the International (b) a person is unable to obtain the necessaries for life
Development Targets of the Millennium Development (c) a person is below poverty line
Goals? (d) a person is poorer than another in the developed
(a) Reducing the number of persons living in extreme world
poverty by one half of 2015 13. Use of energy requirements (calorie) as a measure of
(b) Universal primary education by 2015 poverty in India was made for the first time by:
(c) Reduction of infant and child mortality by 2/3rd by (a) Amartya Sen
2015 (b) Dandekar and Rath
(d) Reducing the total population by 1/3rd (c) Planning Commission
3. Structural unemployment arises due to: (d) T. N. Srinivasan
(a) deflationary conditions 14. Human Poverty Index (HPI) measures the deprivation as
(b) heavy industry bias a composite index of:
(c) shortage of raw materials (a) longevity, nutrition and knowledge
(d) inadequate productive capacity (b) knowledge, basic needs and standard of living
4. ‘Disguised unemployment’ refers to: (c) longevity, standard of living and sanitation
(a) persons with no jobs (d) longevity, knowledge, and standard of living
(b) more persons employed for a job which a few can 15. After the launch of New Economic Policy:
accomplish (a) both urban and rural poverty increased
(c) unemployment among women (b) rural poverty increased initially but declined in
(d) unemployment of people above 60 years of age 1993-94
5. This type of unemployment can occur even in a situation (c) both rural & urban poverty decreased
of full employment. (d) rural poverty increased but urban poverty declined
(a) Structural unemployment 16. Aam Admi Bima Yojana provides social security to-
(b) Functional unemployment (a) All labours in rural areas
(c) Cyclical unemployment (b) All landless labours living below poverty line in
(d) Disguised unemployment rural areas
6. Usual status (US) unemployment is calculated with (c) All labours in urban areas
reference to a period of: (d) All labours in both rural as well as urban areas
(a) one year (b) indefinite 17. CAPART is related with-
(c) one month (d) one week (a) Assisting and evaluating rural welfare programmes
7. Chronic unemployment is measured using: (b) Computer hardware
(a) US data (b) CWS data (c) Consultant service of export promotion
(c) None of the two (d) Both (a) and (b) (d) Controlling pollution in big industries
8. The Planning Commission was set up in accordance with 18. Which day has been declared as ‘Balika Diwas’ (Girl Day)
the directive principles in: by the Ministry of Woman and Children Development?
(a) Article 38 (b) Article 39 (a) April 5, every year
(c) Article 42 (d) 51 A (b) July 9, every year
9. Structural Planning refers to: (c) October 9, every year
(a) laying down broad goals and strategies (d) December 9, every year
(b) centralised planning 19. Sector wise, maximum employment in the public sector
(c) fixing flexible targets in
(d) changing existing institutions or creating new ones (a) electricity, gas, and water
10. First Plan adopted (b) community, social, and personal services
(a) Lewis-Fei model (b) Mahalanobis model (c) finance, insurance, and real estate
(c) Harrod Domar model (d) Keynesian model (d) transport, storage, and communication
E-32 Planning, Unemployment & Poverty in India

20. Which plan was suspended one year before the time (d) The nation’s priorities have shifted away from
schedule? industrial development to rural development
(a) First Plan (1951-56) (b) Third Plan (1966-71) 29. Various employment programmes have been launched in
(c) Sixth Plan (1980-85) (d) Fifth Plan (1974-79) India, namely, IRDP, NREP, RLEQP, FWP. They are
21. Rajeev Gandhi Scheme for Empowerment of Adolescent meant to improve
Girls named to ‘Sabla’ includes the girls of the age (a) rural employment
group– (b) urban employment
(a) 9 to 12 years (b) 10 to 15 years (c) rural and urban employment
(c) 11 to 18 years (d) 15 to 18 years (d) educated employment
22. Which Indian plan ensured high growth rate as compared 30. Among the following who are eligible to benefit from
with targeted growth rate ? the “Mahatma Gandhi National Rural Employment
(a) Fifth Plan (b) Fourth Plan Guarantee Act”?
(c) Second Plan (d) Eighth Plan (a) Adult members of only the scheduled caste and
23. What was the aim of Antyodaya Programme? scheduled tribe households
(a) Elimination of Urban Poverty (b) Adult members of below poverty line (BPL)
(b) Improving the standards of scheduled castes households
(c) Uplifting minorities (c) Adult members of households of all backward
(d) Helping the poorest among poor communities
24. Economic survey in India is published officially, every (d) Adult members of any household
year by the 31. Inclusive growth as enunciated in the Eleventh Five Year
(a) Reserve Bank of India Plan does not include one, of the following:
(b) Planning Commission of India (a) Reduction of poverty
(c) Ministry of Finance, Government of India (b) Extension of employment opportunities
(d) Ministry of Industries, Government of India (c) Strengthening of capital market
25. The schemes of Urban Micro-Enterprises, Urban Wage (d) Reduction of gender inequality
Employment and Housing, and Shelter Upgradation are 32. In the context of India’s Five Year Plans, a shift in the
part of
pattern of industrialization, with lower emphasis on
(a) Integrated Rural Development Programme
heavy industries and more on infrastructure begins in
(b) Nehru Rozgar Yojana
(a) Fourth Plan (b) Sixth Plan
(c) Jawahar Rozgar Yojana
(c) Eighth Plan (d) Tenth Plan
(d) Prime Minister’s Rozgar Yojana
33. Deputy chairman of the Planning Commission is the:
26. The Employment Assurance Scheme envisages financial
(a) Prime Minister
assistance to rural areas for guaranteeing employment to
(b) Planning Minister
at least
(c) Holds the rank of a cabinet minister
(a) 50 per cent of the men and women seeking jobs in
(d) Economist of repute
rural areas
34. Overall aim of Indian plans is
(b) 50 per cent of the men seeking jobs in the rural areas
(c) one man and one women in a rural family living (a) industrial development
below the poverty line (b) economic growth with social justice
(d) one person in a rural landless house hold living (c) socialist pattern of society.
below the poverty line (d) regional prosperity
27. The growth rate of which one of the following sectors has 35. Ninth Plan put emphasis on
very low employment elasticity? (a) universal education
(a) manufacturing (b) construction (b) growth with social justice and equity
(c) financial services (d) mixed farming (c) removal of rural unemployment
28. The planning process in the industrial sector in India (d) decentralization
has assumed a relatively less important position in the 36. _____ sector was the largest contributor to GDP during
nineties as compared to that in the earlier period. Which the Ninth Plan?
one of the following is not true in this regard? (a) Manufacturing
(a) With the advent of liberalization, industrial (b) Trade
investments/development have largely been placed (c) Information technology
within the domain of private and multinational (d) Financial services
sectors. 37. Durgapur, Bhilai and Rourkela steel plants were
(b) With markets assuming a central place, the role of established during _____ Plan.
central planning in many sectors has been rendered (a) Second (b) Third
redundant (c) Fourth (d) First
(c) The focus of planning has shifted to sectors like 38. ____ Plan had the fastest growth rate.
human resource development, infrastructure, (a) First (b) Fifth
population control and welfare. (c) Sixth (d) None
Planning, Unemployment & Poverty in India E-33

39. National Development Council is concerned with 47. In which one the following Five Year Plans the actual
(a) implementing state plans growth performance in India in respect of GDP(at factor
(b) approving and assessing major development projects cost) was less than the target set ?
in India (a) Sixth Five Year Plan
(c) approval of five-year plans (b) Seventh Five Year Plan
(d) implementing community development programmes (c) Eighth Five Year Plan
40. Which of the following is not a millennium development (d) Ninth Five Year Plan
goal? 48. How would the size of the labour force and the rate of
(a) The eradication of extreme poverty and hunger. unemployment be affected if people retrenched from
their jobs did not actively seek employment?
(b) Universal primary and secondary education.
(a) The labour force would be smaller and the
(c) Gender equality and empowerment of women.
unemployment rate would be lower.
(d) Reducing the child mortality rate. (b) The labour force would be smaller and the
41. Which plan emphasised optimum utilisation of capacities, unemployment rate would be higher.
improvement in productivity and establishment of sunrise (c) There would be no change in the labour force and
industries”? the unemployment rate would be higher.
(a) Fifth Plan (b) Sixth Plan (d) The labour force would be smaller and there would
(c) Second Plan (d) Seventh plan be no change in the unemployment rate.
42. Which of the following programmes is specially directed 49. The natural rate of unemployment is likely to fall if:
towards helping women in difficult circumstances or in (a) unemployment benefits increase
distress? (b) income tax increases
(a) Swadhar (c) more training is available for the unemployed
(b) Swayamsiddha (d) geographical immobility increases
(c) Mahila Samriddhi Yojana 50. To reduce cyclical unemployment the government might:
(d) Rashtriya Mahila Kosh (a) increase the budget surplus
43. PEARL is a programme related to (b) increase the balance of payments deficit
(a) improvement of gems and jewellery export (c) increase the budget deficit
(b) pet awareness and rescue (d) reduce government expenditure
(c) knowledge sharing on urban reforms and city 51. The Economic Planning Committee was established in
the chairmanship of
governance
(a) J.L. Nehru
(d) quality enhancement of processed food for export
(b) Dr. Rajendra Prasad
44. Indicative planning (c) Ramkrishna Mudaliyar
(a) replaces the market system (d) K.C. Niyogi
(b) is a method of controlling the 52. The second five year plan was based on
(c) economy by setting short term goals (a) P.C. Mahalnobis Model
(d) can work only when capital account convertibility (b) Nehruvian Model
operates works through the market (c) Harro-Domar Model
45. Which one of the following statements is not correct? (d) None of these
(a) Under the Targeted Public Distribution System, the 53. What was the main objective of the Third Five year Plan?
families below Poverty Line are provided 50 kg (a) To make economy self dependent
of food grains per month per family at subsidized (b) To ensure social development
price. (c) For abolition of poverty
(b) Under Annapurna Scheme, indigent senior citizens (d) To establish heavy industries
of 65 years of age or above eligible for National Old 54. Which five year plan was focused on poverty and
Age Pension but not getting pension can get 10 kg unemployment for the first time?
of food grains per person per month free of cost. (a) Third Five Year Plan
(c) Ministry of social justice and empowerment has a (b) Fourth Five Year Plan
scheme in which indigent people living in welfare (c) Fifth Five Year Plan
(d) Sixth Five Year Plan
institutions like orphanages are given 15 kg of
55. Who among the following has written the book named
foodgrains per person per month of BPL rates.
‘Indian Economy: Gandhian Blue print”?
(d) Ministry of Human Resource Development gives
(a) Acharya Vinoba Bhave
financial support to mid-day meal scheme for the (b) Morarji Desai
benefit of Class I to V students in government or (c) Jai Prakash Narayan
government aided school. (d) Charan Singh
46. Which Five- Year Plan had an objective of “Rapid 56. A shift in pattern of Industrialisation with lower emphasis
industrialization with particular emphasis on development of the heavy industry and move on infrastructure begins
of basic and heavy industries” ? in which five year plan?
(a) First (b) Second (a) Fifth plan (b) Sixth plan
(c) Third (d) Fourth (c) Seventh plan (d) Eight plan
E-34 Planning, Unemployment & Poverty in India

57. The government introduced an agricultural strategy which (a) Sixth Five Year Plan
gave rise to green revolution in India under the plan (b) Seventh Five Year Plan
(a) Third Five Year Plan (b) Fourth Five Year Plan (c) Eighth Five Year Plan
(c) Fifth Five Year Plan (d) Sixth Five Year Plan (d) Ninth Five Year Plan
58. Agriculture, Irrigation and Power Projects were given 70. The ‘Bombay Plan’ drafted by GD Birla and JRD Tata
highest priority in which among the following plans? emphasized:
(a) First Five Year Plan (b) Second Five Year Plan (a) that the economy should be left to the dynamic
(c) Third Five Year Plan (d) Fourth Five Year Plan investments by the private sector in heavy industries,
59. Who among the following works as Ex-officio secretary etc.
of National Development Council? (b) the public sector investment in infrastructure and
(a) Secretary of Finance Ministry heavy industries
(b) Vice Chairman of Planning Commission (c) annual planning
(c) Secretary of Planning Commission (d) that the private sector should foot the Bill for
(d) Secretary of Planning & Implementation Ministry
intensive and low return investments in the industrial
Secretary of Planning Commission
sector.
60. Socio-economic planning is a part of ____?
71. Which one among the following statements regarding the
(a) Union List (b) State List
Eighth five year plan in India is not correct?
(c) Concurrent List (d) Reserved List
61. In which five year plan self reliance as an object of (a) The plan was postponed by two years because of
planning was emphasized? political upheavals at the centre
(a) First Five Year Plan (b) It aimed at high growth of both agriculture and
(b) Second Five Year Plan manufacturing sectors
(c) Third Five Year Plan (c) Its emphasis was on growth in export and import,
(d) Fourth Five Year Plan improvement in trade and current account deficit
62. Who among the following is NOT a part of National (d) It set before itself the two principal objectives of
Development Council? ‘growth with stability’ and ‘growth with justice’
(a) Finance Commission of India Chairman 72. By which one of the following years does the 11th Five
(b) Vice Chairman of Planning Commission Year Plan aim at achieving 10% rural tele-density in
(c) Secretary of Planning Commission India from the present 1.9%?
(d) Secretary of Planning & Implementation Ministry (a) 2009 (b) 2010
63. Which five year plan Infrastructure as an object of (c) 2011 (d) 2012
planning was emphasized? 73. When the productive capacity of the economic systems of
(a) First Five Year Plan (b) Second Five Year Plan a state is inadequate to create sufficient number of jobs, it
(c) Third Five Year Plan (d) Sixth Five Year Plan is called
64. Which five year plan is also known as Mahalanobis (a) seasonal unemployment
model plan? (b) structural unemployment
(a) First Five Year Plan (b) Second Five Year Plan (c) disguised unemployment
(c) Third Five Year Plan (d) Sixth Five Year Plan (d) cyclical unemployment
65. Disguised unemployment generally means 74. The main objective of the 12th Five-Year Plan is
(a) Large number of people remain unemployed [CSAT-2014-I]
(b) Alternative employment is not available (a) inclusive growth and poverty reduction
(c) Marginal productivity of labour is zero (b) inclusive and sustainable growth
(d) Productivity of worker is low (c) sustainable and inclusive growth to reduce
66. Which Five-Year Plan had an objective of “Rapid
unemployment
industrialization with particular emphasis on development
(d) faster, sustainable and more inclusive growth
of basic and heavy industries” ?
75. ‘Pradhan Mantri Jan-Dhan Yojana’ has been launched
(a) First (b) Second
for [CSAT-2015-I]
(c) Third (d) Fourth
67. In which of the following years was the planning (a) providing housing loan to poor people at cheaper
commission set up in India ? interest rates
(a) 1947 (b) 1948 (b) promoting women’s Self-Help Groups in backward
(c) 1949 (d) 1950 areas
68. Who is the ex-officio Chairman of the Planning (c) promoting financial inclusion in the country
Commission of India ? (d) providing financial help to the marginalized
(a) Vice-President of India communities
(b) The Prime Minister of India 76. The Government of India has established NITI Aayog to
(c) The Finance Minister of India replace the [CSAT-2015-I]
(d) The Law Minister of India (a) Human Rights Commission
69. In which one the following Five Year Plans the actual (b) Finance Commission
growth performance in India in respect of GDP(at factor (c) Law Commission
cost) was less than the target set ? (d) Planning Commission
Planning, Unemployment & Poverty in India E-35

Select the correct answer using the codes given below:


Statement Based MCQ (a) 1 only (b) 2 and 3
(c) 1 and 3 (d) 1, 2 and 3
1. Which of the two employment programmes are being 7. With reference to “Aam Admi Bima Yojana’’, consider the
merged in newly introduced Prime Minister’s Employment following statements
Generation Programme? 1. The member insured under the scheme must be the
1. PMRY 2. NREP head of the family or an earning member of the family
3. REGP 4. RLEGP in a rural landless house-hold.
Choose the right option 2. The member insured must be in the age group of 30 to
(a) 1 only (b) 1 and 2 65 years.
(c) 1 and 3 (d) 2 and 4 3. There is a provision for free scholarship for upto two
2. Which of the following is/are correct? children of the insured who are studying between
1. Bulk of unemployment in India is in rural areas. classes 9 and 12.
2. The disguised unemployment in agricultural sector is Which of the statements given above is/are correct?
perennial. (a) 1 only (b) 2 and 3
3. Industrialization has rendered several people jobless in (c) 1 and 3 (d) 1, 2 and 3
India. 8. Consider the following statements regarding Indian Planning:
(a) 1 and 2 (b) 1, 2 and 3 1. The Second Five-Year Plan emphasized on the
(c) 1 only (d) 2 and 3 establishment of heavy industries.
3. Which of the following were the priority objectives of the 2. The Third Five-Year Plan introduced the concept of
Eighth plan? import substitution as a strategy for industrialization.
1. Universalization of elementary education. Which of the statements given above is/are correct?
2. Growth and diversification of agriculture. (a) 1 only (b) 2 only
3. Containment of population growth. (c) Both 1 and 2 (d) Neither 1 nor 2
4. Gradual privatization of most public sector undertakings. 9. Planning Commission of India
Choose your answer from the following codes 1. was set up in 1950
(a) 1, 2 and 4 (b) 2, 3 and 4 2. is a constitutional body.
(c) 1, 3 and 4 (d) 1, 2 and 3 3. is an advisory body
4. With reference to the government’s welfare schemes, 4. is a government department
consider the following statements. (a) 1 and 2 (b) 2 and 3
1. Under the Antyodaya Anna Yojana, the foodgrains are (c) 1 and 3 (d) 3 only
available to the poorest of the poor families at ` 2 kg 10. _____ is/are not matched correctly?
for wheat and ` 3 kg of rice. 1. First Plan 1950-55
2. Under the National Old Age Pension Scheme, the 2. Third Plan 1961-66
old and destitute are provided ` 75 month as Central 3. Fourth Plan 1966-71
Pension, in addition to the amount provided by most 4. Seventh Plan 1985-90
State Governments. (a) 1 and 2 (b) 3 only
3. Government of India has allocated 25 kg foodgrains (c) 1 only (d) 1 and 3
per below poverty line family per month, at less than 11. Basic objectives of Indian plans are
half the economic cost. 1. economic growth
Which of these statements are correct? 2. self-reliance
(a) 1 and 2 (b) 1 and 3 3. employment generation
(c) 2 and 3 (d) 1, 2 and 3 4. population growth
5. Which of the following can aid in furthering the Government’s (a) 1, 2 and 4 (b) 1, 2 and 3
objective of inclusive growth ? (c) 2 and 3 (d) 1, 2, 3 and 4
1. Promoting Self-Help Groups. 12. ____ is an achievement of Indian planning.
2. Promoting Micro Small and Medium Enterprises. 1. development of infrastructure
3. Implementing the Right to Education Act. 2. diversification of industry and exports
Select the correct answer using the codes given below: 3. high growth in national in-come
(a) 1 only (b) 1 and 2 4. control over prices
(c) 2 and 3 (d) 1, 2 and 3 (a) 1 and 2 (b) 1, 2 and 3
6. The Multi-dimensional Poverty Index developed by Oxford (c) 1, 2 and 4 (d) 2 and 3
Poverty and Human Development Initiative with UNDP 13. Which plans and features are wrongly matched?
support covers which of the following? 1. First Plan - Community development projects
1. Deprivation of education, health, assets and services at 2. Second - Heavy industries
household level. 3. Third - Green Revolution
2. Purchasing power parity at national level. 4. Fourth Plan
3. Extent of budget deficit and GDP growth rate at (a) 1 and 4 (b) 3 and 4
national level. (c) 3 only (d) 1 and 3
E-36 Planning, Unemployment & Poverty in India

14. Fourth Plan failed due to (2) The third five year plan aimed to achieve self-
1. Indo-China conflict sufficiency in foodgrains and increase agricultural
2. Bangladesh refugee influx production to meet the requirements of industry and
3. bad monsoons exports.
4. bad prices situation Which of the statements given above is/are correct?
(a) 1, 3 and 4 (b) 1, 2 and 3 (a) 1 only (b) 2 only
(c) 2, 3 and 4 (d) 1, 2, 3 and 4 (c) Both 1 and 2 (d) Neither 1 nor 2
15. National Rural Employment Guarantee Act ensures that the 22. Consider the following statements about Indian Economy?
1. Centre bears 75% of the cost of wages of unskilled (1) During 11th Five Year Plan, the foodgrain
manual workers. production steadily increased.
2. State government provides 100 days of work to every (2) During 11th Five Year Plan, the average inflation
member of a household in a financial year. on the wholesale price Index is above 10%.
(a) 1 only (b) 2 only Which of the statements given above is/are correct?
(c) Both 1 and 2 (d) Neither 1 nor 2 (a) only 1 (b) only 2
16. Consider the following statements : (c) Both 1 and 2 (d) Neither 1 nor 2
1. The period of the 10th Five Year Plan is 2003-2008. 23. Consider the following statements :
2. The 10th Five year Plan approach towards tourism signifies (1) MNREGA was launched in the 11th five year plan.
a distinct shift from that adopted in previous plans. (2) Indira Awas Yojana was launched in the 9th Five
Which of the statements given above is/are correct ? Year Plan.
(a) 1 only (b) 2 only Which of the statements given above is/are correct?
(c) Both 1 and 2 (d) Neither 1 nor 2 (a) only 1 (b) only 2
17. Which among the following is not correct with regard to
(c) Both 1 and 2 (d) Neither 1 nor 2
24. Consider the following statements :
Sampoorna Garmeena Rozgar Yojana?
(1) The Ninth Five Year Plan was launched in the 50th
1. The cash component of the programme is borne
year of India’s independence.
exclusively by the Central Government.
(2) For the first time in the Indian economy the GDP
2. Foodgrains are provided free of costs to the States/Union
growth rate of the Ninth Five Year Plan was set at
Territories. 7%.
Select the answer using the code given below: Which of the statements given above is/are correct?
(a) 1 only (b) 2 only (a) 1 only (b) 2 only
(c) Both1 and 2 (d) Neither 1 Nor 2 (c) Both 1 and 2 (d) Neither 1 nor 2
18. _____ was the priority objectives of the 10th Plan? 25. Consider the following statements :
1. 7.92% growth of CPP (1) In the 11th Five Year Plan, the growth rate of the
2. Increasing literacy rate to 75 per cent export was always positive.
3. Five years of schooling to all children by 2007 (2) In the middle of the 11th Five Year Plan, the import
4. Creating 100 million jobs in 5 years growth was negative due to external factors.
(a) 1, 2 and 4 (b) 2, 3 and 4
Which of the statements given above is/are correct?
(c) 1, 3 and 4 (d) 1, 2 and 3
(a) 1 only (b) 2 only
19. The National Development Council (NDC) :
(c) Both 1 and 2 (d) Neither 1 nor 2
(1) discusses progress of the National plan.
26. Consider the following statements :
(2) suggests the ways to achieve goals of the national (1) Indo-china war had hampered the proper progress
plan. and implementation of Fourth Five Year Plan in
(3) gives guidelines to formulate the National plan. India.
(4) suggests plan allocation. (2) In the Indian Economy, the plan Holiday took place
Consider the above statements, select the correct answer during 1966 to 1969
from the codes given below : Which of the statements given above is/are correct?
Codes : (a) 1 only (b) 2 only
(a) 1, 2 and 3 (b) 2, 3 and 4 (c) Both 1 and 2 (d) Neither 1 nor 2
(c) 1, 2 and 4 (d) All of the above 27. Consider the following statements :
20. Inclusive growth would necessitate : (1) The state sets broad parameters and goals for the
(a) Development of infrastructural facilities economy.
(b) Revival of agriculture (2) The targets to be achieved are broadly set by the
(c) Increase availability of social services such as state.
education and health. (3) The plan is made for the specific time period of
(d) All the above. about 15 years.
21. Consider the following statements regarding Indian Which of the statements given above is/are correct
planning. about the Indicative planning?
(1) The second five year plan emphasized on the (a) 1, 2 and 3 (b) 1 and 2
establishment of heavy industries. (c) 1 and 3 (d) only 1
Planning, Unemployment & Poverty in India E-37

28. Consider the following statements about the LPG model 33. In context of ‘Indira Awaas Yojana’, consider the
of economic development process in India? following statements:
(1) Disinvestment of profit making public sector (1) Indira Awaas Yojana seeks to provide shelter to
Enterprises. rural BPL households.
(2) Permitting private sectors to establish Industrial (2) Allotment under this housing scheme is made only
Units without taking a licence. in the name of the female member of the household.
(3) Chronically sick industries were referred to BIFR Which of the following is/are correct?
for the formulation of revival or rehabilitation plan. (a) Only1 (b) Only 2
Which of the processes given above is/are taken after (c) Both 1 and 2 (d) Neither 1 nor 2
adoption of LPG model? 34. A person qualifies for insurance under the Aam Aadmi
(a) 1, 2 and 3 (b) 1 and 3 Bima Yojana only if he ______
(c) 2 and 3 (d) only 2 1. Is earning member of rural household
29. Consider the following two statements and answer the 2. Is Landless
question– 3. Belongs to BPL category
I. The rural poverty alleviation programme will 4. Between 15- 59 years of age which among the above
henceforth run on mission mode. statements is / are correct?
II. National Rural Livelihood Mission is centrally (a) Only 1 (b) 1 & 2
sponsored programme implemented by the states. (c) 1, 2 , 3 & 4 (d) 1 , 3 & 4
(a) Only I is correct 35. Consider the following:
(b) Only II is correct 1. Low per capita income
(c) Both I and II are correct 2. Low rate of capital formation
(d) None is correct 3. Working force largely in the tertiary sector
30. Which of the statement[s] is true regarding the NDC Which among the above is/ are the characters of an
1. to prescribe guidelines for the formulation of the underdeveloped economy?
National Plan, including the assessment of resources (a) 1 & 2 (b) 2 & 3
for the Plan; (c) 1 & 3 (d) 1, 2 & 3
2. to consider the National Plan as formulated by the 36. Consider the following statements:
Planning Commission; 1. Indira Gandhi Matritva Sahyog Yojana (IGMSY) has
3. to consider important questions of social and been launched to improve the health and nutrition
economic policy affecting national development; status of pregnant, lactating women and infants.
and 2. The scheme envisages Cash Incentives for the above
4. to review the working of the Plan from time to time beneficiaries
and to recommend such measures as are necessary 3. All Government Women Employees are beneficiaries
for achieving the aims and targets set out in the of the IGMSY
National Plan.
Which among the above statements is / are correct?
(a) Only 1, 2,and 3 (b) Only 2, 3
(a) Only 1 & 2 are correct
(c) Only 1 and 2 (d) All of them
(b) Only 2 & 3 are correct
31. Who among the following constitute the National
(c) Only 1 is correct
Development Councils?
(d) All are correct
1. The Prime Minister of India
37. Which among the following is not correct with regard to
2. The Leader of Opposition
3. Minister of Union Cabinet Sampoorna Garmeena Rozgar Yojana?
4. Chief Minister of the States 1. The cash component of the programme is borne
Select the correct answer using the given code below. exclusively by the Central Government.
(a) 1 and 2 only 2. Foodgrains are provided free of costs to the States/
(b) 1, 2, 3 and 4 Union Territories.
(c) 1, 3 and 4 only Select the answer unsing the code given below:
(d) 1, 2 and 3 only (a) 1 only (b) 2 only
32. Consider the following statements: (c) Both 1 and 2 (d) Neither 1 Nor
1. Bulk of employment in India is in rural areas. 38. Consider the following statements
2. The disguised unemployment in agricultural sector 1. Food for Work Programme was launched in India
is perennial. during the 10th Five Year Plan.
3. Industrialization has rendered several people jobless 2. The Planning commission in India is a constitutional
in India. Which of the statement(s) given above is/ body.
are correct? Which of the statements given above is/are correct?
(a) 1 and 2 (b) 1 and 3 (a) Only 1 (b) Only 2
(c) 2 and 3 (d) 1, 2 and 3 (c) Both 1 and 2 (d) Neither 1 nor 2
E-38 Planning, Unemployment & Poverty in India

39. The 11th Five Year Plan strategy to raise agricultural 3. The major objective of third plan is continued
output envisages which of the following? emphasis on basic industries together with an
1. Greater attention to land reforms. increased outlay of 35% in agriculture and applied
2. Double the rate of growth of irrigated area. sectors.
3. Promote animal husbandry and fishery. Which among the following statement is correct:
4. Interest free credit to the farmers. (a) 1 only (b) 2 only
Select the correct answer using the codes given below (c) 3 only (d) 3 and 2
(a) 1 and 3 (b) 2 and 3 45. Consider the following statement
(c) 1, 2 and 3 (d) 2 and 4 1. The plan holiday was declared for three years as
40. ‘Inclusive growth’ is a phrase used in India’s there was discontinuity in the planning process.
1. 9th Plan 2. 10th Plan 2. Plan period was from 1965-1968
th
3. 11 Plan 4. 12th Plan Which among the following Statement is correct:
Select the correct answer using the codes given below. (a) 1 only (b) 2 only
(a) 1, 2 and 3 (b) 2 and 4 (c) 1 and 2 (d) 1 and 3
(c) 3 and 4 (d) Only 4 46. In _____ demand for labour falls short to the supply due
41. Consider the following statement: to rapidly growing population and their immobility
1. The Planning Commission was established in 1950, 1. Structural Unemployment
by Article 39 of the Directive Principles of the 2. Disguised Unemployment
Constitution of India. 3. Seasonal Unemployment
2. Prime Minister, Narendra Modi dissolved the Choose the correct Code
planning commission in his first Independence Day (a) 1 only (b) 2 only
speech in 2014. It has since been replaced by a new (c) 3 only (d) 2 and 3
institution named niti Aayog. 47. Which of the following statements is/are incorrect?
Which among the following statement is not true? 1. Open Unemployment: It refers to an unemployment
(a) 1 only (b) 2 only that occurs at certain seasons of the years. The
(c) 1 and 2 (d) Neither 1 nor 2 period of such unemployment varies from state to
42. NITI AAYOG is defined as: state, depending upon the methods of farming, the
1. The National Institution for Transforming India condition of soil, the type and numbers of crops
aayog grown, etc.
2. It has replaced YojanaAayog by Prime Minister, 2. Frictional Unemployment: It refers to temporary
Narendra Modi unemployment that exists during the period, wherein
3. Niti Aayog provides less opportunity than Yojana workers leave one job and join some other.
Aayog 3. Cyclical Unemployment: It is associated with the
Which among the following statement is true? downswing and depression phases 9 business cycle.
(a) 1 only (b) 2 only It is the most common type of unemployment in the
(c) 1 and 2 (d) 1 and 3 developed capitalist economies.
43. The difference between Niti Aayog and Yojana Aayog is Select the incorrect definition using the code given below:
as follows: (a) 1 only (b) 2 only
1. In Niti Aayog there are 3 full time members while (c) 3 only (d) 2 and 1
Yojana Aayog has 8 members. 48. Scheme ______ is a Government of India program aimed
2. NITI is a think-tank and does not have the power at providing 24x7 uninterrupted power supply to all
to impose policies while Yojana impose policies on homes in Rural India
states and tied allocation of funds with projects it 1. Deendayal Disabled Rehabilitation Scheme
approved. 2. Deen Dayal Upadhyaya Gram JyotiYojana
3. NITI has power to allocate funds while Yojana 3. Digital IndiaProgramme
doesn’t. Select the name of the scheme
Which among the following statement is true? (a) 1 only (b) 2 only
(a) 1 and 2 (b) 1 and 3 (c) 3 only (d) 2 and 3
(c) 2 and 3 (d) 1, 2 and 3 49. Consider the following statements
44. Consider the following statement 1. Absolute Poverty: It refers to the total number
1. The major objective of first plan is to increase National of people living below Poverty line. As per this
Income by 25% through rapid industrialisation. This measure, around 30% of India’s population is below
plan was developed by Professor Mahalanobis. poverty line.
2. The major objective of second plan is to stimulate 2. Reative Poverty: It refers to Poverty of people,
balanced economic development while correcting in comparison to other people, regions in the nation.
imbalances caused by World War II and partition. Which among the following statement is/ are incorrect?
The plan targeted the socio-economic development (a) 1 only (b) 2 only
but the outcome was very poor. (c) Neither 1 nor 2 (d) Both 1 and 2
Planning, Unemployment & Poverty in India E-39

50. Which among the following is the objective of Prime 56. Which of the following were the priority objectives of the
Minister RojgarYojana (PMRY) Eighth plan?
1. To provide self-employment to educated unemployed 1. Universalization of elementary education.
by setting up of sever lakh micro-enterprises 2. Growth and diversification of agriculture.
2. To promote micro-enterprises and helping the rural 3. Containment of population growth.
poor into self-help group. 4. Gradual privatization of most public sector
3. To create employment and durable assets in rural undertakings.
areas. Choose your answer from the following codes
4. Improving the quality of life of people in rural areas. (a) 1, 2 and 4 (b) 2, 3 and 4
Choose the correct code (c) 1, 3 and 4 (d) 1, 2 and 3
(a) 1 only (b) 2 only
(c) 3 only (d) 4 only Matching Based MCQ
51. Consider the following statements
1. Since 1993-94, the benefit of Indira AwasYojana DIRECTIONS (Qs. 57-66) : Match List-I with List-II and
is being provided even to those rural poor of non- select the correct answer using the codes given below the lists.
schedule caste/schedule tribe who are living below
57. List-I List-II
the poverty line.
(A) Development (1) UN India Human
2. A minimum of 60% of funds is to be utilized for
Programme Development Report
construction of houses for the SC/ST people.
(B) National Council of (2) India Development
3. From 1995-96, IAY benefits have been extended to
Applied Economic Report
widows or next to kin of defence personnel killed in
Research
action.
(C) Indira Gandhi Institute (3) World Development
4. Benefits have also been extended to ex-servicemen
of Development Report
and retired members of Para military forces as long
Research
as they fulfil the normal eligibility conditions of IAY.
(D) World Bank (4) Human Report
Which of the following is / are correct
Development
(a) 1 and 2 (b) 3 and 4
(a) A – 4 ; B – 1 ; C – 2 ; D – 3
(c) 1 and 4 (d) 1, 2, 3, 4
52. Whose objective is to facilitate credit support to poor (b) A – 4 ; B – 2 ; C – 1 ; D – 3
women for this socio-economic upliftment. (c) A – 2 ; B – 3 ; C – 4 ; D – 1
(a) RashtriyaMahilaKosh (d) A – 2 ; B – 1 ; C – 4 ; D – 3
(b) National Rural Health Mission 58. Match List I with List II and select the correct answer
(c) Village Grain Bank Scheme using the codes given below the list
(d) Annapurna Scheme List-I List-II
53. Consider the following statement regarding usual status (Five Year Plan) (Objective)
1. One year 2. Indefinite A. First Plan 1. Growth with social justice
3. One month 4. One week B. Third Plan 2. Sustainable inclusive growth
Usual status (US) unemployment is calculated with C. Fifth Plan 3. Self reliant and generating
reference to a period of? economy
(a) 1 only (b) 2 only D. Twelfth plan 4. Agriculture, Irrigation and
(c) 3 only (d) 4 only power projects
54. Consider the following statements: Codes :
1. Indira Gandhi Matritva Sahyog Yojana (IGMSY) has A B C D
been launched to improve the health and nutrition (a) 2 4 3 1
status of pregnant, lactating women and infants. (b) 2 3 4 1
2. The scheme envisages Cash Incentives for the above (c) 4 3 1 2
beneficiaries (d) 1 3 2 1
3. All Government Women Employees are beneficiaries 59. Match the items in List-I with List-II:
of the IGMSY List-I List-II
Which among the above statements is / are correct? A. Reproductive 1. Police Research
(a) 1 and 2 (b) 2 and 3 Child Health
(c) 1 only (d) 1, 2, 3 B. The National Rural 2. Trauma
55. Which of the two employment programmes are Employment
being merged in newly introduced Prime Minister’s Guarantee Act
Employment Generation Programme?
C. Post Traumatic 3. Mother
1. PMRY 2. NREP
3. REGP 4. RLEGP Stress Disorder and Child
Choose the right option D. Bureau of Police 4. Employment at
(a) 1 only (b) 1 and 2 Research and village level
(c) 1 and 3 (d) 2 and 4 Development
E-40 Planning, Unemployment & Poverty in India

Codes : 63. Match the important dates


A B C D Dates Event
(a) 2 1 3 4 I. 1934 a. Planning commission set
(b) 3 4 2 1 up
(c) 4 3 2 1 II. 1944 b. Interim government sets
(d) 1 2 3 4 up the Planning Advisory
60. Match List-I with List-II and select the correct Board
answer from the codes given below – III. 1946 c. M. Visvesvaryya, in his
List-I List-II book Planned Economy
A. Sarva Shiksha 1. 1987 of India, advocates the
necessity of planning in
Abhiyan
the country much before
B. Saakshar Bharat 2. 1988
Independence
Mission
IV. 1950 d. Planning and Development
C. Operation Blackboard 3. 2001 Department set up under
D. National Literacy 4. 2009 the chairmanship of A.
Mission Dalal
Codes : (a) I-a, II-b, III-c,IV-d
A B C D (b) I-c, II-d, III-b, IV-a
(a) 3 4 1 2 (c) I-d, II-c, III-b, IV-a
(b) 4 3 2 1 (d) I-a, II-d, III-a, IV-b
(c) 1 2 3 4 64. Match the plans with the years
(d) 1 2 4 3 Plan Year
61. Match the following I. First Plan a. 1961-1966
List-I List-II II. Second Plan b. 1951-1956
(Five Year Plan) (Emphasis) III. Third Plan c. 1956-1961
A. First 1. Food security and women (a) I-a, II-b, III-c
empowerment (b) I-b, II-c, III-a
B. Second 2. Heavy industries (c) I-c, II-b, III-a
(d) I-a, II-c, III-b
C. Fifth 3. Agriculture and
65. Match the following
community development Date of Launch Scheme
D. Ninth 4. Removal of poverty I. 2007 a. Aam Admi Bima Yojana
Codes : II. 2009 b. Bachat Lamp Yajana
A B C D III. 2015 c. Atal Pension Yojana
(a) 1 2 4 3 (a) I-a, II-b, III-c
(b) 1 4 2 3 (b) I-c, II-b, III-a
(c) 3 2 4 1 (c) I-b, II-c, III-a
(d) 3 4 2 1 (d) I-c, II-c, III-b
62. Match the members of NitiAayog: 66. Match the following
Column-A Column-B Year of starting Name of programme
I. Chairperson a. Arvind Panagariya I. 1971-72 a. Pilot Intensive Rural
II. CEO b. Narendra Modi Employment
III. Vice Chairperson c. Nitin Gadkari II. 1972-73 b. Crash Scheme for Rural
IV. Special Invitees d. Smt. Sindhushree Khullar Employment
(a) I-b, II-d, III-a,IV-c III. 1973-74 c. Food for work Programme
IV. 1977-78 d. Drought Prove Areas
(b) I-a, II-b, III-c, IV-d
Programme
(c) I-c, II-b, III-d, IV-a (a) I-a, II-b, III-c, III-d (b) I-b, II-a, III-d, IV-c
(d) I-c, II-b, III-d, IV-a (c) I-d, II-c, III-b, IV-a (d) I-a, II-d, III-a, IV-b
Planning, Unemployment & Poverty in India E-41

EXERCISE-1 poor and enact wide ranging social and economic


reforms.
1. (c) 2. (d) 3. (d) 4. (b) 5. (c) 6. (a)
55. (d) Charan Singh wrote the book named, “Indian
7. (d) 8. (b) 9. (d) 10. (c) 11. (c) 12. (a)
Economy: Gandhian Blue Print”. He was the fifth
13. (b) 14. (d) 15. (b) 16. (b) 17. (a) 18. (d)
Prime Minister of India, serving from 1979 until
19. (b) 20. (d) 21. (c) 22. (d) 23. (d) 24. (c)
25. (b) 26. (c) 27. (c) 28. (d) 29. (c) 1980.
30. (d) All adult members of the household who registered 56. (b) Sixth five year plan (1980–85) marked the beginning
can apply for work. To register, they have to : Be of economic liberalisation.
local residents In Sixth Five year plan, a shift in pattern of
31. (c) Inclusive growth is ‘broad-based growth’, ‘shared Industrialisation with lower emphasis of the heavy
growth’, and ‘pro-poor growth’. It excludes the industry and move on infrastructure begins.
capital markets which is left to operate by itself in 57. (a) Under the third five year plan, the government
the open market. introduce an agricultural strategy which gave rise to
32. (c) It was during eighth plan period (1992-97) that green revolution in India. Under the third plan the
emphasis was put on infrastructure development. goal was to establish self reliance and self generating
33. (c) 34. (b) 35. (b) 36. (a) 37. (a) 38. (a) economy. But the earlier plans had shown that the
39. (c) 40. (b) 41. (b) 42. (a) 43. (c) 44. (d) agricultural production was the limiting factor so
45. (a) agriculture was given top priority .
46. (b) The Second Five Year Plan’s broad objective was 58. (a) 59. (c) 60. (c) 61. (d)
establishment of a socialistic pattern of society. It 62. (a) 63. (d) 64. (b) 65. (c)
aimed at rebuilding rural India, development of 66. (b) The Second Five Year Plan’s broad objective was
basic and heavy industries and enlarging scope of establishment of a socialistic pattern of society. It
public sector. aimed at rebuilding rural India, development of
47. (d) The revised targeted annual rate of growth of 9th basic and heavy industries and enlarging scope of
plan was 6.5% while the actual growth rate was public sector.
5.5%. 67. (d) In March 1950, Government of India constituted a
48. (b) statutory body with the Prime Minister of India as
49. (c) Supply-side measures can be used to reduce the its chairman called Planning Commission of India.
natural rate of unemployment. In this case training 68. (b)
enables more employees to accept jobs thereby 69. (d) The revised targeted annual rate of growth of 9th
reducing the natural rate of unemployment. plan was 6.5% while the actual growth rate was
50. (c) Low interest rates might encourage more spending 5.5%.
and reduce cyclical unemployment. 70. (b) 71. (c)
51. (a) After India gained independence, a formal model 72. (d) The aim of 11th Five Year Plan is to connect
of planning was adopted, and the Planning every village by telephone and provide broadband
Commission, was established on 15 March 1950, connectivity to all villages by 2012.
with Prime Minister Jawaharlal Nehru as the 73. (d) Cyclical unemployment is unemployment that results
chairman. when the overall demand for goods and services
52. (a) The second five year plan was based on in an economy cannot support full employment. It
P.C. Mahalnobis Model. The Mahalanobis model
occurs during periods of slow economic growth or
is a Neo-Marxist model of economic development,
during periods of economic contraction.
created by statistician Prasanta Chandra Mahalanobis
74. (d) 12th Five Year Plan of the Government of India aims
in 1953. Mahalanobis became essentially the key
at faster, sustainable and more inclusive growth.
economist of India’s Second Five Year Plan.
75. (c) Pradhan Mantri Jan-Dhan Yojana (PMJDY) is
53. (a) The main objective of the third five year plan was to
National Mission for Financial Inclusion to ensure
make economy self-dependent. The third Five-year
Plan stressed on agriculture and improvement in the access to financial services, namely, Banking/
production of wheat so as to be self-sufficient. Savings & Deposit Accounts, Remittance, Credit,
54. (c) Fifth five year plan was focused on poverty and Insurance, Pension in an affordable manner.
unemployment for the first time. The slogan of 76. (d) The Government of India has established NITI
‘poverty abolition’ was given by Indira Gandhi in Aayog to replace the Planning Commission. The
1971 and it was implemented during the fifth five Union Government of India announced formation of
year plan(1974–79). Gandhi promised to reduce NITI Aayog on 1 January 2015 and the first meeting
poverty by targeting the consumption levels of the of NITI Aayog was held on 8 February 2015.
E-42 Planning, Unemployment & Poverty in India

EXERCISE-2 MNREGA was launched in 200 select districts in


1. (c) 2. (b) 3. (d) 4. (d) 2006. However Indira Awas Yojana was lauched
5. (d) The concept of the inclusive growth is based on both in the 9th Five Year Plan to provide housing for the
economic and social empowerment of ordinary and rural poor in India.
under privileged individuals. Thus all the statement 24. (c) The Ninth Five Year Plan was launched in the
are correct. 50th year of India’s independence. Under the plan
6. (a) Deprivation of education, health, assets and services for the first time in the Indian Economy the GDP
at household level growth rate was set at 7%.
7. (c) The statements 1st and 3rd are correct, but 2nd 25. (b) Regarding the 11th Five Year Plan, in the middle
statement is wrong because, the member should be of the plan, the import growth was negative due to
aged between 18 and 59 years not 30 and 65 years. external factors and the growth rate of the export
8. (a) Second Five-Year Plan (1956-1961); The fourth was negative during certain periods.
Five-Year Plan (1969-74) introduced the concept of 26. (b) Despite big investments during the first three Plans
import substitution as a strategy for industrialization. the living standards of the poor could not be raised
9. (c) 10. (d) 11. (b) 12. (a) 13. (b) and poverty and inequity in distribution of state
14. (c) 15. (b) 16. (d) 17. (b) 18. (d) resources remained stark. Consequently, the period
19. (a) National Development Council aims to prescribe between 1966 and 1969 marked the shift from a
guidelines for the formulation of the National Plan, ‘growth approach’ to a ‘distribution from growth
including the assessment of resources for the Plan; approach’. Looking at the failures and pitfalls the
to consider the National Plan as formulated by planners suspended the impending Fourth Plan,
the Planning Commission; to consider important which was due in 1966, until 1969 for a revision of
questions of social and economic policy affecting objectives and targets. This was called as the ‘Plan
national development; and.to review the working of Holiday’ extending from 1 April 1966 to 31 March
the Plan from time to time and to recommend such 1969.
measures as are necessary for achieving the aims 27. (b) Indicative planning is a form of economic planning
and targets set out in the National Plan. implemented by a state in an effort to solve the
20. (d) Inclusive growth is a concept which involves problem of imperfect information in market and
equitable allocation of resources during the process mixed economies in order to increase economic
of economic growth with benefits incurred by every performance. Regarding the indicative planning,
section of society. Inclusive growth necessitates state sets broad parameters and goals for the
development of infrastructural facilities, revival of economy and the targets to be achieved are broadly
agriculture and also increases availability of social set by the state.
services such as education and health. 28. (a) LPG model of economic development process in
21. (c) The Second Five Year Plan heralded in a true sense India involves disinvestment of profit making public
the Socialist Project of the then Prime Minister, sector enterprises, permitting private sectors to
Jawaharlal Nehru. The economic policies of Nehru establish industrial units without taking a licence and
were heavily influenced by the erstwhile Union chronically sick industries were referred to BIFR
of Soviet Socialist Republics (USSR) which had for the formulation of revival or rehabilitation.
followed the path of speedy industrialisation to 29. (c) 30. (d) 31. (c) 32. (a) 33. (b)
expand the manufacturing base of its economy. 34. (b) 35. (a) 36. (a) 37. (b)
The Second Five year Plan focused mainly on 38. (a) According to The National Food for Work
heavy industry as against the First Plan which was Programme, food subsidy should be better targeted
essentially an agricultural plan. This was done to through targeted public distribution system and
boost domestic production and manufacturing of specific programmes for the poor like Food for
goods. Third plan aimed to achieve self-sufficiency Work Programme. The National Food for Work
in foodgrains and to increase agricultural production Programme was launched on 14 November 2004
to meet the requirements of industry and exports. in 150 of the most backward districts of India with
22. (c) During The eleventh plan ( 2007-12)the food grain the objective of generating supplementary wage
production steadily increased and the average employment. The Planning commission is not a
inflation on the wholesale price Index was above constitutional body.
10%. 39. (c) 11th Five Year Plan’s initiatives for agriculture
23. (b) The Mahatma Gandhi National Rural Employment were:
Guarantee Act (MNREGA) is an Indian law that • Focusing on land reform issues
aims to guarantee the ‘right to work’ and ensures • Doubling the rate of growth of irrigated area
livelihood security in rural areas by providing at • Promoting animal husbandry and fishery
least 100 days of guaranteed wage employment in • Providing easy access to credit at affordable rates
a financial year to every household whose adult 40. (c) Inclusive growth is a phrase used in India for 11th
members volunteer to do unskilled manual work. (2007-2012) and 12th (2012-2017) Five year plans.
Planning, Unemployment & Poverty in India E-43

41. (d) The Planning Commission was established in 1950, speech in 2014. It has since been replaced by a new
by Article 39 of the Directive Principles of the institution named nitiaayog.
Constitution of India. The planning commission 42. (c) Nitiaayog (The National Institution for Transforming
(yojanaaayog) was an institution in the government India AAYOG) established by Prime Minister
of India, which formulated India’s five-year plans, Narendra modi (chairperson) to replace the planning
among other function. Prime Minister Narendra commission. Nitiaayog will provide opportunities
that the previous planning commission structure
modi announced his intention to dissolve the
lacked, to represent the economic interests of the
planning commission in his first Independence Day
state governments and union territories of India.
43. (a)
S. No. Points of Difference NITI Aayog Planning Commission
1 Financial clout To be an advisory body, or a think-tank. Enjoyed the powers to allocate funds to
The powers to allocate fund vested in the ministries and state governments
finance ministry.
2 Full-time members Three full-time members. had eight full-time members
3 States’ role Includes the Chief Ministers of all States States’ role was limited to the National
and the Lieutenant Governors of all Union Development Council and annual interaction
territories in its Governing Council, during Plan meetings
devolving more power to the States of the
Union.
4 Member secretary To be known as the CEO and to be Secretaries or member secretaries were
appointed by the prime minister appointed through the usual process
5 Part-time members To have a number of part-time members, Full Planning Commission had no provision
depending on the need from time to time for part-time members
6 Constitution Governing Council has state chief ministers The commission reported to National
and lieutenant governors. Development Council that had state chief
ministers and lieutenant governors.
7 Organization New posts of CEO, of secretary rank, and Had deputy chairperson, a member secretary
Vice-Chairperson. Will also have two full- and full-time members.
time members and part-time members as
per need. Four cabinet ministers will serve
as ex-officio members.
8 Participation Consulting states while making policy and Policy was formed by the commission and
deciding on funds allocation. Final policy states were then consulted about allocation of
would be a result of that. funds.
9 Allocation No power to allocate funds Had power to decide allocation of government
funds for various programs at national and
state levels.
10 Nature NITI is a think-tank and does not have the Imposed policies on states and tied allocation
power to impose policies. of funds with projects it approved.
44. (c) The major objective of first plan is to stimulate 46. (a) Structural Unemployment: In this type of
balanced economic development while correcting unemployment demand for labour falls short to the
imbalances caused by World War II and partition. supply due to rapidly growing population and their
The plan targeted the socio-economic development immobility.
but the outcome was very poor. The major Objective 47. (a) Seasonal Unemployment: It refers to an
unemployment that occurs at certain seasons of the
of second plan is to increase National Income by
years. The period of such unemployment varies
25% through rapid industrialisation. This plan was
from state to state, depending upon the methods of
developed by Professor Mahalanobis. The major farming, the condition of soil, the type and numbers
objective of third plan is continued emphasis on of crops grown, etc.
basic industries together with an increased outlay of Frictional Unemployment: It refers to temporary
35% in agriculture and applied sectors. unemployment that exists during the period, wherein
45. (a) After the disastrous experience of the Third Plan, workers leave one job and join some other.
a plan holiday was declared for three years. This Cyclical Unemployment: It is associated with the
period was named as Plan Holiday as there was downswing and depression phases 9 business cycle.
discontinuity in the planning process. Gap in Plan It is the most common type of unemployment in the
Period (1966-69) developed capitalist economies.
E-44 Planning, Unemployment & Poverty in India

48. (b) Deen Dayal Upadhyaya Gram Jyoti Yojana is a 61. (c) 1. First Five Year Plan (1951-56)- Agriculture
Government of India program aimed at providing and community development
24x7 uninterrupted power supply to all homes in 2. Second Five Year Plan (1956-61)- Heavy
Rural India. It was launched in 2015. Industry
49. (c) Absolute Poverty: It refers to the total number 3. Fifth Five Year Plan (1974-79)- Removal of
of people living below Poverty line. As per this Poverty
measure, around 30% of India’s population is below 4. Ninth Five Year Plan (1997-2002)- Food
poverty line. Relative Poverty: It refers to Poverty Security and woman empowerment
of people, in comparison to other people, regions in 62. (a) The various members of NITIAAYOG are:
the nation.  Chairperson: Prime Minister Narendra Modi
50. (a) The objective of Prime Minister Rojgar Yojana  Ceo: Smt. Sindhu shree Khullar, IAS
(PMRY) is to provide self-employment to educated  Vice Chairperson: Arvind Panagariya
unemployed by setting up of sever lakh micro-  Special invitees: Nitin Gadkari, Smriti Zubin
enterprises.
Irani and Thawar Chand Gehlot
51. (d) 1. Since 1993-94, the benefit of Indira
63. (b) 1934: M. Visvesvaryya, in his book Planned
AwasYojana is being provided even to those
Economy of India, advocates the necessity of
rural poor of non-schedule caste/schedule tribe
planning in the country much before Independence.
who are living below the poverty line.
1944: Planning and Development Department set up
2. A minimum of 60% of funds is to be utilized for
construction of houses for the SC/ST people. under the chairmanship of A. Dalal.
3. From 1995-96, IAY benefits have been 1946: Interim government sets up the Planning
extended to widows or next to kin of defence Advisory Board.
personnel killed in action. 1947: Economic Programme Committee set up
4. Benefits have also been extended to ex- under the chairmanship of Jawaharlal Nehru.
servicemen and retired members of Para 1950: Planning Commission set up.
military forces as long as they fulfil the normal 64. (b) First plan – 1951-56, Second Plan – 1956-61, Third
eligibility conditions of IAY. plan – 1961-66
52. (a) The objective of Rashtriya Mahila Kosh is to 65. (d) Aam Admi Bima Yojana was launched in 2007, Atal
facilitate credit support to poor women for this socio- Pension Yojana was launched on May 9, 2015 and
economic upliftment. Bachat Lamp Yojana was launched in 2009.
53. (a) Usual status (US) unemployment is calculated with 66. (b)
reference to a period of one year. S. Name of the Year of Main Objectives
54. (a) Indira Gandhi Matritva Sahyog Yojana (IGMSY) No. Programme Starting
has been launched to improve the health and
1. Crash Scheme 1971-72 Generation of new employment
nutrition status of pregnant, lactating women and
for Rural rural development.
infants. The scheme envisages Cash Incentives for
Employment
the above beneficiaries.
55. (c) 56. (d) 57. (a) 2. Pilot Intensive 72-73 Construction work in Villages.
58. (c) First plan – agriculture, Irrigation and power projects Rural
Third Plan – self reliance and generating economy Employment
Fifth Plan – growth with social justice 3. Drought 73-74 To develop natural resources
Twelfth plan – sustainable inclusive growth Prove Areas in drought prove rural areas.
59. (b) Programme
60. (a) Sarva Shiksha Abhiyan, 2001; Saakshar Bharat
4. Food for work 77-78 To provide food for work in
Mission, 2009; Operation Blackboard, 1987.
Programme development process
National Literacy Mission, 1988.
AGRICULTURE
3
Introduction
Agriculture has always been one of the most important sectors of Indian economy, be it pre- independence or post- independence
periods. This is further proved by the large number of Indians whose livelihood depends on agriculture. Indian agriculture has
a fairly successful history. It is now first in the world in the production of milk, pulses, jute and many fruits; second in rice,
wheat, sugarcane, cotton, etc. and a leading producer of spices, plantation crop, livestock, fishes and poultry.

Agriculture

Growth and Agriculture Points to


Introduction National Major Land
Productivity Insurance remember
Agriculture Agriculture Reforms
policy Revolutions
Public Distribution
Major Cooperative AICIL
Features of Agriculture System
Agriculture Farmaing
Indian in five NAIS
Programmes Green
Agriculture year plans Revolutios
Rural Credit Agricultural FCI
Second and Extension Marketing TPDS
NFSM Green Services
Revolution Agricultural
RKVY
White Price Policy
NHM Cooperative
Revolution
Credit WTO and
ISOPOM
Blue Societies Agriculture
NMSA Revolution subsidies
NABARD
Evergreen
RADP
Revolution Kisan
NeGPA Credit Green Box
Card Subsidies
NMFP
Commerical Blue Box
Banks Subsidies
NAFED Amber Box
Subsidies
TRIFED
NCDC
E-46 Agriculture

Features of Indian Agriculture •• Indian agriculture is still largely dependent on


the uncertainties of monsoon for its irrigational
•• Agriculture is the primary occupation in India as it requirements. However, India’s water resources,
provides direct livelihood to more than 49% (2015) of if fully harnessed, can irrigate more than 50% of our
its labour force. In India, 75% of below the poverty line cultivated area.
(BPL) population lives in rural areas, and is directly or
indirectly dependent on agriculture. Growth and Productivity
•• Agriculture contributes to more than 17.9% (2015) of
GDP, although this share has progressively declined •• The total irrigated area increased from less than one
from 57% in 1950-51. In developed countries, like the million hectares per annum before the green revolution to
about 2.5 million hectares per annum during the 1970’s.
UK and USA, the share of agriculture in GDP is only
The total gross irrigated area now is 80 million hectares.
around 2%.
•• Total food grains (cereals and pulses) production increased
•• India accounts for 7.68 % (2015) of total global
from 48.1 million tonnes in 1950-51 to an expected 257.07
agricultural output. India is the second largest producer
million tonnes in 2014-15. This remarkable progress
of agricultural products in the world.
which the nation has achieved in food self-sufficiency is
•• Agriculture contributes to around 10% of the total shown in the given table.
value of India’s commodity exports. Thirteen
•• As evident in the table, overall growth in food grains
major commodities including tea, coffee, tobacco, production has been impressive, assisted largely by
cashew, spices, raw cotton and sugar are the primary technological breakthrough. However, one disturbing
agricultural exports in India. Almost 30% of tea and aspect is the year-to-year fluctuation in cereals output
50% of coffee and jute produced in the country are which affects the employment and income of the poor
exported. In addition to this, credit must be given to who depend solely on on-farm activities. This emphasizes
export of manufactured goods using agricultural raw the need for the government to procure and store food
materials, which further accounts for another 15% of grains in years of surplus and release the same through
India’s exports. PDS in drought years in order to avoid famine conditions.
•• Indian agriculture has been able to improve its per •• Although production has increased several folds,
capita net availability of food-grains to 491.2 grams Indian agriculture is still lagging behind because of low
(2014) from 395 grams in 1950s. productivity.
Change in Area and Production of Crops During 2012-13 and 2013-14
Area (MHA) Production (MT)
Name of Crops
2012-13 2013-14 change 2012-13 2013-14 change
Wheat 30.00 30.61 +0.61 93.51 95.60 +2.09
Rice (Rabi) 3.84 4.38 +0.54 12.87 14.19 +1.32
Sorghum 3.79 3.55 –0.24 2.44 3.21 +0.77
Maize 1.46 1.49 +0.03 6.06 6.40 +0.34
Barley 0.69 0.76 +0.07 1.75 1.92 +0.17
Total coarse cereals 5.94 5.80 –0.14 10.25 11.53 +1.28
Gram 8.52 9.66 +1.14 8.83 9.79 +0.96
Urad 0.69 0.75 +0.06 0.47 0.41 –0.06
Moong 0.74 0.65 –0.09 0.40 0.31 –0.09
Total Pulses 13.30 14.66 +1.36 12.43 13.52 +1.09
Total food grains 53.09 55.45 +2.36 129.06 134.84 +5.78
Groundnut 0.79 1.08 +0.29 1.51 2.12 +0.61
Rapeseed &Mustard 6.36 6.63 +0.27 8.03 8.25 +0.22
Total Rabi Oilseeds 8.16 8.63 +0.47 10.15 10.96 +0.81
Total Area /Production 61.25 64.08 +2.84 139.21 145.80 +06.59

It is evident that the yield per hectare of food grains has shown variety of seeds, plant protection measures, fertilizers and
remarkable increase in the post Green Revolution period. irrigation. The growth was less impressive in the case of rice
The growth is comparatively less in the case of wheat where and minimal in pulses, as the latter is still grown in marginal
technology could provide an optimum mix of high-yielding lands under rain-fed conditions in vast areas of the country.
Agriculture E-47

Agriculture in Five Year Plans


Five year Plan Major Features
1 (1951-56)
st
•• Launch of the Community development Programme, abolition of Zamindari system, campaigns for
growth in food and other related areas like fisheries, forestry, animal husbandry, soil conservation,
etc. were the major features.
•• Growth in agriculture was 2.71%.
2nd (1956-61) •• Industrial sector was given more importance in this plan.
•• Industrial sector was given more importance in this plan.
•• Agricultural Expenditure was only 20% of the actual plan expenditure.
•• The agricultural growth, however, was high at 3.15%.
3rd (1961-66) •• Achieving self- sufficiency in foodgrains and increase in agricultural production was one of the main
aims of this plan.
•• Higher priority was given to agricultural and allied areas as compared to industrial development.
•• However, the plan did not achieve its goals and agricultural growth fell to 0.73%.
•• Land reforms, land ceiling and Green Revolution were some of the major initiatives in this plan.
Annual Plans •• Priority was given to minor irrigation projects and High Yielding Variety of seeds was preferred so as
(1966-69) to increase agricultural productivity.
•• Agricultural growth was high at 4.16%.
4 (1969-74)
th
•• The results of the introduction of Green revolution and HYV seeds were good.
•• Expenditure on agriculture was 22% of annual expenditure.
•• Agricultural growth was 2.57%.
5th (1974-79) •• Emphasis was laid on spread of HYV seeds, use of fertilizers, pesticides and insecticides to increase
production.
•• Expenditure on agriculture was around 21% of annual expenditure.
•• Agricultural growth was 3.28%.
6th (1980-85) •• It was realised by this plan that growth of Indian economy depends on rural and agricultural development.
•• The growth rate in agricultural production was a high 4.3% against a target of 3.8%.
•• Overall growth in agricultural sector was 2.52%.
7th (1985-90) •• Expenditure on agriculture was 22% of annual expenditure.
•• Growth in agriculture was 3.47%
8th (1992-97) •• The growth target was 4.1% but the agricultural sector showcased an impressive growth of 4.68%.
9th (1997-2002) •• This plan was a failure in the agricultural sector and it registered an agricultural growth rate of 2.44%.
10th (2002-07) •• Against a target of 4%, the average agricultural growth rate was only 2.3%.
11th (2007-12) •• The major emphasis was on increasing agricultural productivity and profitability by making available
affordable institutional credit, farm mechanisation, biotechnology, cold storages, and marketing.
•• Growth in agriculture was 3.5%.
12 (2012-17)
th
•• This plan, like its predecessors, has a target of 4% agricultural growth rate, with growth in food-grains
at 2% and non- food grains at 5.6%.
•• The plan puts an emphasis on improvement in technology, use of public- private partnership, greater
road connectivity, development of horticulture, dairying, and other related agricultural fields.

National Agricultural Policy 1. Over 4 % annual growth rate aimed over next two decades.
2. Greater private sector participation through contract
The National Agriculture Policy aims to realise the vast farming.
untapped potential for growth in Indian agriculture, developing 3. Price protection for farmers.
rural infrastructure, promote value addition, accelerate the 4. National agricultural insurance scheme to be launched.
growth of agro business, create employment in rural areas, 5. Dismantling of restrictions on movement of agricultural
secure a fair standard of living for the farmers and agricultural commodities throughout the country.
workers and their families, discourage migration to urban areas 6. Rational utilisation of country’s water resources for
and face the challenges arising out of economic liberalization optimum use of irrigation potential.
and globalisation. The major features of the new agricultural 7. High priority to development of animal husbandry,
policy are: poultry, dairy and aquaculture.
E-48 Agriculture

8. Capital inflow and assured markets for crop production. related to “sustainable agriculture” in the context of risks
9. Exemption from payment of capital gains tax on associated with climate change. It hopes to achieve its aims by
compulsory acquisition of agricultural land. preparing appropriate adaptation and mitigation strategies for
10. Minimise fluctuations in commodity prices. enhancing livelihood opportunities, ensuring food security and
11. Continuous monitoring of international prices. contributing to economic stability.
12. Plant varieties to be protected through a legislation. Rainfed area Development Programme (RADP):
13. Adequate and timely supply of quality inputs to farmers. RADP was launched by the government as a pilot scheme
14. High priority to rural electrification. under RKVY, focusing on small and marginal farmers and
15. Setting up of agro-processing units and creation of off- farming systems. It targets integrated farming, on- farm water
farm employment in rural areas. management, storage, marketing and value addition of farm
produce in order to enhance farmers’ income in rain-fed areas.
Major Agriculture Programmes
National e-Governance Plan in Agriculture
The Central government supplements the efforts of the state
governments through centrally sponsored schemes. Some of NeGPA
these major schemes are as follows : This aims to achieve rapid development of agriculture in
India through ICT enabled  multiple delivery channels such
National Food Security Mission (NFSM) as Internet, Government Offices, Touch Screen Kiosks, Krishi
NFSM was launched in 2007-08 to increase the production of Vigyan  Kendras , Kisan Call Centres,  Agri-Clinics, Common
rice, wheat and pulses by 10, 8 and 2 million tonnes respectively, Service Centres, Mobile Phones for ensuring timely access to
by the end of 11th plan through productivity enhancement and agriculture related information for the farmers of the country. 
area enhancement, enhancing farm level economy and creating This includes information on pesticides, fertilizers, seeds, soil
employment opportunities. This has been extended to the 12th health, weather forecast, fishery inputs, drought relief and
plan too with additional production targets. management, etc.
Rashtriya Krishi Vikas Yojana (RKVY) National Mission on Food Processing (NMFP)
RKVY was launched in 2007-08 for the 11th plan to incentivize NMFP was launched in 2012. It is a centrally sponsored
states to enhance investment in agriculture and allied sectors scheme for giving greater role to states/ UTs, decentralised
to achieve 4% growth rate. The government has approved administration, better outreach and effective supervision and
continuation of the RKVY scheme in the 12th plan whereby the monitoring. The NMFP would also provide flexibility to states/
funding will be routed into three components- UTs in the selection of beneficiaries, location of projects, etc.
(i) production growth, for development of food processing industry in India.
(ii) infrastructure and assets, and
(iii) sub-schemes and flexi-fund. Major Agricultural Revolutions
National Horticulture Mission (NHM) Green Revolution
NHM was launched in 2005-06 for promotion of holistic Introduction
growth of horticulture sector, including fruits, vegetables,
root and tuber crops, mushroom, spices, flowers, aromatic The world’s worst recorded disaster the Bengal famine, which
plants, cashew and cocoa. Central share would be 85%, while hit India in year 1943 claimed approximately 4 million lives.
the states will contribute 15% of the share. From 2014-15, It was, therefore, first priority of the government to ensure
the Mission for Integrated Development in Horticulture food security for all. This led to various measures taken by
(MIDH) has been started by bringing all ongoing schemes government to increase production of food grain in the country.
related to horticulture under a single umbrella scheme. The green Revolution which began in year 1967/68 focused
on expansion of farming areas and use of high yield variety
Integrated Scheme For Oilseeds, Pulses, Oil seeds along with adoption of double cropping in the existing
Palm And Maize (Isopom) farmland.
ISOPOM provides flexibility to states in implementation based It included a technology package. Comprising use of HYV
on a regionally differentiated approach for promoting crop of two staple cereals (rice and wheat), improved irrigation,
diversification and providing a focus to the programme. This proper utilisation of fertilizer and pesticides and associate
scheme is under implementation in the country for increasing management skills. M.S. Swaminathan is considered father
area, production and yield of these crops from 2004-05. The of Green Revolution in India.
pulses component was merged with NFSM w.e.f. April 2010.
Objectives of the Green Revolution
National Mission for Sustainable Agriculture •• To ensure food security by increasing production of main
(NMSA) crops like wheat, rice, etc.
NMSA is one of the eight missions outlined under National •• To increase agriculture productivity.
Action Plan for Climate Change. It seeks to address issues •• To enhance modernisation of farm practices.
Agriculture E-49

Impact of Green Revolution the dairy development through the co-operative societies.
•• The Green Revolution has resulted in phenomenal increase Prof. Verghese Kuerin was the father of White Revolution
in the production of ‘wheat’ gain maximum benefit from in India. White revolution helped India become not only self
green Revolution. sufficient but also the largest dairy producer of the world.
It works through a network of co-operative societies which
•• This revolution led to prosperity of farmers, especially
are owned and managed by the milk producers. The co-
those who were having more than 10 hectares of land.
operative societies were most successful in the Anand District
•• Increased production of food grains resulted in reduction of Gujarat. The main objectives of the co-operative society
in imports. Also, sometimes India exported foodgrains. are the procurement, transportation, storage of milk at the
•• This revolution increased farmer’s income and farmer’s chilling plants. These co-operatives, apart from providing
invested surplus income to increase agriculture productivity. financial help, also provide consultancy. The increase in milk
The phenomenon of Green Revolution in India led to an production has also been termed as Operation Flood.
increase in yields due to improved agronomic technology.
The first phase of green revolution included introduction of Blue Revolution
high-yielding varieties of crops and application of modern This is very similar to green revolution, but the focus here is
agricultural techniques. This led to an increase in production on aquaculture and water preservation. It has been used in
needed to make India self-sufficient in food grains. The India for several years to increase the number of fresh water
production of wheat produced the best results in fuelling self- fishes.
sufficiency of India. Other positive effects of this phenomenon
were reduction in imports of food grains, prosperity of farmers, Evergreen Revolution
development of industries, overall growth of economy and The emphasis in this revolution is on sustainable agriculture by
many more. means of organic and green agriculture. Dr. MS Swaminathan
gave this concept. It includes the use of integrated pest
Second Green Revolution management, nutrient supply and resource management.
Post green revolution, due to the rise in use of chemical
pesticides and fertilizers, there were negative effects on the Yellow Revolution
soil and the land such as land degradation, thereby becoming Yellow Revolution refers to a marked increase in the production
ecologically untenable. Other challenges also became of oilseeds in India. It started in year 1986-87 and within a
apparent. India achieved an increase in production of food decade India became a leading producer of oilseeds by the
grains so much so as to have an overflowing buffer stock, but years 1996-97.
the economic access to food was a huge problem. We had to This revolution can be attributed to governmental support
export excess food which was not consumed by Indians due to especially the setting up of the Technology mission on oilseeds
lack of economic power. in 1986. Factors that played an important role in bringing
To counter these and other such problems, a Second Green yellow Revolution are:
Revolution is visualised and it will largely depend on Indo-US •• Institution support and linkages.
collaboration. •• Provision of extension and training in oilseeds technology.
The Second Green Revolution seeks to minimise post-harvest •• Governments support price policy.
wastage, improve storage and help Indian farmers meet •• Use of hybrid and improved varieties of seeds.
the phyto-sanitary conditions so that they can participate
•• Provision of plant genetic resources.
productively in the global agricultural trade. The first Green
Revolution was assisted by research undertaken by public
institutions such as Universities. However, the Second Green Land Reforms
Revolution (appropriately also called Gene Revolution, Factors like the size of holdings, the pattern of ownership, the
because of the predominance of Biotechnology) will be directed method of inheritance and security of tenure have a definite
by proprietary research and governed by Intellectual Property impact on investment in agriculture. In pre-Independent India
Rights. the following three types of land tenure system existed:
Moreover, the research so far has been confined to crops of •• The Zamindari System It prevailed in Bengal, Bihar,
high value meant solely for the market. No public or private Orissa and North Madras. It was introduced by the East
sector has invested significantly in new genetic technologies for India Company and under it a class of revenue collectors,
the so-called orphan crops such as cowpea, millet, sorghum, called zamindars, was created. These zamindars acted
etc. which are critical for the food supply and livelihoods of as intermediaries between the cultivators and the
the world’s poorest people. The Second Green Revolution on government. Revenue was ‘settled’ by the government
the other hand seeks to focus on such crops and increases their with zamindars, either permanently or temporarily. As
productivity too. long as the zamindar assured the remittance of the settled
revenue to the government he was free to fix and extort
White Revolution any revenue from the tenant. This led to gross abuses and
It occurred in India in 1970, when the National Dairy exploitation and spread of rural poverty in Bengal and
Development Board (NDDB) was established to organise Bihar.
E-50 Agriculture

•• Mahalwari System It was introduced in North India It is the last type of farming which was sought to be encouraged
in terms of which the village community was jointly in the five-year plans. However, no breakthrough could be
responsible for payment of rent. made due to:
•• Ryotwari System It was prevalent in parts of Madras, •• the farmer’s reluctance to alienate his land which is
Bombay province and Assam. Under this system, the usually the only inheritance he has;
cultivator paid the revenues directly to the state without •• unscrupulous elements using the co-operative farming
an intermediary. concept to circumvent land ceiling;
•• lack of political and administrative will.
Purpose
The purpose of land reforms was to achieve the following three Rural Credit and Extension Services
objectives:
•• to increase production by ensuring security of tenure to Three types of loans are provided to Indian farmers to meet
the cultivator; their financial requirements -
•• to increase the purchasing power of the rural population (i) Short term loans
and thereby boost the demand for industrial products; and
These are provided for a period of less than 15 months
•• to achieve social justice along with economic growth
to meet out expenses of routine farming and domestic
through a single set of measures rather than through
consumptions. This type of loan is demanded by farmers
different, and sometimes contradictory, measures.
for purchasing seeds, fertilizers and for meeting family
Major Land Reform Measures Taken after requirements.
Independence (ii) Medium term loans
Several important land reform measures were brought about These are provided for a period of 15 months to 5 years
by the government after Independence, like. to purchase agricultural equipments, animals and for land
•• Abolition of intermediaries like zamindars, jagirdars, improvements.
etc. It resulted led in several states promulgating laws
for putting an end to ‘absentee landlordism’. As a result, (iii) Long term loans
about 30 lakh tenants acquired land ownership over an These are provided for a period of more than 5 years.
area of 62 lakh acres throughout the country. This type of loan is taken by farmers to purchase land and
•• Imposition of ceiling laws It laid down the maximum expensive agricultural equipments and for repayment of
land that can be owned by a land holder (which was old loans.
subsequently amended to ‘holding’ by a family with effect During 2004-05 (up to December 2004) commercial
from 1972). The excess land was to be surrendered to the banks accounted for 61 % of the total institutional credit
government. to agriculture, RRBs for 11 % and co-operatives for 28
•• Consolidation of holding It was introduced as a measure %.
of improving farming efficiency. It made considerable
progress in Punjab, Haryana and Western U.P. However, Co-operative Credit Societies (CCS)
it did not have much effect in the southern and eastern These are the most important source of rural credit. More than
states. 88,000 primary agricultural credit societies provided short-
The land reforms are included in the Ninth Schedule of the term loans of Rs. 34,520 crores during 2000-01.
Constitution, thereby making these laws immune to judicial Co-operatives are organised on a three-tier system. Each state
challenge. However, implementation of these laws requires far has a State Co-operative Bank (SCB) and Central Co-operative
stronger political will than is required in including them in the Banks (CCBs) in each district with the Primary Agricultural
Ninth Schedule. Credit Societies (PACS) as the lower tier. NABARD funding to
PACS is routed only through state co-operative banks.
Co-operative Farming In addition to the above, Land Development Banks (LDBs)
Co-operative farming had been advocated by Mahatma Gandhi provide long-term rural credit for land improvement, soil
as far back as in 1942. Complete benefits of agriculture can be conservation and other investments of a capital nature. LDBs
acquired through co-operative farming. have now been renamed as State Co-operative Agricultural
and Rural Development Banks (SCARDBs). They raise their
Co-operative farming can take the form of:
funds through long-term debentures offering state government
•• Co-operative tenant farming, where the society holds guarantee, and refinancing from NABARD.
the land and leases it to individual members;
•• Co-operative collective farming, where land is National Bank for Agriculture and Rural
irrevocably surrendered to the collective; Development (NABARD)
•• Co-operative joint farming, where the farmers pool NABARD is the apex institution for providing credit facility to
their land and reap the economies of scale, although the agricultural and rural areas. It came into existence on July 12,
ownership continues to remain with the individual farmer. 1982 and took over the functions of the erstwhile Agricultural
Agriculture E-51

Credit Development (ACD), Rural Planning and Credit Cell Government (50%), State Government (15%) and the
(RPCC) of RBI and the Agricultural Refinance Development sponsoring bank (35%). There were 196 RRBs covering
Corporation (ARDC). 427 districts with a total of 14475 branches. With a
NABARD is associated with policy, planning, operation and view towards consolidating and strengthening RRBs, the
even monitoring levels for providing agricultural credit. It has Government of India initiated a process of amalgamation
been jointly set up by the government and the Reserve Bank of RRBs, in a phased manner.
of India.
NAFED
Objectives of NABARD National Agricultural Co-operative Marketing federation of
•• To provide refinance to banks and other credit institution India was established in 1958. It is the highest co-operative
in rural areas. organistaion at the national level. Its major functions are
•• To promote institutional development. procurement, distribution, export and import of selected
•• To evaluate, monitor and inspect client banks. agricultural commodities.
•• To act as a coordinator of rural credit institution.
TRIFED
•• To act as a regulator for cooperative banks and RRBs.
The Tribal Co-operative Marketing Development Federation
•• To maintain close link with RBI.
of India Limited came into existence in 1987. It is a national-
Most of the functions of RBI in the area of agricultural credit
level apex organization functioning under the administrative
have been taken over by NABARD.
control of Ministry of Tribal Affairs, Govt. of India. 
Function of NABARD
NCDC
1. It frames policies and guidelines for rural financial
National Cooperative Development Corporation was set up
institutions.
by an Act of Parliament in 1963 as a statutory Corporation
2. It provides credit facilities to issuing organisation and
under Ministry of Agriculture and Farmers Welfare. It aims to
monitors flow of ground level rural credit.
provide financial assistance to co-operatives for infrastructure
3. It helps RRBs to prepare development action plans. and business development, for their economic upliftment,
4. It provides financial support for the training institutes of along with capacity building interventions.
co-operative banks, commercial banks and RRBs.
5. It undertakes inspection of RRBs, Co-operative banks, Agricultural Insurance
rural development banks and apex credit co-operative
societies. Many crop insurance schemes are available in our country
such as,
Kisan Credit Card
NABARD formulated a model scheme for issue of Kisan
AICIL
Credit Cards (KCCs) to farmers so that they may use them to Agriculture Insurance Company of India Limited was
readily purchase agricultural inputs such as seeds, fertilizers, incorporated in 2002 and is under the administrative control
pesticides, etc. These are operated by commercial banks, of Ministry of Finance, Government of India, and under
RRBs and co-operative banks. the operational supervision of Ministry of Agriculture,
The scheme, introduced in 1998-99, has made rapid progress Government of India. It seeks to provide insurance coverage
with the banking system. The scheme has helped in augmenting and financial support to the farmers in the failure of any of the
the flow of short-term crop loans for seasonal agricultural notified crop, to encourage the farmers to adopt progressive
operations of farmers. farming practices, high value in-puts and higher technology;
From January 31, 2006, the scheme has been extended to all to help stabilize farm incomes, particularly in disaster years. 
types of loan requirements of borrowers of State Co-operative
NAIS
Agriculture Rural Development Banks (SCARDBs). It covers
short-term/medium-term credit and long-term credit for National Agricultural Insurance Scheme is a central sector,
agriculture and allied activities and a reasonable component of government sponsored crop insurance scheme, in operation
consumption credit within the overall limit sanctioned to the since 1999. This scheme aims at tackling the issue of
borrowers. production risk faced by the agricultural sector. It provides
financial support to the  farmers  in the failure of any of the
Commercial Banks crops. The AICIL is the implementing agency of this scheme.
•• The CBs finance rural credit directly through Regional NAIS has been further modified as MNAIS with the aim of
Rural Banks (RRBs). further helping the farm sector.
•• Direct financing is mainly through crop loans and term
loans for equipment and machinery. Agricultural Marketing
•• In 1975, five RRBs were set up to provide direct loans to Increase in agricultural production alone will not bring about
small and marginal farmers, rural artisans and agricultural prosperity for the farmers. It is essential that agricultural
labourers. The RRBs were co-sponsored by Central produce fetches a remunerative price too.
E-52 Agriculture

The Indian system of agricultural marketing suffers from a Agriculture Price Policy
number of defects, as a consequence of which the Indian farmer
is deprived of a fair price for his produce. The following are The main objectives of an agriculture price policy are:
the primary defects in the system of agricultural marketing in •• To ensure that the producer gets a minimum
India. remunerative price.
•• To maintain a reasonable terms of trade between
(i) Inadequate Warehouses agricultural sectors;
Unscientific methods of storage lead to considerable •• To maintain the general price level and protect the non-
wastage. The setting up of Central Warehousing producing consumer from violent fluctuations in price
Corporation and State Warehousing Corporations has of food grains.
improved the situation. During the pre-green revolution period, due to the shortage
in domestic production, the terms of trade were in favour
(ii) Lack of grading and standardisation of the agriculture sector. However, with the surplus of food
Different varieties of agricultural produce are not graded grains in the years following the green revolution, the policy
properly and hence, do not meet official standardisation shifted towards protecting the farmers from the prospect of
requirements fully. non- remunerative prices.
The Commission for Agricultural Costs and Prices (CACP)
(iii) Inadequate transport facilities formerly known as the Agricultural Prices Commission
Most of the roads are not cemented and therefore, are analyzes the input costs and recommends the minimum
unfit for motor vehicles. Thus, the produce has to be support prices for 24 major crops. MSP announced by
the government is that price at which government is ready
carried on slow moving transport vehicles like bullock
to purchase the crop from farmers directly if the crop prices
carts, and this is one of the reasons that farmers sell their
become lower than MSP. As a result, market price of the crop
products to the nearest mandis even if prices offered there never comes down from the level of MSP. The minimum
are fairly low. price security gives incentives to farmers to increase their
production. However, historically, the actual support prices
(iv) Presence of a large number of middlemen are fixed far above the recommended prices due to intense
The chain of middlemen in the agricultural marketing lobbying by the interested parties. Procurement price and
system is so prevalent that the share of farmers has minimum support price are different from each other.
reduced substantially. Procurement price (PP) is that price at which government
purchases the crop after harvesting, while Minimum
(v) Malpractices in unregulated markets Support price (MSP) is the minimum price at which
Even after so many years since the advent of technological government declares it will buy the crop. Since 1968-69, the
changes in the Indian agricultural system, the number of MSP is usually the procurement price.
unregulated markets in the country is substantially large. Apart from fixing support prices, government action in
Agents or brokers, taking advantage of the ignorance and holding the price consists of:
illiteracy of farmers, use unfair means and cheat them. Buffer stocking, which is the practice of holding large
Another malpractice in the mandis relates to the use of stocks by government agencies like Food Corporation of
wrong weights and measures. India (FCI) and releasing the stocks in the market to counter
price rise. The buffer stock also helps to maintain the Public
(vi) Inadequate market information Distribution System (PDS).
Most of the farmers have virtually no contact with the Import of food grains in case shortage is apprehended. There
has not been an occasion to resort to imports since 1996
mandis. As a result, they accept whatever price the
largely because of the vast stock of food grain accumulated
trader offers to them. The government is using radio and
by the FCI.
television media to broadcast market prices regularly so
as to tackle this very problem.
WTO and Agricultural Subsidies
(vii) Inadequate credit facilities The WTO Agreement on Agriculture (AoA), 1995 permitted the
The Indian small and marginal farmers are poor and developed countries to continue to provide farm subsidies, but
lack staying power. They try to sell off the produce under certain restrictions. In WTO terminology, agricultural
immediately after the crop is harvested though prices at subsidies have been segregated into various ‘boxes’:
that time are very low.
Green Box subsidies
Warehousing and marketing facilities are essential to
It includes amounts spent on research, disease control,
ensure that these defects are overcome. The Government
infrastructure and food security. These also include direct
has come out with solutions such as agriculture price payments made to farmers such as income support that do not
policy and maintaining buffer stocks so as to assist stimulate production. These are not considered trade distorting
marketing of agricultural produce. and are encouraged.
Agriculture E-53

Blue Box subsidies reaching the truly needy and as a system for keeping general
consumer prices in check. More than 80 % of grain to the Public
It includes direct payments to farmers to limit production and
Distribution System is provided by the Punjab, Haryana and
certain government assistance to encourage agriculture and
western Uttar Pradesh.
rural development in developing countries.
Blue Box subsidies are seen as being trade distorting. Food Corporation of India (FCI)
Amber Box subsidies The Food Corporation of India (FCI) was established in
1965, as the public sector marketing agency responsible for
It includes all agricultural subsidies that do not fall into either
implementing government price policy through procurement
blue or green boxes. These include government policies of and public distribution operations. It was responsible for
Minimum support Prices (MSP) for agricultural products securing for the government a commanding position in the
or any help directly related to production quantities (e.g. food-grain trade. By 1979, the Corporation was operating in
power, fertiliser, seeds, pesticides, irrigation, etc.). These all states as the sole agent of the central government in food-
are subject to reduction commitment to the de-minimus level grain procurement. The Corporation uses the services of state
of agricultural outputs- to 5% for developed and 10% for government agencies and co-operatives in its operations.
developing countries.
FCI is the sole repository of food-grains reserved for the
India insisted that developed countries should first dismantle Public Distribution System. Food-grains, primarily wheat and
their agricultural subsidy structure before asking developing rice, account for between 60 and 75 % of the Corporation’s
countries to open up their market for farm imports. Our total annual purchases.
emphasis has been on making farm trade fair before it is made
free. TPDS Targeted
Targeted PDS (TPDS) means targeting the PDS to the poor.
Public Distribution System This system started when the procurement and issue prices of
PDS items saw a rise with the ushering in of economic reforms
Objectives of Public Distribution System in the 1990s. Thus, while till 1992 any one could avail of
The objectives of “public Distribution system” are as follows: the PDS, beginning from 1996-97, the TPDS targeted only
•• To protect the low income groups by guaranteeing the poor households across all regions. The below poverty line
supply of certain minimum quantity of food grains at (BPL) households were identified and were provided with
affordable price. food-grains at very low prices. From March 2000, the above
•• To ensure equitable distribution of food grains. poverty line (APL) cardholders had to pay a much increased
•• To control price fluctuation of essential commodities in price. The ‘poorest of the poor’ category was established by
the open market. the ‘Antyodaya’ programme. Those under this category are
entitled to avail rice and wheat at prices lower than those for
Features BPL households.
Features of Public Distribution system are: Points to Remember
1. PDS is a system of distribution of selected essential
After independence Indian Government undertook various
commodities through ‘fair price shops’ which are operated
measures to improve the condition of Agriculture.
by private dealers.
•• The ‘Land Reform’ measures and ‘Green Revolution’
2. Items which are distributed through PDS are rice, wheat,
were the greatest achievements of the Indian
Sugar, edible oil and kerosine
Government in enhancing the agricultural production
3. The purpose of PDS is to offer basic minimum quantity of and productivity.
essential commodities at lowest price to poorer sections of
•• Between 1950 and 1990, there was a substantial
society.
increase in the agricultural productivity. As a result of
4. The required commodities are acquired by the government Green Revolution, India became self sufficient in food
through procurement or import and a buffer stock is production.
maintained. •• Land Reforms resulted in abolition of Zamindari system.
Public Distribution System (PDS) was conceived as a primary •• The proportion of GDP contributed by agriculture
social welfare and poverty alleviation programme of the changed significantly, but not the population depending
government to ensure price stabilisation in the grain market. on it.
Achievements •• Around 65% of the country’s population continued to
be employed in agriculture, even till as late as 1990’s.
Government’s objective of providing reasonable prices for
Agricultural output could have increased with much
basic food commodities like food grains, sugar, edible oil and
less people working in this sector, but industrial and
kerosene is achieved through the Public Distribution System
service sectors required the extra people involved in
of India, the largest PDS of the world. Channelling basic
agriculture. This was regarded as a major failure of
food commodities through the PDS serves as a conduit for
policies followed during the planning periods.
E-54 Agriculture

1. First Agricultural University in India was established in 11. When the Kisan Credit Card schemes for the farmers
(a) Jabalpur (b) Kanpur started in India?
(c) Faizabad (d) Pant Nagar (a) 1995–1996 (b) 1998–1999
2. Which one of the following agencies is not included in (c) 2005–2006 (d) 2007–2008
the operation of the Kisan Credit Cards? 12. Which one of the following apex co-operative organisation
(a) Co-operative Banks at the national level of India?
(b) Regional Rural Banks (a) TRIFED (b) NAFED
(c) Scheduled Commercial Banks (c) FCS (d) RRBs
(d) NABARD 13. Which one of the following co-operative societies is
3. Which one of the following is the objective of the responsible for the production of fertilizers?
(a) NAFED (b) IFFCO
National Watershed Development project for rainfed
(c) NCDC (d) TRI FED
areas of India?
14. Which one of the following states is the first state to
(a) Sustainable farming systems
impose Agriculture Income Tax in India?
(b) Integrated Watershed Management (a) Madhya Pradesh (b) Uttar Pradesh
(c) Reclanation of Forest Area (c) West Bengal (d) Bihar
(d) Both ‘a’ and ‘b’ 15. Which one of the following five year plans has the highest
4. Which one among the following Agricultural sectors is growth rate in Agriculture sector in India ?
covered under the market Intervention scheme (MIS) of (a) Sixth Five Year Plan
the Indian Government? (b) Seventh Five Year Plan
(a) Forestry (b) Irrigation (c) Eighth Five Year Plan
(c) Horticulture (d) Animal Husbandry (d) Ninth Five Year Plan
5. The price at which the Government purchase foodgrains 16. National Seed Policy, 2002 was based on the
for maintaining the public distribution system and for recommendation of
building up buffer stocks are known as (a) Kelkar Committee
(a) Minimum support prices (b) M.V Rao Committee
(b) Procurement prices (c) Y.V Reddy Committee
(c) Issue prices (d) Abhijit Sen Committee
(d) Coiling prices 17. When was the National Food Security Mission (NFSM)
6. Which one of the following agencies assigns the launched in India?
Agricultural Income Tax to states in India? (a) Rabi Season (1999–2000)
(a) Inter – State council (b) Kharif Season (1999–2000)
(b) National Development Council (c) Rabi Season (2007–2008)
(c) Agriculture Finance Corporation (d) Kharif Season (2007–2008)
(d) Finance commission 18. The Bonded Labour System (Abolition) Act was legislated
7. Which one of the following agencies of Indian in
Government implements the price support scheme (PSS)? (a) 1974 (b) 1976
(a) FCI (c) 1977 (d) 1978
(b) NAFED 19. In which of the following years was the Food Corporation
of India (FCI) set up ?
(c) Agriculture pricing agency of India
(a) 1955 (b) 1960
(d) None of the above
(c) 1965 (d) 1970
8. Which one of the following is not included in the
20. Rashtriya Krishi Vikas Yojna was launched in the year?
Agriculture and Allied Services of Indian Economy? (a) 2003 (b) 2004
(a) Fishing (b) Forestry (c) 2006 (d) 2007
(c) Logging (d) Food Processing 21. “Everything else can wait, but not agriculture”. The
9. In which five year plan in Indian Economy, the targets statement is attributed to
for the crop function were not fixed for the first time- (a) Jagjivan Ram (b) J.L. Nehru
(a) Seventh five year plan (c) Mahatma Gandhi (d) Sardar Patel
(b) Eighth five year plan 22. The head office of the National Bank for Agriculture and
(c) Ninth five year plan Rural Development (NABARD) located in
(d) Tenth five year plan (a) Lucknow (b) Hyderabad
10. The importance of agriculture in Indian Economy is (c) New Delhi (d) Mumbai
indicated by its contribution to which of the following? 23. Which is not a source of Agriculture finance in India?
(a) National income and employment (a) Co-operative societies
(b) Industrial development and international trade (b) Commercial Banks
(c) Supply of foodgrains (c) Regional Rural Banks
(d) All of the above (d) None of these
Agriculture E-55

24. Which one of the following methods of soil conservation 32. The central nodal agency for implementing the price
is most effective in arid areas? support operations for commercial crops is:
(a) Mulching (b) Shelter belt (a) NAFED (b) NABARD
(c) Gully plugging (d) Terracing (c) TRIFED (d) FCI
25. NABARD was established in the 33. Which among the following has been discontinued after
(a) Fourth Five Year Plan the Cabinet decision in small saving schemes?
(b) Fifth Five Year Plan (a) National Saving Certificates (NSC)
(c) Sixth Five Year Plan (b) Kisan Vikas Patras (KVP)
(d) Eighth Five Year Plan (c) Monthly Income Scheme (MIS)
26. In India, which of the following has the highest share (d) Recurrent Deposit Schemes
in the disbursement of credit to agriculture and allied 34. Which of the following is an apex financing agency for
activities? the institutions providing investment and production
(a) Commercial Banks credit for promoting the various developmental activities
(b) Co-operative Banks in rural areas ?
(a) RBI (b) NABARD
(c) Regional Rural Banks
(c) SIDBI (d) RRB
(d) Microfinance Institutions
35. In 1982, the Rural Planning and Credit Cell (RPCC)
27. An objective of the National Food Security Mission is to
of Reserve Bank of India was merged to which of the
increase the production of certain crops which are
following banks?
(a) Rice and wheat only (a) SIDBI (b) NABARD
(b) Rice, wheat and pulses only (c) IDBI (d) RRB
(c) Rice, wheat, pulses and oil seeds only 36. Which among the following states is the highest producer
(d) Rice, wheat, pulses, oil seeds and vegetables of Soya bean in India?
28. What will be the ultimate impact on the poor and landless (a) Uttar Pradesh (b) Andhra Pradesh
farmers if all of a sudden, government decides to raise the (c) Rajasthan (d) Madhya Pradesh
Minimum Support Prices of rice & foodgrains inexorably 37. Which among the following is largest producer of wheat
or excessively? in India?
(a) The poor farmers will become rich (a) Punjab (b) Uttar Pradesh
(b) The markets will crash (c) Rajasthan (d) Madhya Pradesh
(c) The poor farmers will get poorer 38. The Rural Infrastructure Development Fund (RIDF),
(d) This will encourage the exports of rice and food which is used by the state governments to develop
grains infrastructure in rural areas, is managed by which among
29. Which among the following is the largest food item the following?
imported by India? (a) IDBI (b) RBI
(a) Vegetable Oils (c) NABARD (d) Rural Ministry of India
(b) Food Grains 39. Which among the following commodity come under the
(c) Milk and Dairy Products preview of Open Market Sale Scheme of the Government
(d) Meat and Poultry products of India to maintain a minimum buffer stock for meeting
30. What is the main motive of the government behind having the unforeseen exigencies like drought, flood and other
a dual price system & setting up of fair price shops? natural calamities and also for providing food grains
(a) To demote speculation and hoarding required for Public Distribution System and the other food
(b) To incentivise the trading of essential commodities grain based welfare programmes of the Government?
(c) To eliminate the monopoly of the traders and (a) Wheat only
speculators (b) Wheat & Rice
(c) Wheat , Rice and Pulses
(d) To make the essential commodities available to the
(d) Wheat , Rice and Oilseeds
weaker sections of the society
40. Which among the following facility has been started in
31. Which among the following is an example of Green Field
India for refinance assistance for small irrigation, IRDP,
Investment? Dairy Development and Mechanism of farms?
(a) Investment made by a real estate company in (a) National Credit Fund
agriculture land to develop it later when the land (b) National Rural Credit Fund
prices increase (c) National Credit Stabilization Fund
(b) Investment made by a company in a new factory (d) Rural Infrastructural Development Fund
complex in a remote land of the country where there 41. Which programme targets integrated farming, on- farm
was no facilities water management, storage marketing and value addition
(c) Investment made by a company to clean up a of farm produce in order to enhance farmers’ income in
cement factory located in populated area because of rainfed areas?
its pollution and using it for a commercial office (a) Integrated Scheme for Oilseeds, Pulses, Oil Palm
purpose and Maize
(d) Investment made by a company to clean up a cement (b) National Mission for Sustainable Agriculture
factory located in populated area because of its (c) Mission for Integrated Development in Horticulture
pollution and using it for a residential purpose (d) Rainfed Area Development Programme
E-56 Agriculture

1. It seeks to minimise post-harvest wastage


Statement Based MCQ 2. It will focus on improved storage
3. It will help the Indian farmers meet phyto-sanitary
1. Index ‘Residex’ is associated with- conditions
1. Share Prices 2. Mutual Fund Prices 4. It will equip the Indian farmers to participate more
3. Price Inflation Index 4. Land Prices fully in global agricultural trade
Choose the right option Which of these statement/s is/are correct?
(a) 1 only (b) 1 and 2
(a) 1, 2, 3 and 4 (b) 1, 3 and 4
(c) 3 only (d) 4 only
(c) 4 only (d) 2, 3 and 4
2. Green Revolution has led to:
8. With reference to India’s food policy, consider the
1. Spurt in production of all food grains.
following statements:
2. Greater regional inequalities
1. It seeks to avert famines by maintaining a buffer
3. reduction in inter-personal inequalities.
stock
4. Increase in production of wheat.
2. It ensures remunerative price for the farmers so that
(a) 1 and 3 (b) 1, 2 and 3
they do not switch crops
(c) 2, 3 and 4 (d) 2 and 4
3. It encourages surplus production for the export
3. Increase in agricultural production is due to:
market
1. increase in cultivable area.
2. improvement in yield 4. It aims to maintain general price line.
3. better cropping pattern Which of these statements are correct?
4. implementation of land reforms: (a) 1, 2, 3 and 4 (b) 1 and 4
(a) 1 and 3 (b) l and 2 (c) 1, 2 and 4 (d) 1 and 2
(c) 1, 2 and 3 (d) 1, 2, 3 and 4 9. The purpose of the tenancy reform legislation was to:
4. Consider the following statements 1. afford security of tenure to tenants
1. Food for Work Programme was launched in India 2. enact fixation of air rent
during the 10th Five Year Plan. 3. conferment of ownership right on tenants in some
2. The Planning commission in India is a constitutional cases
body. (a) 1 only (b) 2 only
Which of the statements given above is/are correct? (c) 1 and 2 (d) 1, 2 and 3
(a) Only 1 (b) Only 2 10. Which of the following are features of Indian agriculture?
(c) Both 1 and 2 (d) Neither 1 nor 2 1. Disguised unemployment
5. The consequences of colonialism on Indian agriculture 2. High productivity
were: 3. Majority of large farmers
1. fragmentation of holdings 4. Dependent on monsoon
2. impoverishment of the peasantry (a) 1 and 2 (b) 2 and 3
3. eviction of land owners for non-payment of dues (c) 2 and 4 (d) 1 and 4
4. better farming practices 11. Indian poverty is said to be predominantly rural. Yet
5. Commercialization of agriculture the problem of urban poverty appears to have reached
(a) 1, 2 and 4 (b) 1, 2, 3 and 5 unmanageable proportions. The paradox is explained by:
(c) 2, 3 and 4 (d) 2, 3 and 5 1. greater visibility of urban poverty
6. With reference to Indian agriculture, consider the
2. migration of rural population to urban areas
following statements:
3. seasonality of agricultural operations
1. Agriculture provides direct livelihood to 59% of the
4. the cycle of drought and floods
labour force in India.
(a) 1 only (b) 1 and 2
2. It contributes more than 20% to the GDP
3. It accounts for about 10% of the total value of (c) 1 and 4 (d) All of these
India’s commodity exports 12. Which of the following statements with regard to Indian
4. Almost 80% of the area under agriculture is Agriculture is/are true?
irrigated. 1. Although rice is the most important cereal being
Which of the above statement/s is/are correct? cultivated in India in terms of total production, wheat
(a) 1, 2, 3 and 4 (b) 1, 2 and 3 is fast catching up.
(c) 3 only (d) 4 only 2. Green revolution did not make an impact on the
7. Consider the following statements regarding the production of pulses.
objectives of the Second Green Revolution announced in (a) 2 only (b) 1 only
2005: (c) 1 and 2 (d) Neither 1 nor 2
Agriculture E-57

13. Consider the following statements: (a) 1, 2 and 3 (b) 1, 2 and 4


1. Regarding the procurement of foodgrains, (c) 1, 3 and 4 (d) 2, 3 and 4
Government of India follows a procurement target 18. Consider the following statements ;
rather than an open-ended procurement policy. 1. The Union Government fixes the Statutory Minimum
2. Government of India announces minimum support Price of sugarcane for each sugar season
prices only for cereals 2. Sugar and sugarcane are essential commodities
3. For distribution under Targeted Public Distribution under the Essential Commodities Act.
System (TPDS), wheat and rice are issued by the Which of the statements .given above is/are correct?
Government of India at uniform Central issue prices (a) 1 only (b) 2 only
to the States/Union Territories. (c) Both 1 and 2 (d) Neither 1 nor 2
Which of the statements given below above is/are correct? 19. Increase in wheat production depends on
(a) 1 and 2 (b) 2 only 1. Area expansion
(c) 1 and 3 (d) 3 only 2. Developing marketing infrastructure
14. Consider the following statements. 3. Better inputs
1. The loans disbursed to farmers under Kisan Credit (a) 1 only (b) 1 and 2
Card Scheme are covered under Rashtiya Krishi (c) 1, 2 and 3 (d) 2 and 3
Bema Yojna of Life Insurance Corporation of India 20. Which statements regarding wheat production is/ are
2. The Kisan Credit Card holders are provided personal incorrect?
accident insurance of ` 50,000 for accidental death 1. Maximum area under wheat is in Uttar Pradesh
and ` 25,000 for permanent disability. 2. Maximum production is in Uttar Pradesh
Which of the statements given above is/are correct? 3. Highest productivity is in Haryana
(a) 1 only (b) 2 only (a) 1 and 3 (b) 2 only
(c) Both 1 and 2 (d) Neither 1 nor 2 (c) 1 and 2 (d) 3 only
15. Consider the following statements: 21. Which of the fallowing are responsible for the decrease
India continues to be dependent on imports to meet the of per capita holding of cultivated land in India?
requirement of oilseeds in the country because 1. Low per capita income.
1. farmers prefer to grow foodgrains with highly
2. Rapid rate of increase of population.
remunerative support prices.
3. Practice of dividing land equally among the heirs.
2. most of the cultivation of oilseed crops continues to
4. Use of traditional techniques of ploughing.
be dependent on rainfall
Select the correct answer using the codes given below
3. oils from the seeds to tree origin and rice bran have
(a) 1 and 2 (b) 2 and 3
remained unexploited
(c) 1 and 4 (d) 2,3 and 4
4. it is far cheaper to import oilseeds than to cultivate
22. Which among the following statements is/are true with
the oilseed crops
regard to WTO membership?
Which of the statements given above are correct?
1. All WTO members automatically receive the ‘most
(a) 1 and 2 (b) 1, 2 and 3
favoured nation’ status.
(c) 3 and 4 (d) 1, 2, 3 and 4
2. Over 75% of WTO members are from developing
16. In India, rural incomes are generally lower than the
urban incomes, which of the following reasons account countries, WTO membership allows them access to
for this? developed markets at the lower tariff.
1. A large number of farmers are illiterate and know Select the correct answer using the codes given below.
little about scientific agriculture (a) 1 only (b) 2 only
2. Prices of primary products are lower than those of (c) Both 1 and 2 (d) Neither 1 nor 2
manufactured products 23. Ministry of Agriculture has decided that despite adequate
3. Investment in agriculture has been low when stocks of wheat and rice, India will not export these
compared to investment in industry. commodities. What are the reason(s) for this decision ?
Codes: 1. Indian variety of wheat and rice are in limited
(a) 1, 2 and 3 (b) 1 and 2 demand abroad.
(c) 1 and 3 (d) 2 and 3 2. National Food Security Bill provides 25 kg of wheat
17. Which of the following are the objectives of the or rice to all BPL families. Huge stock of grains will
Commission for Agricultural Costs and Prices (CACP)? be needed for this initiative.
1. To stabilize agricultural prices 3. As nations recover from global recession, a formula is
2. To ensure meaningful real income levels to the being worked out, where imports and exports will be
farmers conducted by exchange of surplus commodities with
3. To protect the interest of the consumers by providing the trading nations. India will use its food stocks for
essential agricultural commodities at reasonable such trade.
rates through public distribution system. (a) 1 only (b) 2 only
4. To ensure maximum price for the farmer (c) 3 only (d) All 1, 2 and 3
E-58 Agriculture

24. As a policy to boost the agricultural sector, the GOI has Which of the statements given above is/are correct?
taken special measures over time. Which of the following (a) 1 only (b) 2 only
are not a measure with a direct impact on the agricultural (c) Both 1 and 2 (d) Neither 1 nor 2
sector? 30. Consider the following statements about the National
1. Setting up of a National Food Processing Bank Cooperative Marketing Federation of India Limited
2. Opening irrigation, sanitation and water projects for (NAFED).
Private Participation. 1. NAFED promotes the inter-state trade as well as
3. Efforts to reduce fiscal deficit to 5.5 per cent export of certain agricultural commodities to many
level of GDP foreign countries.
(a) 1 only (b) 2 only 2. NAFED is responsible for the movement of the
(c) 3 only (d) 1 and 2 essential commodities from surplus areas to scarcity
25. Consider the following statements areas.
1. The recent decision of Government of India to Which of the statements given above is/are correct?
partially decontrol the sugar industry gives the (a) 1 only (b) 2 only
millers the freedom to sell sugar in open market (c) Both 1 and 2 (d) Neither 1 nor 2
and removes their obligation to supply sugar at 31. Consider the following statements about the Agriculture
subsidised rates to ration shops. Income Tax
1. Agriculture Income Tax is levied and collected by
2. C. Rangarajan panel also suggested decontrolling of
the Union Government.
sugar industry in India.
2. Agriculture Income Tax is levied throughout the
Which of the statements given above is/ are correct?
country in India.
(a) Only 1 (b) Only 2
Which of the statements given above is/are correct?
(c) Both 1 and 2 (d) Neither 1 nor 2
(a) 1 only (b) 2 only
26. Consider the following statements :
(c) Both 1 and 2 (d) Neither 1 nor 2
1. National Agricultural Insurance Scheme (NAIS) was
32. Consider the following statements :
launched in the Sixth Five Year Plan of the Indian 1. In India, more than 85% agriculture workers are
Economy. mainly casual labourers.
2. The growth rate of the primary sector of the Indian 2. Agriculture labourers in India are mainly dominated
Economy sector never reached 8% during the by the backward classes.
planning history of Indian economy. Which of the statements given above is/are correct?
Which of the statements given above is/are correct? (a) 1 only (b) 2 only
(a) 1 only (b) 2 only (c) Both 1 and 2 (d) Netiher 1 nor 2
(c) Both 1 and 2 (d) Neither 1 nor 2 33. Consider the following statements :
27. With reference to the agriculture sector of India, consider 1. Regional Rural Banks grant direct loans and
the following statements advances to marginal farmers and rural artisans.
1. Rural infrastructure Development Fund disperses 2. NABARD is responsible for laying down policies
loans to the states to complete the minor irrigation and to oversee the operations of the RRBs.
projects. Which of the statements given above is/are correct?
2. Rural infrastructure Development Fund is managed (a) 1 only (b) 2 only
by the public sector Commercial Banks. (c) Both 1 and 2 (d) Neither 1 nor 2
Which of the statements given above is/are correct? 34. Consider the following statements :
(a) 1 only (b) 2 only 1. National project for Cattle and Buffalo Breeding
(c) Both 1 and 2 (d) Neither 1 nor 2 (NPCBB) was started for genetic improvement.
28. Consider the following statements : 2. National Project for Cattle and Buffalo Breeding
1. Central warehousing corporation is responsible for (NPCBB) was launched in 2000.
the construction and running of the godowns for Which of the statements given above is/are correct?
storage of agriculture products. (a) 1 only (b) 2 only
2. Central warehousing corporation of India was (c) Both 1 and 2 (d) Neither 1 nor 2
established in 1957. 35. Consider the following statements about National
Which of the statements given above is/are correct? Watershed Development project for rainfed areas
(a) 1 only (b) 2 only launched in India?
(c) Both 1 and 2 (d) Neither 1 nor 2 1. National Watershed Development Project for rainfed
29. Consider the following statements : areas was launched in 1990-91.
1. Agriculture including allied activities accounted for 2. National Watershed Development Project for rainfed
about 55% employment in the country according to areas was launched throughout the country.
census 2011. Which of the statements given above is/are correct?
2. The growth in agriculture and allied sectors is a (a) 1 only (b) 2 only
necessary condition for inclusive growth in India. (c) Both 1 and 2 (d) Neither 1 nor 2
Agriculture E-59

36. Consider the following statements : 1. Paper


1. In the Eleventh Five Year Plan, the average 2. Hank Yarn
foodgrains production was more than 200 million 3. Fertilizers
tonnes. 4. Petroleum and Petroleum Products
2. The main reason for the increase in the production of 5. Cotton and Jute seeds
the coarse cereals in 2000s is mainly due to increase (a) 1, 2 and 3 (b) 1, 3 and 4
in demand of coarse cereals as a nutri-food. (c) 3, 4 and 5 (d) 2, 3, 4 and 5
Which of the statements given above is/are correct? 43. Consider the following statements:
(a) 1 only (b) 2 only
1. 14 major dairying states account for more than 90%
(c) Both 1 and 2 (d) Neither 1 nor 2
of India’s Milk Production
37. Consider the following statements :
1. India is the 4th largest producer of Natural Rubber 2. National Dairy Plan was launched to improve the
in the world. quality of Milch animals in these 14 states
2. India in the world, shares more than 8.0% in the Which among the above statements is / are correct?
Natural Rubber Production. (a) Only 1 is correct
Which of the statements given above is/are correct? (b) Only 2 is correct
(a) 1 only (b) 2 only (c) Both 1 and 2 are correct
(c) Both 1 and 2 (d) Neither 1 nor 2 (d) Neither 1 nor 2 is correct
38. Consider the following statements : 44. Consider the following statements:
1. India is the 6th largest producer of coffee. 1. Among Fruits, Largest Area under cultivation in
2. India’s share in Global Area under coffee is about India is that of Banana
2%. 2. Among Fruits, Largest Production is India is that of
3. India contributes about 4% to world coffee Mango
production as well as in the International trade. Which among the above is / are correct statements?
Which of the statements given above is/are correct? (a) Only 1 is correct
(a) 1 only (b) 2 only
(b) Only 2 is correct
(c) 1 and 2 (d) 1, 2 and 3
(c) Both statements are correct
39. Consider the following statements :
(d) Both statements are incorrect
1. The farm power availability in India is much lower
than that of Korea. 45. Consider the following statements:
2. India is meeting 80% of its urea requirement through 1. Both Coffee and Tea prefer slightly acidic Soil
indigenous production. 2. Both Coffee and Tea show winter dormancy
Which of the statements given above is/are correct? Which among the above statements is / are correct?
(a) 1 only (b) 2 only (a) Only 1 is correct
(c) Both 1 and 2 (d) Neither 1 nor 2 (b) Only 2 is correct
40. Consider the following: (c) Both 1 and 2 are correct
1. Uneconomic & Fragmented Holdings (d) Neither 1 nor 2 is / are correct
2. Inefficient Labor Force 46. Consider the following statements:
3. No use of HYV seeds 1. Most of the wool produced in India is not enough to
Which among the above is/ are the reasons for low fulfill demand of the country
productivity in Indian Agriculture? 2. Most of the wool produced in India is NOT of
(a) Only 1 (b) Only 1 & 2
apparel grade
(c) Only 2 & 3 (d) 1, 2 & 3
Which among the above statements is are correct ?
41. Public Distribution System (PDS) means distribution
(a) Only 1 is correct
of essential commodities to a large number of people
(b) Only 2 is correct
through a network of Fair Price Shops on a recurring
(c) Both 1 and 2 are correct
basis. The Commodities distributed under PDS are:
(d) Neither 1 nor 2 is correct
(1) Wheat (2) Rice
(3) Sugar (4) Pulses 47. Consider the following statements:
(5) Kerosene 1. All kinds of Development Banks in India appeared
Select the correct option: Post-Independence
(a) 1,2 and 4 (b) 1, 2, 3 and 4 2. NABARD is the only agricultural Development
(c) 1, 2, 3 and 5 (d) All of the above Bank in India
42. The government has powers under the Essential Which among the above statements is / are correct?
Commodities Act, 1955 (EC Act) to declare a commodity (a) Only 1 is correct
as an essential commodity to ensure its availability to (b) Only 2 is correct
people at fair price. The Commodities covered under the (c) Both 1 and 2 are correct
Act include: (d) Neither 1 nor 2 is correct
E-60 Agriculture

48. Consider the following statements: 54. In the context of food and nutritional security of India,
1. Price Stabilization Fund Scheme was launched for enhancing the ‘Seed Replacement Rates’ of various crops
Coffee, Tea, Rubber and Tobacco growers in the helps in achieving the food production targets of the
country future. But what is/ are the constraint/ constraints in
2. In this scheme Government procures the commodities its wider/greater implementation?
when the prices fell below a certain level to support 1. There is no National Seeds Policy in place.
the growers 2. There is no participation of private sector seed
Which among the above statements is/ are correct ? companies in the supply of quality seeds of
vegetables and planting materials of horticultural
(a) Only 1 is correct
crops.
(b) Only 2 is correct
3. There is a demand-supply gap regarding quality
(c) Both 1 and 2 are correct
seeds in case of low value and high volume crops.
(d) Neither 1 nor 2 is correct Select the correct answer using the code given below.
49. Consider the following: (a) 1 and 2 (b) 3 only
1. Prevent Black marketing (c) 2 and 3 (d) None of these
2. Trade and earn revenue 55. Which among the below are the products in which India
3. Become the sole controller of food grain supply is 1st in the world?
Which among the above is/ are the primary objective I. Milk
of the government to buy food grains directly from the II. Pulses
farmers? III. Spices
(a) Only 1 (b) Only 1 & 3 IV. Jute
(c) Only 2 & 3 (d) 1, 2 & 3 (a) I, II & III (b) I, II & IV
50. Public Distribution System (PDS) is operated under the (c) II, III &IV (d) All the above
responsibility of the: 56. What are the features of Indian agriculture?
1. Central Government I. Agriculture is the primary occupation in India as
2. State Governments it provides direct livelihood to 70% of its labour
(a) Only 1 (b) Only 2 force.
(c) Both 1 and 2 (d) Neither 1 nor 2 II. Agriculture contributes to about 10% of the total
51. Arrange the following in the descending order of the value of India’s commodity exports.
largest agricultural imports of India: III. Almost 30% of tea and 50% of coffee and jute
produced in the country are exported.
1. Pulses
(a) I,II,III (b) I & II
2. Wood and Wood Products
(c) II & III (d) All the above
3. Edible Vegtable Oils
57. What are long term loans?
(a) 1-2-3 (b) 1-3-2
I. These are provided for a period of less than 15
(c) 2-1-3 (d) 3-2-1 months to meet out expenses of routine farming and
52. Vegetable Oil has been India’s largest non-oil import domestic consumptions
commodity. In this context please consider the following II. These are provided for a period of 15 months to 5
reasons. years to purchase agricultural equipments, animals
1. The Food grain are grown more in India because of and for land improvements.
the remunerative support prices III. These are provided for a period of more than 5
2. The Oil seed cultivation in India is mostly dependent years.
upon the Rain fed Areas (a) Only II (b) Only I
3. The import of Vegetable oil has been cost effective (c) Only III (d) None of the above
Which among the above statements hold correct ? 58. What is National Food Security Mission?
(a) 1 & 2 I. Mission to increase the production of rice, wheat and
(b) 2 & 3 pulses by 10, 8 and 2 million tonnes respectively.
(c) 1 & 3 II. To enhance investment in agriculture and allied
(d) All are correct statements sectors to achieve 4% growth rate.
53. Which among the following gives a justification about III. For promotion of holistic growth of horticulture
sector, including fruits, vegetables, root and tuber
the fact that “ Green Revolution has not been green as it
crops, mushroom, spices, flowers, aromatic plants,
made out to be” ? cashew and cocoa.
1. It confined only to wheat IV. Provides flexibility to states in implementation
2. It confined to selected areas only based on a regionally differentiated approach for
3. It helped only big farmers promoting crop diversification and providing a focus
Choose the correct options: to the programme.
(a) Only 1 (b) 1 & 2 (a) II (b) III
(c) 1, 2 & 3 (d) 2 & 3 (c) IV (d) I
Agriculture E-61

59. What is a Green Box subsidy? growth, infrastructure and assets, and sub-schemes
I. These include direct payments to farmers to limit and flexi-fund.
production and certain government assistance to (a) Only I (b) Only II
encourage agriculture and rural development in (c) All the above (d) None of the above
developing countries 64. Which is correct with respect to second green revolution?
II. These include government policies of Minimum I. It seeks to minimize post-harvest wastage
support Prices (MSP) for agricultural products or II. Improve storage and help Indian farmers meet the
any help directly related to production quantities (for phyto-sanitary conditions so that they can participate
eg. power, fertiliser, seeds, pesticides, irrigation productively in the global agricultural trade.
etc.). III. It is assisted by research undertaken by public
III. These include amounts spent on research, disease institutions such as Universities.
control, infrastructure and food security IV. It is directed by proprietary research and governed
(a) I & III (b) Only II by Intellectual Property Rights.
(c) Only III (d) III & II (a) I & II (b) I, II & III
60. Which statement is correct according to the growth and (c) I, II & IV (d) All the above
productivity of agriculture? 65. Which of the following is correct statement?
I. The total irrigated area increased from less than I. Agriculture is the primary occupation in India as it
one million hectares per annum before the green provides direct livelihood to 59% of its labour force
revolution to about 2.5 million hectares per annum II. In India, 75% of below the poverty line (BPL)
during the 1970’s population lives in rural areas, and is directly or
II. The total gross irrigated area now is 40 million indirectly dependent on agriculture.
hectares.
III. Agriculture contributes to more than 13.7% (2013)
III. The yield per hectare of food grains has shown
of GDP, although this share has progressively
remarkable increase in the pre Green Revolution
declined from 57% in 1950-51.
period.
IV. In developed countries, like the UK and USA, the
(a) Only I (b) I & II
share of agriculture in GDP is only around 2%.
(c) I & III (d) None of the above
(a) I & II (b) I & III
61. Which statement is correct regarding NAIS?
(c) I, II & III (d) All the above
I. It is a central sector, government sponsored crop
66. What are the factors that have a definite impact on
insurance scheme, in operation since 1999.
investment in agriculture?
II. It provides financial support to the farmers in the
I. Size of holdings
failure of any of the crops.
II. The pattern of ownership
III. The AICIL is the implementing agency of this
scheme. III. The method of inheritance and security of tenure
(a) I & II (b) Only II (a) Only I (b) I & II
(c) Only III (d) All the above (c) All the above (d) None of the above
62. Which of the following statement is correct? 67. Which of the following statement is correct according to
I. ISOPOM provides flexibility to states in Zamindari System?
implementation based on a regionally differentiated I. The Zamindari System prevailed in Bengal, Bihar,
approach for promoting crop diversification and Orissa.
providing a focus to the programme. II. It was introduced by the East India Company
II. This scheme is under implementation in the country and under it a class of revenue collectors, called
for increasing area, production and yield of these zamindars, was created.
crops from 2009-10 III. Revenue was ‘settled’ by the government with
III. The pulses component was merged with NFSM zamindars, either permanently or temporarily.
w.e.f. April 2013. IV. Revenue was ‘settled’ by the.
Which is correct? (a) I, II & IV (b) I & II
(a) I & II (b) Only I (c) I & III (d) I, II & III
(c) Only II (d) All the above 68. Which statement is correct regarding Ryotwari system?
63. Which of the following is correct regarding Rashtriya I. System was introduced in North India in terms
Krishi Vikas Yojana? of which the village community was jointly and
I. RKVY was launched in 2007-08 severally responsible for payment of rent.
II. Aim was to incentivize states to enhance investment I. Zamindars acted as intermediaries between the
in agriculture and allied sectors to achieve 4% cultivators and the government.
growth rate. II. Revenue was ‘settled’ by the government with
III. The government has approved continuation of the zamindars, either permanently or temporarily.
RKVY scheme in the 12th plan whereby the funding (a) I & II (b) Only III
will be routed into three components- production (c) Only II (d) None of the above
E-62 Agriculture

69. What were the objectives of Land reforms? may use them to readily purchase agricultural inputs
I. to increase production by ensuring security of tenure such as seeds, fertilizers, pesticides, etc.
to the cultivator; II. These are operated by only commercial banks.
II. to increase the purchasing power of the rural III. The scheme has helped in augmenting the flow
population and thereby boost the demand for of short-term crop loans for seasonal agricultural
industrial products; and operations of farmers.
III. to achieve social justice along with economic growth IV. From January 31, 2006, the scheme has been
through a single set of measures rather than through extended to all types of loan requirements of
different, and sometimes contradictory, measures. borrowers of State Cooperative Agriculture Rural
(a) I & II (b) I, II & III Development Banks (SCARDBs).
(c) I & III (d) None of the above (a) I & II (b) Only IV
70. Which of the following sentences are correct? (c) I, III & IV (d) All the above
I. Second Green Revolution is also called Gene 75. Which of the statement is true?
Revolution. I. Co-operative farming had been advocated by
II. Cowpea and Millets are called Orphan crops. Mahatma Gandhi as far back as in 1942.
III. Second green revolution is directed by proprietary II. The phenomenon of Green Revolution in India led
research. to an increase in yields due to improved agronomic
(a) I & II (b) I, II & III technology.
(c) I & III (d) All the above III. The third Green Revolution is called Gene
71. What is a Blue Box subsidy? Revolution
I. These include direct payments to farmers to limit IV. The first phase of green revolution included
production and certain government assistance to introduction of high-yielding varieties of crops and
encourage agriculture and rural development in application of modern agricultural techniques
developing countries (a) I, II & III (b) I & III
II. These include government policies of Minimum (c) II & III (d) I, II & IV
support Prices (MSP) for agricultural products or any 76. What are short term loans?
help directly related to production quantities (for eg.
I. These are provided for a period of less than 15
power, fertiliser, seeds, pesticides, irrigation etc.).
months to meet out expenses of routine farming
III. These include amounts spent on research, disease
and domestic consumptions.
control, infrastructure and food security
II. These are provided for a period of 15 months to 5
(a) I & III (b) Only I
years to purchase agricultural equipments, animals
(c) Only III (d) III & II
and for land improvements.
72. What is Co-operative tenant farming?
III. These are provided for a period of more than 5
I. Where the society holds the land and leases it to
years.
individual members.
(a) Only II (b) Only I
II. Where land is irrevocably surrendered to the
(c) Only III (d) None of the above
collective.
III. Where the farmers pool their land and reap the 77. Which statement is correct with regard to 2nd Five year
economies of scale, although the ownership plan?
continues to remain with the individual farmer. I. Industrial sector was given more importance in this
(a) I & II (b) Only I plan.
(c) Only III (d) None of the above II. Agricultural Expenditure was only 20% of the actual
73. Which of the statement is correct regarding Land plan expenditure.
Development Banks? III. The agricultural growth, however, was high at
I. It provides long-term rural credit for land 3.15%.
improvement, soil conservation and other (a) Only I (b) Only II
investments of a capital nature. (c) I & II (d) All the above
II. LDBs have now been renamed as State Co-operative 78. What is procurement price?
Agricultural and Rural Development Banks I. It is that price at which government purchases the
(SCARDBs) crop after harvesting, while MSP is the minimum
III. They raise their funds through long-term debentures price at which government declares it will buy the
offering state government guarantee, and refinancing crop.
from NABARD. II. It is that price at which people purchases the crop
(a) I & II (b) II & III after harvesting, while MSP is the maximum price
(c) Only II (d) All the above at which government declares it will buy the crop.
74. Which of the following statements are correct in III. It is the price at which people purchase the product
accordance to Kisan Credit Cards? from the government
I. NABARD formulated a model scheme for issue of (a) Only I (b) II & III
Kisan Credit Cards (KCCs) to farmers so that they (c) Only III (d) None of the above
Agriculture E-63

79. What are Co-operative Credit Societies? 85. What is Co-operative joint farming?
I. They are apex institution for providing credit facility I. Where the society holds the land and leases it to
to agricultural and rural areas. individual members
II. These are the most important source of rural credit. II. Where land is irrevocably surrendered to the
III. They are finance rural credit directly through collective;
Regional Rural Banks (RRBs). III. Where the farmers pool their land and reap the
(a) I & II (b) Only III economies of scale, although the ownership
(c) Only II (d) None of the above continues to remain with the individual farmer.
80. Which of the statement is correct regarding White (a) I & II (b) Only II
revolution? (c) Only III (d) None of the above
I. It occurred in India in 1970. 86. What are the primary defects in the system of agricultural
II. Prof. Verghese Kuerin was the father of White marketing in India?
Revolution in India. I. Inadequate warehouses
III. It works through a network of co-operative II. Lack of grading and standardization
societies which are owned and managed by the milk III. Inadequate transport facilities
producers. IV. Presence of a large number of middlemen
(a) I & II (b) I & III (a) I & II (b) I, II & III
(c) Only I (d) All the above (c) Only IV (d) All the above
81. What is Co-operative collective farming? 87. What are the main objectives of an agriculture price
I. Where the society holds the land and leases it to policy?
individual members. I. To ensure that the producer gets a minimum
II. Where land is irrevocably surrendered to the collective. remunerative price.
III. Where the farmers pool their land and reap the II. To maintain a reasonable terms of trade between
economies of scale, although the ownership agricultural sectors.
continues to remain with the individual farmer. III. To maintain the general price level and protect the
(a) I & II (b) Only II non-producing consumer from violent fluctuations
(c) Only III (d) None of the above in price of food grains.
82. What is NABARD? (a) Only I (b) I & II
I. They are apex institution for providing credit facility (c) All the above (d) None of the above
to agricultural and rural areas. 88. What are medium term loans?
II. These are the most important source of rural credit. I. These are provided for a period of less than 15
III. They provide finance rural credit directly through months to meet out expenses of routine farming and
Regional Rural Banks (RRBs). domestic consumptions
(a) I & II (b) Only I II. These are provided for a period of 15 months to 5
(c) Only II (d) None of the above years to purchase agricultural equipments, animals
83. NABARD took over the function of which all institutes? and for land improvements.
I. Agricultural Credit Development (ACD) III. These are provided for a period of more than 5
II. Rural Planning and Credit Cell (RPCC) of RBI years.
III. Agricultural Refinance Development Corporation (a) Only II (b) Only I
(ARDC) (c) Only III (d) None of the above
IV. State Co-operative Agricultural and Rural 89. What is correct statement with regard to buffer stocking?
Development Banks (SCARDBs). I. It is the practice of holding large stocks by
(a) I, II & III (b) I & II government agencies like Food Corporation of
India (FCI) and releasing the stocks in the market to
(c) Only IV (d) All the above
84. What are the functions of NABARD? counter price rise
I. It is associated with policy, planning, operation and II. The buffer stock also helps to maintain the Public
even monitoring levels for providing agricultural Distribution System (PDS).
credit. (a) Only I (b) Only II
II. Its primary task is to function as refinancing (c) Both I & II (d) None of the above
institution for all types of lending for agricultural 90. What is the WTO Agreement on Agriculture (AoA)?
and rural development. I. It permitted the developed countries to continue to
III. In addition to this, it provides term credit to state provide farm subsidies, but under certain restrictions.
co-operative banks, regional rural banks, land II. It permitted the developing countries to continue to
development banks and state governments (only provide farm subsidies, but under certain restrictions.
for share capital contribution to co-operative credit III. It does not permit the developed countries to
societies). continue to provide farm subsidies.
(a) Only I (b) I & II (a) Only III (b) Only I
(c) II & III (d) All the above (c) Only II (d) None of the above
E-64 Agriculture

91. What is Rashtriya Krishi Vikas Yojana? I. The Food Corporation of India (FCI) was established
I. Mission to increase the production of rice, wheat and in 1975.
pulses by 10, 8 and 2 million tonnes respectively. II. It was responsible for securing for the government
II. To enhance investment in agriculture and allied a commanding position in the food-grain trade.
sectors to achieve 4% growth rate. III. By 1979, the corporation was operating in all states
III. For promotion of holistic growth of horticulture as the sole agent of the central government in food-
sector, including fruits, vegetables, root and tuber grain procurement.
crops, mushroom, spices, flowers, aromatic plants, (a) I & III (b) Only I
(c) I & II (d) II & III
cashew and cocoa.
96. Which statement is correct regarding TPDS?
IV. Provides flexibility to states in implementation
I. It is targeted for rich people.
based on a regionally differentiated approach for II. It is targeted to poor people.
promoting crop diversification and providing a focus III. This system started when the procurement and issue
to the programme. prices of PDS items saw a rise with the ushering in
(a) II (b) III of economic reforms in the 1990s.
(c) IV (d) I (a) I & III (b) Only III
92. What are the land reforms taken after Independence? (c) II & III (d) None of the above
I. Abolition of intermediaries like zamindars, jagirdars, 97. What were the measurements taken by government to
etc., which resulted in several states promulgating improve the condition of agriculture?
laws for putting an end to ‘absentee landlordism’. I. The ‘Land Reform’ measures and ‘Green
II. Imposition of ceiling laws which lay down the Revolution’ were the greatest achievements of the
maximum land that can be owned by a land holder Indian Government in enhancing the agricultural
III. Consolidation of holding was introduced as a production and productivity.
measure of improving farming efficiency II. Between 1950 and 1990, there was a substantial
(a) I & II (b) I & III increase in the agricultural productivity. As a result
(c) Only I (d) All the above of Green Revolution, India became self sufficient in
food production.
93. What is an AmberBox subsidy?
III. Land Reforms resulted in abolition of zamindari
I. There include direct payments to farmers to limit
system.
production and certain government assistance to IV. The proportion of GDP contributed by agriculture
encourage agriculture and rural development in changed significantly, but not the population
developing countries. depending on it.
II. These include government policies of Minimum (a) I & II (b) I, II & IV
support Prices (MSP) for agricultural products or (c) Only III (d) All the above
any help directly related to production quantities (for 98. Which statement is correct regarding Evergreen
eg. power, fertiliser, seeds, pesticides, irrigation Revolution?
etc.). I. The emphasis in this revolution is on sustainable
III. There include amounts spent on research, disease agriculture by means of organic and green
control, infrastructure and food security. agriculture.
(a) I & III (b) Only I II. Dr. MS Swaminathan gave this concept.
(c) Only II (d) III & II III. It includes the use of integrated pest management,
94. Which statement is correct regarding public distribution nutrient supply and resource management.
system? (a) I & II (b) Only II
I. Public Distribution System (PDS) was conceived (c) Only III (d) All the above
99. What is National Horticulture Mission?
as a primary social welfare and poverty alleviation
I. Mission to increase the production of rice, wheat and
programme of the government to ensure price
pulses by 10, 8 and 2 million tonnes respectively.
stabilization in the grain market. II. To enhance investment in agriculture and allied
II. Government’s objective of providing reasonable sectors to achieve 4% growth rate.
prices for basic food commodities like food grains, III. For promotion of holistic growth of horticulture
sugar, edible oil and kerosene is achieved. sector, including fruits, vegetables, root and tuber
III. It is the second largest system in the world. crops, mushroom, spices, flowers, aromatic plants,
IV. More than 50 per cent of grain to the Public cashew and cocoa.
Distribution System is provided by the Punjab, IV. Provides flexibility to states in implementation
Haryana and western Uttar Pradesh. based on a regionally differentiated approach for
(a) I & II (b) Only II promoting crop diversification and providing a focus
(c) Only III (d) III & IV to the programme.
95. Which statement is correct regarding Food Corporation (a) II (b) III
of India? (c) IV (d) I
Agriculture E-65

100. Which statement is correct regarding AICIL? 102. Match the following states with Land Tenure System:
I. It was incorporated in 2002 and is under the States Land Tenure System
administrative control of Ministry of Finance, I. Bengal, Bihar a. Ryotwari System
Government of India, and under the operational II. North India b. Zamindari system
supervision of Ministry of Agriculture, Government III. Madras, Bombay c. Mahalwari system
of India. (a) I – b, II – c, III – a (b) I – a, II – b, III – c
II. It seeks to provide insurance coverage and financial (c) I – c, II – b, III – a (d) None of the above
support to the farmers in the failure of any of the 103. Match the followingtotal institutional credit to agriculture:
notified crop. Banks Percentage
III. It encourages the farmers to adopt progressive I. Commercial bank a. 28%
farming practices, high value in-puts and higher II. RRB’s b. 61%
technology; to help stabilize farm incomes,
III. Co-operative banks c. 11%
particularly in disaster years.
(a) I – b, II – c, III – a (b) I – a, II – b, III – c
(a) I & II (b) II & III
(c) Only III (d) All the above (c) I – c, II – a, III – b (d) I – b, II –a, III – c
101. Match the following 104. In the context of food and nutritional security of India,
List I List II enhancing the ‘Seed Replacement Rates’ of various crops
(Five Year Plan) (Emphasis) helps in achieving the food production targets of the
A. First 1. Food security and women future. But what is/are the constraint/ constraints in its
empowerment wider/greater implementation? [CSAT 2014 - I]
B. Second 2. Heavy industries 1. There is no National Seeds Policy in place.
C. Fifth 3. Agriculture and community 2. There is no participation of private sector seed
development companies in the supply of quality seeds of
D. Ninth 4. Removal of poverty vegetables and planting materials of horticultural
Codes : crops.
A B C D 3. There is a demand-supply gap regarding quality
(a) 1 2 4 3 seeds in case of low value and high volume crops.
(b) 1 4 2 3 Select the correct answer using the code given below.
(c) 3 2 4 1 (a) 1 and 2 (b) 3 only
(d) 3 4 2 1 (c) 2 and 3 (d) None
E-66 Agriculture

EXERCISE-1 10. (d) The importance of agriculture in Indian Economy


1. (d) G. B. Pant University of Agriculture and Technology is indicated by its contribution to national income,
(GBPUA&T, Pant Nagar,Uttara Khand) is the first industrial development and supply of foodgrains. It
agricultural university of India. It was inaugurated contributes around 13.7% to GDP , supplies raw
by Jawahar Lal Nehru on 17 November 1960. material for development of industries along with
2. (d) A Kisan Credit Card is a credit card to provide supplying food grains for livelihood.
affordable credit for farmers. It was started by 11. (b) A Kisan Credit Card is a credit card to provide
the Government of India, Reserve Bank of India affordable credit for farmers. It was started by the
(RBI), and National Bank for Agricultural and Reserve Bank of India (RBI), and National Bank for
Rural Development (NABARD) in 1998–99 to Agricultural and Rural Development (NABARD) in
help farmers access timely and adequate credit. 1998–99 to help farmers access timely and adequate
The credit is available to farmers in most of the credit.
banks like commercial banks cooperative banks 12. (b) The Tribal Cooperative Marketing Development
and regional rural banks. However in day to day Federation of India Limited (TRIFED) started
operations NABARD is not included. functioning in 1988 under the Administrative
3. (d) Sustainable farming system and Integrated watershed control of the Ministry of Tribal Affairs, Govt. of
systems are the objective of the National Watershed India. FCS is food and supplies department. RRB
Development project for rainfed areas of India. The is regional rural bank . All the above three are not
scheme of National Watershed Development Project apex bodies at national level. However National
for Rainfed Areas(NWDPRA) was launched in Agricultural cooperative Marketing Federation of
1990–91 in 25 States and 2 Union Territories based on India Ltd. (NAFED) is one of the Central Nodal
twin concepts of integrated watershed management Agencies and apex body which implements PSS.
and sustainable farming systems. Its objective 13. (b) Indian Farmer Fertilizer cooperative Ltd (IFFCO)
are conservation, development and sustainable is the co-operative society responsible for the
management of natural resources, enhancement of production of fertilizers. Indian Farmers Fertiliser
agricultural production in a sustainable manner, Co-operative Limited (IFFCO) was registered on
restoration of ecological balance and creation of November 3, 1967 as a Multi-unit Co-operative
sustained employment opportunities for the rural Society.
community. 14. (d) Bihar is the first state to impose agricultural income
4. (c) The Market Intervention Scheme (MIS)is an tax in India. Agricultural income tax is levied on
adhoc scheme under which horticultural commodities the income from Agriculture. At present agriculture
and other agricultural commodities falls which are is subjected to two direct taxes and they are
perishable in nature and are not covered under the Agricultural Income tax and Land Tax. They are
minimum price support scheme. levied by the state governments. Bihar was the first
5. (b) The price at which the Government purchases state in India to levy a tax on agricultural income in
foodgrains for maintaining the public distribution 1938.
system and for building up buffer stocks are known 15. (c) The growth performance of the agriculture sector
as procurement prices. has been fluctuating across the plan periods. It
6. (d) Finance commission witnessed a growth rate of 4.8 per cent during the
7. (b) In Order to help the farmers in getting remunerative Eighth plan period (1992–97).
prices for their produce with a view to encourage 16. (b) National seed policy was based on the
higher investment and as also to increase production recommendation of M.V. Rao committee in
and productivity of a commodity, the government 2002. The Policy was meant to “strengthen the
declares Minimum Support Price (MSPs) for 25 seed industry’’ in view of the liberalisation in the
notified agricultural commodities for each Kharif & farm sector under the World Trade Organisation.
Rabi crop season.National Agricultural cooperative The policy seeks to provide intellectual property
Marketing Federation of India Ltd (NAFED) is one protection to stimulate investment in research and
of the Central Nodal Agencies which implements development of new plant varieties.
PSS. 17. (c) The National Development Council (NDC) on
8. (d) Agriculture(crops) and allied sectors are like 29th May, 2007 adopted a resolution to launch a
forestry, logging and fishing. Food processing is Food Security Mission comprising rice, wheat
not included in the Agriculture and allied services of and pulses to increase the production of rice by 10
Indian Economy. million tons, wheat by 8 million tons and pulses
9. (d) Tenth five year plan by 2 million tons by the end of the Eleventh Plan
Agriculture E-67

(2011–12). Accordingly, a centrally sponsored plan by a special act by the parliament and its main
Scheme, ‘National Food Security Mission’, had focus was to uplift rural India by increasing the
been launched from the rabi season 2007–08 to credit flow for elevation of agriculture & rural non
operationalize the above mentioned resolution. farm sector.
18. (b) The Bonded Labour System (Abolition) Act, 1976 26. (a) In India, commercial banks have the highest share in
was enacted to provide for the abolition of bonded the disbursement of credit to agriculture and allied
labour system with a view to prevent the economic activities.The commercial banks disburse around
and physical exploitation of the weaker sections of 60% credit followed by cooperative banks around
the society. It freed all the bonded labourers from 30% and RRB and others.
bondage with simultaneous liquidation of their debts. 27. (b) ‘National Food Security Mission’, had been
It made the practice of bondage a cognisable offence launched from the rabi season 2007–08 to increase
punishable by law. The Act is being administered the production of rice by 10 million tons, wheat by
and implemented by the Ministry of Labour and the 8 million tons pulse by 2 million tons decided by
State governments concerned. National Development Council.
19. (c) The Food corporation of India (FCI) was set up 28. (c) 29. (a) 30. (d) 31. (b) 32. (a) 33. (b)
in 1965. Its objectives are Effective price support 34. (b) 35. (b) 36. (d) 37. (b) 38. (c) 39. (b)
operations for safeguarding the interests of the 40. (c)
farmers. Distribution of food grains throughout 41. (d) RADP was launched by the government as a pilot
the country for public distribution system and scheme under RKVY, focusing on small and
maintaining satisfactory level of operational and marginal farmers and farming systems. It targets
buffer stocks of foodgrains to ensure National Food integrated farming, on- farm water management,
Security. storage marketing and value addition of farm
20. (d) Rashtriya Krishi Vikas Yojana was launched in produce in order to enhance farmers’ income in
August 2007 as a part of the 11th Five Year Plan rainfed areas.
by the Government of India. Launched under the EXERCISE-2
aegis of the National Development Council, it seeks
1. (d) 2. (d) 3. (c)
to achieve 4% annual growth in agriculture through
4. (a) According to The National Food for Work Programme,
development of Agriculture and its allied sectors
during the period under the 11th Five Year Plan food subsidy should be better targeted through targeted
(2007–11). public distribution system and specific programmes for
21. (b) The statement, “Everything else can wait, but not the poor like Food for Work Programme. The National
agriculture” is attributed to J.L. Nehru. Nehru Food for Work Programme was launched on 14
mentioned these words after independence . He said November 2004 in 150 of the most backward districts
this in the context of the Bengal Famine of 1942–43 of India with the objective of generating supplementary
and the acute food scarcity prevailing in the country wage employment. The Planning commission is not a
in 1947. constitutional body.
22. (d) The head office of the National Bank for agriculture 5. (d) 6. (b) 7. (a) 8. (c) 9. (d) 10. (d)
and Rural Development (NABARD) is located in 11. (b) 12. (c) 13. (d) 14. (b) 15. (d) 16. (a)
Mumbai. It helps farmers access timely and adequate 17. (a) 18. (c) 19. (d) 20. (d)
credit. 21. (b) Rapid rate of increase of population and practice of
23. (d) Co-operative societies, Commercial Banks and dividing land equally among the heirs are responsible
Regional Rural Banks are source of agriculture for the decrease of per capita holding of cultivated land
finance in India. They provide finance under in India.
various schemes run by central government and 22. (b) 23. (b) 24. (c)
state governments to purchase seeds, implements, 25. (c) The decision to partially decontrol the sugar sector
fertilizer, pesticides etc. was taken by the Cabinet Committee on Economic
24. (b) The shelter belt method of soil conservation is most Affairs (CCEA). This gives millers the freedom to
effective in arid areas. A shelterbelt is a barrier of sell in the open market and remove their obligation
trees or shrubs. The term “field shelterbelt” is used
to supply the sweetener at subsidised rates to ration
to distinguish between rows of trees or shrubs on
agricultural fields from those planted in other ways: shops. The Rangarajan Committee has recommended
around farmyards or livestock facilities (farmstead total decontrol of the sugar industry by doing away
shelterbelts). It controls soil erosion and traps the with the levy sugar obligation, release mechanism
moisture which is necessary to grow crop in such and freeing of export-import.
regions. 26. (d) In the year 1999 the National Agricultural Insurance
25. (c) National Bank for Agriculture and Rural Scheme was introduced by replacing comprehensive
Development (NABARD) is an apex development crop Insurance Scheme. NAIS covers all food
bank in India having headquarters based in Mumbai. crops (cereals and pulses), oilseeds, horticultural
It was established on 12 July, 1982 in sixth five year and commercial crops. It covers all farmers, both
E-68 Agriculture

loanees and non-loanees, under the scheme. Growth 35. (a) The scheme of National Watershed Development
rate of primary sector reached above 8% with Project for Rainfed Areas(NWDPRA) was launched
green revolution. Both the given statements are not in 1990-91 in 25 States and 2 Union Territories
correct. based on twin concepts of integrated watershed
27. (a) Rural Infrastructure Development Fund (RIDF) was management and sustainable farming systems.
instituted in NABARD with an announcement in the At present, this scheme is being implemented as
Union Budget 1995-96 with the sole objective of a programme of Centrally Sponsored Scheme of
giving low cost fund support to State governments Macro Management of Agriculture in 28 States and
and state owned corporations for quick completion 2 Union Territories.
of ongoing projects relating to medium and minor 36. (c) In the Eleventh Five Year Plan, the average
irrigation, soil conservation, watershed management foodgrains production was more than 200 million
and other forms of rural infrastructure. tonnes, and during 2000s production of the coarse
28. (c) CWC is a Warehousing Agency in India, established cereals increased mainly due to increase in demand
in 1957. It provides logistics support to the agricultural of coarse cereals as a nutri-food.
sector. CWC is operating 464 Warehouses across 37. (c) India is the fourth largest producer, of natural
the country with a storage capacity of 10.54 million rubber and fifth largest consumer of natural rubber
tonnes providing warehousing services for a wide and synthetic rubber together in the world. India in
range of products ranging from agricultural produce the world, shares more than 8.0% in the Natural
to sophisticated industrial products. Rubber Production.
29. (c) The growth in agriculture and allied sectors is a 38. (d) India is the 6th largest producer of coffee in the
necessary condition for inclusive growth in India. world having a share of around 2% in terms of
Agriculture including allied activities accounted for global area. India contributes about 4% to world
about 55% employment in the country according to coffee production as well as in the International
census 2011. trade.
30. (c) National cooperation marketing federation of India 39. (c) The farm power availability in India is much lower
limited NAFED promotes cooperative marketing, than that of Korea. India is meeting 85% of the urea
promotes the inter-state trade as well as export of fertilizer through indigenous production. However,
certain agricultural commodities to many foreign we are deficient in phosphatic and potassic resources
countries. It is responsible in stabilizing prices of and around 90% is imported.
commodities and thus responsible for the movement 40. (a)
of the essential commodities from surplus areas to 41. (c) The commodities distributed under PDS are as
scarcity areas. follows :- • Wheat • Rice • Sugar • Kerosene
31. (d) Agricultural income tax is levied on the income from 42. (d) There are seven broad categories of essential
Agriculture. At present agriculture is subjected to commodities covered by the Act. These are (1)
two direct taxes and they are agricultural Income Drugs; (2) Fertilizer, inorganic, organic or mixed;
tax and Land Tax. They are levied by the state (3) Foodstuffs, including edible oilseeds and oils; (4)
governments. Not all states levy agricultural income Hank yarn made wholly from cotton; (5) Petroleum
tax. and petroleum products; (6) Raw jute and jute
32. (c) Agriculture labourers in India are mainly dominated textile; (7) (i) seeds of food-crops and seeds of fruits
by the backward classes and more than 85% and vegetables; (ii) seeds of cattle fodder; and (iii)
agriculture workers are mainly casual labourers. jute seeds. Recently cotton seed was also included in
33. (c) National Bank for Agriculture and Rural the list.
Development (NABARD) is an apex development 43. (a) 44. (d) 45. (a) 46. (c) 47. (d) 48. (a)
bank in India . It was established on 12 July 1982 in 49. (a) 50. (c) 51. (d) 52. (a) 53. (b) 54. (b)
sixth five year plan and its main focus was to uplift 55. (d) 56. (c)
rural India by increasing the credit flow for elevation 57. (c) These are provided for a period of more than 5 years.
of agriculture and rural non-farm sector and laying This type of loan is taken by farmers to purchase
down policies and to oversee the operations of the land and expensive agricultural equipments and for
RRBs. Moreover Regional Rural Banks grant direct repayment of old loans.
loans and advances to marginal farmers and rural 58. (d) NFSM was launched in 2007-08 to increase the
artisans. So both statements are correct. production of rice, wheat and pulses by 10, 8 and
34. (c) Government of India has initiated a major 2 million tonnes respectively, by the end of 11th
programme ‘National Project for Cattle and Buffalo plan through productivity enhancement and area
Breeding’ (NPCBB) from October 2000. The enhancement, enhancing farm level economy
NPCBB envisages genetic upgradation on priority and creating employment opportunities. This has
basis with a focus on development and conservation been extended to the 12th plan too with additional
production targets.
of important indigenous breeds.
Agriculture E-69

59. (c) Green Box subsidies include amounts spent on 68. (d) Ryotwari System was prevalent in parts of Madras,
research, disease control, infrastructure and food Bombay province and Assam. Under this system,
security. These also include direct payments made the cultivator paid the revenues directly to the state
to farmers such as income support that do not without an intermediary.
stimulate production. These are not considered trade 69. (b) 70. (d)
distorting and are encouraged. 71. (b) Blue Box subsidies include direct payments to
60. (a) The total irrigated area increased from less than farmers to limit production and certain government
one million hectares per annum before the green assistance to encourage agriculture and rural
revolution to about 2.5 million hectares per annum development in developing countries. Blue Box
during the 1970’s. The total gross irrigated area subsidies are seen as being trade distorting.
now is 80 million hectares. 72. (b) In Co-operative tenant farming, the society holds
61. (d) National Agricultural Insurance Scheme is a central the land and leases it to individual members.
sector, government sponsored crop insurance 73. (d) Land Development Banks (LDBs) provide long-
scheme, in operation since 1999. This scheme aims
term rural credit for land improvement, soil
at tackling the issue of production risk faced by the
conservation and other investments of a capital
agricultural sector. It provides financial support to
nature. LDBs have now been renamed as State
the farmers in the failure of any of the crops. The
Co-operative Agricultural and Rural Development
AICIL is the implementing agency of this scheme.
NAIS has been further modified as MNAIS with the Banks (SCARDBs). They raise their funds through
aim of further helping the farm sector. long-term debentures offering state government
62. (b) ISOPOM provides flexibility to states in guarantee, and refinancing from NABARD.
implementation based on a regionally differentiated 74. (c) NABARD formulated a model scheme for issue of
approach for promoting crop diversification and Kisan Credit Cards (KCCs) to farmers so that they
providing a focus to the programme. This scheme is may use them to readily purchase agricultural inputs
under implementation in the country for increasing such as seeds, fertilizers, pesticides, etc. These
area, production and yield of these crops from 2004- are operated by commercial banks, RRBs and co-
05. The pulses component was merged with NFSM operative banks.
w.e.f. April 2010. 75. (d)
63. (c) RKVY was launched in 2007-08 for the 11th plan 76. (b) These are provided for a period of less than 15
to incentivize states to enhance investment in months to meet out expenses of routine farming and
agriculture and allied sectors to achieve 4% growth domestic consumptions.
rate. The government has approved continuation 77. (d) Industrial sector was given more importance in this
of the RKVY scheme in the 12th plan whereby plan, Agricultural Expenditure was only 20% of the
the funding will be routed into three components- actual plan expenditure, The agricultural growth,
production growth, infrastructure and assets, and however, was high at 3.15%.
sub-schemes and flexi-fund. 78. (a) Procurement price is that price at which government
64. (c) The second Green Revolution seeks to minimize purchases the crop after harvesting, while MSP is
post-harvest wastage, improve storage and help the minimum price at which government declares
Indian farmers meet the phyto-sanitary conditions so it will buy the crop. Since 1968-69, the MSP is
that they can participate productively in the global usually the procurement price.
agricultural trade. The first Green Revolution was 79. (c) These are the most important source of rural credit.
assisted by research undertaken by public institutions 80. (d)
such as Universities. However, the second Green 81. (b) Co-operative collective farming, where land is
Revolution (appropriately called Gene Revolution,
irrevocably surrendered to the collective
because of the predominance of Biotechnology) will
82. (b) NABARD is the apex institution for providing
be directed by proprietary research and governed by
credit facility to agricultural and rural areas. It
Intellectual Property Rights.
came into existence on July 12, 1982 and took over
65. (d)
66. (c) Factors like the size of holdings, the pattern of the functions of the erstwhile Agricultural Credit
ownership, the method of inheritance and security Development (ACD), Rural Planning and Credit
of tenure have a definite impact on investment in Cell (RPCC) of RBI and the Agricultural Refinance
agriculture. Development Corporation (ARDC).
67. (d) The Zamindari System prevailed in Bengal, Bihar, 83. (a)
Orissa and North Madras. It was introduced by 84. (d) Its primary task is to function as refinancing
the East India Company and under it a class of institution for all types of lending for agricultural and
revenue collectors, called zamindars, was created. rural development. In addition to this, it provides
These zamindars acted as intermediaries between term credit to state co-operative banks, regional
the cultivators and the government. Revenue was rural banks, land development banks and state
‘settled’ by the government with zamindars, either governments (only for share capital contribution to
permanently or temporarily. co-operative credit societies).
E-70 Agriculture

85. (c) Co-operative joint farming, where the farmers policy through procurement and public distribution
pool their land and reap the economies of scale, operations. It was responsible for securing for the
although the ownership continues to remain with the government a commanding position in the food-grain
individual farmer. trade. By 1979, the corporation was operating in all
86. (d) 87. (c) states as the sole agent of the central government in
88. (a) These are provided for a period of 15 months to 5 food-grain procurement.
years to purchase agricultural equipments, animals 96. (c) Targeted PDS (TPDS) means targeting the PDS to
and for land improvements. the poor. This system started when the procurement
89. (c) Buffer stocking, which is the practice of holding and issue prices of PDS items saw a rise with the
large stocks by government agencies like Food ushering in of economic reforms in the 1990s.
Corporation of India (FCI) and releasing the stocks 97. (d)
in the market to counter price rise. The buffer stock 98. (d) The emphasis in this revolution is on sustainable
also helps to maintain the Public Distribution System agriculture by means of organic and green
(PDS). agriculture. Dr. MS Swaminathan gave this concept.
90. (b) The WTO Agreement on Agriculture (AoA), 1995 It includes the use of integrated pest management,
permitted the developed countries to continue to nutrient supply and resource management.
provide farm subsidies, but under certain restrictions 99. (b) NHM was launched in 2005-06 for promotion of
91. (a) RKVY was launched in 2007-08 for the 11th plan holistic growth of horticulture sector, including
to incentivize states to enhance investment in fruits, vegetables, root and tuber crops, mushroom,
agriculture and allied sectors to achieve 4% growth spices, flowers, aromatic plants, cashew and cocoa.
rate. The government has approved continuation 100. (d) Agriculture Insurance Company of India Limitedwas
of the RKVY scheme in the 12th plan whereby incorporated in 2002 and is under the administrative
the funding will be routed into three components- control of Ministry of Finance, Government of
production growth, infrastructure and assets, and India, and under the operational supervision of
sub-schemes and flexi-fund. Ministry of Agriculture, Government of India. It
92. (d) seeks to provide insurance coverage and financial
93. (c) Amber Box subsidies include all agricultural support to the farmers in the failure of any of the
subsidies that do not fall into either blue or green notified crop.
boxes. These include government policies of 101. (c) 1. First Five Year Plan (1951-56)- Agriculture and
Minimum support Prices (MSP) for agricultural community development
products or any help directly related to production 2. Second Five Year Plan (1956-61)- Heavy Industry
quantities (for eg. power, fertiliser, seeds, pesticides, 3. Fifth Five Year Plan (1974-79)- Removal of
irrigation etc.). Poverty
94. (a) Public Distribution System (PDS) was conceived
4. Ninth Five Year Plan (1997-2002)- Food Security
as a primary social welfare and poverty alleviation
and woman empowerment.
programme of the government to ensure price
102. (a) 103. (a)
stabilization in the grain market. Government’s
objective of providing reasonable prices for basic 104. (b) We have a National Seed policy made in 2002. So 1
food commodities like food grains, sugar, edible is not a constraint. The private sector produces high-
oil and kerosene is achieved through the Public priced seeds but in lower volume. It supplies nearly
Distribution System of India, the largest PDS of the the entire hybrid seeds required for vegetables. So 2
world. is not a constraint. In the case of low value and high
95. (d) The Food Corporation of India (FCI) was established volume crops such as wheat and rice, farmers tend
in 1965, as the public sector marketing agency
to use their own preserved seeds as there is demand
responsible for implementing government price
and supply gap. So 3 is a constraint.
FISCAL AND MONETARY
4
POLICY

Introduction
Fiscal Policy deals with the taxation and expenditure decisions of the goverment covered in the annual budget. Monetary Policy
deals with the supply of money in the economy and the rate of interest. In India, the goverment deals with fiscal policy, while
the Central bank (RBI) is resoponsible for monetary policy.

Fiscal policy (i) a systematic effort to simplify tax structure and tax laws
(ii) a deliberate move to a regime of reasonable direct tax
Fiscal policy or budgetary policy refers to the use by rates and better administration and enforcement.
the government of the various instruments such as
The budget or the annual financial statement of the government
taxation, expenditure and borrowing in order to achieve
gives expression to its fiscal policy. Under Article 112 of the
the objectives of balanced economic development, full
Indian Constitution, the President shall cause to be laid, before
employment or to establish a welfare state. In the context of both the Houses of Parliament a financial statement at the
economic liberalization, the major themes of the fiscal policy commencement of every financial year. The General Budget is
comprises: presented in Lok Sabha by the Minister of Finance.
E-72 Fiscal and Monetary Policy

Union budget or Annual financial statement is a statement (central excise, custom duty, corporation tax) dividends of
of estimated receipts and expenditures of the Government of public sector units (PSU’s) and expenditure of the Government.
India. It has to be placed before Parliament for every financial
Capital Account consists of all capital receipts and expenditure
year, i.e. April 1 to March 31 & being presented at 11 a.m.
such as domestic and foreign loans, loan repayment, foreign, etc.
on the last working day of February. The annual financial
statement gives the following details:
(a) An outline of the results of the last financial year
Expenditure
compared with the previous budget estimates. Total expenditure of the government can be classified into
(b) Government forecasts of receipts and payments for the two categories- Developmental or Non-developmental.
next year. Developmental expenditure includes government spending
(c) Proposed changes in taxes and expenditure allocations. with the aim of creating economic and social infrastructure
For instance, the Government can change the proportion like transport, roads, communication, hospitals, school, etc.
of revenue collected from direct taxes and indirect taxes Non-developmental does not directly contribute to
or it might increase public expenditure on defence and development of economy, for example expenditure for loan
decrease that of social services. repayment, interest payable on internal and external loans,
The budget shows the receipts and payments of the defence expenditure, subsidies, etc. In the Indian budget
Government under three heads: management, this classification is not used.
(i) Consolidated fund In India, the public expenditure is of two types – Plan and Non-plan.
It consists of all revenues and loans received by the
government. Article 114(3) of the Constitution declares Plan Expenditures
that no money can be taken out of Consolidated Fund on Expenditure on central plans such as agriculture, rural
India without the approval of the Lok Sabha. development, irrigation, transport, communications, environment
(ii) Contingency fund and welfare schemes are considered plan expenditure.
It comprises of the sum placed at the disposal of the
President to meet unforeseen expenditure. Non-plan Expenditures
(iii) Public Account Non-plan expenditure is further divided into Revenue
It consists of receipts and payments, which are in the expenditure, which includes interest payments, subsidies,
form of deposit account with the Government, such as defence expenditure and Capital expenditure, which includes
provident funds, small savings, etc. loans to PSUs, states, foreign governments.
There are two parts of budget, i.e. Revenue Account and In short, all asset creating and productive expenditure is
Capital Account. part of plan expenditure, and all non- productive, consumptive
and non- asset building expenditure is part of non- plan
Revenue Account contains all current receipts, such as taxation, expenditure.

Public Expenditure

Non–Plan (Expenditures Plan (Expenditures


not envisaged in envisaged in
5 year plan documents) 5 year plan documents)

Central Assistance
Central Palns for Plans of States
Revenue Capital & UTs.

Police Social Services


Interest Pensions Loans to public
Payments Enterprises Transport & others
Broadcasting Agriculture
Education Technology &
Loans to States Environment
Industries & UTs & Foreign
Agricultural Defence Revenue Governments. Rural Development
Subsidies
Expenditure
Grants to States &,
UTs & Foreign
Governments
Fiscal and Monetary Policy E-73

On the recommendations of the Sukhomoy Chakravarti such, the Kelkar panel in 2012 on fiscal consolidation
Committee, from the financial year 1987-88, Indian budget recommended a series of measures like disinvestment, raising
started being classified as plan and non-plan expenditure, diesel prices, auction of spectrum, pruning some plan schemes
instead of developmental and non-developmental. and rationalising of subsidies.
Primary Deficit = Fiscal Deficit–Interest Payments.
Deficit
India started using this term since 1997-98. Primary deficit is
It means shortage. The gap between Revenue and Expenditure considered a very useful tool in helping bring more transparency
is called Deficit. in the government’s pattern of expenditure. It shows the current
state of government finances. If interest payments are deducted
Types of Deficit
from fiscal deficit, then it will obviously show a lesser deficit
Revenue deficit means the excess of current revenue expenditure for that year as the interest payments are on account of loans
over current revenue receipts. Revenue deficit indicates that taken in the past and not in the present year.
the government cannot meet its current expenditure from its Monetised deficit refers to that part of deficit for which the
current revenue. government borrows from the RBI. To meet the government’s
Revenue Deficit= Revenue expenditure – Revenue receipts such requirements, the RBI prints fresh currency, as a result
Budget deficit is the overall deficit, i.e. the excess of total of which the economy gets monetised. This term was adopted
expenditure over total revenues. It includes both capital and by India in 1997-98.
revenue items in receipts and expenditure. Traditionally, Budget 2015-16
deficit financing in Indian budgets had meant filling this gap.
Budget Deficit= Total expenditure – Total receipts Tax revenue (income and 9,19,842 crores
corporate taxes)
Fiscal deficit is budget deficit plus borrowings and other
liabilities. Previously, when budgetary deficit was the prime Non-tax revenue (customs, 2,21,733 crores
consideration, borrowings from the market and receipts from excise duties and service
national savings, provident funds, etc. were being treated as taxes)
capital receipts. To this extent, government’s actual deficit Total Revenue Receipts 11,41575 crores
was being understood. In other words, fiscal deficit indicates
the total borrowing requirements of the government from all Fiscal Deficit 3.9% (555649 Cr.)
sources, whereas budgetary deficit only indicated government’s Revenue Deficit 2.8% (394472 Cr.)
borrowing from RBI. Primary Deficit 0.7% (99504 Cr.)
Fiscal deficit = Revenue receipts (net tax revenue and
non tax revenue capital receipts (only Non-Planned expenditure 13,12,200 crores
recoveries of loans and other receipts) – Planned expenditure 4,65,277 crores
Total expenditure (Plan and non-plan)
Total Expenditure 17,77,477 crores
or
Fiscal deficit = Budget deficit + Government’s market Taxes
borrowing and liabilities.
Taxes are the main source of government revenues. The pri-
The fiscal deficit situation shows whether the government is
mary purpose of taxation is the mobilisation of resources and
spending beyond its income. India has, unfortunately, been a channelising the same for productive investment. Taxation can
country prone to constant and high fiscal deficit situations. A also be used as a measure to promote equity and reduce dispar-
high fiscal deficit implies high indebtedness of the government ities or to encourage or discourage consumption of particular
and a deficit above 3% in the Indian context means an alarming items. Taxation is in the nature of a compulsory levy and there
situation for the government finances. is no quid pro quo between the amount paid and the services
In India, a high fiscal deficit is mainly due to high revenue provided by the government.
deficit which implies that it is mainly due to the government’s Taxes are broadly divided into two - direct and indirect.
day to day expenditure being more than its day to day income, Taxes like income tax and property tax are direct taxes, while
such that it has to borrow to meet its daily requirements which, excise duty and sales tax are examples of indirect taxes. The
by definition, is unhealthy and dangerous, because borrowing difference between the two is that in the case of direct taxes
is good only if it is for productive purposes. Hence, the FRBM the burden or ‘incidence’ has to be borne by the taxpayers
Act, 2004 laid down that the government’s revenue deficit themselves whereas in the case of an indirect tax, the burden
should be brought down to zero and its fiscal deficit should not can be shifted to another person. For example, a trader who is
be allowed to exceed 3% of the GDP by 2008-09, but this has the assessee in the case of sales tax shifts the burden on to the
still not been achieved. purchasers by recovering the sales tax from them.
A high fiscal deficit is also inflationary because it is mainly Another difference between indirect and direct taxes is that
due to the government’s high non plan expenditure which the rich and the poor will have to pay the same rate of indirect
is unproductive. Besides, a high fiscal deficit imposes huge tax while a direct tax can be made progressive by prescribing
burden by way of repayment of interest and principal. As different rates of tax for different income levels.
E-74 Fiscal and Monetary Policy

Income Corpora Wealth Commodities Minimum Excise Custom Service


Duties VAT GST
Tax the Tax Tax Transaction Tax Alternate Tax Duties Tax

DIRECT INDIRECT

TYPES

TAX

Classification

Proportional Progressive Regressive


Taxation Taxation Taxation
Taxes can also be categorized as progressive, regressive and Corporation tax
proportional taxes.
It is the tax on income (profit) of the companies. In 1996, gover
(1) Progressive Tax introduced Minimum Alternate Tax (MAT) on companies
Progressive tax means rates of tax increase for increasing which escaped the corporation tax net by using the provisions
values or volumes on which the tax is levied. of exemptions, deductions incentives, depreciation and so on.
Income tax is a progressive tax as it has exemptions for
very small incomes, low rates for the first slab of taxable (B) Indirect Taxes
income, and higher rates for the largest incomes.
Central excise duty
Indirect taxes can be progressive if there are exemptions or
low tax rates for goods heavily consumed by the poor, and The commodities which are produced within the country levied
higher rates on luxury items, mostly consumed by the rich. by central excise duty. However, commodities on which state
India has adopted this system for income tax. This is pro- governments impose excise duties (e.g. liquor, drugs) are
poor way of taxation and is popular in the whole world. exempted from the central excise duty. In recent years large
number of goods has come under excise duty. Moreover, the
(2) Regressive tax
rates of these duties have also been increasing.
Regressive tax is one where the proportion of tax paid falls
as income rises. Customs duty
The most regressive tax is a poll tax, levied at a fixed rate
It is the tax imposed on commodities imported into India
per person regardless of income. A tax system can be made
(import duty) or those exported from India (export duty). Since
regressive by having indirect taxes levied at relatively high
imposing duties on exports reduced the competitive position
rates on goods heavily consumed by the poor.
of the country, the government withdrew export duties. In
(3) Proportional Tax recent times; there has been considerable increase in revenue
Proportional tax is one by which the revenue collected from import duties because of heavy imports of iron and steel,
rises proportionally with income. A tax system could be petroleum products, chemicals, etc. In the new requirements
made approximately proportional by having a uniform rate under WTO, import duty will be the only means to curb inputs.
of income tax with very few exemptions, and indirect taxes Custom duty had been the target revenue generator since
levied at similar rates on as many goods and services as independence. But in recent years the excise duty has overtaken
possible. the custom duty as the largest revenue earner.
At some level, progressive and regressive taxes have to
be made proportional, otherwise there will be no limit to Service tax
increase or decrease as the case may be. It is a tax imposed on the person, who avails any specified
service. It was introduced in 1994-95 to address the
Indian Tax Structure asymmetric and distortionary treatment of goods and services
in tax framework and to widen the tax net. The number of
(A) Direct taxes
services liable for taxation was raised from 3 in 1994-95 to 119
Income Tax in 2011-12. However, this concept of a list of services liable
for taxation was changed by the Budget 2012. In this budget,
It is the tax levied directly on the income of the people by the
the government revamped the taxation provisions for services
Central Government.
Fiscal and Monetary Policy E-75

by introducing a new system of taxation of services in India. (vi) A non-vatable additional tax, not exceeding 1% on inter-
In the new system all services, except those specified in the state supply of goods would be levied by the Centre and
negative list, are subject to taxation. retained by the originating state at least for a period of
two years.
Value Added Tax (VAT)
(vii) CGST, SGST, and IGST would be levied at rates to be
It was introduced in France to overcome the cascading effect recommended by the Goods and Services Tax Council
of several taxes-from raw material to the final product in
(GSTC) which will be chaired by the Union Finance
the process of production. In VAT, the tax on all inputs can
Minister and will have Finance Ministers of states as its
be deducted from the excise paid on the output so taxes are
members.
levied on the value added to each stage in the production
process. VAT system of taxation is adopted in more than 150 (viii) GST would apply to all goods and services except alcohol
countries including Australia and Canada. The main advantage for human consumption.
of VAT over any other form of indirect tax is that it shifts (ix) GST on petroleum products would be applicable from a
the tax base towards the point of final consumption from the date to be recommended by the GST Council.
first point of sale. It thus ensures the “tax neutrality” of the
production decisions. VAT is introduced in India in 2005. The (x) Tobacco and tobacco products would be subject to the
basic features of the tax include 2 rates of 4% for comman CST. In addition, the Centre could continue to levy
consumption commodities and inputs and 12.5% for the Central excise duty.
others. Some essential items are exempted and precious metals (xi) A common threshold exemption would apply to both
are taxed at 1%. CGST and SGST Taxpayers with a turnover below it
would be exempt from GST. Compounding option (i.e.to
Goods and Services Tax* pay tax at a flat rate on turnover without credits) would
be available to small taxpayers below a certain threshold.
The introduction of the GST would be a significant step in the
field of indirect tax reforms in India. By subsuming a large However, a taxable person falling within the limit of
number of central and state taxes into a single tax, it would threshold or compounding could opt to pay tax at the
mitigate cascading or double taxation in a major way and pave normal rate in order to be part of the input tax credit
the way for a common national market. From the consumer’s chain.
point of view, the biggest advantage would be in terms of a (xii) The list of exempted goods and services would be kept
reduction in the overall tax burden on goods, which is currently to a minimum and it would be harmonized for the Centre
estimated at 25%- 30%. Introduction of the GST is also and states as far as possible.
expected to make Indian products competitive in domestic and
(xiii) Exports would be zero-rated.
international markets. Studies show that this would instantly
spur economic growth. Because of its transparent character, it (xiv) Credit of CGST paid on inputs may be used only for
is expected that the GST would be easier to administer. paying CGST on the output and the credit of SGST paid
The broad features of the proposed GST model are as follows: on inputs may be used only for paying SGST. In other
(i) GST would be applicable on supply of goods or services words, the two streams of input tax credit (ITC) cannot
as against the present concept of tax on the manufacture be cross utilized, except in specified circumstances of
or on sale of goods or on provision of services. inter-state supplies, for payment of IGST.
(ii) GST would be a destination-based tax as against the Over the past four decades, the value added tax (VAT) has
present concept of origin-based tax. been an important instrument of indirect taxation, with 130
(iii) It would be a dual GST with the Centre and the states countries having adopted it, resulting in one-fifth of the world’s
simultaneously levying it on a common base. The GST tax revenue. Tax reform in many of the developing countries
to be levied by the Centre would be called Central GST has focused on moving to VAT.Federal countries like Canada,
(CGST) and that to be levied by the states would be New Zealand, and Australia have successfully adopted the
called State GST (SGST). GST into their structure. Implementation of a comprehensive
(iv) An integrated GST (1GST) would be levied on inter-state GST in India is expected, ceteris paribus, to lead to efficient
supply (including stock transfers) of goods or services. allocation of factors of production thus bringing about gains in
This would be collected by the Centre so that the credit GDP and exports. This would translate into enhanced economic
chain is not disrupted. welfare and higher returns to the factors of production, viz.
(v) Import of goods or services would be treated as inter- land, labour, and capital. However, in the near term, as GST
state supplies and would be subject to IGST in addition replaces a number of state-level and Central taxes, revenue
to the applicable customs duties. gains may not be significant.
* Source: Department of Revenue and and NCAER 2015-16 Budget Estimate- Gross Tax receipts are estimated
Working Paper No. 103 titled ‘Moving to Goods and to be ` 14,49,490 crore. Devolution to the States is estimated
Services Tax in India-Impact on India’s growth and to be ` 5,23,958 crore. Share of Central Government will be
International Trade’, 2009. j` 9,19,842 crore.
E-76 Fiscal and Monetary Policy

There are three techniques of deficit financing

9% s
om
Cust

ve x
Re on-ta
Borrowings

10 nue
24%

%
N
Creation Of Issue Of New Accumulated
cise New Money Currency Cash Balance
n Ex
Unio es 10%
Duti
Income tax Servi External aid and borrowings
14% ta
l & oth ce Tax
pi er Ta A developing country often resorts to foreign aid if it finds that
a xes
tc % 9%
e b 4 domestic sources are not large enough. But a country going for
-d pt Corporation tax
on ei
N ec 20% foreign aid has to take two precautions:
R
(i) Keeping the borrowing level low so that country does not
fall in a debt trap, and
Share of Govt. Earnings In
(ii) Keeping foreign aid strings-free.
Union Budget 2015-16
External grants and borrowings are different things. External
aids and grants may come free or with very low or even zero
DEFICIT FINANCING interest rates. However, these may come with many terms
The process of bridging the gap between the revenue and and conditions attached which are usually not good for a
expenditure is called deficit financing. In other words, Deficit country’s economy and autonomy. External borrowings means
financing refers to the ways in which the budgetary gap is taking loans from other countries. External borrowing is often
financed. preferred as it brings foreign currency which may help the
government in various ways. It is also preferred over internal
Deficit financing was first done in the USA in 1930s as a tool borrowings because if the government itself starts borrowing
to get out of the effects of the Great Depression. India tried from the banks of the country, there might not be enough left
this in 1969 and it gradually became a routine phenomenon in for other borrowers.
Indian fiscal management.
Objectives of Deficit financing Internal Borrowings
1. It is used as a tool for meeting financial needs of These are not usually preferred because they might hamper the
government, especially in times of war. investment scenario of the public and corporate sector of India.
But, it may be resorted to as and when required.
2. In under-developed countries, deficit-financing has been
considered essential for financing the plans of economic Printing currency
development.
It is usually the last resort for the government in managing its
3. It is used for the mobilisation of surplus, non- utilised and deficit. It might help the government in times of need but it
idle resources in the economy. should be undertaken only in case of extreme necessity as it has
4. It is used as an instrument of economic policy for many damaging effects on the economy. It increases inflation
removing the conditions of depression and to raise the proportionally. It may also lead to a pressure on the government
level of output and employment. for an upward revision in salaries of government employees,
which in turn will lead to an increase of government’s
Public debt
expenditure, further necessitating printing of currency and
Governments in developing countries borrow internally under more inflation. Thus, it may begin a vicious cycle. Moreover,
various attractive schemes of capital accumulation. Public debt it also has the obvious drawback in the fact that it cannot help
has three components: in case of expenditures that are to be done in foreign currency.
(i) Internal debt: It includes market loans from banks and
financial institutions, short-term borrowings on treasury Black Money
bills and other bonds and certificates issued by the
It is unaccounted money which is concealed from tax authorities.
government.
All illegal economic activities are dealt with this Black Money.
(ii) Other internal liabilities: It includes small saving Hawala market has deep roots with this black money. Black
schemes, provident fund, reserve fund of the railways, money creates parallel economy. It puts an adverse pressure
post and telegraph on which the Central Government has on equitable distribution of wealth and income in the economy.
to pay interest.
Some of the reasons for the spread of black money in India
(iii) External debt: It includes loan from foreign countries and are:
international financial institutions like the World Bank,
• The shortage and consequent black marketing during the
IMF, ADB, etc. war years and the troubled days of partition;
Fiscal and Monetary Policy E-77

• The launching of the five-year plans with large expenditure Finance Commission
on projects and the consequent enlargement of bureaucracy;
• The regime of controls over economic activity providing Under Article 280 of the Constitution, the President appoints a
scope for corrupt practices; Finance Commission every five years to determine:
• Heavy taxation and cumbersome procedures which prompt • the distribution of net proceeds of taxes to be shared
the evasion; between Centre and states;
• Rent control and other regressive laws which led to • the principles which should govern the payment by Union
concealment of actual values in real estate transaction; of grants-in-aid to states; and
• Dishonest foreign trading involving under-invoicing of • any other matter concerning financial relations between
exports and over-invoicing of imports; centre and states.
There are numerous deleterious effects of black money. It Finance Commission is a constitutional body. However, its
distorts the real picture of the economic system. It leads to recommendations, called awards, are only advisory in nature
investment of scarce resources in unproductive areas such as and not binding on Central and State governments. After the
real estate, bullion and precious stones. It acts against the poor 73rd and 74th amendments, Finance Commissions are required
and those with fixed incomes. Most importantly, the existence to recommend steps to supplement resources of local bodies also.
of black money distorts the value system of the society and The latest Finance Commission was set up on January 2,
affects the moral fabric of the society in the long run. 2013, under the chairmanship of Dr. Y. V. Reddy, former
There is no one solution for the evils of black money. The RBI Governor. This is the 14th Finance Commission. Other
most obvious remedy is to simplify the tax structure so that members include Prof. Abhijit Sen, Ms. Sushma Nath, Dr. M.
from a feeling of extortion, the taxpayer is made to feel a sense Govinda Rao and Dr. Sudipto Mundle. The recommendations
of participation in nation building. This requires considerable of the commission will apply on the period 2015-20.
scaling down of the rates and easy settlement of pending cases. Finance Commissions Appointed So Far:
Easing of import controls, especially of quantitative types,
Finance Commission Chairman Operational
and allowing liberal import of consumer goods subject to
Duration
payment of duty is another measure. Removal of restrictive
laws like rent control and urban land ceilings, which have only First K.C. Neogy 1952-57
encouraged the generation of black money, will obviate the Second K. Santhanam 1957-62
need for entering into concealed transactions. Third A.K. Chanda 1962-66
The Black Money and Imposition Act, 2015, which came Fourth P.V. Rajamannar 1966-69
into effect from 1 july 2015, lends 90-days compliace Fifth Mahavir Tyagi 1969-74
window. This gives the person having undisclosed foreign Sixth K.Brahmananda 1974-79
assets and income, a chance to come clean by declaring all Reddy
such assets and paying a total of 60% tax and penalty.
Seventh J.M. Shelat 1979-84
General Anti Avoidance Rules (GAAR) Eighth Y.B. Chavan 1984-89
GAAR has been introduced as a very important component of Ninth N.K.P. Salve 1989-95
Direct Tax code with the objective of preventing such deals Tenth Late Shri KV Pant 1995-2000
and transactions that are carried out to evade and avoid paying
Eleventh A.M.Khusro 2000-2005
taxes. In other words, GAAR seeks to prevent such transactions
that are carried out by way of aggressive tax planning so as to Twelfth C. Rangarajan 2005-2010
avoid paying taxes. GAAR has been prompted by practices Thirteenth Dr. Vijay L Kelkar 2010-2015
of ‘round- tripping’ whereby a company operating in India Fourteenth Yega Venugopal 2015-2020
may deliberately incorporate its office in a tax haven country, Reddy
moves its assets there and invests back in India, thereby
avoiding paying tax in India. Monetary Policy
The announcement to implement GAAR from 2012-13 caused
panic among foreign investors and led to massive outflow of Monetary policy refers to the set of measures adopted by the
foreign funds, which led the government to set up Shome Central bank for monetary management.
committee to review GAAR. The Committee recommended, Monetary Policy Objectives
among other things, postponement of GAAR and also • Stability of external value Fluctuation in exchange rate
recommended that it should not be imposed on investments of a currency affects foreign trade and investment. It is,
from Mauritius and Singapore. It also recommended that therefore, important that the rate of exchange is maintained
GAAR should not be imposed if the tax liabilities are less without violent fluctuations.
than 3 crores. It also suggested doing away with the arbitrary
powers given to tax authorities in India. • Maintenance of domestic price level Fluctuation in
prices affects investment decisions. It also leads to
E-78 Fiscal and Monetary Policy

increasing income disparities. However, monetary policy or gold or govt. approved securities before providing credit
alone cannot ensure the maintenance of domestic prices, to the customers. Here by approved securities we mean bond
as several other factors such as erratic monsoons, changes and shares of different companies. Statutory liquid ratio is
in tastes, fluctuation in world prices, etc. affect domestic determined and maintained by RBI in order to control the
prices. expansion of bank credit. It is determined as percentage total
• Reducing the impact of business cycles (slumps and demand and percentage of time liabilities. Time liabilities refer
booms) by manipulation of credit and interest policy. to liabilities, which the commercial banks are liable to pay to
However, economists are not of the same opinion on the customers on demand or pay within one month’s time, due
whether business cycles are primarily caused by monetary to completion of maturity period. The maximum limit of SLR
factors. is 40%. At present, the SLR is 21.5%.
Indian Monetary Policy If any Indian bank fails to maintain the required level of SLR,
Planned economic development adopted by India required an then it becomes liable to pay penalty to RBI. The defaulter
active monetary policy. The two stated aims of this policy bank pays penal interest at the rate of 3% per annum above
were: the bank rate on the shortfall amount for that particular day.
• boost economic development. If the defaulter bank continues to default on the next working
day then the rate of penal interest can be increased to 5% per
• control inflationary pressure.
annum above the bank rate. This restriction is imposed by RBI
The Reserve Bank of India is the nodal agency for on banks to make funds available to customers on demand as
implementing the monetary policy. RBI has defined its
soon as possible. Gold and Government securities (Gilts) are
monetary policy in terms of “adequate financing of economic
included along with cash because they are highly liquid and
growth and at the same time ensuring reasonable price
safe assets.
stability”. The agreement on Monetary Policy framework
between the Government and the RBI dated february 20, 2015 RBI can increase the SLR to contain inflation, suck liquidity
defines the price stability objective explicitly in terms of the from the market, and to safeguard the customers’ money.
target for inflation- as measured by the consumer price index- Open Market Operations
combined (CPI-C) in near to medium term, i.e.
This refers to the RBI buying and selling eligible securities to
(a) below 6% by january 2016, and regulate money supply. Traditionally RBI was not resorting to
(b) 4% for the financial year 2016-17 and all subsequent years. this method. However, after the large inflow of foreign funds
RBI instruments to achieve a stable monetary policy since 1991, RBI has had to step in to sterilize the flow to avoid
includes: excess liquidity.
Bank Rate It is the rate at which RBI rediscounts the bills of Repo and Reverse Repo
exchange. In practice, it is the rate at which RBI lends to other Repo (Repurchase Option) and Reverse Repo are instruments
commercial banks. It thus acts as a signal to the economy on
used by RBI in day-to-day liquidity management. Repo rate is
the direction of the monetary policy. Bank rate had a limited
the rate at which RBI lends to commercial banks and Reverse
impact in the period before economic reforms (1991) when
Repo is the rate at which RBI borrows from commercial
RBI would determine the interest rate structure. However, with
the delegating of this power to the commercial banks (except banks. In inflationary situations, RBI can hike the Reverse
interest rates in priority sectors) the importance of bank rate Repo rate and absorb the excess liquidity in the market.
has been revived. RBI uses changes in Bank Rate to regulate Similarly, in case there is a requirement to increase liquidity
fluctuations in exchange rate and domestic inflation. Now-a- into the system, RBI can reduce the Repo rate which will lead
days the Prime Lending Rate (PLR) is decided by commercial to release of money into the market. RBI occasionally resorts
banks with reference to bank rate and the deposit position of to the Repo rate to fine-tune the liquidity position, without
each bank. At the moment, this rate is 7.75%. resorting to major policy instruments such as changes in
Cash Reserve Ratio (CRR) CRR and Bank Rate. However, markets are bound to react to
frequent changes in the Repo rates and this will be reflected to
Every commercial bank is required to keep a certain percentage
frequent in corresponding changes in the deposit and lending
of its demand and time liabilities (deposits) with the RBI
rates of commercial banks.
(either as cash or book balance). The RBI varies this ratio to
change the liquidity of market. RBI is empowered to fix the l Margin Requirement- refers to difference between the
CRR at a rate ranging between 3% per cent and 15%. Like securities offered and amount borrowed by the banks.
the Bank Rate, CRR is also subject to frequent changes as RBI l Rationing of Credit - RBI controls the credit granted/
intervenes from time to time to correct monetary or exchange allocated by commercial banks.
rate imbalances. This ratio, currently, is 4%. l Moral suasion - Psychological means and informal
Statutory Liquidity Ratio (SLR) means of selective credit control.
Statutory liquidity ratio refers to the amount that the l Direct Action - Refers to the step taken by the RBI
commercial banks require to maintain in the form of cash, against banks don’t fulfil conditions and requirements.
Fiscal and Monetary Policy E-79

CREDIT CONTROL MEASURES

Quantitative Qualitative

Bank Open Market Cash Statutory Margin Rationing Moral Direct


Rate Policy Operations Reserve liquidity Requirements of Credit Suasion Action
Ratio Ratio

Capital Account Convertibility (CAC) While India made the rupee fully convertible under current
account, it was felt that the economy was not yet ready for
A currency is said to be convertible when it can be freely capital account convertibility. The countries which went in
exchanged for another currency at market rates. Transaction for hasty CAC in the 1980s had to face financial crisis. The
of current account includes dealing with payments relating to thinking was that trade and domestic liberalization process
foreign trade, travel and other services. Capital account deals should be completed before CAC can be thought of. Many
with transaction in financial assets. While India has made the economists feel that CAC will only help the drain of Indian
Indian rupee fully convertible on current account, it is yet capital, which will get invested abroad. The present policy is
to accept capital account convertibility as a goal. Broadly, to go slow on CAC.
capital account convertibility would mean freedom for firms The RBI appointed the Committee on Capital Account
and residents to freely buy into overseas assets such as equity, Convertibility (CAC) with S.S Tarapore as chairman. The
bonds, property, and acquire ownership of overseas firms Committee submitted its report in 1997 setting a three-phase
besides free repatriation of proceeds by foreign investors. time-table (1997-98, 1998-99, 1999-2000) to achieve CAC.
India is reluctant to increase the pace capital account Committee, recommended that capital account convertibility
convertibility by taking experience of Mexico. It was one of would be greatly beneficial for India by making available
the largest recipients of foreign private capital ($29 billion in large funds (which would promote economic growth) and
1993). But, with the devaluation of its currency and the collapse improved access to world financial markets, bringing about
of the exchange rate, much of the capital fled the country. This an improvement of the country’s financial system in the
catastrophic collapse was due to capital account convertibility, global context, besides allowing Indians to acquire and hold
which encouraged even Mexican residents to convert their international securities and assets.
capital into dollars, precipitating the crisis. Similar situation Before attaining full convertibility, the CAC recommended,
arose in the late 1997 in South East Asian countries also where India was to fulfill certain important preconditions:
their local currencies were devaluated by 58% in one month.
Fiscal consolidation: Reduction in gross fiscal deficit from
There is a difference in concept between ‘Full Currency 4.5% of GDP in 1997-98 to 3.5 % in 1999-2000.
Convertibility’ and a ‘Fully Open Capital Account’. Currency
convertibility refers to the absence of any restriction on the Mandated inflation rate: The mandated rate of inflation for 3
holding of foreign currencies by residents and of the national years should be an average of 3 to 5 %.
currency by foreigners, and on free conversion between Based on the recommendations of the Committee, the Foreign
currencies. It does not preclude restrictions on the type and Exchange Regulation Act (FERA) was repealed and replaced
quantity of non-currency assets that residents can hold abroad with Foreign Exchange Management Act (FEMA) in June
or foreigners can hold in the country. An Open Capital 2000. FERA was considered a very rigid regulation and
Account, on the other hand, is the absence of restrictions promoted conservation of foreign exchange. FEMA in turn
on non-currency asset holdings, and can exist without free encouraged the formation of a foreign exchange market and
conversion of the currency. facilitated trade.
E-80 Fiscal and Monetary Policy

Exercise - 1
1. The process of budget making after re-evaluating every 10. Fiscal deficit means:
item of expenditure in every financial year is known as- (a) Total receipts minus expenditure
(a) Performance Budgeting (b) Total receipts minus interest payments on external
(b) Deficit Budgeting debt
(c) Zero Based Budgeting (c) Revenue receipts minus ex-penditure
(d) Fresh Budgeting (d) Revenue receipts minus defense expenditure
2. Which of the following is not viewed as a national debt? 11. Value-added tax is
(a) Provident Fund (a) ad valorem tax on domestic final consumption collected
(b) Life Insurance Policies at all stages between production and the point of final
(c) National Saving Certificate sale
(d) Long-term Government Bonds (b) ad valorem tax on final consumption collected at
3. Tarapore Committee submitted its report on “Full manufacturing level
Convertibility on Rupee” in- (c) tax on final consumption collected at the consumption
(a) Current account stage
(b) Capital account (d) special tax levied by states on products from other
(c) Both in current as well as in capital account states
(d) Special Drawing Rights (SDRs) 12. States’ debt does not include:
4. CENVAT is related to- (a) loos from State Bank of India
(a) Sales Tax (b) Excise Duty (b) loans from the Central government
(c) Custom Duty (d) Service Tax (c) provident fund
5. Plan expenditure in India is met by- (d) treasury bills issued to inter-national financial
(a) Internal debt and other resources institutions
(b) Assistance from Aid India Club 13. An instrument of qualitative credit control is
(c) Assistance from IMF (a) Open market operations
(d) Assistance from OECD countries (b) variable reserve ratio
6. High fiscal deficit is cause for concern for any economy. (c) Bank rate
What does it denote? (d) Credit rationing
(a) It is a measure of the borrowing of an economy 14. Budget deficit is:
(b) It is total expenditure less total receipts excluding (a) excess of total expenditure including loans net of
borrowings lending over revenue receipts
(c) It reflects the decrease in tax collections for the year (b) difference between revenue receipts and revenue
(d) it means the lack of liquidity and earnings for the expenditure
economy (c) difference between all receipts and all expenditure
7. With reference to the Finance Commission of India, (d) fiscal deficit less interest payments
which of the following statements is correct ? 15. Current account in the Balance of Payment comprise is
(a) It encourages the inflow of foreign capital for ___?
(a) difference of total exports and imports in one year
infrastructure development
(b) invisible account like tourism, shipping, insurance,
(b) It facilitates the proper distribution of finances
etc.
among the Public Sector Undertakings
(c) transactions like net external assistance, NRI deposits,
(c) It ensures transparency in financial administration
etc.
(d) None of the statements (a), (b) and (c) given above
(d) trade balance plus invisible balance
is correct in his context
16. Fiscal stability means that, other things remaining
8. Which one at the following is responsi­ble for the
constant
preparation and presentation of Union Budget to the
(a) debt-GDP ratio declines over time
Parliament?
(b) both debt and GDP decrease over time
(a) Department of Revenue
(c) debt and GDP increase at the same rate
(b) Department of Economic Affairs
(d) debt increases but GDP remains the same
(c) Department of Financial Services
17. Which is incorrect about convertibility?
(d) Department of Expenditure
(a) Exchange rate should be based on forces of demand
9. In India, the tax proceeds of which one of the following
and supply.
as a percentage of gross tax revenue has significantly
(b) Exchange rate should show the strength of the
declined in the last five years?
economy.
(a) Service tax (b) Personal income tax
(c) Discourage black market transactions.
(c) Excise duty (d) Corporation tax
(d) RBI would become a direct player.
Fiscal and Monetary Policy E-81

18. ____ is not a non-Plan expenditure? 24. The banks are required to maintain a certain ratio between
(a) Interest payment their cash in hand and total assets. This is called:
(b) Subsidies (a) SBR (Statutory Bank Ratio)
(c) Central assistance for states and UT plans (b) SLR (Statutory Liquid Ratio)
(d) Defense expenditure (c) CBR (Central Liquid Reserve)
19. Which one of the following statements regarding the levying, (d) CLR (Central Liquid Reserve)
collecting, and distribution of Income Tax is correct? 25. The sales tax you pay while purchasing a toothpaste is a
(a) The Union levies, collects, and distributes the (a) tax imposed by the Central Government
proceeds of income tax between itself and the states. (b) tax imposed by the Central Government but collected
(b) The Union levies, collects, and keeps all the by the State Government
proceeds of income tax itself. (c) tax imposed by the State Government but collected
(c) The Union levies and collects the tax but all the by the Central Government
proceeds are distributed among the states (d) tax imposed and collected by the State Government
(d) Only the surcharge levied on income tax is shared 26. According to the provisions of the Fiscal Responsibility
between the Union and the states. and Budget Management
20. The diagram shows a relationship between inflation and Act., 2003 and FRBM Rules, 2004, the Government is
unemployment for an economy. under obligation to present three statements before the
parliament along with the Annual Budget. Which one of
the following is not one of them?
(a) Macroeconomic Framework Statement
(b) Fiscal Policy Strategy Statement
(c) Medium-term Fiscal Policy Statement
(d) Statement showing Short term Fiscal Policy
27. As per the Economic Survey 2007-2008, which one
of the following is the largest source of revenue of the
Government of India?
(a) Excise Duty (b) Personal income Tax
Which of the following policies would move the economy (c) Corporation Tax (d) Customs Duties
from A to B? 28. Fiscal Policy in India is formulated by
(a) Reduce interest rate and increase income tax rates (a) the Reserve Bank of India
(b) Reduce interest rates and decrease income tax rates (b) the Planning Commission
(c) Increase interest rates and decrease income tax rates (c) the Finance Ministry
(d) the Securities and Exchange Board of India
(d) Increase interest rates and increase Income tax rates
29. Corporation tax is imposed by
21. If a government budgets for a surplus and there is an
(a) State Government
unexpected increase in the level of economic activity,
(b) Central Government
which of the following is likely to occur?
(c) Local Government
(a) There will be an increase in tax revenues and an (d) State as well as Central Government
increase in the budget surplus 30. Convertibility of rupee implies [CSAT 2015-I]
(b) There will be an increase in tax revenues and a (a) being able to convert rupee notes into gold
decrease in the budget (b) allowing the value of rupee to be fixed by market
(c) There will be a decrease in tax revenues and an forces
increase in the budget surplus (c) freely permitting the conversion of rupee to other
(d) There will be a decrease in tax revenues and a currencies and vice versa
decrease in the budget surplus (d) developing an international market for currencies in
22. In India, deficit financing is used for raising resources for India
(a) economic development 31. When the Reserve Bank of India reduces the Statutory
(b) redemption of public debt Liquidity Ratio by 50 basis points, which of the following
(c) adjusting the balance of payments is likely to happen? [CSAT 2015-I]
(d) reducing the foreign debt (a) India’s GDP growth rate increases drastically
23. Which one of the following is the correct statement? (b) Foreign Institutional Investors may bring more
Service tax is a/an: capital into our country
(a) direct tax levied by the Central Government. (c) Scheduled Commercial Banks may cut their lending
(b) indirect tax levied by the Central Government. rates
(c) direct tax levied by the State Government. (d) It may drastically reduce the liquidity to the banking
(d) indirect tax levied by the State Government. system
E-82 Fiscal and Monetary Policy

Exercise - 2
Statement Based MCQ ten years in consonance with India’s demographic
dividend.
3. Devolution of a specified share of central taxes to
1. After 1947, development and non-development
local bodies as grants.
expenditures have increased, the increase in the former Select the correct answer using the codes given below
being more. Nondevelopment expenditure involves (a) 1 only (b) 2 and 3
1. interest payments 2. subsidies (c) 1 and 3 (d) 1, 2 and 3
3. defence 4. irrigation 7. Which statements about indirect taxes in India are true?
(a) 1 and 2 (b) 1 only 1. Yield from indirect taxes is more than that from direct
(c) 1, 2 and 3 (d) 2, 3 and 4 taxes.
2. Consider the following statements in respect of Financial 2. Indirect taxes have grown faster than direct taxes after
Commission: 1947.
1. It is mandatory to appoint a Finance Commission 3. Indirect taxes are ultimately paid for by persons who
every five years. do not actually pay taxes to the government.
2. Finance Commission lays down the principles 4. Increase in indirect taxes is good in a developing
governing grant-in-aid to states. country.
3. Finance Minister is the ex-officio Chairperson of the (a) 1, 2 and 4 (b) 1 and 2
Finance Commission. (c) 2 only (d) 1, 2 and 3
8. Which are the pre-requisites required by the Indian
4. The award given by the Finance Commission is
economy to implement convertibility of rupee on trade
binding on Central and State governments. account as suggested by the Rangarajan Committee?
Which of the above statements are correct? 1. There should be comfortable foreign exchange resources
(a) 1, 2, 3 and 4 (b) 1, 2 and 4 2. Low rate of inflation.
(c) 1, 2 and 3 (d) 1, 3 and 4 3. Mechanism by which the government can pass on the
3. Increase in net RBI credit for Central Government changes in the price of imported goods to the consumers
represents- 4. SLR and CRR must be low.
1. Budgetary Deficit 2. Revenue Deficit (a) 1 and 2 (b) 1, 2 and 3
3. Fiscal Deficit 4. Monetised Deficit (c) 1 and 4 (d) All of the above
Choose the right option 9. Consider the following statements
(a) 1 only (b) 1 and 2 Full convertibility of the rupee may mean
(c) 3 only (d) 4 only 1. Its free float with the international currencies.
4. Which of the following is the main aim of Indian Monetary 2. Its direct exchange with any other international
Policy? currency at any prescribed place inside and outside
the country.
1. Control inflationary pressure
3. It acts just like any other international currency.
2. Boost economic development Which of these statements are correct?
(a) 1 only (b) 2 only (a) 1 and 2 (b) 1 and 3
(c) 1 and 2 (d) Neither 1 nor 2 (c) 2 and 3 (d) 1, 2 and 3
5. With reference to the Indian Public Finance, consider the 10. Which of the following is/are type of Budget?
following statements. 1. Capital budget 2. Revenue budget
1. External liabilities reported in the Union Budget are (a) 1 only (b) 1 and 2
based on historical exchange rates. (c) 2 only (d) Neither 1 nor 2
2. The continued high borrowing has kept the real 11. Which of the following are among the non-plan expenditures
interest rates high in the economy. of the Government of India?
3. The upward trend in the ratio of Fiscal Deficit to 1. Defence expenditure
GDP in recent years has an adverse effect on private 2. Subsidies
investments. 3. All expenditures linked with the previous plan periods
4. Interest payments is the single largest component 4. Interest payment
of the non-plan revenue expenditure of the Union
Codes:
Government.
(a) 1 and 2 (b) 1 and 3
Which of these statements are correct?
(c) 2 and 4 (d) 1, 2, 3 and 4
(a) 1, 2 and 3 (b) 1 and 4
(c) 2, 3 and 4 (d) 1, 2, 3 and 4 12. Consider the following statements:
6. Which of the following is /are among the noticeable The Indian rupee is fully convertible:
features of the recommendations of the Thirteenth Finance 1. in respect of Current Account of Balance of payment
2. in respect of Capital Account of Balance of payment
Commission?
1. A design for the Goods and Services Tax, and a 3. into gold
compensation package linked to adherence to the Which of these statements is/are correct ?
proposed design. (a) 1 alone (b) 3 alone
2. A design for the creation of lakhs of jobs in the next (c) 1 and 2 (d) 1, 2 and 3
Fiscal and Monetary Policy E-83

13. With reference to Union Budget, which of the following 1. Hawala market has deep roots with this black money
is/are covered under Non-Plan Expenditure? 2. It is unaccounted money which is concealed from tax
1. Defence expenditure authorities
2. Interest payments 3. All legal economic activities are dealt with this Black
3. Salaries and pensions Money
4. Subsidies 4. It puts an adverse pressure on equitable distribution of
Select the correct answer using the code given below. wealth and income in the economy
(a) 1 only (b) 2 and 3 only (a) 1 only (b) 1 and 2
(c) 1, 2, 3 and 4 (d) None (c) 3 only (d) 1, 2 and 4
14. Which of the following are included in the category of 21. Which of the following is the tax imposed on commodities
direct tax in India? imported into India (import duty) or those exported from
1. Corporation tax 2. Tax on income India (export duty)?
3. Wealth tax 4. Customs duty 1. Customs duty 2. Central excise duty
5. Excise duty 3. Incorporate duty
Select the correct answer using the codes given below (a) 1 only (b) 2 only
(a) 1, 2 and 3 (b) 1, 2, 4 and 5 (c) 3 only (d) 2 and 3
(c) 2 and 3 (d) 1, 3, 4 and 5 22. Which among the following statements is incorrect in
15. Which of the following refers to that part of deficit for
regards to Statutory liquid ratio?
which the government borrows from the RBI?
1. Statutory liquid ratio refers to the amount that the
1. Primary deficit 2. Secondary deficit
commercial banks require to maintain in the form
3. Regulatory deficit 4. Monetised deficit
of cash, or gold or govt. approved securities before
(a) 1 only (b) 1 and 2
providing credit to the customers
(c) 3 only (d) 4 only
2. Statutory liquid ratio is determined and maintained by
16. Which of the following statements is/are correct in terms
RBI in order to control the expansion of bank credit
of Direct and Indirect taxes?
1. Excise duty and sales tax are examples of indirect 3. At present, the SLR is 4%
taxes 4. It is determined as percentage total demand and
2. Taxes like income tax and property tax are direct taxes percentage of time liabilities
3. There is no difference between Direct and Indirect tax (a) 1 only (b) 1 and 2
Select the correct answer using the code given below: (c) 3 only (d) None of the above
(a) 1 only (b) 2 only 23. Which of the following statements is/are correct in regards
(c) 1 and 2 (d) 1, 2 and 3 to Revenue budget?
17. Which of the following refers to the freedom for firms and 1. It consists of all capital receipts and expenditure
residents to freely buy into overseas assets? such as domestic and foreign loans, loan repayment,
1. Capital Account Consumption foreign and etc
2. Capital Across Convertibility 2. It consists of all current receipts, such as taxation,
3. Capital Account Continuity dividends of public sector units (PSU’s) and
4. Capital Account Convertibility expenditure of the government
(a) 1 only (b) 1 and 2 (a) 1 only (b) 2 only
(c) 3 only (d) 4 only (c) 1 and 2 (d) Neither 1 nor 2
18. Which of the following refers to the RBI buying and selling 24. With reference to ‘Central excise duty’, which of the
eligible securities to regulate money supply? following statements is/are correct?
1. Repo and Reverse Repo 1. Commodities on which state governments impose
2. Open Market Operations excise duties are exempted from the central excise
3. Response and Reverse Repo duty
4. Relative Market Operations 2. In recent years large number of goods has come under
(a) 1 and 2 (b) 2 only excise duty. Moreover, the rates of these duties have
(c) 3 only (d) 1, 2 and 4 also been increasing
19. Which of the following is/are the major Objectives of 3. Commodities which are produced within the country
Deficit financing? levied by central excise duty
1. Used as an instrument of economic policy (a) 1 only (b) 1 and 2
2. It is used as a tool for meeting financial needs of
(c) 3 only (d) 1, 2 and 3
government
25. Mobilization of resources and channelizing the same for
3. Used for the mobilization of surplus, non- utilized and
idle resources in the economy productive investment is the primary purpose of
(a) 1 only (b) 1 and 2 1. Expenditure 2. VAT
(c) 3 only (d) 1, 2 and 3 3. Taxation 4. CRR
20. Which of the following statements is incorrect in regards (a) 1 only (b) 2 only
to Black money? (c) 3 only (d) 1, 2 and 4
E-84 Fiscal and Monetary Policy

26. Which of the following is the process of bridging the gap 35. Which of the following statements is incorrect about Repo
between the revenue and expenditure? and Reverse Repo?
1. Multiple financing 2. Bridge financing 1. At present, the repo rate is 11.75%
3. Accurate financing 4. Deficit financing 2. At present, the reverse repo rate is 5.75%
(a) 1 only (b) 2 only 3. Repo rate is the rate at which RBI lends to commercial
(c) 3 only (d) 4 only banks
27. In which year was Service tax introduced?
4. Reverse Repo is the rate at which RBI borrows from
1. 1983-84 2. 1994-95
3. 1967-68 4. 2003-2004 commercial banks
(a) 1 only (b) 2 only (a) 1 only (b) 1 and 2
(c) 3 only (d) 4 only (c) 1, 3 and 4 (d) None of the above
28. Which of the following means rates of tax increase for 36. Which of the following is/are the components of Public
increasing values or volumes on which the tax is levied debt?
1. Progressive tax 2. Proportional tax 1. External debt
3. Regressive tax 4. Indirect tax 2. Other internal liabilities
(a) 1 only (b) 2 only 3. Internal debt
(c) 3 only (d) 1, 2 and 4 (a) 1 only (b) 1 and 2
29. Which of the following is not a technique of Deficit (c) 3 only (d) 1, 2 and 3
financing? 37. Choose the correct one from the below expressions
1. Internal Borrowings
1. Fiscal deficit = Budget deficit – Government’s market
2. Counting Currency
3. Printing Currency borrowing and liabilities
4. External aid and borrowings 2. Fiscal deficit = Budget deficit + Government’s
(a) 1 only (b) 2 only market borrowing and liabilities
(c) 2 and 3 (d) 1, 2, 3 and 4 3. Fiscal deficit = Revenue expenditure – Budget
30. Where was VAT introduced? receipts
1. France 2. USA 4. Fiscal deficit = Revenue expenditure + Budget
3. Australia 4. China receipts
(a) 1 only (b) 1 and 2 (a) 1 only (b) 2 only
(c) 3 only (d) 1, 2 and 3 (c) 3 only (d) None of the above
31. Which of the following taxes is the one by which the 38. Regressive tax can be defined as
revenue collected rises proportionally with income? 1. The one by which the revenue collected rises
1. Regressive tax 2. Progressive tax proportionally with income
3. Corporate tax 4. Proportional tax
2. The rates of tax increase for increasing values or
(a) 1 only (b) 2 only
(c) 3 only (d) 4 only volumes on which the tax is levied
32. Which of the following statements is/are correct with 3. The one where the proportion of tax paid falls as
reference to Primary deficit? income rises
1. India started using this term since 1997-98 (a) 1 only (b) 2 only
2. Primary deficit is fiscal deficit minus interest payments (c) 3 only (d) 1, 2 and 3
3. It shows the current state of government finances 39. Which of the following precautions has to be taken by a
(a) 1 only (b) 1 and 2 country going for foreign aid?
(c) 3 only (d) 1, 2 and 3 1. Keeping foreign aid strings-free
33. In the context of economic liberalization, which of the 2. Keeping the borrowing level low so that country does
following is/are the major themes of the fiscal policy? not fall in a debt trap
1. A deliberate move to a regime of reasonable direct tax (a) 1 only (b) 2 only
rates and better administration and enforcement (c) Both 1 and 2 (d) Neither 1 nor 2
2. A systematic effort to simplify tax structure and tax 40. Which of the following is a type of the Public expenditure
laws in India?
(a) 1 only (b) 2 only 1. Plan 2. Non-plan
(c) 1 and 2 (d) Neither 1 nor 2 (a) 1 only (b) 2 only
34. With reference to ‘Cash Reserve Ratio’, which of the (c) 1 and 2 (d) Neither 1 nor 2
following statements is/are correct? 41. Choose the correct one from the following.
1. The RBI varies Cash Reserve Ratio to change the 1. Revenue deficit = Revenue expenditure – Revenue
liquidity of market receipts
2. CRR is subject to frequent changes as RBI intervenes 2. Revenue deficit = Revenue receipts – Revenue
from time to time to correct monetary or exchange expenditure
rate imbalances 3. Revenue deficit = Revenue receipts – Total
3. CRR currently is 4% expenditure
4. RBI is empowered to fix the CRR at a rate ranging 4. Revenue deficit = Revenue expenditure – Total
between three per cent and 15 per cent receipts
(a) 1 only (b) 1 and 2 (a) 1 only (b) 1 and 2
(c) 3 only (d) 1, 2, 3 and 4 (c) 3 only (d) 1, 2, 3 and 4
Fiscal and Monetary Policy E-85

42. Annual financial statement has to be placed before (a) 1 only (b) 2 only
parliament for every financial year i.e. (c) 3 only (d) 4 only
1. January 1 to December 31 51. Which of the following has been introduced as a very
2. March 31 to April 1 important component of Direct Tax code with the objective
3. April 1 to March 31 of preventing such deals and transactions?
(a) 1 only (b) 2 only 1. General Avoidance Rules
(c) 3 only (d) None of the above 2. General Anti Affect Rules
43. Consider the following: 3. General Anti Avoidance Rules
1. Market borrowing 4. General Arm Affect Rules
2. Treasury bills (a) 1 only (b) 2 only
3. Special securities issued to RBI (c) 3 only (d) 1, 2 and 4
Which of these is/are components(s) of internal debt? 52. Which of the following details can be obtained by Annual
(a) 1 only (b) 1 and 2 financial statement?
(c) 2 only (d) 1, 2 and 3 1. Government forecasts of receipts and payments for
44. Consider the following statements with regard to Statutory the next year
Liquidity Ratio (SLR) 2. An outline of the results of the last financial year
1. To meet SLR, Commercial banks can use cash only. compared with the previous budget estimates
2. SLR is maintained by the banks with themselves. 3. Proposed changes in taxes and expenditure allocations
3. SLR restricts the banks leverage in pumping more (a) 1 only (b) 1 and 2
money into the economy. (c) 3 only (d) 1, 2 and 3
Which of the statements given above is/are correct? 53. Excess of total expenditure over total revenues is termed as
(a) 1, 2 and 3 (b) 1 and 3 1. Revenue deficit 2. Fiscal deficit
(c) 2 and 3 (d) only 2 3. Budget deficit 4. Overall deficit
45. Which among the following is/are the nodal agency for (a) 1 only (b) 1 and 2
implementing the monetary policy? (c) 3 only (d) 4 only
1. State Bank of India 54. Which of the following may lead to a pressure on
the government for an upward revision in salaries of
2. Reserve Bank of India
government employees?
3. Union Bank
1. Internal Borrowings
(a) 1 only (b) 1 and 2
2. External aid and borrowings
(c) 2 only (d) 1, 2 and 3
3. Counting Currency
46. Which of the following refers to the set of measures adopted
4. Printing currency
by the central bank?
(a) 1 only (b) 1 and 2
1. Monetary policy 2. GAAR
(c) 3 only (d) 4 only
3. Finance Commission 4. Black Money 55. Which of the following is the tax on income of the
(a) 1 only (b) 1 and 2 companies?
(c) 3 only (d) 4 only 1. Corporation tax
47. Which of the following refers to the use by the government 2. Reliable tax
of the various instruments such as taxation, expenditure and 3. Compensatory tax
borrowing in order to achieve the objectives of balanced (a) 1 only (b) 2 only
economic development etc? (c) 3 only (d) 1, 2 and 3
1. Annual financial statement 56. Who among the following appoints a Finance Commission
2. Fiscal policy under Article 280 of the Constitution?
3. Revenue budget 1. Prime Minister 2. Finance Minister
(a) 1 only (b) 2 only 3. President 4. Public
(c) 3 only (d) 1, 2 and 3 (a) 1 only (b) 2 only
48. Which among the following is/are the main objective of (c) 3 only (d) 3 and 4
Monetary Policy? 57. Which of the following statements is/are correct as per
1. Maintenance of domestic price level Article 114(3) of the Constitution?
2. Reducing the impact of business cycles 1. No money can be taken out of consolidated fund
3. Stability of external value without the approval of the Rajya Sabha
(a) 1 only (b) 1 and 2 2. No money can be taken out of consolidated fund
(c) 3 only (d) 1, 2 and 3 without the approval of the Lok Sabha
49. The highest rate of income tax before 1975 was 3. Money can be taken out of consolidated fund without
1. 72.85 2. 97.25 any approval
3. 46.89 4. 27.14 Select the correct answer using the code given below:
(a) 1 only (b) 2 only (a) 1 only (b) 2 only
(c) 3 only (d) 3 and 4 (c) 3 only (d) 1, 2 and 3
50. In which of the following years, did govt introduce 58. Which of the following statements of the government gives
Minimum Alternate tax on companies? expression to its fiscal policy?
1. 1996 2. 1949 1. Annual financial statement
3. 1972 4. 2005 2. Deficit financing
E-86 Fiscal and Monetary Policy

3. Open capital account (a) I-c, II-a, III-b (b) I-b, II-c, III-a
(a) 1 only (b) 1 and 2 (c) I-a, II-d, III-b (d) I-b, II-a, III-c
(c) 3 only (d) 1, 2 and 3 64. List-I List-II
59. Which of the following is the rate at which RBI lends to (A) Partial (1) Unified exchange rate,
commercial banks? convertibility applied for export and
1. Corporate rate 2. Economy rate import of goods only, visible
3. Bank rate 4. Growth rate transaction of BoP.
(a) 1 only (b) 1 and 2 (B) Convertibility (2) Currency can be
(c) 3 only (d) 4 only on trade account converted on all accounts.
(C) Convertibility (3) 60% of foreign exchange
Matching Based MCQ on current account to be converted at market
rates and rest at official rate.
DIRECTIONS (Qs. 60-67) : Match List-I with List-II and (D) Full convertibility (4) Full convertibility in
select the correct answer using the codes given below the lists. all visible and invisible
60. Match columns A and B wherein Column B defines Column A transaction of goods and
Column A Column B services.
I. Public Account a. Consists of all revenues (a) A-1, B-2, C-3, D-4 (b) A-3, B-1, C-4, D-2
and loans received by the (c) A-4, B-2, C-1, D-3 (d) A-1, B-4, C-3, D-2
government 65. Match columns A and B wherein Column B defines Column A
II. Consolidated fund b.  Comprises of the sum Column A Column B
placed at the disposal of the
President to meet unforeseen Capital a. Includes interest payments,
I.
expenditure expenditure subsidies, defence expenditure
III. Contingency fund c.  Consists of receipts and b. Includes loans to PSUs, states,
payments, which are in II. Plan expenditure
foreign governments
form of deposit account
c. Includes expenditure on central
with the government, such
plans such as agriculture,
as provident funds, small Revenue
savings, etc III. rural development, irrigation,
expenditure
(a) I-c, II-a, III-b (b) I-a, II-b, III-c transport, communications,
(c) I-a, II-d, III-b (d) I-b, II-a, III-c environment and welfare schemes
61. Match columns A and B wherein column B shows the tax (a) I-c, II-a, III-b (b) I-b, II-c, III-a
to GDP ratio for respective year in Column A (c) I-a, II-d, III-b (d) I-b, II-a, III-c
Column A Column B 66. In the context of Indian economy, which of the following is/are
I. 1950-51 a. 10.60% the purpose/purposes of ‘Statutory Reserve Requirements’?
II. 2007-08 b. 6% (CSAT 2014-I)
III. Present c. 11.89% 1. To enable the Central Bank to control the amount of
(a) I-c, II-a, III-b (b) I-a, II-c, III-b advances the banks can create
(c) I-b, II-c, III-a (d) I-b, II-a, III-b 2. To make the people’s deposits with banks safe and
62. Match columns A and B Column A liquid
Column A Column B 3. To prevent the commercial banks from making
I. Planned expenditure a. 2,21,733 Crores excessive profits
II. Non-Planned b. 4,65,277 4. To force the banks to have sufficient vault cash to
Crores expenditure meet their day-to-day requirements
III. Tax revenue c. 13,12,200 Crores
Select the correct answer using the code given below.
IV. Non-tax revenue d. 9,19,842 Crores
(a) 1 only (b) 1 and 2 only
(a) I-c, II-a, III-b, IV-d (b) I-b, II-c, III-d, IV-a
(c) I-a, II-c, III-d, IV-b (d) I-d, II-c, III-d, IV-a (c) 2 and 3 only (d) 1, 2, 3 and 4
63. Match columns A and B wherein Column B defines Column A 67. With reference to Indian economy, consider the following.
(CSAT 2015-I)
Column A Column B
1. Bank rate 2. Open market operations
External debt a. Includes small saving schemes,
I. 3. Public debt 4. Public revenue
of the provident fund, reserve fund railways
Which of the above is/are component/ components of
b. Includes loan from foreign countries
II. Internal debt Monetary Policy?
and international financial institutions
(a) 1 only (b) 2, 3 and 4
Other internal c. Includes market loans from banks and
III. (c) 1 and 2 (d) 1, 3 and 4
liabilities financial institutions
Fiscal and Monetary Policy E-87

EXERCISE-1 maintain with itself at any given point of time in


1. (c) 2. (c) 3. (b) 4. (a) 5. (a) 6. (b) the form of liquid assets like cash in hand, current
7. (d) The Commission shall make recommendations as to balances with other banks and first class securities
the following matters, namely : which can be turned into cash (gold, cash or other
(i) The distribution between the Union and the States approve securities). This ratio at present is 25%.
Some assets have to be in liquid form to take care of
of the net proceeds of taxes which are to be, or may
financial emergencies which every bank has to face.
be, divided between them under Chapter I Part XII
It regulates the credit growth in India.
of the Constitution and the allocation between the
25. (d) Taxes on tooth paste come under GST which is
States of the respective shares of such proceeds;
administered by State government.Sales tax is paid
(ii) The principles which should govern the grants-in-aid to sales tax authority in the state from where the
of the revenues of the States out of the Consolidated goods are moved.
Fund of India and the sums to be paid to the States 26. (d) The Act requires the government to lay before
which are in need of assistance by way of grants- the parliament three policy statements in each
in-aid of their revenues under article 275 of the financial year namely Medium Term Fiscal Policy
Constitution for purposes other than those specified Statement; Fiscal Policy Strategy Statement and
in the provisions to clause (1) of that article; and Macroeconomic Framework Policy Statement.
(iii) The measures needed to augment the Consolidated 27. (c) As per economic survey 2007-2008 corporation tax
Fund of a State to supplement the resources of the is the largest source of revenue of the Government
Panchayats and Municipalities in the State on the of India.
basis of the recommendations made by the Finance 28. (c) The Department of Economic Affairs (DEA) under
Commission of the State. Ministry of Finance is the nodal agency of the Union
8. (b) The DEA or the Department of Economic Affairs Government to formulate and monitor country’s
is also responsible for preparation and presentation economic policies and programmes having a bearing
to the Parliament of Central Budget and the Budgets on domestic and international aspects of economic
for the State Governments under President’s Rule management.
and Union Territory Administration. 29. (d) Corporation Tax is imposed by State as well as
9. (c) The excise duty’s share in the total tax revenue, Central Government. Corporation tax is imposed on
which was 41.3 percent in 1992-93, declined to 25.1 the income or capital of some types of legal entities.
percent in 2006-07. The customs duty’s share in the The taxes may also be referred to as income tax.
total tax revenue, which was 31.9 percent in 1992- 30. (c) Convertibility of rupee implies freely permitting
93, fell to 17.5 percent in 2006-07, as a result of the conversion of rupee to other currencies and
massive structuring on excise and customs. vice versa. Currency Convertibility is the ease with
10. (c) 11. (a) 12. (d) 13. (d) 14. (c) 15. (d) which a country’s currency can be converted into
16. (a) 17. (d) 18. (c) 19. (a) 20. (d) 21. (a) gold or another currency.
22. (a) Deficit financing refers to the difference between 31. (c) When the Reserve Bank of India reduces the
expenditure and receipts. In public finance, it means Statutory Liquidity Ratio by 50 basis points; the
the govt. is spending more than what it is earning. Scheduled Commercial Banks may cut their lending
Deficit financing is a necessary evil in a welfare rates.
state as the states often fail to generate tax revenue EXERCISE-2
which is sufficient enough to take care of the
1. (c) 2. (b) 3. (d)
expenditure of the state. The basic intention behind
4. (c) Planned economic development adopted by India
deficit financing is to provide the necessary impetus
to economic growth by artificial means. required an active monetary policy. The two
23. (b) All taxes which are the personal liability of an stated aims of this policy were to boost economic
assessee come under direct taxes. They include development and control inflationary pressure
income tax, professional tax, wealth tax, securities 5. (c)
transaction tax, commodity transaction tax and the 6. (a) A design for the Goods and Services Tax, and a
like. On the other hand, the taxes which a person compensation package linked to adherence to the
can recover from some other person but the liability proposed design
of which remains of the person collecting such taxes 7. (d) 8. (b) 9. (d)
are indirect taxes. These are custom duty, excise, 10. (a) There are two types of budgets i.e., Revenue budget
service tax, vat, CST and the like and Capital budget.
24. (b) SLR or the Statutory Liquidity Ratio is that ration Revenue budget contains all current receipts, such as
of total deposits which a commercial bank has to taxation, (central excise, custom duty, corporation
E-88 Fiscal and Monetary Policy

tax) dividends of public sector units (PSU’s) and 20. (c) Black money deals with all illegal economic activities
expenditure of the government. Capital budget 21. (a) Customs duty is the tax imposed on commodities
consists of all capital receipts and expenditure such imported into India (import duty) or those exported
as domestic and foreign loans, loan repayment, from India (export duty). Since imposing duties
foreign and etc. on exports reduced the competitive position of the
11. (d) Non-plan expenditures include non-developmental country, the government withdrew export duties
expenditure (interest payment, subsidies, defence 22. (c) At present, the SLR is 21.5%
expenditure, civil administration), developmental 23. (b) Revenue budget contains information about taxation
expenditure and expenditure incurred on projects such as central excise, custom duty, corporation tax
which remained unfinished in the earlier plans. etc
12. (a) In respect of capital account of balance of payment, 24. (d) The commodities which are produced within the
the Indian rupee is partially convertible after 1991. country levied by central excise duty. However,
13. (c) Non-plan expenditure covers interest payments, commodities on which state governments impose
subsidies (mainly on food and fertilisers), wage and excise duties (e.g., liquor, drugs) are exempted
salary payments to government employees, grants to from the central excise duty
States and Union Territories governments, pensions, 25. (c) Taxation is used for mobilizing and channelizing
police, economic services in various sectors, resources for productive investment. It can also
defence, loans to public enterprises, loans to States, be used as a measure to promote equity and
Union Territories and foreign governments. reduce disparities or to encourage or discourage
14. (a) Corporation Tax, Wealth Tax and Income Tax are consumption of particular items
in the category of direct tax. 26. (d) The process of bridging the gap between the revenue
15. (d) Monetised deficit was adopted by India in 1997- and expenditure is called deficit financing. In other
98. It refers to that part of deficit for which the words, Deficit financing refers to the ways in which
government borrows from the RBI. To meet the the budgetary gap is financed
government’s such requirements, the RBI prints 27. (b) Service tax was introduced in 1994-95 to address
fresh currency, as a result of which the economy the asymmetric and distortionary treatment of goods
gets monetised and services in tax framework and to widen the tax
16. (c) In the case of direct taxes the burden or ‘incidence’ net
has to be borne by the taxpayers themselves whereas 28. (a) Income tax is a progressive tax as it has exemptions
in the case of an indirect tax, the burden can be for very small incomes, low rates for the first slab
shifted to another person of taxable income, and higher rates for the largest
17. (d) A currency is said to be convertible when it can incomes
be freely exchanged for another currency at market 29. (b) There are three techniques of Deficit financing:
rates. Transaction of current account includes Printing Currency, Internal Borrowings and
dealing with payments relating to foreign trade, External aid and borrowings. Counting Currency is
travel and other services. Capital account deals with not a technique of Deficit financing
transaction in financial assets. While India has made 30. (a) It was introduced in France to overcome the
the Indian rupee fully convertible on current account, cascading effect of several taxes-from raw material
it is yet to accept capital account convertibility as a to the final product in the process of production
goal. Broadly, capital account convertibility would 31. (d) Proportional tax is one by which the revenue
mean freedom for firms and residents to freely buy collected rises proportionally with income. A tax
into overseas assets such as equity, bonds, property, system could be made approximately proportional
and acquire ownership of overseas firms besides by having a uniform rate of income tax with very
free repatriation of proceeds by foreign investors few exemptions, and indirect taxes levied at similar
18. (b) Open Market Operations refers to the RBI buying rates on as many goods and services as possible
and selling eligible securities to regulate money 32. (d) Primary deficit is fiscal deficit minus interest
supply. Traditionally RBI was not resorting to this payments. India started using this term since
method. However, after the large inflow of foreign 1997- 98. Primary deficit is considered a very
funds since 1991, RBI has had to step in to sterilize useful tool in helping bring more transparency in
the flow to avoid excess liquidity the government’s pattern of expenditure. It shows
19. (d) In under-developed countries, deficit-financing has the current state of government finances. If interest
been considered essential for financing the plans payments are deducted from fiscal deficit, then it
of economic development. It is used as a tool for will obviously show a lesser deficit for that year as
meeting financial needs of government, especially in the interest payments are on account of loans taken
times of war. It is used for the mobilization of surplus, in the past and not in the present year
non- utilized and idle resources in the economy. It 33. (c) Fiscal policy comprises of several major themes
is also used as an instrument of economic policy for 34. (d) Every commercial bank is required to keep a
removing the conditions of depression 4 to raise the certain percentage of its demand and time liabilities
level of output and employment. (deposits) with the RBI (either as cash or book
Fiscal and Monetary Policy E-89

balance). The RBI varies this ratio to change the 48. (d) Objectives of Monetary Policy are:
liquidity of market. RBI is empowered to fix the Stability of external value: Fluctuation in exchange
CRR at a rate ranging between three per cent and rate of a currency affects foreign trade and
15 per cent. Like the Bank Rate, CRR is also investment. It is, therefore, important that the rate of
subject to frequent changes as RBI intervenes from exchange is maintained without violent fluctuations.
time to time to correct monetary or exchange rate Maintenance of domestic price level: Fluctuation in
imbalances. This ratio, currently, is 4% prices affects investment decisions. It also leads to
35. (a) At present, the repo rate is 6.75% increasing income disparities. However, monetary
36. (d) Public debt has three components - Internal debt, policy alone cannot ensure the maintenance of
Other internal liabilities and External debt
domestic prices, as several other factors such as
37. (b) Fiscal deficit is budget deficit plus borrowings and
erratic monsoons, changes in tastes, fluctuation in
other liabilities.
world prices etc., affect domestic prices.
Fiscal deficit = Budget deficit + Government’s
market borrowing and liabilities. Reducing the impact of business cycles (slumps
The fiscal deficit situation shows whether the and booms) by manipulation of credit and interest
government is spending beyond its income. India policy. However, economists are not of the same
has, unfortunately, been a country prone to constant opinion on whether business cycles are primarily
and high fiscal deficit situations. A high fiscal deficit caused by monetary factors.
implies high indebtedness of the government and 49. (b) The highest rate of income tax was 97.25, before
a deficit above 3% in the Indian context means an 1975. Moderate rates of income tax encourage
alarming situation for the government finances savings, faster growth and motivate voluntary
38. (c) Regressive tax is one where the proportion of tax compliance to tax regime. Over the years, the rates
paid falls as income rises. The most regressive tax is have been brought down
a poll tax, levied at a fixed rate per person regardless 50. (a) In 1996, govt introduced minimum Alternate tax
of income. A tax system can be made regressive by (MAT) on companies which escaped the corporation
having indirect taxes levied at relatively high rates tax net by using the provisions of exemptions,
on goods heavily consumed by the poor deductions incentives, deprecation and so on
39. (c) A country going for foreign aid has to take several 51. (c) General Anti Avoidance Rules (GAAR) has been
precautions. However two major precautions are: introduced as a very important component of Direct
Keeping the borrowing level low so that country
Tax code with the objective of preventing such deals
does not fall in a debt trap; and Keeping foreign aid
and transactions that are carried out to evade and
strings-free
avoid paying taxes. In other words, GAAR seeks
40. (c)
41. (a) Revenue deficit means the excess of current revenue to prevent such transactions that are carried out by
expenditure over current revenue receipts. Revenue way of aggressive tax planning so as to avoid paying
deficit indicates that the government cannot meet its taxes
current expenditure from its current revenue. 52. (d) Annual financial statement gives various information
Revenue deficit= Revenue expenditure – Revenue for the current, last and next year
receipts 53. (c) Budget deficit is the overall deficit i.e., the excess
42. (c) of total expenditure over total revenues. It includes
43. (d) Treasury Bills are money market instruments to both capital and revenue items in receipts and
finance the short term financial requirements of expenditure. Traditionally, deficit financing in
the Government of India. These are discounted Indian budgets had meant filling this gap
securities and are issued at a discount to face value. 54. (d) Printing currency is usually the last resort for the
44. (c) SLR used by bankers indicates the minimum government in managing its deficit. It might help
percentage of deposits that the banks have to the government in times of need but it should be
maintain in the form of gold, cash or other approved undertaken only in case of extreme necessity as
securities. it has many damaging effects on the economy. It
45. (c) The Reserve Bank of India is the nodal agency for increases inflation proportionally. It may also lead to
implementing the monetary policy. RBI has defined a pressure on the government for an upward revision
its monetary policy in terms of “adequate financing
in salaries of government employees, which in turn
of economic growth and at the same time ensuring
will lead to an increase of government’s expenditure,
reasonable price stability”.
further necessitating printing of currency and more
46. (a) Monetary policy refers to the set of measures
adopted by the central bank (RBI) for monetary inflation
management 55. (a) Corporation tax is the tax on income/profit of the
47. (b) It is used to achieve the objectives of balanced organizations. In India, at one time, corporation tax
economic development, full employment or to was quite high
establish a welfare state Economics takes care of 56. (c) Under Article 280 of the Constitution, the President
various needs and wants of life appoints a Finance Commission every five years
E-90 Fiscal and Monetary Policy

57. (b) Lok Sabha approval is mandatory in order to take Expenditure


out money from the Consolidated fund Non-Planned expenditure 13,12,200 crores
58. (a) Fiscal policy is expressed by the budget Planned expenditure 4,65,277 crores
59. (c) Bank rate is an instrument of monetary policy which Total 17,77,477 crores
is the rate at which RBI lends to commercial banks 63. (b) Internal debt, Other internal liabilities and External
60. (a) The budget shows the receipts and payments of the debt are three components of Public debt wherein
government under three heads. All revenues and Internal debt includes market loans from banks
loans received by the government comes under and financial institutions, short-term borrowings
Consolidated fund, sum placed at the disposal of the on treasury bills and other bonds and certificates
President to meet unforeseen expenditure falls under issued by the government. External debt includes
Contingency fund and receipts & payments which loan from foreign countries and international
financial institutions like the World Bank, IMF,
are in form of deposit account with the government
ADB, etc. Other internal liabilities includes small
comprises Public Account
saving schemes, provident fund, reserve fund of the
61. (c) The tax to GDP ratio (centre and states together) railways, post and telegraph on which the central
was 6 percent in 1950-51, rose to 11.89 in 2007-08 government has to pay interest
and is currently around 10.60% 64. (b)
62. (b) A summary of the expected Income and Expenditure 65. (b) All asset creating and productive expenditure is
data of the Union Budget 2015-16 is given in the part of plan expenditure, and all non- productive,
Table. It shows a revenue deficit of 2.8 percent and consumptive and non- asset building expenditure is
part of non- plan expenditure. Non-plan expenditure
fiscal deficit of 3.9 percent is further divided into revenue expenditure and capital
Fiscal and Monetary Policy expenditure
Budget 2015-16(Revenue Receipts) 66. (b) Because Reserve Requirements are designed as
Tax revenue (income and 9,19,842 Crores “precautionary measures” and not to stop banks from
corporate taxes) “excessive” profit. This eliminates (c) and (d).
67. (c) The RBI implements the monetary policy through
Non-tax revenue (customs, excise 2,21,733 Crores open market operations, bank rate policy, reserve
duties and service taxes) system, credit control policy, moral persuasion and
Total 11,41,575 Crores through many other instruments.
MONEY SUPPLY AND
5
INDIAN FINANCIAL SYSTEM

Introduction
A well established financial system plays very important role in economic development of any country. A financial system
consists of financial institutions, financial markets, financial instruments and financial services. This system provides a frame-
work by which savings and surplus funds are mobilized in a productive manner.
A financial system server as a link between savers and investors
It promotes the capital formation by bringing together supply of savings and demand for funds.
This system provides detailed information about the players in the market such as individuals, corporate houses, government
agencies etc.
It also provides a mechanism for controlling risks involved in managing savings and allocating funds.
It covers the whole gamut of demand for and supply of funds for productive purposes. The financial system promotes eco-
nomic development through mobilising savings and channelising these to investment avenues.
The Indian financial system consists of both short term and long term finances.

Indian Financial System

Introduction Capital Money Supply Insurance Micro


Sector
Money Market Inflation Indian Banking Finance
Market System
Securities Financial LIC GI C IRDA
Institution Inflation
Call Money Gilt in India Evolution Commodity
Market Edged SEBI Types of RBI Futures
Corporate DFI Inflation
Treasury Bill Market
Securities Objectives
Market Calculation
ICICI Function NBFIs
Commercial Bill Of Inflation of RBI
IDBI
Market Impact of Composition Exim
Collateral Loan Inflation Bank
Market Control of RRB NHB
Certificate of Inflation Private SIDBI
Banks
Deposit and Deflation NABARD
Co-operative
Commercial paper Bank
Markets Checking SCB
Money Market Deflation Banking Systems in India
Instruments Banking Sector Reforms
E-92 Money Supply and Indian Financial System

MONEY MARKET
Money market refers to the short term (6-12 months) (iv) The call money market is highly sensitive and competitive
borrowings and lending, whereas market. As such, it acts as the best indicator of the
Capital market refers to longer term lending/ borrowing. liquidity position of the organised money market.
Money market refers to that mechanism whereby borrowers (v) The rate of interest in the call money market is highly
manage to obtain short term loan- able funds on the one hand, unstable. It quickly rises under the pressures of excess
and lenders succeed in getting credit worthy borrowers for demand for funds and quickly falls under the pressures of
their money on the other. In this way, any institution or person excess supply of funds.
who is willing to provide short-period monetary debt becomes (vi) The call money market plays a vital role in removing
a part of the money market. the day-to-day fluctuations in the reserve position of the
Under Indian money market, the Reserve Bank of India individual banks and improving the functioning of the
occupies the central position because it regulates and controls banking system in the country.
the credit supply of the country.
Ordinarily, the Indian money market is divided into two parts: 2. Treasury Bill Market
1. The Unorganised sector. The Treasury bill market deals in treasury bills which are
2. The Organised sector. the short-term (i.e. 91, 182 and 364 days) liability of the
Government of India. Theoretically, these bills are issued to
Unorganised Sector meet the short-term financial requirements of the government.
But, in reality, they have become a permanent source of funds
The Unorganised sector is formed of the unregulated non-bank to the government. Every year, a portion of treasury bills are
financial intermediaries, indigenous bankers, and moneylenders. converted into long-term bonds. Treasury bills are of two types:
(i) Ad hoc (ii) Regular
Organised Sector (i) Ad hoc : Ad hoc treasury bills are issued to the state
The organised sector includes the State Bank of India and governments, semi-government departments and foreign
its associated banks, 19 nationalised banks, Regional Rural central banks. They are not sold to the banks and the
Banks, Co-operative Banks, Non-governmental sectors and general public, and are not marketable.
other banks, whereas the unorganised sector includes the (ii) Regular : The regular treasury bills are sold to the banks
money-lenders and indigenous bankers. and public and are freely marketable. Both types of ad
Organised sector includes Reserve bank of India, Private banks, hoc and regular treasury bills are sold by Reserve Bank of
public sector banks, developmental banks and other non banking India on behalf of the Central Government.
financial companies (NBFCs) such as life insurance corporation The Treasury bill market in India is underdeveloped as
of India (LIC), Unit trust of India (UTI), the International compared to the Treasury bill markets in the U.S.A. and the
Finance Corporation, IDBI and the co-operative sector. U.K. In the U.S.A. and the U.K., the treasury bills are the
The organised sector of Indian money market can be further most important money market instrument:
classified into the following sub-markets: (a) Treasury bills provide a risk-free, profitable and highly
liquid investment outlet for short-term, surpluses of
1. Call Money Market various financial institutions;
(b) Treasury bills from an important source of raising fund
The most important component of organised money market
for the government; and
is the call money market. It deals in call loans or call money
(c) For the central bank the treasury bills are the main
granted for one day. Since the participants in the call money
instrument of open market operations.
market are mostly banks, it is also called interbank call money
market. On the contrary, the Indian Treasury bill market has no
dealers except the Reserve Bank of India. Besides the Reserve
The banks with temporary deficit of funds form the demand
Bank, some treasury bills are held by commercial banks, state
side and the banks with temporary excess of funds form the
government and semi-government bodies.
supply side of the call money market.
The main features of Indian call money market are as follows: But, these treasury bills are not popular with the non-bank
(i) Call money market provides the institutional arrangement financial institutions, corporations, and individuals mainly
because of absence of a developed Treasury bill market.
for making the temporary surplus of some banks available
to other banks which are temporary in short of funds.
(ii) Mainly the banks participate in the call money market. 3. Commercial Bill Market
The State Bank of India is always on the lenders’ side of Commercial bill market deals in commercial bills issued by
the market. the firms engaged in business. These bills are generally of
(iii) The call money market operates through brokers who three months maturity. A commercial bill is a promise to pay
always keep in touch with banks and establish a link a specified amount in a specified period by the buyer of goods
between the borrowing and lending banks. to the seller of the goods.
Money Supply and Indian Financial System E-93

The seller, who has sold his goods on credit draws the bill range of money market instruments and give investors greater
and sends it to the buyer for acceptance. After the buyer or flexibility in the deployment of their short-term surplus funds.
his bank writes the word ‘accepted’ on the bill, it becomes a
marketable instrument and is sent to the seller. 6. Money Market Instruments
The seller can now sell the bill (i.e. get it discounted) to his
Chakravarthy committee and Narasimhan committee
bank for cash. In times of financial crisis, the bank can sell
recommended certain money market instruments to reform
the bills to other banks or get them rediscounted from the
Indian money market. Some of these are:
Reserved Bank.
In India, the bill market is undeveloped as compared to the • 182 days treasury bills which are sold through fortnightly
same in advanced countries like the U.K. There is absence auctions. They carry attractive rates of interest and
of specialised institutions like acceptance houses and discount practically no risk and are therefore popular with
houses, particularly dealing in acceptance and discounting commercial banks.
business. • 364 days treasury bills were also introduced in 1992.
• Dated government securities with maturities up to 10
4. Collateral Loan Market years have also been introduced primarily to develop a
secondary market.
Collateral loan market deals with collateral loans i.e. loans
• Money market mutual funds have been permitted to be
backed by security. In the Indian collateral loan market, the
floated by commercial banks.
commercial banks provide short- term loans against government
securities, shares and debentures of the government, etc. • Repurchase options (repos) and reverse repos have been
introduced in order to even out sharp fluctuations in the
money market. Repos provide an opportunity for RBI
5. Certificate of Deposit and to repurchase government securities from commercial
Commercial Paper Markets banks. Reverse repos are government securities sold
Certificate of Deposit (CD) and Commercial Paper (CP) through auction at fixed cut-off rate of interest.
markets deal with certificates of deposit and commercial • Liquidity Adjustment Facility (LAF) refers to RBI’s policy
papers. These two instruments (CD and CP) were introduced of using Repos and Reverse Repos to adjust liquidity on a
by Reserve Bank of India in March 1989 in order to widen the day-to-day basis.

CAPITAL MARKET
Capital market deals with long-term finance (more than The main players in Indian capital markets are:
365 days) funds. It includes all facilities and institutional • Banks, indigenous and commercial.
arrangements available for borrowing and lending of term • Insurance companies
funds (including medium-term). • Development Finance Institutions (DFI),
The difference between money market and capital market is • Non-Banking Finance Companies, (NBFCs)
not so much in the institutions involved as in their term of • Non-Banking Financial Institutions.
borrowing or lending. Long-term funds are raised either by
The capital market may be divided into
borrowing from certain institutions or by issuing securities.
(i) the securities market; and (ii) the financial institutions.
Capital Market
Securities Market
Main Players Securities Financial The securities market is further divided into the gilt-edged
Market Institutions market and the corporate securities market.

Gilt Edged Securities (G-sec)


Gilt Edged Corporate IFCI
Securities
Indian capital market consists of market for gilt-edged securities
Securities
ICICI guaranteed by the government (Central or state). These
securities are issued by local authorities (like city corporations,
Primary Secondary IDBI municipalities and port trusts) and autonomous government
Market Market undertaking like the state electricity boards, development banks,
etc. The RBI plays a dominant role in the gilt-edged market
Stock Exchange Over the through its open market operations. Mostly financial institutions
(23) Counter Exchange that have statutory obligations to keep a certain portion of their
investments in government securities buy these. RBI trades in
National Regional G-sec as part of its open market operations.
(21)
Corporate Securities
NSE BSE
ISE Others It refers to the stocks, shares, debentures, etc. of companies
(15) (6) which are traded in stock market forming primary market. For
E-94 Money Supply and Indian Financial System

inducing the public to invest their savings in new issues, services ISE
of specialised institutions (underwriters and stock brokers) are Interconnected Stock Exchange of India (ISE) is a
required. This kind of underwriting (guaranteeing purchase of collaboration of India’s 15 regional stock exchanges (RSEs),
a new issue at a fixed price) was non-existent in India until and was set up in 1998. The RSEs were provided more power
the establishment of the ICICI in 1951. Other institutions like and reach through this. It is a web based exchange.
the LIC, IDBI, UTI, IFCI and GIC, commercial banks, stock
Indo Next is a new stock exchange, set up to promote liquidity
brokers and investment funds soon became the participants in
to stocks of small and medium enterprises (SMEs) was
the new issues market.
launched in 2005 jointly by the BSE and FISE (Federation
Secondary market, deals in securities already issued by of Indian Stock Exchanges, representing 18 regional stock
companies. The main purpose of such a market is to provide exchanges). It is better known as BSE Indo Next.
liquidity (easy convertibility into cash) to such securities.
The secondary market has two segments: stock exchange and Approved Stock Exchanges in India
over -the-counter exchange. 1. Bhuvneshwar Stock Exchange, Bhuvneshwar.
2. Calcutta Stock Exchange, Calcutta.
Stock Exchange 3. Cochin Stock Exchange, Cochin.
A stock exchange is an organised market, i.e association of 4. Delhi Stock Exchange, Delhi.
persons or firms to regulate and supervise all transactions, 5. Guwahati Stock Exchange, Guwahati.
rules, regulations and standard practices to govern all market 6. Hyderabad Stock Exchange, Hyderabad.
transactions, authorised stock brokers and an exchange floor 7. Jaipur Stock Exchange, Jaipur.
where stock broker or their authorised agents meet during 8. Canara Stock Exchange, Mangalore.
fixed business hours to buy and sell securities. The stock 9. Ludhiana Stock Exchange, Ludhiana.
exchange is an institution for orderly buying and selling of 10. Chennai Stock Exchange, Chennai.
listed securities. Listed securities are those that appear on the 11. M.P. Stock Exchange, Indore.
approved list of a stock exchange. (A particular security is 12. Magadh Stock Exchange, Patna.
listed or quoted when it is incorporated in the register of the 13. Pune Stock Exchange, Pune.
stock exchange so that it can be bought / sold there.) There are 14. U.P. Stock Exchange, Kanpur.
23 stock exchanges in the country out of which 20 (including 15. Vadodara Stock Exchange, Vadodara.
Bombay Stock Exchange) are regional and two national. 16. Koyambtour Stock Exchange, Coimbatore.
17. Merrut Stock Exchange, Meerut.
NSE 18. Mumbai Stock Exchange, Mumbai.
19. Over the Counter Exchange of India, Mumbai.
National Stock Exchange (NSE) was set up in 1984. NSE grew 20. National Stock Exchange, Mumbai.
rapidly to become India’s biggest exchange. The NSE share 21. Ahmedabad Stock Exchange, Ahmedabad.
index is known as Nifty. There is a 50 share index, S and P 22. Bangalore Stock Exchange, Bangalore.
CNX 50 (Nifty fifty), S & P CNX 500 (500 share index). Nifty 23. Capital Stock Exchange, Kerala Ltd., Tiruvananthapuram,
junior is the second-tier index of 50 stocks. The NSE became Kerala
the first stock exchange in India to admit overseas shareholders. On July 9, 2007 SEBI has withdrawn units approval from
Saurashtra Stock Exchange, Rajkot due to its passive working.
BSE Hence the number of approved stock exchanges has reduced to 23.
Bombay Stock Exchange (BSE), India’s oldest stock exchange Securities and Exchange Board of India (SEBI):
formally came into being in 1887 and was a regional exchange The SEBI was set up in 1988 in place of the Controller
till 2002 when it became a national exchange. It became a of Capital Issue who was a functionary of the Ministry of
corporate entity, BSE Ltd. in 2005. Finance. It was made a statutory body in 1992. It is the main
The BSE has four indices. Sensex or sensitive Index has 30 regulatory institution of the Indian capital market.
stocks. BSE-200 is a 200 stock share index (including the Sensex SEBI is managed by six members - one chairman (nominated
stocks). Dollex is its dollar version. BSE-500 is a 500-stock by Central Government), two members (officers of central
index which was developed in 1999 and represents several major ministries) one member (from RBI) and remaining two
industries, with special emphasis on IT sector. The National members are nominated by Central Government. The office
Index of 100 stocks quoted on a national level gives a wider of SEBI is situated at Mumbai with its regional offices at
representation of the stock market. It includes the Sensex stocks. Kolkata, Delhi and Chennai. In 1988 the initial capital of
SEBI was 7.5 crore which was provided by its promoters
Over the Counter Exchange of India (OTCEI) was set up (IDBI, ICICI, IFCI). This amount was invested and with it’s
in 1989, but began to trade only in 1992. The OTCEI deals interest amount day-to-day expenses of SEBI are met.
in securities that are not listed on a stock exchange. These The SEBI is authorised to:
unlisted securities are basically securities of small companies
(with paid -up capital between 30 lakh and 25 crore) and • oversee the working of stock exchanges;
have a limited market. Their prices are determined through • regulate merchant banks and mutual funds;
direct negotiations between stock brokers and not by bidding • register and regulate intermediaries such as stock brokers;
as in stock exchanges. With the prevalence of screen-based • curb fraudulent and unfair trade practices including
trading, the relevance of regional stock exchanges has insider trading;
diminished. • promote the development of a healthy capital market.
Money Supply and Indian Financial System E-95

Development Finance Institutions (DFIs) modernising industrial enterprises; encouraging private


capital participation (both domestic and external) and help to
The Industrial Finance Corporation of India (IFCI):
develop capital markets. Its main aim was to give term loans
The IFCI was the first DFI to be established in 1948 under in rupee and foreign exchange, underwrite issue of shares and
an Act of Parliament. It was originally a shareholders’ debentures and directly subscribe to these issues. Among all
Corporation, but in 1993 it was converted into a company under
the DFIs, ICICI had the most spectacular growth. In a ‘reverse
the Companies Act. It grants loans and advances to industrial
merger’, ICICI was merged with the ICICI Bank in March
concerns, repayable normally within 25 years; guarantees
2002 and became the first ‘universal bank’ in the country.
loans raised by industrial units; subscribes to and underwrites
the issue of stocks and shares by industrial concerns; and Industrial Development Bank of India (IDBI)
extends guarantees for deferred payments by importers. It was The IDBI was established under an Act of Parliament in
converted into a public limited company in 1993 to provide it 1964 for providing direct term finance to large industrial
more operational freedom. However, huge NPAs have made units and assisting small and medium units through banks
IFCI a sick financial institution. and State Finance Corporations (SFCs). It was converted
Industrial Credit and Investment Corporation of India (ICICI): into a company by the 2003 Act and became a universal bank
The ICICI was established in 1955 for promoting and like ICICI through reverse merger with the IDBI Bank.

MONEY SUPPLY
Money supply is the stock of liquid assets held by the public, M1 measure represents the most liquid form of money among
which can be freely exchanged for goods and services. The four money stock measures adopted by RBI. As we move
money supply includes notes and coins to bank deposits. It from M1 to M4, the liquidity gets reduced. In other words,
includes all economic units (households, firms and institutions) M4 possesses the lowest liquidity among all these measures.
but excludes the producers of money (such as the government The reduction in liquidity indicates the shifting from ‘medium
and the banking system as this is not in circulation). As it of exchange’ to ‘store of value’. All these four money stock
measures are not of equal importance. Their relative importance
excludes the money held by the government in its treasuries
varies from the point of view of monetary policy
and the money with the banking system, money supply is bound Generally, in developed countries, the bank deposits are the
to be smaller than the total stock of money in the country’s most important component in money supply, while due to
economic system. less banking habits in under-developed countries people want
The four concepts of money used in calculating money to keep their money in the most liquid form i.e., currency.
M3 is the most important component among all money stock
supply are known as the money stock measures or measures
measures and is generally termed as ‘Broad money’.
of monetary aggregates. These are In economics, the money supply or money stock is the
1. M1 → Money with the Public (currency notes and coins) total amount of monetary assets available in an economy at a
+ Demand deposits of banks (on current and saving specific time. There are several ways to define ‘money’ but
bank accounts) + Other demand deposits with RBI. It standard measures usually include currency in circulation
is highly liquid and banks will not be able to run their and demand deposits.
Money supply data are usually recorded and published by
lending programmes on this basis.
the government or the central bank of the country. Public
2. M2 → M1 + Saving bank deposits with Post-offices. and private sector analyst have long monitored changes in
3. M3 → M1 + Term deposits with the bank. money supply because of its possible effect on the price level
4. M4 → M3 + All deposits of Post-offices. inflation, exchange rate and the business cycle.

INFLATION
Inflation is an increase in price of goods. It can be seen as increases in the money supply (monetarism). Inflation affects
a devaluation of the worth of money. A crucial feature of all segments of the economy. It hurts persons having fixed
inflation is that price rises are sustained. Once only increase incomes, as they can purchase less number of goods in the
in the rate of, say, value-added tax, will immediately put up same amount of money. It affects those like retired persons
prices, but this does not represent inflation, unless the indirect who depend on interest income on their savings. It upsets
effects of the VAT rise have repercussions of prices in periods macro-economic decision making, as the uncertainty regarding
after the direct effects. future costs can affect projections.
Accounts of the causes of inflation are numerous. The most It may appear that inflation benefits borrowers, as it leads to
popular arguments are that it is caused by excess demand in a fall in the real cost of capital. But this is only a temporary
the economy (demand-pull inflation), that it is caused by high phase and the interest rate is bound to go up to compensate for
costs (cost-push inflation) and that it results from excessive the inflation.
E-96 Money Supply and Indian Financial System

Inflation In India Calculation of Inflation


In India inflation is a structural as well as a monetary Changes in price level are measured by the following:
phenomenon. In the short term, localised demand-supply • Wholesale Price Index
imbalances in wage goods, often due to seasonal variations • Consumer Price Index
in production- coupled with market rigidities and regulatory • Gross Domestic Product (GDP) Deflator
failures have supported inflation that have resulted in a The Wholesale Price Index (WPI) is a weighted average
more widespread impact on the consumers than the initial of indices covering 676 commodities, which are traded in
inflationary impulse. In the medium to long-term, the primary, manufacturing and fuel and power-sectors. WPI is
movement and outcome of monetary aggregates such as the thus a measure of inflation on an economy-wide scale. Services
money supply and interest rates of the financial systems have do not figure in this, as there is usually no wholesale price
influenced aggregate demand and consequently changes in for services. The weightage for primary articles is only 20
price levels in the economy. The latter considerations and percent. A revised WPI with 1993-94 as the base year has been
the influence of global commodity prices on the domestic adopted. WPI figures are available on weekly basis with a gap
prices have become more important with the opening and of about two weeks, and are, therefore, useful in continuous
growing integration of the Indian economy with the rest of monitoring of prices for policy making.
the world. Consumer Price Index (CPI) is the retail price average of a
basket of goods and services directly consumed by the people.
Types of Inflation It is computed separately for the following three groups:
1. Demand Pull Inflation occurs when excessive demand (i) Industrial workers - 260 commodities
for commodities increases their prices. This is the (ii) Urban Non-Manual Employees-180 commodities.
conventional inflationary situation of ‘too much money (iii) Agricultural labourers - 60 commodities
chasing too few goods’. The main demand-pull factors in These indices, however, did not cover all the segments of the
India are: population and thus, did not reflect the correct picture of the
(i) Mounting government expenditure price behaviour of the whole country.
(ii) Deficit financing and increase in money supply A result, there was a strong need for compiling a CPI for
entire urban and rural population of the country to measure
(iii) Role of black money
the inflation in Indian economy based on CPI. Thus, now
(iv) Rapid population growth. Central Statistics Office (CSO) of the Ministry of Statistics
2. Cost Push Inflation occurs when wages and other costs and Programme Implementation has started compiling a new
rise and the producers are successful in passing on the series of CPI:
higher costs to the consumers in the shape of higher (a) CPI for the entire urban population viz CPI (Urban); 225
prices. The main cost -push factors in India are : commodities.
(i) Fluctuations in output and supply in both agriculture (b) CPI for the entire rural population viz CPI (Rural) 250
and industry sectors. Fluctuations in output of food commodities.
grains have been a major factor responsible for rise (c) Consolidated CPI for Urban + Rural will also be
in food-grain prices as well as general price. In the compiled based on above two CPIs
same way, the supply of manufactured goods also These reflect the changes in the price level of various goods
did not increase adequately in last few years. Power and services consumed by the Urban and rural population.
breakdowns, strikes and lock-outs and shortage of These new indices are now compiled at State / UT and all
transport facilities have been the major constraints India levels.
responsible for lowering production of manufactured The CPI inflation series is wider in scope than the one based on
goods. With ever-rising demand for manufactured the wholesale price index (WPI), as it has both rural and urban
goods, the producers are in a position to hike the figures, besides state-wise data. The new series, with 2010 as
prices of their products the base year, also includes services, which is not the case with
(ii) Problem of hoarding by traders and black the WPI series.
marketeers. A comparison of this new series with WPI is given below:
(iii) Taxation which gives the traders an opportunity to WPI CPI - New Series
raise the prices of goods, the proportion of which is wef Feb 2012
often more than the levy of taxes.
Base Year 2004-05 2010
(iv) Administered Prices.
Elemenetary Items 676 200 (Weighted items)
(v) Hike in Oil Prices.
3. Stagflation is a situation when high rates of inflation Weightage of Food products 243 49.71
coexist with high rate of unemployment (a combination of (%)
stagnation and inflation). Weightage of Energy products 14..91 9.49
4. Hyperinflation refers to a situation of runaway inflation (%)
reaching even up to 100 percent per annum. It also refers Weightage of Miscellaneous Services 26.31
to rapid inflation in which prices increase so fast that Items (%) not
money loses its importance as a medium of exchange. included
Money Supply and Indian Financial System E-97

GDP deflator, which distinguishes between physical growth Policy Measure to Control Inflation
in output and price rise, and gives an accurate picture of the
over-all price level. The issue of inflation is addressed from both demand and
The GDP Deflator is arrived at by dividing GDP at current supply sides. Demand management implies putting a check
prices by GDP at constant prices in terms of base year prices on the demand of the public for goods and services. Demand
(1993-94 in India). This indicates as to how much of the management is achieved by measures such as postponing
growth in GDP in a year is due to price rise and how much public expenditure, reducing excess liquidity either through
due to increase in output. GDP deflator is now available only taxes or saving schemes and restrictions on ad hoc treasury
annually with a gap of one year. bills. While such measures help contain the money supply,
there is a danger that these will contract the economy and lead
Theoretically, the growth in physical output in an economy has
to an increase in unemployment. Rationalisation of excise and
to be matched by a corresponding growth in monetary flows.
import duties of essential commodities leads to higher burden
Any mismatch between the two gets reflected in the growth
on poor.
rate and price changes. Monetary policy aims at managing the
balance between growth in real flows and growth in monetary RBI assists in controlling inflation through monetary measures
flows. such as quantitative and selective credit controls and by
manipulating the Cash Reserve Ratio (CRR) and the Statutory
Causes of Inflation Liquidity Ratio (SLR). These are the monetary policies adopted
The inflation occurs due to two main factors: by government.
(a) Increase in demand for goods and services. On the supply side, the mechanism of Public Distribution
(b) Decrease in the supply of goods & services System (PDS) ensures availability of essential commodities
(a) Factors causing an Increase in demand for goods & for the vulnerable sections of society. This helps maintain
services: price levels. Fixation of maximum prices helps to eliminate
(i) Increase in public expenditure the incentive for hoarding and speculative activity in food-
(ii) Increase in private expenditure grains. Control over private trade in food-grains and adoption
(iii) Increase in exports of Open General Licence (OGL) to ease the imports of sugar,
(iv) Reduction in taxation pulses, etc., in case of shortages are also some of the common
(v) Rapid growth of population measures undertaken. Coupled with this is the open market
(vi) Black money sale of rice and wheat resorted to by FCI from its buffer stock
(vii) Deficit financing in times of price rise.
(viii) Cheap money policy Inflation and Price Control in the Post-reform Period
(ix) Increase in consumer spending One of the major achievements of economic reforms of
(x) Department of Tax internal debts. 1991 was the removal of restrictions on production and
(b) Factors causing Decrease in supply of goods and prices. The direction was to move from a regulated to a
services: market-related environment. Contrary to expectations, the
(i) Shortage of supplies of factors removal of restrictions did not lead to a spurt in prices.
(ii) Industrial disputes However, inflationary pressure remained moderately high
(iii) Natural calamities caused, paradoxically, by the very success of the reform
(iv) Loop-sided Production process, which brought in large foreign investment leading
(v) Hoarding by consumers an increase in money supply. Foreign inflows need to be
(vi) Hoarding by traders ‘sterillised’ by the RBI by withdrawing from circulation
(vii) Operation of Law of Diminishing Returns. an equivalent amount of rupee either through open market
operation or through regulating bank credit. At the same
time, it was important to maintain the correct exchange
Impact of Inflation rate for the rupee so that its appreciation does not make
1. Inflation is the most regressive form of taxation as it our exports uncompetitive. Following are some of the fiscal
affects the poor and vulnerable sections of the society policies adopted by government.
the most. Such a situation leads to increasing income • Release of buffer stock of food grains to maintain the price
disparities. level;
2. Inflation dampens exports by making our products
expensive and, conversely, makes imports attractive. • Import of some essential commodities like edible oils;
Such a situation may warrant formal or informal • Strict fiscal and monetary discipline
devaluation of the currency in order to make our exports • Compensating farmers for the loss on account of withdreawal
competitive. offertilizer subsidy, through upward revision of procurement
3. Inflation leads to recession, as people with fixed incomes prices.
set apart an increasing share of their income to meet • To maintain price stability, the Central Issue Price for rice
the growing costs of essential commodities, leaving ans wheat has not been revised sonce july 2002. There has
very little for expenditure on non-essential terms. The
been a continuous reduction in the import duty on edible oils.
production of such items has to be reduced, leading to
shutdowns and recession.
E-98 Money Supply and Indian Financial System

Inflation and Price Control in the Post-reform (ii) Merit goods: It includes primary education, immunisation,
public health programme, etc. Not only individual beneficiary
Period
but society at large benefits by making these goods available.
One of the major achievements of economic reforms of 1991 Providing subsidy for these goods is essential.
was the removal of restrictions on production and prices. (iii) Non-merit goods: In non merit goods the benefit goes
The direction was to move from a regulated to a market- directly to the individual while the costs are borne by the
related environment. Contrary to expectations, the removal society. Pollution caused by automobile emission is an
of restrictions did not lead to a spurt in prices. However, example of non-merit goods.
inflationary pressure remained moderately high caused,
paradoxically, by the very success of the reform process, which
brought in large foreign investment, leading an increase in
Deflation
money supply. Foreign inflows need to be ‘sterilised’ by the RBI “Deflation is that state in which the value of money rises and
by withdrawing from circulation an equivalent amount of rupee the price of goods and services falls.”
either through open market operation or through regulating The state of deflation may appear in the economy due to
following reasons-
bank credit. At the same time, it was important to maintain
(i) When the Government withdraws money from
the correct exchange rate for the rupee so that its appreciation circulation.
does not make our exports uncompetitive. Following are some
(ii) When Government imposes heavy direct taxes or takes
of the fiscal policies adopted by government. heavy loans from the public (voluntary or compulsory
• Release of buffer stock of food grains to maintain the or both).
price level;
• Import of some essential commodities like edible oils; (iii) When the Central Bank sells the securities in open
• Strict fiscal and monetary discipline market (which reduces the quantity of money in
• Compensating farmers for the loss on account of circulation).
withdrawal of fertilizer subsidy, through upward revision (iv) When Central Bank controls the credit money and
of procurement prices. adopts various measures such as increase in CRR,
• To maintain price stability, the Central Issue Price for credit rationing and direct action.
rice and wheat has not been revised since July 2002. (v) When the Central Bank increases the Bank rate (which
There has been a continuous reduction in the import duty curtails the quantity of credit in the economy).
on edible oils. (vi) When state of over-production (excess supply over
demand) takes place in the economy.
Public Goods, Merit Goods and Subsidies Measures of Checking Deflation
Subsidies are a major burden on the national budget. There (i) Increasing money supply.
are inevitable leakages in subsidies and much of it does not (ii) Promote credit creation by the banks.
reach the intended beneficiaries. In many cases, the benefits
(iii) Curtailment in taxes so as to increase the purchasing
are taken by the undeserving. Most of our agricultural and
power of the people.
food subsidies suffer from this drawback. In order to assist
(iv) Increasing the public expenditure and the employment
proper targeting of subsidies, goods are classified into three
opportunities in the economy.
categories:
(i) Public goods: It includes national defence, police, general (v) Increasing the money supply in circulation by repayment
administration, etc. These services are available to all and of old public debts.
the enjoyment by one is not at the expense of another. No (vi) Providing of economic subsidy by the Government to
price can be put on these services. the industrial sector of the economy.

INDIAN BANKING
Reserve Bank of India was nationalised on January 1, 1949. 4. The State Bank of Mysore
For the co-ordinated regulation of Indian banking, the Indian 5. The State Bank of Saurashtra
Banking Act was passed in March 1949. By Act, the Reserve 6. The State Bank of Patiala
Bank of India was granted extended powers for the inspection 7. The State Bank of Travancore
of non-scheduled banks. For extending banking facilities in In order to have more control over the banks, 14 large
the rural areas the Imperial Bank of India was partially commercial banks, whose reserves were more than 50 crores
nationalised on July 1, 1955 and it was named as the State each, were nationalised on 19th July, 1969. The nationalised
Bank of India. Along with SBI other 8 (at present 7) banks banks are as follows:
were converted as its associate banks which form what is 1. The Central Bank of India
named as the State Bank Group. They are as follows- 2. Bank of India
1. The State Bank of Bikaner and Jaipur (In the beginning 3. Punjab National Bank
the State Bank of Bikaner and the State Bank of Jaipur 4. Canara Bank
were separate. But they were merged and named as the 5. United Commercial Bank
State Bank of Bikaner and Jaipur). 6. Syndicate Bank
2. The State Bank of Hyderabad 7. Bank of Baroda
3. The State Bank of Indore 8. United Bank of India
Money Supply and Indian Financial System E-99

9. Union Bank of India 1. Andhra Bank


10. Dena Bank 2. Punjab and Sindh Bank
11. Allahabad Bank 3. New Bank of India
12. Indian Bank 4. Vijaya Bank
13. Indian Overseas Bank 5. Corporation Bank
14. Bank of Maharashtra 6. Oriental Bank of Commerce
After one decade, on April 15, 1980, those 6 private sector On 4th September, 1993 the Government merged the New Bank
banks whose reserves were more than 200 crores each were of India with Punjab National Bank and as a result of this the
nationalised. These banks are as: total number of nationalised bank got reduced from 20 to 19.
Evolution of the Indian Banking Industry

Trigger Phases Major


Events Changes

Beginning of institutional  Birth of joint stock banking


banking with 3 joint stock companies
banks  Introduction of deposit banking
Phase 1 and bank branches
Pre Nationalisation Phase  Presidency banks and other joint
stock banks formed setting the
foundation of modern banking
Nationalisation of Imperial system.
Band and 20 other scheduled
commercial banks  State Bank of India formed out of
imperial bank
 20 SCBs nationalised in two
Phase 2
Era of Nationalisation phases
and Consolidation  Directed credit programmes on
the rise
Acceptance of  Introduction of social banking
recommedations of the
Narasimham Committee  Major changes in prudential
regulations
 Interest rates deregulated
Phase 3  Statutory presumption of
Introduction of Indian
resources eased more private
Financial & Banking Sector
Reforms and Partial sector players came in
Liberalisation strengthened the system as a
Hike in
whole
the FDI celling for
banking sector and declaration
of roadmap for  FDI ceiling for the banking sector
liberalisation increased to 74% from 49 %
Phase 4  Roadmap for inclusion of foreign
Period of increased
banks declared
Liberalisation
 More liberal branch licensing
policy followed.
Phase 4
Continues-more
liberalisation expected
With the change of the Indian economy to a higher growth As certain rigidities and weaknesses were found to have
trajectory, the provision of adequate and timely availability developed in the banking system during the late eighties,
of bank credit to the productive sectors of the economy has the Government felt that these had to be addressed to enable
acquired importance. As public sector banks still own about 71 the financial system to play its role in ushering in a more
% of the assets of the banking system, they continue to play efficient and competitive economy. Accordingly, a high-level
an important role in responding to the changes in the economic Committee under the Chairmanship of Shri M. Narasimham
environment. As the banking regulator and supervisor and as on the Financial System (CFS), was set up on August 14, 1991
the monetary policy authority, the Reserve Bank of India (RBI) to examine all aspects relating to the structure, organisation,
continues to guide the banking system, including foreign, functions and procedures of the financial systems. Based on the
private sector and public sector banks, to meet emerging recommendations of the Committee a comprehensive reform
economic challenges. of the banking system was introduced in 1992-93.
E-100 Money Supply and Indian Financial System

Another high-level Committee, under the Chairmanship of Shri issued by Ministry of Finance are circulated through it.
M. Narasimham was constituted by the Government of India The Reserve Bank has adopted the Minimum Reserve
in December 1997 to review the record of implementation of System for the note issue. Since 1957, it maintains gold
financial system reforms recommended by the CFS in 1991 and and foreign exchange reserves of 200 crores, of which at
chart the reforms necessary in the years ahead. The Committee least 115 crores should be in gold.
submitted its report to the Government in April 1998. 2. Banker to the Government :
The second important function of the Reserve Bank is to
Reserve Bank of India (RBI) act as the Banker, Agent and Adviser to the Government.
It is the Central Bank of the country. The Reserve Bank of It performs all the banking functions of the State and
India was established in 1935 with a capital of 5 crore. This Central Government and it also tenders useful advice
capital of 5 crore was divided into 5 lakh equity shares to the Government on matters related to economic and
of 100 each. In the beginning the ownership of almost all monetary policy. It also manages the public debt for
the share capital was with the non-government shareholders. the Government. The RBI also channelises bank credit
In order to prevent the centralisation of the equity shares in in favour of priority sectors. RBI has thus developed
the hands of a few people, the Reserve Bank of India was institutions like IDBI, SIDBI, NABARD, NHB, etc.
nationalised on January 1, 1949. The RBI also promotes cooperative banking and the bill
market.
The general administration and direction of RBI is managed by
a Central Board of Directors consisting of 20 members which 3. Banker’s Bank :
includes 1 Governor, 4 Deputy Governors, The Reserve Bank performs the same function for
1 Government official appointed by the Union Government of other banks as these banks ordinarily perform for their
India to give representation to important stratas in economic customer. It regulates, penalizes and helps banks as and
life of the country. Besides, 4 directors are nominated by when needed.
the Union Government to represent local boards. Apart from 4. Controller of Credit :
the central board there are 4 local boards also and their head The Reserve Bank undertakes the responsibility of
offices are situated in Mumbai, Chennai, Kolkata and New controlling credit created by the commercial banks.
Delhi. 5 members of local boards are appointed by the Union To achieve this objective it makes extensive use of
Government for a period of 4 years. The local boards work quantitative and qualitative techniques to control and
according to the instructions and orders given by the Central regulate the credit effectively in the country.
Board of Directors, and from time to time they also tender 5. Custodian of Foreign Reserves :
useful advice on important matter. The head office of Reserve For the purpose of keeping the foreign exchange rates
Bank of India is in Mumbai. At present, Raghuram Rajan is the stable, the Reserve Bank buys and sells foreign currencies
Governor of Reserve Bank of India. and also protects the country’s foreign exchange funds.
6. Other Functions :
Objectives of RBI
The bank performs a number of other developmental
• The objectives of the RBI is to regulate the issue of bank works. These works include the function of being a
notes and keeping of reserves with a view of securing clearing house, arranging credit for agriculture (which
monetary stability in India and generally to operate the has been transferred to NABARD), collecting and
currency and credit system of the country to its advantage. publishing the economic data, (weekly, monthly and
• The formulation, framework and institutional architecture annual reports are published by the RBI in the field of
of monetary policy in India have evolved around these the country’s currency and finance status), buying and
objectives - maintaining price stability, ensuring adequae selling of Government securities and trade bills, giving
flow of credit to sustain growth momentum and securing loans to the Government, buying and selling of valuable
financial stability. commodities, etc. It also acts as the representative of
• The responsibility for ensuring financial stability has Government in I.M.F. and represents the membership of
involved the vesting of extensive power in and operational India.
objectives for the RBI for regulation and supervision of As mentioned abve, RBI also acts as Controller of Credit, and
financial system and its constituents, the money, debt and in this role makes extensive use of quantitative and qualitative
foreign exchange segments of the financial markets in techniques. These are as follows :
India and the payment and settlement system.
Quantitative Credit Control Measures
Functions of Reserve Bank These measures ae those through which the Central bank of
1. Issue of Notes : Country regulates and controls the total quantum. i.e. volume
The Reserve Bank has the monopoly over issuing of loans and credit given by commercial banks.
of notes in the country. It has the sole right to issue • Bank Rate : It is that rate of interest at which the Central
currency notes of various denominations except one Bank provides financial assistance to commercial banks.
rupee note. The Reserve Bank acts as the only source of When bank rate is raised, it is called “dear money policy”
legal tender of money because even the one rupee notes which implies that commercial banks would also raise the
Money Supply and Indian Financial System E-101

rate of interest at which they would lend to various sectors • Statutory Liquidity Ratio (SLR): It is that ratio of the total
in the economy. On the other hand, when bank rate is deposits of liquid funds, consisting mainly of government
lowered, it is called “cheap money policy” Generally, bank securities and, to some extent, cash in hand. It mainly
rate may be raised during period of inflation, and may be serves the purpose of lending to government , and to a
lesser extent, it also prevents a panic run on the banks.
lowered during a period os slowdown and recession.
• Repo and Reverse Repo rate: Under repo rate, the RBI
• Cash Reserve Ratio (CRR) : It is that ratio of total undertakes to repurchase government securities from
deposits of a bank which a bank has to keep in cash with banks,, with a commitment that it will give them back
the central bank. When this ratio is raised, it is called a to the banks. Thus, under Repos, the RBI lends to
policy of “credit squeeze” and when lowered, it is known commercial banks by repurchasing government securities
as a policy of “credit liberalisation” it may generally be lying with them and therefore, repos imply injection of
raised during inflation and lowered during a slowdown. liquidity in the banking system.
• Open market Operations: These are conducted by the RBI
by selling and buying government securities from banks. Composition of Banking System in
Generally, the RBI may sell government securities during India
inflation, so as to mop up excess funds lying with banks. The Indian banking system consists of commercial banks both
It may buy these during a period of slowdown to release in public and private sector, Regional Rural Banks (RRBs) and
liquidity in the banking system. co-operative banks.
Indian Banking Industry

Scheduled Unscheduled
Banks Banks

Scheduled Scheduled
Commercial Co-operative
Banks Banks

SBI & Public


Associates Sector
Rural Urban
National Banks Banks (PSBs) Co-operative Co-operative
(including IDBI) Banks Banks
Foreign Banks
Regional Rural Signal
Short term Long term Multi-state
Banks
Other Scheduled
State, district and SCARDBs &
Commercial Banks
primary level PCARDBs
Cooperative Banks
Regional Rural Banks (RRBs) incorporated in India and so have their head offices in India.
RRBs were set up with the objective of developing the rural Foreign banks, on the other hand, have been incorporated in
economy by providing credit and encouraging other productive foreign countries and have their head offices outside India.
activities in the rural areas.
Public sector banks sponsor RRBs, subscribing to the share Co-operative Banks
capital. The RRBs meet the credit needs of the weaker sections– These banks are so called because they have been organised
small and marginal farmers, artisans, small entrepreneurs, etc. under the provisions of the co-operative society’s law of the
The RRBs accumulated huge losses and fared badly in their states.
recovery of loans; the government allowed them to grant loans State co-operative Banks (SCB) : In terms of organisational
to non-priority sectors as well so as to improve their financial
set-up of the co-operative banking system, the state co-
position.
operative bank (SCB) in each state (co-operation being a state
Private Sector Banks subject) are at the apex level; at the district level are the central
These banks in India are of two kinds- Indian and foreign. co-operative banks (CCBs); and at the village level, there are
The Indian private sector banks are those which have been primary agricultural credit societies (PACS).
E-102 Money Supply and Indian Financial System

Scheduled and Non-Scheduled Banks RRBs and state co-operative banks are scheduled banks. The
non-scheduled banks are those which have not been included
The Reserve Bank of India has divided the banks as scheduled
in the Second Schedule of the RBI Act, 1934. While in 1960-
and non-scheduled banks. The scheduled banks are those which
61, there were 256 non-scheduled banks, most have been now
have a paid-up capital and reserves of an aggregate value of not
amalgamated with bigger banks.
less than Rs 5 lakh. All commercial banks–Indian and foreign,
Banking System in India
Indicators 2010 2011 2012 2013
Number of 163 163 169 151
Commercial Banks
Number of Branches 88,203 94,019 102,377 109,811
Aggregate Deposits ` 44928 billion (US$660 ` 52078 billion (US$770 ` 59091 billion (US$870 ` 67504.54 billion
billion) billion) billion) (US$1.0 trillion)
Bank Credit ` 32448 billion (US$480 ` 39421 billion (US$580 ` 46119 billion (US$660 ` 52605 billion (US$780
billion) billion) billion) billion)
Deposit as percentage 78% 78% 78% 79%
to GNP (at factor cost)

Credit Deposit Ratio 74% 76% 79% 79%

Narsimhan Committee on Banking Sector • Banks should classify their assets into four categories,
Reforms: namely-standard, sub-standard, doubtful and loss.
The major recommendations of the committee are as follows : • Banks should conform to BASEL norms and have a capital
• Banks should be free to open or close a branch, except adequacy ratio (CAR) of 8% by 1998.
in rural areas, where banking cannot depend on a bank’s • Priority sector lending should be brought down from 40%
choice, Also, there should be no further branch expansions. to 10%
• Nationalisation of banks should not take place any more. • There should be a four-tier banking structure (international,
• Private and foreign banks should be set up to promote national, regional and local).
competition.
Non performing assets:
• There should be a phased reduction of CRR and SLR
• Speedy computerization of banks. As per recommendations of the Narasimham Committee, it
• Setting up of special “ debt recovery tribunals” has been decided that credit facilities granted by banks will be
classified into performing and non-performing assets (NPA).
• Setting up of Asset Reconstruction of companies for not
performing assets (NPAs). NPA is a loan (whether term loan, cash credit, overdraft, or
bills discounted), which is in default for more than six months.
• A board for financial supervision should be set up.
In case of such assets, the income should be shown only on
• Banks should adopt provisioning norms to keep provisions
against bad loans in their books. receipt and not in the bank’s book on a due basis.

INSURANCE SECTOR
The Insurance sector was nationalised in two stages. General Insurance Corporation of
India (GIC)
Life Insurance Corporation of India
The GIC was formed in November 1972 upon the nationalisation
(LIC) of general insurance business. The 107 private companies
The LIC was established on first of September 1956 after operating in the field were grouped together into four :
nationalising the existing private insurance companies. LIC has (i) National Insurance Company,
a strong social security scheme which is funded by the Social (ii) United India Insurance Company,
Security Fund (SSF) set up and administrated by it since 1989- (iii) Oriental Insurance Company, and
90. The LIC runs three schemes supported by SSF - Janashree (iv) New India Assurance Company,
with GIC as the holding company. These companies can
Bima Yojana, Krishi Shramik Samajik Suraksha Yojana and
compete among themselves for all types of general insurance
Shiksha Sahayog Yojana. These schemes benefit persons and except aviation insurance and crop insurance which are the
families below the poverty line. monopoly of GIC.
Money Supply and Indian Financial System E-103

Insurance Regulatory and Some of the functions of the authority include:


• To protect the interest of and secure fair treatment to
Development Authority policy holders.
IRDA is a multimember nodal agency and is vested with • To bring about speedy and orderly growth of the
regulatory powers in respect of the insurance sector similar insurance industry.
to those vested in SEBI in respect of the capital markets and • To ensure speedy settlement of genuine claims and to
of RBI for the banking sector. The IRDA Act, 1999 cleared prevent frauds and malpractices.
the way for private sector entry into the insurance business.
• Promote fairness, transparency and orderly conduct in
The IRDA has powers to lay down prudential norms and financial markets dealing with insurance.
investment guidelines. It has ended the monopoly of LIC/
GIC in the insurance sector, by permitting private players to
enter, as recommended by Malhotra Committee.

COMMODITY FUTURES MARKET


India has a long history of commodity futures trading. A The MCX recorded the highest turnover in terms of value
commodity includes all types of goods. Futures trading is of trade during 2009, followed by National Commodity and
organised in such goods or commodities as is permitted by the Derivatives Exchange Ltd. (NCDEX) and National Multi-
government. A commodity exchange which is an association or Commodity Exchange of India Ltd. (NMCE) respectively.
a company or any corporate body organizing future trading in
commodities Forwards Market Commission (FMC)
A future contract is a type of ‘forward contract’ which means
FMC is a statutory body set up by the Forwards Contracts
a contract for the delivery of goods that is not a ready delivery
(Regulation) Act, 1952. It functions under the administrative
contract. A ready delivery contract provides delivery of goods
control of Department of Consumer Affairs, Ministry of
and payment of price either immediately or within a period not
Consumer Affairs, Food and Public Distribution. The
exceeding 11 days. The ready delivery contracts are known as
Commission is a Regulatory body set up to ensure financial
“spot” or “cash” contracts.
integrity, market integrity and protection and promotion of the
All contracts in commodities providing delivery of goods and interest of consumers.
payment of price after 11 days are forward contracts which may
be either “specific delivery contracts” or “ futures contracts”.
There are four commodity exchanges working in the country.
Foreign Portfolio Investments
They are NMCE (National Multi Commodity Exchange) - The government has allowed Foreign Institutional Investors
the oldest commodity exchange in the country which became (FIIs) to invest in Indian stock markets since September 1992.
operational in 2002, MCX (Multi Commodity Exchange), These FIIs are foreign pension funds, mutual funds, investment
NCDEX (National commodity and Derivatives Exchange) trusts, asset management companies (AMCs) and portfolio
and ICEX (Indian Commodity Exchange). ICEX is the managers. FIIs have generally been actively participating in
latest commodity exchange which becomes operational since the Portfolio Investment Scheme, although there are charges
November 27, 2009. India bulls of private sector and MMT that they have brought greater volatility to the Indian stock
of the public sector are the main promoter companies of this market. Net FII inflow at end of 2012 stood at US $ 31.01
exchange. billion (as per the economic Survey 2012-13). 1759 FIIs are
currently registered with SEBI.

NON-BANKING FINANCIAL INSTITUTIONS (NBFIS)


NBFIs work in similar ways as banks, but there are certain in view of the difficulties expressed by the NHB and EXIM
differences Bank, their aggregate borrowing limit has been enhanced to 11
According to the Economic Survey, there are four Institutions, times of their net owned funds (NOF) for one year (for NHB
namely: up to 30 September, 2012 and for EXIM Bank up to 31 March,
(i) Export Import Bank of India (EXIM Bank), 2012, subject to review.)
(ii) National Bank for Agriculture and Rural Development
(NABARD), Export-Import Bank of India (EXIM
(iii) National Housing Bank (NHB) and Bank)
(iv) Small Industries Development Bank of India (SIDBI). EXIM bank in India was established on January 1, 1982 for
These institutions are regulated by the RBI as all-India FIs. The financing, facilitating and promoting foreign trade in India.
outstanding of total resources mobilized at any point of time Besides, EXIM Bank also discharges duties of co-ordinating
by an FI, including funds mobilized under the ‘umbrella limit’, the activities of various financial institutions, providing
as prescribed by the RBI, should not exceed 10 times its net finances for export and imports of goods and services. Besides
owned funds as per its latest audited balance sheet. However, India, this bank also manages finances to third world countries
E-104 Money Supply and Indian Financial System

for export and import of goods and services. The Government It is also free to obtain loans in foreign currencies from foreign
of India wholly owns EXIM Bank of India. institutions. SIDBI will provide loans (both in Indian and
Foreign Currencies) under its ‘Single Window Service’.
National Housing Bank (NHB) While IDBI continues to be the single largest shareholder of
National Housing Bank was established in July 1988 as wholly SIDBI, the State Bank of India and LIC follow as the next two
owned subsidiary of RBI. NHB is the apex banking institution large share holders in that order.
providing finances for houses. The statutory mandate of
NHB covers promotional, developmental and regulatory National Bank for Agriculture and
aspects of housing finance with focus on developing a sound
housing finance system. NHB amended its Act called NHB
Rural Development (NABARD)
(Amendment) Act, 2000 which came into force on June 12, It is an apex depevlomental bank in India. Setting up of
2000. NHB has made a number of efforts to promote the NABARAD was recommended by the committee to review
supply of real resources like land and building material. arrangements for institutional credit for agriculture and rural
NHB has been permitted to mobilise resources by issuing development, set up by the RBI, under the chairmanship of Shri
bonds and debentures. NHB can obtains short term loan for B.Sivaraman. It was established in 1982 and its main focus has
18 months period from RBI and obtain long-term loan from been on the upliftment of rural India by increasing credit flow
National Housing Credit Fund constituted by RBI. It can also into agriculture and rural non-farm sector. It has been entrusted
obtain loans from various financial institutions of India and with matters concerning policy, planning and operations in the
abroad. It can also obtain loans in foreign currencies. Besides, field of credit for agriculture and other economic activities
NHB can accept long term deposits from Central Government in rural areas in India. NABARAD is active in developing
and other institutions. financial inclusion policy and is a member of the Alliance for
Financial Inclusion.
A major activity of NHB includes extending financial assistance
to eligible institutions in the housing sector by way of refinance Micro Finance
and direct finance. The NHB is the regulator and supervisor of Financial Inclusion is the delivery of financial services at
Housing Finance Companies (HFCs) in the country. an affordable cost to vast sections of the disadvantaged and
low income groups. It is crucial to innovate and provide
Small Industries Development Bank means to include the financially disadvantaged by ensuring
of India (SIDBI) access to financial services, and timely and adequate credit.
Financial inclusion can be described as the provision of
Small Industries Development Bank of India (SIDBI) was
established as wholly owned subsidiary of IDBI under the affordable financial services, like access to payments and
Small Industries Development Bank of India Act, 1989 as the remittance facilities, savings, loans and insurance services
principal financial institution for promotion, financing and by the formal financial system to those who are excluded.
development of industries in the small scale sector. SIDBI The Self-Help Group (SHG)-bank linkage programme
started its operations from April 2, 1990. It oversees the activity continued to be the main micro-finance model by which
of agencies which provide finances to small enterprises. It’s the formal banking system reaches micro-enterpreneurs
headquarter is situated at Lucknow. 5 Regional and 21 Branch (including farmers). Launched in 1992 as a pilot project,
Offices have also been started in different parts of the country. it has since proved its efficacy as a mainstream programme
All duties related to small enterprises which were performed for banking by the poor who mainly comprise the marginal
by IDBI, have been shifted to SIDBI. SIDBI provides farmers, landless labourers, artisans and craftsmen and
assistance to the small scale industrial sector in the country others engaged in small businesses like hawking and vending
through other institutions like State Financial Corporations in the rural areas. The main advantages of the programme are
(SFC), Commercial Banks, and State Industrial Development timely repayment of loans to banks, reduction in transaction
Corporations, etc. Besides share capital, SIDBI can increase costs both to the poor and the banks, doorstep “saving and
its resources by taking loans from the Government of India and credit” facility for poor and exploitation of the untapped
RBI. SIDBI can also obtain loans from Indian Capital market. business potential of the rural areas.
Money Supply and Indian Financial System E-105

Exercise - 1
1. Demand management to control inflation may lead to: 9. Bank rate refers to:
(a) increase in government expenditure (a) the rate at which banks lend to its prime borrowers
(b) increase in unemployment due to contraction of the (b) the maximum rate that banks can pay for deposit
economy (c) the rate at which banks rediscount bills with RBI
(c) expansion of credit (d) the rate of interest charged on inter-bank transactions
(d) creation of excess liquidity 10. Open market operation of RBI refers to:
2. Which of the following is not a function of the Securities (a) buying and selling of shares
and Exchange Board of India (SEBI)? (b) auctioning of foreign exchange
(a) Supervising the working of the Stock Exchanges (c) trading in securities
(b) Underwriting new capital issues (d) transaction in gold
(c) Regulating merchant banks and mutual funds 11. According to the Chakravarthy Committee, one of the
(d) promoting the development of a healthy capital principal causes affecting price stability in India is:
(a) existence of black money
market
(b) violent fluctuation in agricultural production
3. It is said that, in order to control inflation, foreign inflow (c) India’s precarious balance of payment position
needs to be sterlised. Sterlisation here refers to: (d) fiscal deficit
(a) ensuring that black money is accounted. 12. International capital flows have been uncertain and
(b) compliance with import-export regulations volatile, causing exchange rate and balance of payment
(c) ensuring that counterfeit currency does not enter crisis among developing countries. Among the various
circulation sources of external capital available to India which source
(d) withdrawing equivalent local currency to maintain a should you consider as the most preferable?
desirable rate of exchange (a) Foreign institutional investment
4. The first public sector bank in India which obtained (b) Portfolio investment
license for Internet Banking from RBI is- (c) External commercial borrowing
(a) Punjab National Bank (d) Foreign direct investment
(b) Oriental Bank of Commerce 13. Which of the following was described as the biggest ever
(c) Corporation Bank cash purchase of the shares of a bank in India?
(a) Purchase of Nedungadi Bank shares by Punjab
(d) State Bank of India
National Bank
5. Which of the following markets are independently
(b) Amalgamation of Global Trust Bank with Oriental
regulated by Forward Market Commission?
Bank of Commerce
(a) Mutual Funds (c) Merger of Centurion Bank of Punjab with HDFC
(b) Commodity Futures Market Bank
(c) Stock Market (d) Purchase of State Bank of India shares from RBI by
(d) Foreign Exchange Markets the Central Government
6. Inflation, in theory, occurs: 14. To know whether the rich are getting richer and the poor
(a) when the price of essential commodities outstrips getting poorer, it is necessary to compare
income (a) the wholesale price index over different periods of
(b) when money supply grows at a higher rate than time for different regions
GDP in real terms (b) the distribution of income of an idential set of
(c) when the exchange rate of a currency falls income recipients in different period of time
(d) when fiscal deficit exceeds balance of payment (c) the distribution of income of different sets of income
recipients at a point of time
deficit
(d) the availability of foodgrains among two sets of
7. Among the supply side measures to contain inflation is:
people, one rich and the other poor, over different
(a) postponing public expenditure
period of time
(b) mopping up excess liquidity through taxation
15. Which of the following statements correctly expresses the
(c) credit control measures of RBI difference between preference shares and equity shares?
(d) maintaining price levels through an effective PDS (a) equity shareholders have no voting right but
8. Which of the following is not true of the Regional Rural preference shareholders have voting rights
Banks (RRBs)? (b) preference shareholders have no have voting rights
(a) They provide direct loans to small and marginal but equity shareholders have voting rights
farmers; (c) preference shareholders have no right to profit
(b) They are co-sponsored by Reserve Bank of India; whereas equity shareholders have a right to profit
(c) They also perform other banking operations; (d) preference shareholders get exemption from taxes
(d) State Governments are share-holders in RRBs. while equity shareholders do not get any exemption
E-106 Money Supply and Indian Financial System

16. Which one of the following governmental steps has (a) Self Regulatory Organisations
proved relatively effective in controlling the double digit (b) Small Revenue Operators
rate of inflation in the Indian economy during recent (c) Securities Roll-back Operators
years? (d) Securities Regulatory Organisations
(a) enhanced rate of production of all consumer goods 23. Debentures and Equity differ in?
(b) streamlined public distribution system (a) Debentures are bonds confirming that money has
(c) pursuing an export oriented strategy
been borrowed; equity is a shareholder’s voting
(d) containing budgetary deficit and unproductive
rights in proportion to his shareholding
expenditure
(b) An equity shareholder cannot withdraw his
17. Which one of the following is not a feature of “Value
investment but debenture holder can withdraw his
Added Tax” ?
money.
(a) It is multi-point destination-based system of taxation.
(c) Equity shares have greater risk compared to
(b) It is a tax levied on value addition at each stage of
debentures which have fixed interest on the amount
transaction in the production distribution chain.
paid.
(c) It is a tax on the final consumption of goods or
(d) Both have the right to vote irrespective of the size of
services and must ultimately be borne by the
their holdings. Debentures are of lower value than
consumer.
equity
(d) It is basically a subject of the central government
24. How do we define the terms bull and bear with regard to
and the state governments are only a facilitator for
stock markets?
its successful implementation.
(a) A bull is an optimistic operator who first buys and
18. Which one of the following statements is an appropriate
then sells shares in expectation of the price going
description of deflation?
up; a bear is a pessimistic market operator who sells
(a) It is a sudden fall in the value of a currency against
the shares in expectation of buying them back at a
other currencies
lower price.
(b) It is a persistent recession in both the financial and
(b) There is nothing significantly different as both
real sectors of economy
operate in the capital market.
(c) It is a persistent fall in the general price level of
(c) Bull is one who first sells a share and then buys it
goods and services
at a lower price; bear means one who first buys and
(d) It is a fall in the rate of inflation over a period of then sells it in expectation of prices going up.
time
(d) A bull is ready to buy any share; a bear only deals
19. A rapid increase in the rate of inflation is sometimes
in government securities.
attributed to the “base effect”. What is “base effect”?
25. What do you understand by the term “Jobber”?
(a) It is the impact of drastic deficiency in supply due to
(a) A stock dealer who deals directly with the public for
failure of crops
which he gets a commission
(b) It is the impact of the surge in demand due to rapid
(b) A broker who buys and sells on behalf of his clients
economic growth
(c) A stock dealer who is a member of a stock exchange
(c) It is the impact of the price levels of previous year
and deals with the public only through the medium
on the calculation of inflation rate
of brokers
(d) None of the statements (a), (b) and (c) given above
(d) A stock broker who deals only with shares and not
is correct in this context
any other form of securities
20. The basic aim of Lead Bank Scheme is that:
26. What are “Futures”?
(a) big banks should try to open offices in each district
(a) Securities or commodities sold or bought with the
(b) there should be stiff competition among the various
assumption of delivery at a later stage
nationalized banks
(b) Shares and debentures bought at a cheaper rate and
(c) individual banks should adopt particular districts for
sold later at high rates
intensive development
(c) Particular types and quantities of securities and
(d) all the banks should make intensive efforts to
com-modities bought for selling at higher prices
mobilize deposits
(d) None of these
21. The International Development Association, a lending
27. An outward shift in the demand for money, other things
agency, is administered by the:
being equal should lead to:
(a) International Bank for Reconstruction and
(a) A lower interest rate but the same quantity of money
Development (b) A higher interest rate but the same quantity of
(b) International Fund for Agricultural Development money
(c) United Nations Development Programme. (c) A higher quantity of money but lower interest rates
(d) United Nations Industrial Development Organization (d) A higher quantity of money but the same interest
22. The acronym SRO, being used in the capital market for rate
various market participants, stands for which one of the 28. A rise in ‘SENSEX’ means:
following? (a) a rise in prices of shares of all companies registered
with Bombay Stock Exchange
Money Supply and Indian Financial System E-107

(b) a rise in prices of shares of all companies registered 35. Open market Operation by RBI are the part of its :
with National Stock Exchange (a) Fiscal Policy (b) Credit Policy
(c) an overall rise in prices of shares of group up (c) Income Policy (d) Monetary Policy
companies registered with Bombay Stock Exchange 36. Which of the following statements is not correct regarding
(d) a rise in prices of shares of all companies belonging the ‘Banking Sector’ of India?
to a group of companies registered with Bombay (a) At present there are 26 Nationalized Banks in India.
Stock Exchange
(b) Foreign Banks and Regional Rural Banks do not
29. Which of the following would best describe the role of a
come under the category of Scheduled Commercial
merchant bank?
Banks.
(a) To monitor the activities of publicly-list companies
(b) To act as banker and financial agent to the federal (c) Banks have the freedom to regulate their own
government Savings Bank Deposit interest rates.
(c) To provide borrowing and leading services, (d) Narsimham Committee is related to Banking Sector
primarily to the business sector reforms.
(d) To provide loans to the corporate sector through
37. The concept of ‘Universal Banking’ was implemented in
receipts of insurance premiums
30. Which action by the Reserve Bank would stimulate the India on the recommendations of:
economy in an economic downturn? (a) Abid Hussain Committee
(a) Selling government securities in the cash market (b) R H Khan Committee
(b) Buying government securities in the cash market (c) S Padmanabhan Committee
(c) Selling foreign currencies on the foreign exchange (d) Y H Malegam Committee
market 38. Which among 1the following is considered to be a part of
(d) Buying foreign currencies on the foreign exchange Shadow Banking in India?
market (a) Business Correspondents
31. Which of the following could be a cause of demand (b) Bankassurance Providers
inflation?
(c) Non-Banking Financial Companies
(a) An increase in the cost of labour
(d) Private Banks
(b) An increase in domestic interest rates
39. Which among the following is the only correct statement?
(c) An increase in the level of consumer spending
(a) Money market meets long term financing needs
(d) An increase in import prices, resulting from a
(b) Recession in industrial sector in India is normally
depreciating dollar
due to fall in exports
32. Other things being equal, what would be the impact of an
(c) Ways and means advances given by RBI are nowhere
increase in net capital inflow on the Indian currency?
related to State’s revenue
(a) There would be an increase in the demand for the
(d) Exchange rate is fixed by RBI
American dollar and an appreciation of the currency.
40. Bring out the only incorrect statement:
(b) There would be a decrease in the demand for the
(a) Reverse Repo operation by RBI aims at injecting/
rupees (`) and a depreciation of the currency.
increasing liquidity
(c) There would be an increase in the supply of the
(b) SDR refers to special drawing rights
American dollar and an appreciation of the currency.
(c) Rupee appreciation results in decrease in imports
(d) There would be a decrease in the supply of the
(d) Increase in inflation rate leads to decline in real
American dollar and a depreciation of the currency.
interest rate
33. If an exporter earns money and deposits that with RBI,
41. Which among the following correctly defines the Net
what will be the ultimate impact on country’s money
Interest Income?
supply?
(a) Money supply will increase (a) Interest earned on advances
(b) Money supply will decrease (b) Interest earned on investments
(c) Money supply will remain unaltered (c) Total interest earned on advances and investment
(d) Money supply will depend upon the current exchange (d) Difference between interest earned and interest paid
rate 42. In which among the following types comes the Interest
34. ‘SHG Bank Linkage Programme’ is a programme which Rate Risk?
encourages India’s banks to lend to self-help groups (a) Credit risk (b) Market risk
(SHGs) composed mainly of poor women, this has (c) Operational risk (d) All the above categories
evolved into an important Indian tool for microfinance. 43. When there is an inflationary trend in the economy, what
This programme was initiated by ? would be trend in the pricing of the Bank Products?
(a) Reserve Bank of India (RBI) (a) Increasing Trend
(b) Agricultural Refinance and Development
(b) Decreasing Trend
Corporation (ARDC)
(c) National Bank for Agriculture and Rural (c) Constant Trend
Development (NABARD) (d) There is no relevance of the inflation in pricing of
(d) Non-Banking Finance Companies (NBFC) the Banking Products
E-108 Money Supply and Indian Financial System

44. From time to time, which among the following body 52. Which among the following gives a precise definition of
publishes the “Exchange Control Manual” in context “ Arbitrage” in Financial World?
with the Foreign Exchange in India? (a) To profit from an existing discrepancy among prices,
(a) Foreign Trade Promotion Board exchange rates, and/or interest rates on Different
(b) Department of Commerce Markets without risk of these changing
(c) Reserve Bank of India (b) to profit from an existing discrepancy among prices,
(d) SEBI exchange rates, and/or interest rates on Same market
45. As per Section 24 (2A) of Banking Regulation Act without risk of these changing
1949, every banking company in India has to maintain (c) To profit from an existing discrepancy among
equivalent to an amount which shall not at the close of prices, exchange rates, and/or interest rates on new
the business on __________ be less than 25% of the total techniques or products in same market.
of its net demand and time liabilities, which is known as (d) All of above
SLR. 53. “A statement of estimated receipts and expenditures
called annual Financial Statement (Budget) has to be
Which among the following is the correct option?
placed before parliament for each financial year.” The
(a) Any Day (b) Any Week above provision has been enshrined in which among the
(c) Any Fortnight (d) Any Month following articles of Constitution of India?
46. Which among the following is NOT Tier I Capital? (a) Article 110 (b) Article 111
(a) Paid up Capital (c) Article 112 (d) Article 113
(b) Statutory Reserves 54. Capitalism is based upon “Laissez-faire system”. What is
“Laissez-faire system”?
(c) Revaluation Reserves
(a) No governmental intervention
(d) Investment Fluctuation Reserves (b) Maximum governmental intervention
47. For which of the following, the Reserve Bank of India has (c) Role of market forces is minimum
stipulated a maximum Capital Adequacy Requirements in (d) Limited government Intervention
India? 55. Which among the following is true about “deficit
(a) Private Sector Banks financing”?
(b) Banks that Undertake Insurance Business. (a) Public expenditure in excess of public revenue
(c) Local Area Banks (b) Public revenue in excess of public expenditure
(d) Scheduled Commercial Banks (c) New currency replaced by old currency
48. Which one of the following Public Sector Bank’s emblem (d) None of above
figures a dog and the words ‘faithful friendly, in it? 56. An excise duty is a charge on which of the following?
(a) Punjab National Bank (a) Export of goods
(b) Syndicate Bank (b) Production of goods
(c) Oriental Bank of Commerce (c) Production or sale of goods
(d) State Bank of India
(d) Consumption of good
49. Which among the following is NOT an instrument of
57. “Bad money (if not limited in quantity ) drives good
qualitative control in India ?
money out of circulation.” The above statement is from
(a) Regulation of the Consumer Credit
which among the following laws?
(b) Rationing of the Credit
(a) Keynes’ law (b) Wagner’s law
(c) Margin Requirements
(c) Gresham’s law (d) Grimm’s law
(d) Variable Costs and Reserves
58. Which among the following is a correct definition of
50. Which among the following would most likely follow if
currency drain?
the Reserve Bank of India effects selling of the securities?
(a) A currency drain is an export of the domestic
(a) The cash resources at the disposal of the commercial
currency
banks increase.
(b) A currency drain is an increase in currency held
(b) The cash resources at the disposal of the commercial
outside the banks.
banks get diminished.
(c) A currency drain is the currency holding by a
(c) The cash resources of the commercial banks remain
parallel economy
unchanged.
(d) None of these
(d) None of the above.
59. Which among the following is a correct definition of
51. The Laffer curve is the graphical representation of:
Fiduciary Issue of notes?
(a) The relationship between tax rates and absolute
(a) The issue of currency notes without metallic
revenue these rates generate for the government.
(b) The inverse relationship between the rate of backing.
unemployment and the rate of inflation in an (b) The issue of currency notes with metallic backing.
economy. (c) The issue of currency notes with partial metallic
(c) The inequality in income distribution. backing.
(d) The relationship between environmental quality and (d) The issue of currency notes with proportional
economic development. metallic backing.
Money Supply and Indian Financial System E-109

60. Which among the following is correct about the Vostro 62. The most active segment of the Money Market in India is
Account? which one of the following?
(a) It is an account maintained by a foreign bank with a (a) Call Money / Notice Money Market
Bank in India in Indian Rupees. (b) Repo / Reverse Repo
(b) It is an account maintained by an Indian bank with a (c) Commercial Paper (CP)
Bank in Foreign country in Foreign Currency. (d) Certificate of Deposit (CD)
(c) It is an account maintained by a Indian Bank Branch 63. An increase in the Bank Rate generally indicates that the :
with a Foreign Bank in Foreign Currency. (a) market rate of interest is likely to fall
(d) It is an account maintained by a Foreign Bank (b) Central Bank is no longer making loans to
Branch with a Indian Bank in Foreign Currency. commercial banks
61. What do we call an arrangement whereby an issuing (c) Central Bank is following an easy money policy
Bank at the request of the Importer (Buyer) undertakes (d) Central Bank is following a tight money policy
to make payment to the exporter (Beneficiary) against 64. What does venture capital mean?
stipulated documents? (a) A short-term capital provided to industries
(a) Bill of Exchange (b) A long-term start-up capital provided to new
(b) Letter of Exchange entrepreneurs
(c) Letter of Credit (c) Funds provided to industries at times of incurring
losses
(d) Bill of Entry
(d) Funds provided for replacement and renovation of
industries
E-110 Money Supply and Indian Financial System

Exercise - 2
Statement Based MCQ Which of these statements are correct?
(a) 1, 2 and 3 (b) 1 and 4
(c) 2, 3 and 4 (d) 1, 2, 3 and 4
1. Consider the following statements :
6. Consider the following statements:
The function of the Reserve Bank of India does not
1. The repo rate is the rate at which other banks borrow
include:
from the Reserve Bank of India.
1. Open market operations
2. A value of 1 for Gini Coefficient in a country implies
2. Monitoring revenue collection
that there is perfectly equal income for everyone in
3. Supervising non-banking finance companies
4. Review of public expenditure its population.
Which of the statement/s given above is/are correct? Which of the statements given above is/are correct?
(a) 1 only (b) 2 and 4 (a) 1 only (b) 2 only
(c) 1, 3 and 4 (d) 1, 2, 3 and 4 (c) Both 1 and 2 (d) Neither 1 nor 2
2. Consider the following statements with regard to 7. Consider the following statements:
Statutory Liquidity Ratio (SLR) : 1. Sensex is based on 50 of the most important stocks
1. To meet SLR, Commercial banks can use cash only. available on the Bombay stock Exchange (BSE).
2. SLR is maintained by the banks with themselves. 2. For calculating the Sensex, all the stock are assigned
3. SLR restricts the banks leverage in pumping more proportional weightage.
money into the economy. 3. New York Stock Exchange is the oldest stock
Which of the statements given above is/are correct? exchange in the world.
(a) 1, 2 and 3 (b) 1 and 3 Which of the statements given above is/are correct?
(c) 2 and 3 (d) only 2 (a) 2 only (b) 1 and 3
3. Consider the following statements. (c) 2 and 3 (d) none
1. The National Housing Bank, the apex institution of 8. Name the 3 commodities in the Wholesale Price Index.
housing finance in India, was set up as a wholly- 1. Primary articles
owned subsidiary of the Reserve Bank of India. 2. Fuel, power, light and lubricants
2. The Small Industries Development Bank of India 3. Manufactured products
was established as a wholly-owned subsidiary of the 4. Food articles and industrial raw materials
Industrial Development Bank of India. (a) 1, 2 and 3 (b) 2, 3 and 4
Which of the statement given above is/are correct? (c) 1, 3 and 4 (d) 1 and 3
(a) 1 only (b) 2 only 9. Which of the following measures would result in an
(c) Both 1 and 2 (d) Neither 1 nor 2 increase in the money supply in the economy?
4. Why is the offering of “teaser loans’’ by commercial 1. Purchase of government securities from the public
banks a cause of economic concern ? by the Central Bank
1. The teaser loans are considered to be an aspect of 2. Deposit of currency in commercial banks by the
sub-prime lending and banks may be exposed to the public
risk of defaulters in future. 3. Borrowing by the government from the Central
2. In India, the teaser loans are mostly given to Bank
inexperienced entrepreneurs to set up manufacturing 4. Sale of government securities to the public by the
or export units. Central Bank
Which of the statements given above is/are correct? Select the correct answer using the codes given below :
(a) 1 only (b) 2 only (a) 1 only (b) 2 and 4
(c) Both 1 and 2 (d) Neither 1 nor 2 (c) 1 and 3 (d) 2, 3 and 4
5. With reference to the Indian Public Finance, consider the 10. A company undertakes a public issue which means that
following statements: _____ .
1. External liabilities reported in the Union Budget are 1. Shares of the company will be issued only through
based on historical exchange rates public sector organizations like banks/central
2. The continued high borrowing has kept the real financial institutions etc.
interest rates high in the economy 2. Shares of the company will be issued to general
3. The upward trend in the ratio of Fiscal Deficit of public through the primary market.
GDP a recent years has a adverse effect on private 3. means some stakeholders/promoters are leaving
investment the company and wish to sell their stock to general
4. Interest payments is the single largest component public.
of the non-plan revenue expenditure of the Union (a) 1 only (b) 2 only
Government (c) 3 only (d) All 1, 2 and 3
Money Supply and Indian Financial System E-111

11. What is ‘Future Trading’? 1. Cutting the frequency of Open Market Operations
1. It is a trade between two stock exchanges wherein 2. Cutting the Cash Reserve Ratio
it is decided to purchase stocks of each other on a 3. Cutting the Repo and Reverse Repo rates
fixed price throughout the year Choose the correct options:
2. It is an agreement between two parties to buy or sell (a) Only 1 (b) Only 1 & 2
an underlying asset in the future at a predetermined (c) Only 2 & 3 (d) 1, 2 & 3
price 18. Which one among the following is an appropriate
3. It is an agreement between stock exchange that they description of deflation?
will not trade in stocks of each other in future or for (a) it is a sudden fall in the value of a currency against
a predetermined period of time other currencies
(a) 1 only (b) 2 only (b) It is a persistent recession in the economy
(c) 3 only (d) All 1, 2 and 3 (c) It is a persistent fall in the general price level of
12. Brent index is associated with : goods and services
(1) crude oil prices (2) copper future prices (d) It is fall in the rate of inflation over a period of time
(3) gold future prices (4) shipping rate index 19. Consider the following statements:
Which of the following is true? 1. Not all banks come under the regulation of Right to
(a) Only 1 (b) Only 2 Information Act
(c) Only 3 (d) Only 4 2. Not all banks come under the Schedule 2 of RBI
13. Which of the following statements is/ are correct? Which among the above statements is / are correct?
1. NIFTY is based upon 50 firms in India. (a) Only 1 is correct
2. NIFTY is governed and regulated by the Reserve (b) Only 2 is correct
Bank of India. (c) Both 1 and 2 are correct
3. NIFTY does not trade in mutual funds. (d) Neither 1 nor 2 is correct
Select the correct answer using the codes given below : 20. Consider the following statements regarding the Office of
(a) Only 1 (b) Only 2 Economic Advisor (OEA):
(c) Only 3 (d) 1 and 3 1. It is attached to the Ministry of Finance.
14. Which of the following measures should be taken when
2. The weekly compilation and Publication of
an economy is going through in inflationary pressures?
Wholesale Price Indices (WPI) is done by the Office
1. The direct taxes should be increased.
of Economic Advisor.
2. The interest rate should be reduced.
Which of the statement(s) given above is/are correct?
3. The public spending should be increased.
(a) Only 1 (b) Only 2
Select the correct answer using the codes given below:
(c) Both 1 and 2 (d) Neither 1 nor 2
(a) Only 1 (b) Only 2
21. Consider the following statements regarding Non-
(c) 2 and 3 (d) 1 and 2
15. Consider the following statements: Banking Finance Companies (NBFCs):
1. High growth will lead to inflation. 1. NBFCs can also engage in Micro-Finance Activities.
2. High growth will lead to deflation. 2. Housing-finance companies form a distinct sub-
Which of the statements given above is/are correct? group of the NBFCs.
(a) Only 1 (b) Only 2 3. The deposit insurance facility of the Deposit
(c) Both 1 and 2 (d) Neither 1 nor 2 Insurance and Credit Guarantee Corporation is not
16. Which of the following are the provisions of SARFAESI available for NBFC depositors.
Act which enables banks to reduce their non-performing Which among the above statement(s) is/are not correct?
assets (NPAs)? (a) Only 1 (b) 1 and 3
1. Enforcement of Security Interest by secured creditor (c) All of there (d) None of these
(Banks/ Financial Institutions). 22. Which among the following are the ‘Credit-Rating
2. Transfer of non- performing assets to asset Agencies’ of India?
reconstruction company which will then dispose of 1. CRISIL 2. CARE
those assets and realise the proceeds. 3. ICRA 4. ONICRA
3. To provide a legal framework for securitization of (a) 1 and 3 (b) 1, 2 and 3
assets. (c) 1, 3 and 4 (d) All of them
4. Assisting banks in making the credibility track 23. Which among the following are the wholly/partly owned
record of customers under Credit Information subsidiaries of the Reserve Bank of India (RBI)?
Bureau of India (CIBIL). 1. Deposit Insurance and Credit Guarantee Corporation
(a) 1 and 2 (b) 1, 2 and 3 (DICGC)
(c) 2, 3 and 4 (d) 1, 2, 3 and 4 2. National Housing Bank (NHB)
17. Recently, one of the well known market analysts made 3. National Bank for Agriculture and Rural
this statement: “We expect the Reserve Bank of India to Development (NABARD)
continue to ease liquidity” Which among the following 4. Bharatiya Reserve Bank Note Mudran Private
instruments can be used by RBI to continue to ease Limited (BRBNMPL)
liquidity? (a) 1 and 4 (b) 1, 2 and 4
(c) 2, 3 and 4 (d) All of them
E-112 Money Supply and Indian Financial System

24. Consider the following statements: Which among the above statements is/ are correct?
1. Increase in deposit rate results in higher savings (a) Only 1 is correct
2. Increase in deposit rate results in fall in credit off (b) Only 2 is correct
takes (c) Both 1 and 2 are correct
3. Increase in deposit rate results in increase in (d) Neither 1 nor 2 is correct
investment 30. Consider the following:
Which among the above statements is / are correct? 1. Commercial Banks 2. Central Bank of India
(a) 1 & 2 (b) 1 & 3 3. Government
(c) 1, 2, 3 (d) Only 1 Which among the above can create money?
25. Consider the following statements: (a) Only 1 (b) Only 1 & 2
1. As per the Basel II accords, Banks have to maintain (c) Only 3 (d) All 1 2 & 3
a minimum 8% CRAR 31. The Non-Votable charges or ‘Charged Expenditure’
2. Out of 8% Banks have to keep a Tier I Capital of included in the Union Budget include :
5% 1. Salary and Allowances of the Presiding Officers of
3. In India, the Banks are required to keep the CAR of the houses of Parliament.
9% at least. 2. Salary and Allowances of the Judges of the Supreme
Which among the above statements is / are correct? Court and High Courts.
(a) Only 1 3. Salary and Allowances of the CAG.
(b) Only 1 & 2 4. Pension of the retired Judges of the Supreme Court.
(c) Only 1 & 3 5. Pension of the retired Judges of High Courts.
(d) All are correct statements Select the option(s) which is/are correct?
26. Consider the following statements: (a) 1 and 2 (b) 1, 2 and 3
1. In Minimum Reserve System, RBI had to keep a (c) 1, 2, 3 and 4 (d) All of them
minimum reserve of Gold worth ` 115 Crores and 32. Which of the following are among the taxes levied
rest in Indian Rupees. exclusively by the Central Government and are
2. The Minimum Requirement of Foreign Securities mentioned in the Union List of the Seventh Schedule of
was diluted when Minimum Reserve System was
the Constitution of India?
launched.
1. Corporation Tax
Which among the above statement is / are correct?
2. Taxes on advertisement in the newspapers
(a) Only 1 is correct
3. Taxes on Agricultural income
(b) Only 2 is correct
4. Taxes on consumption/sale of electricity
(c) Both of them are correct
(a) Only 2 (b) Only 4
(d) Both of them are incorrect
(c) 1 and 2 (d) 3 and 4
27. Consider the following steps:
33. Consider the following taxes:
1. Banking Department gets currency from the Issue
1. VAT paid during purchase of a tyre tube for a
Department
2. Government puts the currency in circulation vehicle
3. Central Government incurs a deficit in its Budget 2. Service Tax paid while making payments of dinner
4. Government Sells Treasury Bills to Banking in a restaurant
Department of RBI 3. Duty paid while importing machinery from abroad
Which among the following is the correct order of the which among the above are “direct taxes”?
above steps? (a) Only 1 (b) 1 & 2
(a) 1 2 3 4 (b) 2 3 4 1 (c) 1, 2 & 3 (d) None of them
(c) 3 4 1 2 (d) 4 3 2 1 34. Consider the following statements:
28. Consider the following statements: 1. Deficit financing essentially involves public
1. In Narrow Banking, Banks just accept deposits and expenditure in excess of public revenue
provide loans. 2. In Deficit financing, the new currency replaces the
2. In Narrow Banking, there is rarely Asset Liability old currency
Mismatch. Which among the above statements is / are correct?
Which among the above statements is / are correct? (a) Only 1 is correct (b) Only 2 is correct
(a) Only 1 is correct (c) Both are correct (d) Both are incorrect
(b) Only 2 is correct 35. Indian Currency Printing Institutions are located in which
(c) Both 1 and 2 are correct of the following places?
(d) None of them is correct (1) Nasik (2) Hoshangabad
29. Consider the following statements: (3) Hyderabad (4) Aurangabad
1. In Banks, the “Staff Security Deposits” come under (5) Devas
Time Liabilities Select the correct option:
2. In Banks. “Cash certificates” come under the (a) 1, 2 and 4 (b) 1, 2, 3 and 4
Demand Liabilities (c) 1, 2, 3 and 5 (d) All of the above
Money Supply and Indian Financial System E-113

36. Consider the following statements: 43. The Reserve Bank of India regulates the commercial
1. There is almost no speculation in the G-sec market banks in matters of:
2. The Investors in the G-Sec Market are predominantly 1. liquidity of assets 2. branch expansion
the institutions 3. merger of banks 4. winding-up of banks
Which among the above statements is / are correct? Select the correct answer using the codes given below.
(a) Only 1 is correct (a) 1 and 4 only (b) 2, 3 and 4 only
(b) Only 2 is correct (c) 1, 2 and 3 only (d) 1, 2, 3 and 4
(c) Both 1 & 2 are correct 44. Which of the following grants / grant direct credit
(d) Neither 1 nor 2 is correct assistance to rural households?
37. Consider the following: 1. Regional Rural Banks
1. Change in the Reserve Requirements
2. National Bank for Agriculture and Rural
2. Taxation
Development
3. Government Spending
Which among the above terms are closest to the Fiscal 3. Land Development Banks
policy ? Select the correct answer using the codes given below.
(a) Only 1 & 2 (b) Only 2 & 3 (a) 1 and 2 only (b) 2 only
(c) Only 1 (d) Only 1 & 3 (c) 1 and 3 only (d) 1, 2 and 3
38. Consider the following statements in context with 45. When the Reserve Bank of India reduces the Statutory
Treasury Bills: Liquidity Ratio by 50 basis points, which of the following
1. They are issued by Government of India on behalf is likely to happen?
of Reserve Bank of India (a) India’s GDP growth rate increases drastically
2. They are mostly for Short term borrowings (b) Foreign Institutional Investors may bring more
3. Treasury Bills can not be purchased by any person capital into our country
resident of India (c) Scheduled Commercial Banks may cut their lending
Which among the above is / are correct ? rates
(a) All are correct (b) 2 & 3 are correct (d) It may drastically reduce the liquidity to the banking
(c) Only 2 is correct (d) Only 3 is correct system
39. With reference to inflation in India, which of the following 46. In the context of Indian economy, which of the following
statements is correct? is/are the purpose/purposes of ‘Statutory Reserve
(a) Controlling the inflation in India is the responsibility
Requirements’?
of the Government of India only
1. To enable the Central Bank to control the amount
(b) The Reserve Bank of India has no role in controlling
the inflation of advances the banks can create
(c) Decreased money circulation helps in controlling 2. To make the people’s deposits with banks safe and
the inflation liquid
(d) Increased money circulation helps in controlling the 3. To prevent the commercial banks from making
inflation excessive profits
40. Consider the following liquid assets: 4. To force the banks to have sufficient vault cash to
1. Demand deposit with the banks meet their day-to-day requirements
2. Time deposit with the banks Select the correct answer using the code given below.
3. Savings deposit with the banks (a) 1 only (b) 1 and 2 only
4. Currency (c) 2 and 3 only (d) 1, 2, 3 and 4
The correct sequences of these assets in the decreasing 47. Consider the following statement regarding concept of
order of liquidity is: money:
(a) 1-4-3-2 (b) 4-3-2-1 1. M1: Money with the Public (currency notes and
(c) 2-3-1-4 (d) 4-1-3-2 coins) + Demand deposits of banks (on current and
41. Convertibility of rupee implies: saving bank accounts) + Other demand deposits
(a) being able to convert rupee notes into gold with RBI. It is highly liquid and banks will not be
(b) allowing the value of rupee to be fixed by market able to run their lending programmes on this basis.
forces
2. M2: M1 + Saving bank deposits with Post-offices.
(c) freely permitting the conversion of rupee to other
3. M3: M2 + Term deposits with the bank.
currencies and vice versa
(d) developing an international market for currencies in 4. M4: M3 + All deposits of Post-offices.
India Which among the following is correct?
42. In the context of Indian economy, consider the following (a) 1, 2, 3, 4 (b) 1, 2, 3
pairs: (c) 1, 3, 4 (d) 1, 2, 4
Term Most Appropriate description 48. The _____ is the total amount of monetary assets available
1. Melt down Fall in stock prices in an economy at a specific time.
2. Recession Fall in growth rate 1. Money Supply 2. Money Stock
3. Slow down Fall in GDP 3. Money Demand 4. Stock in Demand
Which of the pairs given above is/are correctly matched? Choose the correct answer from the above code:
(a) 1 only (b) 2 and 3 (a) 1 and 2 (b) 3 and 4
(c) 1 and 3 (d) 1, 2 and 3 (c) 1 and 3 (d) 2 and 4
E-114 Money Supply and Indian Financial System

49. Consider the following statement: 56. The definition of Wholesale Price Index (WPI) is as
1. Inflation is a decrease in price of goods follows:
2. Inflation is caused by excess demand in economy 1. The WPI is a weighted average of indices covering
3. Inflation results from excessive increase in the 676 commodities, which are traded in primary,
money supply manufacturing and fuel and power-sectors.
4. Inflation effects all segments of the economy 2. It is the retail price average of a basket of goods and
Which among the following statement is/are incorrect? services directly consumed by the people.
(a) 1 only (b) 2 only Choose the correct definition.
(c) 3 only (d) 4 only (a) 1 only (b) 2 only
50. Consider the following statement regarding inflation: (c) 1 and 2 (d) None of the Above
1. Inflation is beneficial for senior citizen 57. What is the difference between Inflation and Deflation?
2. Inflation is beneficial for fixed income citizen 1. Inflation is an increase in price of goods while
3. Inflation is beneficial for borrowers Deflation is that state in which the value of money
Choose the person who gets the benefit of inflation. rises and the price of goods and services falls.
(a) 1 only (b) 1 and 2 2. Deflation is an increase in price of goods while
(c) 2 only (d) 3 only Inflation is that state in which the value of money
51. Which among the followings is the type of inflation? rises and the price of goods and services falls.
1. Demand Pull Inflation 3. Inflation is that state in which the value of money
2. Cost Push Inflation rises and the price of goods and services falls while
3. Stagflation deflation is an increase in price of goods.
4. Hyperinflation Choose the correct difference between Inflation and
Choose the correct type. Deflation.
(a) 1, 2, 3, 4 (b) 1, 2 (a) 1 only (b) 2 only
(c) 3, 4 (d) 1, 4 (c) 3 only (d) 1, 2, 3
52. What is the main cost-push factor in India? 58. The state of deflation appears in the economy due to
1. Problem of hoarding by traders and black marketeers various reason:
2. Taxation which gives the traders an opportunity to 1. When the Government withdraws money from
circulation.
raise the prices of goods, the proportion of which is
2. When Government imposes heavy direct taxes or
often more than the levy of taxes
takes heavy loans from the public
3. Administered Prices
3. When the Central Bank buys the securities in open
4. Hike in Oil Prices
market
Choose the correct code.
Choose the correct reason.
(a) 1, 2, 3 (b) 1, 2, 3, 4
(a) 1 and 2 (b) 2 and 3
(c) 1, 2 (d) None of the Above
(c) 1 and 3 (d) 1, 2, 3
53. How is the price level measured?
59. What are the measures of checking deflation?
1. Wholesale Price Index 1. Increasing money supply
2. Consumer Price Index 2. Promote credit creation by the banks.
3. Gross Domestic Product (GDP) Deflator 3. Curtailment in taxes so as to increase the purchasing
4. Business Price Index power of the people.
Choose the correct code. Choose the correct measure.
(a) 1, 2, 3, 4 (b) 1, 2, 3 (a) 1 only (b) 2 only
(c) 1, 2 (d) 1 (c) 3 only (d) 1, 2, 3
54. Consider the following statement: 60. Which among the following is the type of organised
1. Increase in private expenditure sector of Indian money market?
2. Increase in exports 1. Call money market
3. Increase in taxation 2. Treasury Bill Market
4. Rapid growth of population 3. Commercial Bill Market
Choose the factor that does not cause an increase in 4. Collateral loan market
demand of goods and services. Choose the correct code.
(a) 1 only (b) 2 only (a) 1 and 2 (b) 3 and 4
(c) 3 only (d) 4 only (c) 1, 2, 3, 4 (d) None of the Above
55. What are the causes of inflation? 61. Consider the following statement:
1. Increase in demand for goods & services 1. Ad hoc treasury bills are sold to the banks and public
2. Decrease in the supply of goods & services and are freely marketable.
3. Decrease in demand for goods & services 2. Regular treasury bills are not sold to the banks and
4. Increase in the supply of goods & services the general public, and are not marketable.
Choose the correct code. Choose the correct code.
(a) 1 and 2 (b) 3 and 4 (a) 1 only (b) 2 only
(c) 1 and 4 (d) 2 and 3 (c) 1 and 2 (d) None of the Above
Money Supply and Indian Financial System E-115

62. Certificate of Deposit (CD) and Commercial Paper were 69. Who all are the main players in the Indian capital market?
introduced by a Bank in March 1989: 1. Insurance companies
1. Reserve Bank of India 2. Development Finance Institutions (DFI)
2. State Bank of India 3. Non-Banking Finance Companies, (NBFCs)
3. HDFC Bank 4. Non-Banking Financial Institutions.
4. ICICI Bank Choose the correct option.
Choose the name of the Bank. (a) 1, 2, 3, 4 (b) 1, 2, 3
(a) 1 only (b) 2 only (c) 1, 2, (d) 1
(c) 3 only (d) 4 only 70. Choose the type of capital market:
63. Which among the following are the Nationalised Banks? 1. Securities Market
1. Bank of India 2. Financial institutions
2. Punjab National Bank 3. Gill-edged market
3. Canara Bank 4. Incorporated securities
4. United Commercial Bank Which among the following code is correct?
Choose the code. (a) 1 and 4 (b) 1 and 3
(a) 1 and 2 (b) 3 and 4 (c) 1 and 2 (d) 3 and 4
(c) 1 and 4 (d) 1, 2, 3, 4 71. What all are local authorities?
64. What are the functions of RBI? 1. City corporations 2. Municipalities
1. Issue of notes 3. Port trusts
2. Banker to the government Choose the incorrect option.
3. Controller of debit (a) 1 only (b) 2 only
4. Custodian of foreign reserves (c) 3 only (d) None of the Above
Choose the incorrect function of RBI. 72. Consider the following statement:
(a) 1 only (b) 2 only 1. Bombay Stock Exchange (BSE) is India’s oldest
(c) 3 only (d) 4 only stock exchange
65. What are the reforms of Narsimham Committee on 2. It formally came into being in 1888
Banking sector? 3. It was a regional exchange till 2002 when it became
1. Nationalisation of banks should not take place any a national exchange
more. Choose the incorrect statement regarding BSE.
2. Private and foreign banks should be set up to (a) 1 only (b) 2 only
promote competition. (c) 3 only (d) None of the Above
3. There should be a phased reduction of CRR and SLR. 73. What are the authorities of SEBI?
Choose the correct option. 1. Oversee the working of stock exchanges
(a) 1 only (b) 2 only 2. Regulate merchant banks and mutual funds
(c) 3 only (d) All of the Above 3. Register and regulate intermediaries such as stock
66. Consider the following statement: brokers
1. As per recommendations of the Narasimham Choose the incorrect answer.
Committee, it has been decided that credit facilities (a) 1 only (b) 2 only
granted by banks will be classified into performing (c) 3 only (d) None of the Above
and non-performing assets (NPA) 74. _______ deals with long-term finance (more than 365
2. NPA is a loan which is in default for more than nine days) funds. It includes all facilities and institutional
months. arrangements available for borrowing and lending of
Choose the incorrect statement. term funds (including medium-term).
(a) 1 only (b) 2 only 1. Capital market
(c) 1 and 2 (d) None of the Above 2. Stock market
67. Consider the following in regards of types of loan: 3. Debit
1. Draft Loan 2. Term Loan 4. Credit
3. Cash credit 4. Overdraft Choose the correct answer.
Choose the one which is not a loan. (a) 1 only (b) 2 only
(a) 1 only (b) 2 only (c) 3 only (d) 4 only
(c) 3 only (d) 4 only 75. Consider the following statement:
68. _______ are conducted by the RBI by selling and buying 1. GIC was formed in November 1972.
government securities from banks. 2. The 107 private companies operating in the
1. Bank rate field were grouped together into four - National
2. Cash reserve ratio Insurance Company, United India Insurance
3. Open market operations Company, Oriental Insurance Company and New
4. Statutory Liquidity Ratio India Assurance Company.
Choose the correct code. Choose the incorrect statement.
(a) 1 only (b) 2 only (a) 1 only (b) 2 only
(c) 3 only (d) 4 only (c) 1 and 2 (d) None of the Above
E-116 Money Supply and Indian Financial System

76. Name of four commodity exchanges working in the Choose the correct statement.
country: (a) 1 only (b) 2 only
1. National Multi Commodity Exchange (c) 3 only (d) All of the Above
2. Multi Commodity Exchange 82. ______ is set up to promote liquidity to stocks of small
3. National commodity and Derivatives Exchange and medium enterprises.
4. Indian Commodity Exchange 1. Indo Next
Choose the correct answer. 2. Interconnected Stock Exchange of India
(a) 1 and 2 (b) 1, 2, 3 3. Over the Counter Exchange of India
(c) 1, 2, 3, 4 (d) None of the Above 4. Bombay Stock Exchange
77. Name of four institutions that are regulated by the RBI as Choose the correct code.
all-India FIs: (a) 1 only (b) 2 only
1. Export Import Bank of India (EXIM Bank) (c) 3 only (d) 4 only
2. National Bank for Agriculture and Rural 83. Consider the following statement:
Development (NABARD) 1. Capital market deals with long-term finance funds.
3. National Housing Bank (NHB) 2. Capital Market includes all facilities and institutional
4. Small Industries Development Bank of India arrangements available for borrowing and lending
(SIDBI) of term funds (including medium-term).
Choose the correct answer. 3. Long-term funds are raised either by borrowing
(a) 1 and 2 (b) 3 and 4 from certain institutions or by issuing securities.
(c) 1, 3, 4 (d) 1, 2, 3, 4 Choose the correct statement.
78. Consider the following statement: (a) 1 only (b) 2 only
1. EXIM bank in India was established on January 1, (c) 3 only (d) All of the Above
1982
2. National Housing Bank was established in July, Matching Based MCQ
1988.
3. SIDBI started its operations from April 2, 1990. DIRECTIONS (Qs. 84-86) : Match List-I with List-II and
Choose the correct code. select the correct answer using the codes given below the
(a) 1 only (b) 2 only lists.
(c) 3 only (d) 1, 2, 3 84. List-I List-II
79. The _________ bank linkage programme continued to (A) Depression (1) Coexistence of inflation and
be the main micro-finance model by which the formal stagnation
banking system reaches micro-entrepreneurs (including (B) Recession (2) Recovery from depression
farmers). (C) Reflation (3) Reduction in production
1. Self-Help Group 2. Indian bank over a short period
3. SBI 4. PNB (D) Stagflation (4) Insufficient demand leading
Choose the correct name of the bank. to idle men and machinery
(a) 1 only (b) 2 only over a long time
(c) 3 only (d) 4 only (5) Reduction in level of economy
80. Consider the following statement. due to falling prices
1. NABARD stands for National Bank for Agriculture (a) A-1; B-2; C-3; D-5 (b) A-4; B-3; C-2; D-5
and Rural Design (c) A-4; B-3; C-2; D-1 (d) A-3; B-4; C-2; D-1
2. NABARD is an apex developmental bank in India. 85. Match the following.
3. Setting up of NABARD was recommended by the Stock exchange name Set up Year
committee to review arrangements for institutional i. NSE a. 1887
credit for agriculture and rural development, set ii. BSE b. 1984
up by the RBI, under the chairmanship of Shri B. iii. OTCEI c. 1989
Sivaraman. iv. ISE d. 1998
Which statement is incorrect? (a) i-a, ii-b, iii-c, iv-d
(a) 1 only (b) 2 only (b) i-b, ii-a, iii-c, iv-d
(c) 3 only (d) 4 only (c) i-d, ii-c, iii-b, iv-a
81. Consider the following statement: (d) i-c, ii-b, iii-a, iv-d
1. IRDA is a multimember nodal agency 86. Match the following.
2. It is vested with regulatory powers in respect of the Goods Example
insurance sector similar to those vested in SEBI in i. Public goods a. Primary education
respect of the capital markets and of RBI for the ii. Merit goods b. National defence
banking sector. iii. Non-merit goods c. Pollution
3. The IRDA Act, 1999 cleared the way for private (a) i-a, ii-b, iii-c (b) i-b, ii-a, iii-c
sector entry into the insurance business. (c) i-c, ii-b, iii-a (d) i-a, ii-c, iii-b
Money Supply and Indian Financial System E-117

EXERCISE-1 48. (b) Syndicate Bank. The symbol of dog implies that
Bank is trustworthy and a friend. Its slogan is : Your
1. (b) 2. (b) 3. (d) 4. (a) 5. (b) 6. (b)
faithful and friendly financial partner.
7. (d) 8. (b) 9. (c) 10. (c) 11. (b) 12. (d)
49. (d) 50. (b)
13. (d) 14. (b) 15. (c) 16. (d)
51. (a) In economics, the Laffer curve is a hypothetical
17. (d) VAT is the State Subject.
representation of the relationship between
18. (c) Deflation is a decrease in the prices of goods and
government revenue raised by taxation and all
services. It occurs when the annual inflation rate
possible rates of taxation. It is used to illustrate the
falls below 0%, which is negative inflation rate.
concept of taxable income elasticity – which taxable
This is different from Disinflation which is a slow-
income will change in response to changes in the
down in the inflation rate. This is a situation when
rate of taxation.
inflation declines to lower levels but prices continue
52. (a) 53. (c) 54. (a)
to rise.
55. (a) Deficit financing, practice in which a government
19. (c) Base effect is almost an ubiquitous term which says
spends more money than it receives as revenue,
that the previous data affects the calculation of the
the difference being made up by borrowing or
current data.
minting new funds. Although budget deficits may
20. (c) The basic aim of Lead Bank scheme is that the
occur for numerous reasons, the term usually refers
bank should adopt particular districts for intensive
to a conscious attempt to stimulate the economy
development by offering loans and banking services.
by lowering tax rates or increasing government
21. (a) International Development Association (IDA) , is
expenditures. The influence of government deficits
a part of the World Bank that helps the world’s
upon a national economy may be very great. It is
poorest countries. It complements the World
widely believed that a budget balanced over the span
Bank’s other lending arm— the International Bank
of a business cycle should replace the old ideal of
for Reconstruction and Development (IBRD)
an annually balanced budget. Some economists have
which serves middle-income countries with capital
abandoned the balanced budget concept entirely,
investment and advisory services. IDA was created
considering it inadequate as a criterion of public
in 1960.
policy.
22. (a) A self-Regulatory organization (SRO) is a non-
56. (c) 57. (c) 58. (b) 59. (a)
governmental organization that has the power
60. (a) 61. (c) 62. (a)
to create and enforce industry regulations and
63. (d) A tight monetary policy is a course of action
standards. The priority is to protect investors
undertaken by Central bank to constrict spending in
through the establishment of rules that promote
an economy, or to curb inflation when it is rising too
ethics and equality.
fast. The increased bank rate increases the cost of
23. (a) 24. (a) 25. (c) 26. (a)
borrowing and effectively reduces its attractiveness.
27. (b) An outward shift in the demand for money, other
64. (b) Venture capital (VC) is a long term financial capital
things being equal should lead to a higher interest
provided to early-stage, high-potential, start up
rate but the same quantity of money. The supply
companies or new companies.
will not increase but with more demand the price
(the interest rate) should increase. EXERCISE-2
28. (c) 29. (c) 30. (b) 31. (c)
1. (b)
32. (a) 33. (a) 34. (c) 35. (d)
2. (c) SLR used by bankers indicates the minimum
36. (b) Foreign Banks and Regional Rural Banks also come
under the category of Scheduled Commercial Banks. percentage of deposits that the banks have to
Now, the banks are free to determine their savings maintain in the form of gold, cash or other approved
bank deposit interest rate, subject to the following securities.
two conditions: 3. (c)
1. Each bank will have to offer a uniform 4. (a) The statement (1) is correct because it includes the
interest rate on savings bank deposits up to ` 1 definition of teaser loans but the statement (2) is not
lakh,irrespective of the amount in the account correct, because in India teaser loan is provided to
within this limit. the home buyers not for setting up manufacturing or
2. For savings bank deposits over ` 1 lakh, a export units.
bank may provide differential rates of interest, 5. (c)
if it so chooses. However, there should not be 6. (a) Repo Rate is the rate at which commercial banks
any discrimination from customer to customer borrow funds from RBI. A reduction in the repo
on interest rates for similar amount of deposit. rate will help banks to get money from the central
37. (b) 38. (c) 39. (c) 40. (c) 41. (d) 42. (b) bank at a cheaper rate. When the repo rate increases
43. (a) 44. (c) 45. (a) 46. (c) 47. (c) borrowing from RBI becomes more expensive.
E-118 Money Supply and Indian Financial System

A value of (0) for Gini Coefficient in a country 32. (c) Taxes on Agricultural income and Taxes on
implies that there is perfect equality in the system. consumption/sale of electricity are levied by the
If the value is 1 then there is complete inequality in State Governments.
the country. 33. (d) 34. (a) 35. (c) 36. (c) 37. (b) 38. (c)
7. (a) The ‘BSE SENSEX’ is a value-weighted index 39. (c) When inflation becomes very high, the RBI
composed of 30 stocks and was started in 1 January, decreases supply of money (to check inflation) by
1986. The origin of the NYSE can be traced to 17 adopting light monetary policy.
May, 1792. When the Buttonwood Agreement was 40. (d)
signed by 24 stock brokers outside 68 Wall Street 41. (c) Convertibility of rupee implies freely permitting
in New York under a buttonwood tree. Amsterdam the conversion of rupee to other currencies and
Stock Exchange (1602) is considered oldest in the vice versa. Currency Convertibility is the ease with
world and was established by the Dutch East India which a country’s currency can be converted into
company. gold or another currency.
8. (a) 9. (c) 10. (b) 11. (b) 42. (c)
12. (a) Brent Crude is a major trading classification of sweet 43. (d) The Reserve Bank of India is the main monetary
light crude oil that serves as a major benchmark authority of the country and beside that, in its
price for purchases of oil worldwide. Brent Crude capacity as the central bank, acts as the bank of
is extracted from the North Sea and comprises the national and state governments. Sometimes it
Brent Blend, Forties Blend, Oseberg and Ekofisk happens that some of the banks close down due to
crudes .The Brent Crude oil marker is also known non recovery of loans or such other issues. In such
as Brent Blend, London Brent and Brent petroleum. conditions people have to suffer as their money
The index represents the average price of trading in is with the bank then. For this reason there is
the 25 day Brent Blend, Forties, Oseberg, Ekofisk provision for winding up of the banking company
(BFOE) market in the relevant delivery month as under the Banking Regulation Act, 1949. The power
reported and confirmed by the industry media. of winding up of Bank lies in the hand of Reserve
13. (d) NIFTY is an Index computed from performance Bank of India.
of top stocks from different sectors listed on NSE 44. (c) Land development bank started financing long term
(National stock exchange). NIFTY consists of 50 loan for more significant rural development activities
companies from 24 different sectors. Indian Capital like rural and cottage industries, rural artisans etc.
Markets are regulated and monitored by the Ministry The main purpose of RRBs is to mobilize financial
of Finance, the Securities and Exchange Board of resources from rural / semi-urban areas and grant
India and the Reserve Bank of India. It does not loans and advances mostly to small and marginal
trade in mutual funds. farmers, agricultural labourers and rural artisans.
14. (a) Cost-push inflation occurs when businesses respond 45. (c) When the Reserve Bank of India reduces the
to rising costs, by increasing their prices to protect Statutory Liquidity Ratio by 50 basis points; the
profit margins. There are many reasons why costs Scheduled Commercial Banks may cut their lending
might rise: Component costs, Rising labour costs and rates.
46. (b) Because Reserve requirements are designed as
higher indirect taxes imposed by the government.
“precautionary measures” and not to stop banks
So, the direct taxes should be increased when an
from “excessive” profit. This eliminates (c) and (d)
economy is going through inflationary pressures.
47. (d) The four concepts of money used in calculating
15. (a) Typically, higher inflation is caused by strong
money supply are known as the money stock
economic growth. If Aggregate demand in an
measures or measures of monetary aggregates.
economy expanded faster than aggregate supply, These are M1, M2, M3, M4.
we would expect to see a higher inflation rate. M3 : M1 + Term deposits with the bank.
If demand is rising faster than supply, then this 48. (a) In economics, the money supply or money stock is
suggests that economic growth is higher than the the total amount of monetary assets available in an
long run sustainable rate of growth. economy at a specific time.
16. (b) 17. (c) 49. (a) Inflation is an increase in price of goods. It can
18. (c) Deflation is defined as a fall in the general price be seen as devaluation of the worth of money. A
level of goods and services. It is a negative rate of crucial feature of inflation is that price rises are
inflation. It means the value of money increases sustained. Once only increase in the rate of, say,
rather than decreases. value-added tax, will immediately put up prices, but
19. (c) this does not represent inflation, unless the indirect
20. (b) The Office of the Economic Advisor (OEA) is an effects of the VAT rise have repercussions of prices
attached office of the Ministry of Commerce & in periods after the direct effects. Accounts of the
Industry. causes of inflation are numerous. The most popular
21. (d) 22. (d) 23. (d) 24. (b) 25. (c) 26. (c) arguments are that it is caused by excess demand
27. (c) 28. (b) 29. (a) 30. (d) 31. (d) in the economy (demand-pull inflation), that it is
Money Supply and Indian Financial System E-119

caused by high costs (cost-push inflation) and that (iii) When the Central Bank sells the securities in open
it results from excessive increases in the money market (which reduces the quantity of money in
supply (monetarism). Inflation affects all segments circulation).
of the economy. (iv) When Central Bank controls the credit money and
50. (d) Inflation benefits borrowers, as it leads to a fall in adopt various measures such as increase in CRR,
the real cost of capital. But this is only a temporary credit rationing and direct action.
phase and the interest rate is bound to go up to (v) When the Central Bank increases the Bank rate
compensate for the inflation. (which curtails the quantity of credit in the economy).
51. (a) Types of inflation are Demand Pull Inflation, Cost (vi) When state of over-production (excess supply over
Push Inflation, Stagflation, Hyperinflation. demand) takes place.
52. (b) the main cost-push factor in India are: 59. (d) Measures of checking deflation are:
(i) Fluctuations in output and supply in both agriculture
(i) Increasing money supply.
and industry sectors. Fluctuations in output of food
(ii) Promoting credit creation by the banks.
grains have been a major factor responsible for rise
in food-grain prices as well as general price. In the (iii) Curtailment in taxes so as to increase the
same way, the supply of manufactured goods also purchasing power of the people.
did not increase adequately in last few years. Power (iv) Increasing the public expenditure and the
breakdowns, strikes and lock-outs and shortage of employment opportunities in the economy.
transport facilities have been the major constraints (v) Increasing the money supply in circulation by
responsible for lowering production of manufactured repayment of old public debts.
goods. With ever-rising demand for manufactured (vi) Providing of economic subsidy by the Government
goods, the producers are in a position to hike the to the industrial sector of the econ.
prices of their products. 60. (c) The type of organised sector of Indian money market
(ii) Problem of hoarding by traders and black are Call Money Market, Treasury Bill Market,
marketeers. Commercial Bill Market and Collateral loan market
(iii) Taxation which gives the traders an opportunity to 61. (d) Ad hoc treasury bills are not sold to the banks and
raise the prices of goods, the proportion of which is the general public, and are not marketable while
often more than the levy of taxes. regular treasury bills are sold by Reserve Bank of
(iv) Administered Prices.
India on behalf of the Central Government.
(v) Hike in Oil Prices.
62. (a) Certificate of Deposit (CD) and Commercial Paper
53. (b) Price level is measured by Wholesale Price Index,
Consumer Price Index, Gross Domestic Product (CP) markets were introduced by Reserve Bank of
(GDP) Deflator. India in March 1989 in order to widen the range
54. (c ) Factors causing an increase in demand for goods & of money market instruments and give investors
services: greater flexibility in the deployment of their short-
(i) Increase in public expenditure term surplus funds.
(ii) Increase in private expenditure 63. (d) The nationalised banks are Central Bank of India,
(iii) Increase in exports Bank of India, Punjab National Bank, Canara Bank,
(iv) Reduction in taxation United Commercial Bank, Syndicate Bank, Bank of
(v) Rapid growth of population Baroda, United Bank of India, Union Bank of India,
(vi) Black money Dena Bank, Allahabad Bank, Indian Bank, Indian
(vii) Deficit financing Overseas Bank Bank of Maharashtra.
(viii) Cheap money policy 64. (c ) The functions of RBI are Issue of Notes, Banker
(ix) Increase in consumer spending to the Government, Banker’s Bank, Controller of
(x) Department of Tax internal debts. Credit, Custodian of Foreign Reserves.
55. (a) The inflation occurs due to two main factors:
65. (d)
(i) Increase in demand for goods & services
66. (b) NPA is a loan (whether term loan, cash credit,
(ii) Decrease in the supply of goods & services
56. (a) The Wholesale Price Index (WPI) is a weighted overdraft, or bills discounted), which is in default
average of indices covering 676 commodities, which for more than six months.
are traded in primary, manufacturing and fuel and 67. (a) Types of loan are term loan, cash credit, overdraft,
power-sectors. or bills discounted.
57. (a) Inflation is an increase in price of goods while 68. (c) Open Market Operations: These are conducted by
Deflation is that state in which the value of money the RBI by selling and buying government securities
rises and the price of goods and services falls. from banks.
58. (a) The reasons for deflation in the economy are: 69. (a) The main players in Indian capital markets are:
(i) When the Government withdraws money from • Banks, Indigenous and commercial.
circulation. • Insurance companies
(ii) When Government imposes heavy direct taxes or • Development Finance Institutions (DFI), and
takes heavy loans from the public (voluntary or • Non-Banking Finance Companies (NBFCs)
compulsory or both). • Non-Banking Financial Institutions.
E-120 Money Supply and Indian Financial System

70. (c) The securities market is further divided into the gilt- 78. (d) EXIM bank in India was established on January 1,
edged market and the corporate securities market. 1982. National Housing Bank was established in
71. (d) Local authorities are city corporations, municipalities July 1988. SIDBI started its operations from April
and port trusts. 2, 1990.
72. (c) Bombay Stock Exchange (BSE) India’s oldest stock 79. (a) The Self-Help Group (SHG) bank linkage
exchange formally came into being in 1887 and programme continued to be the main micro-finance
was a regional exchange till 2002 when it became a model by which the formal banking system reaches
national exchange. micro-entrepreneurs (including farmers).
73. (d) The SEBI is authorised to: 80. (a) NABARD stands for National Bank for Agriculture
• oversee the working of stock exchanges; and Rural Development.
• regulate merchant banks and mutual funds; 81. (d) IRDA is a multimember nodal agency. It is vested
• register and regulate intermediaries such as with regulatory powers in respect of the insurance
stock brokers; sector similar to those vested in SEBI in respect
• curb fraudulent and unfair trade practices of the capital markets and of RBI for the banking
including insider trading; sector.
• promote the development of a healthy capital The IRDA Act, 1999 cleared the way for private
market. sector entry into the insurance business.
74. (a) Capital market deals with long-term finance (more 82. (a) Indo Next is a new stock exchange, set up to promote
than 365 days) funds. It includes all facilities and liquidity to stocks of small and medium enterprises.
institutional arrangements available for borrowing 83. (d) Capital market deals with long-term finance (more
and lending of term funds (including medium-term). than 365 days) funds. It includes all facilities and
75. (d) The GIC was formed in November 1972 consequent institutional arrangements available for borrowing
upon the nationalisation of general insurance
and lending of term funds (including medium-term).
business. The 107 private companies operating in
The difference between money market and capital
the field were grouped together into four - National
market is not so much in the institutions involved
Insurance Company, United India Insurance
as in their term of borrowing or lending. Long-term
Company, Oriental Insurance Company and New
funds are raised either by borrowing from certain
India Assurance Company, with GIC as the holding
institutions or by issuing securities.
company.
84. (b) Recession is slow down in effective demand for
76. (c ) There are four commodity exchanges working
goods and services, slow down in the economy
in the country. They are NMCE (National Multi
implies a short run decline in the growth rate.
Commodity Exchange) - the oldest commodity
85. (b) Stock exchange name Set up Year
exchange in the country which became operational in
2002, MCX (Multi Commodity Exchange), NCDEX (i) NSE a. 1984
(National Commodity and Derivatives Exchange) (ii) BSE b. 1887
and ICEX (Indian Commodity Exchange). (iii) OTCEI c. 1989
77. (d) According to the Economic Survey, there are four (iv) ISE d. 1998
Institutions, namely the Export Import Bank of 86. (b) Public goods include national defence, police,
India (EXIM Bank), National Bank for Agriculture general administration, Merit goods include primary
and Rural Development (NABARD), the National education, immunisation, public health programme,
Housing Bank (NHB) and Small Industries Non-merit goods include pollution caused by
Development Bank of India (SIDBI). automobile emission.
FOREIGN TRADE AND 6
INVESTMENT IN INDIA

Introduction
Foreign trade is exchange of capital, goods, and services across international borders or territories, which involves the activities
of the government and individuals.
In most countries, it represents a significant share of gross domestic product (GDP). Foreign trade in India, includes all
imports and exports to and from India. At the level of Central Government it is administered by the Ministry of Commerce
and Industry.

Foreign Trade and Investment

Introduction Foreign Exchange India’s Foreign Balance of India’s Balance


Reserves Trade Payments of Payments

Gold
Trends 2
Special Drawing Problems
Right (SDR) Protectionist
Foreign Currency Policies
Assests (FCA) External Debt
Reserve Tranche
Export Promotion
Position (RTP)
Exchange Rate
Trade Policy
History India’s Export India’s Import Special Economic
Profile Profil e Zone (SEZ)
Foreign Direct
Investment
Introduction Structure of Balance
of Payments

Current Account
Capital Account
Reserve Account Balance
E-122 Foreign Trade and Investment in India

FOREIGN EXCHANGE RESERVES


Foreign exchange reserves also called forex reserves or FX Fixed Exchange rates
reserves are assets held by a central bank or other monetary
Fixed rates are currency values which are tied to a precious
authority, usually in various reserve currencies and used
to back its liabilities e.g. the local currency issued, and the metal such as gold, or anchored to another currency, like the
various bank reserves deposited with the central bank by the US Dollar. This method was brought by the International
government or by financial institutions. Monetary Fund (IMF). The IMF system involved the US$ as
Why do countries keep foreign exchange reserves? the anchor for the system with the US$ given a specific value
In the past, during the Bretton Woods system — an international in terms of gold, and other currencies were then given a value
monetary system formed after the second world war, foreign in terms of the US$, such as £1 = $2.40. India was part of this
exchange reserves were used by countries through their regime too and in 1948, 1$ was equal to Rs. 3.30. However,
central banks to maintain the external value of their currencies the system collapsed in 1971 for a variety of reasons, including
at fixed rate. Subsequently, with the collapse of this system,
the build up of US debts abroad as a result of the need to fund
the focus changed. Reserves are now generally maintained by
countries for meeting their international payment obligations the war in Vietnam, inflation in the USA and growing doubts
— both short and long terms, including sovereign and about the stability of the US$.
commercial debts, financing of imports, for intervention in the
foreign currency markets during periods of volatility, besides Floating Exchange Rates
helping to boost the confidence of the market in the ability of Under a floating system, a currency can rise or fall due to
a country to meet its external obligations and to absorb any changes in demand or supply of currencies on the foreign
unforseen external shocks, contingencies or unexpected capital exchange market. The advantages of floating exchange rates are
movements.
flexibility and automatic adjustment in case of balance of trade
The foreign exchange reserves of India comprise of three
disequilibrium. Under a floating regime, imports and exports
elements:
can readjust to move the balance of payments back towards
(i) Gold,
(ii) Special Drawing Rights (SDR), a desirable equilibrium. Exogenous shocks, like the financial
(iii) Foreign Currency Assets (FCA). crisis of 2008-09, can occur from time to time and floating
exchange rates can help the readjustment process. Moreover,
Gold policymakers are free to devalue or revalue to achieve specific
objectives, such as stimulating jobs and growth and reducing
It accounts for only 5% of our foreign exchange assets.
inflationary pressure.
Special Drawing Rights (SDR) Managed Exchange Rates
An international type of monetary reserve currency, created This is a combination of fixed and floating rates. In today’s
by International Monetary Fund (IMF) in 1969 which operates economic situation, almost all countries follow this system of
as a supplement to the existing reserves of member countries.
exchange rate determination. The governments usually let the
It is also known as “paper gold”. Created in response to
concerns about the limitations of gold and dollars as the sole market determine the exchange rates but intervene whenever
means of settling international accounts, SDRs are designed to needed.
augment international liquidity by supplementing the standard The success of foreign exchange management by RBI can
reserve currencies. Its value is based on a basket of five key be seen in the fact that from US$ 5.8 billion in March 1991
international currencies and SDRs can be exchanged for freely (enough to meet the import requirements for three weeks) the
usable currencies. The basket of five international currencies
country’s reserves have grown to around US$ 352 billion by
includes US dollar, euro, Chinese yuan, Japanese yen and
British pound. December 2015. It is pertinent to note here that some of the
increase in the reserve is attributed to the weakness of the
Foreign Currency Assests (FCAs) dollar in the post-Iraq war scenario.

Foreign currency assests include foreign exchange reserves


less gold holdings, special drawing rights and India’s reserve Reserve Tranche Position
position in the IMF. Each member of the IMF is assigned a quota, part of which is
Foreign Exchange Management payable in SDRs or Specified usable currencies and part in the
Foreign exchange management can be managed in three member’s own currency. The difference between a member’s
possible ways: Fixed currency rates, floating currency rates quota and the IMF’s holdings of its currency is a country’s
and managed exchange rates. Reserve Tranche position (RTP).
Foreign Trade and Investment in India E-123

INDIA’S FOREIGN TRADE


History of Foreign Trade in India the year 2014, India was the 19th largest exporter (with a share
of 1.7%) and 12th largest importer (with a share of 2.5%) of
•• Historically, India ran a trade surplus for centuries merchandise trade in the world. In Commercial services, India
together through export of spices, handicrafts, textiles, is the 6th largest exporter (with a share of 3.3%) and 7th largest
etc. No restrictions on imports or exports were officially importer (with a share of 2.9%). Service sector has been a major
maintained. financer for India’s merchandise trade deficit. Service surplus
•• Before India got independence, import of goods from financed around 53% of merchandise trade deficit during
great Britain received official encouragement through 2013-14.
Imperial preferences. India has been pursuing a policy of market diversification
•• Statutorily, it was the Sea Customs Act, 1878 that directing her export promotion efforts at Asia and ASEAN,
provided the basis for implementing the official bias in Latin America and Africa through Focus Market Initiatives
favour of imports from Britain. and bilateral trade agreements.
•• The Government of India Act, 1935 granted Central
government the exclusive legislative powers to regulate India’s Export Profile & Exports
import of goods into India and export of goods from
India. However, this power was used when the Imports Growth
and Exports (Control) Act, 1947 was enacted. Indian exports have grown at a robust annual average rate of
•• The initial life of the Imports and Exports (Control) Act, 21.3% (CAGR) between FYO3 and FY13; the value of exports
1947 was three years but it was extended from time to rising from USD 52.7 billion in FY03 to 300.2 billion in FY13.
time till 1971.
•• To protect industries from influx of imported goods the Commodity Basket : Exports
government issued the Imports (Control) Order 1955 •• Agriculture and allied products have registered high
allowing most of the imports only against an import growth of 22.2% (CAGR, FY03-FY13) and stood at USD
licence. 40.6 billion in FY13.
•• In 1976 many changes were made to the Imports and •• Major Agriculture and allied commodities exported by
Exports (Control) Act, 1947. This act gave the Central India include basmati rice, marine products, meat and
Government wider powers to prohibit, restrict and control meat products, oil meals, spices, wheat and sugar.
the imports and Exports Trade. •• At USD 183.6 billion in FY13, manufactured goods have
Beginning mid-1991, the Government of India introduced registered growth of 18.4% (CAGR, FY03-FY13).
a series of reforms to liberalise and globalise the Indian •• Major manufactured goods, exported by India include
economy. Reforms in the external sector of India were intended engineering goods, chemicals and related products,
to integrate the Indian economy with the world economy. textiles and ready-made garments.
India’s approach to openness has been cautious, contingent on •• The exports of ores and minerals has grown by 12.1%
achieving certain preconditions to ensure an orderly process (CAGR, FY03-FY13), to touch USD 5.6 billion in
of liberalization and ensuring macroeconomic stability. This FY13, while export of petroleum and crude products has
approach has been vindicated in recent years with the growing registered exponential growth of 41.9% (CAGR, FY03-
incidence of financial crises elsewhere in the world. All the
FY13) to stand at USD 60.2 billion in FY13.
same, the policy regime in India in regard to liberalization of
In terms of share in overall exports, the share of agriculture
the foreign sector has witnessed very significant change.
and allied products, ores and minerals and other commodities
The Import policies prior to 1992 contained an Open General
has moved in a range-bound fashion. However, there has
Licence under which specific goods could be imported and
been some perceptible shift from the manufactured goods to
exported by specific categories of importers and exporters
the petroleum and crude products segment between FY03 and
subject to fulfilment of certain conditions.
FY13.
In 1992, the policy was amended to open general licence and
allow imports and exports of all goods without a licence, except Direction of Exports
those specifically mentioned in a small negative list.
India’s largest exports partner has been Asia. Exports to Asia
Foreign Trade is one of the significant macro fundamental have grown by more than 23.0% (CAGR), from USD 22.2
variables of an economy. India, until recently, was billion in FY03 to USD 150.4 billion in FY13.
predominantly a primary goods exporting and an industrial
The next largest export destination is Europe (USD 58.8 billion
goods importing country.
in FY13), followed by America (USD 53.4 billion).
In 1950s, India’s share in the world trade was 1.78% which was
decline to 0.59% in 1990 and remained low for many years.
India’s share in world trade is currently around 2% (2015) and
Imports’ Profile & Import Growth
our country has set for itself the ambitious target of gaining As a growing country, India’s imports demand has consistently
3.5% of world trade by 2020. As per the rankings of WTO for been on the rise.
E-124 Foreign Trade and Investment in India

Barring a marginal dip in the year of recession in FY10, when •• Non-POL items on the other hand, have growth by 24.8%
imports fell from USD 299.3 billion to USD 287.6 billion, over this period, to stand at USD 321.3 billion in FY13.
imports to the country have always been on the rise.
In the last decade import growth has been high at 26.0%; from Direction of Imports
USD 61.4 billion in FY03 to USD 490.3 billion in FY13. A major part of India’s imports are sourced from Asia itself
(57.7% of overall imports which are due to the POL bill.
Commodity Basket: Imports Imports from Asia stood at USD 283.0 billion in FY13,
•• Commodity imports of the country are classified as POL registering growth of 26.0% over FY03 (CAGR, FY03-13).
(petroleum, oil and lubricants) and non-POL items. In fact, there appears to be substantial difference in the value
•• Import of POL items has grown at 28.5% (CAGR, FY03- of imports from Asia and the next largest import partner of the
FY13), from USD 17.6 billion in FY03 to USD 169.0 country. After Asia, the next largest import source is Europe
billion in FY13. (USD 91.7 billion), followed by America (USD 58.2 billion).

BALANCE OF PAYMENTS
Introduction Current Account
Balance of payments (BOP) is statistical statement of all This account is the summary of all international trade
transactions made between one particular country and all other transactions of the domestic country in one year. It records the
countries during a specified period of time. BOP compares following 3 items:
the dollar difference of the amount of exports and imports, (i) Visible items of trade
including all financial exports and imports. The balance of exports and imports of goods is called the
•• A negative balance of payments means that more money balance of visible trade, e.g. Tea, Coffee, etc.
is flowing out of the country than coming in. The balance (ii) Invisible Trade
of payment is maintained by Central Bank of India, i.e. The balance of exports and imports of services is called
Reserve Bank of India. the balance of invisible trade.The invisbles are divided
into three categories: (a) Services- insurance, travel,
•• Balance of payments may be used as an indicator of
transportation, miscellaneous (like communication,
economic and political stability. For example, if a country
construction, financial, software, etc.), (b) Income, and
has a consistently positive BOP, this could mean that
(c) Transfers (grants, gifts, remittances, etc.).
there is significant foreign investment within that country.
(iii) Unilateral transfers
It may also mean that the country does not export much
of its currency. Unilateral transfers are receipts which residents of a
country make without getting anything in return, e.g.
•• BOP indicates trade balance, foreign investments and
gifts, etc.
investment by foreigners. Even a negative BOP does
The net balance of visible trade, invisible trade and of
not signify unfavourable climate for the economy. It is
unilateral transfers is the balance on current account.
unfavourable only if the economy lacks the means to fill
Current account in India is fully convertible now. It means
the gap created by negative BOP.
that if someone requires foreign exchange for current
Balance of trade and balance of payment purposes, it will be made available to him at official
exchange rate and thus, an unprohibited outflow of foreign
Balance of trade takes into account only those transactions
exchange is possible now. Current account convertibility
arising out of exports and imports (the visible items). It does not was a result of India’s obligations under Article VIII of
consider the exchange of services rendered such as shipping. IMF which prohibits restrictions on exchange on current
Balance of payment takes into account the exchange of both international transactions.
visible and invisible items. Hence, the balance of payments
represents a better picture of a country’s economic transactions Capital Account
with the rest of the world than the Balance of trade. This account is the summary of foreign capital transactions.
On the credit side of this account receipt of foreign exchange
Structure of Balance of Payments due to Foreign Direct Investment (FDI), Foreign Capital
Accounts Investment (FCI) and Foreign Borrowing (FB) is recorded.
On the debit side of capital account payment of foreign
A balance of payments statement is a summary of a nation’s exchange due to Direct Investment Abroad (DIA), Portfolio
total economic transaction undertaken on international account. Investment Abroad (PIA) and Foreign Lending (FL) is
It is usually composed of three sectors: recorded.
1. Current account, While India made the rupee fully convertible under current
2. Capital account, account, it was felt that the economy was not yet ready for
3. Reserve account balance. capital account convertibility (CAC). The countries which
Foreign Trade and Investment in India E-125

went in for hasty capital account convertibility in the 1980s The Reserve Account is also the indicator of Forex Reserves
had to face financial crisis. The thinking was that trade and of the country. If surplus in the Capital Account is more than
domestic liberalization process should be completed before deficit in the Current Account, there is net increase in the
full capital convertibility can be thought of. Many economists Forex Reserves of the country at the end of the year. On the
feel that CAC will only help the drain of Indian capital, other hand if deficit in the current account is more than surplus
which will get invested abroad. After the recommendations of in the Capital Account then there is net decrease in Foreign
RBI appointed S.S Tarapore Committee on Capital Account Reserves of the country at the end of the year.
Convertibility (CAC), India is slowly moving towards full
convertibility, but with constant precautions. India still has Balance of Payment (BOP) Crisis
partial convertibility(40:60) but many reforms undertaken by If international reserves of a country are not enough to balance
India hint at eventual movement towards CAC. a combined deficit in current and capital account on a sustained
basis, then the phenomenon is called a BOP crisis. It can be
Reserve Account Balance tackled by exporting more or by limiting imports through tariffs,
This is the adjusting account in balance of payment. It makes quotas, etc. Contractionary fiscal and monetary policies can
an adjustment between current account balance and capital also tackle the crisis through lower import demand with fall in
account balance. If the deficit in the current account is followed average income levels. Another short term solution is currency
by surplus in capital account than the excess foreign exchange devaluation which encourages exports and discourages costlier
is diverted into capital account to current account so that deficit imports.
in the current account is eliminated. The remaining surplus However, for the Indian economy most of these solutions are
in the capital account is transferred to the Reserve account not realistic in the present context. The most potential solution
and recorded on the credit of reserve account. Therefore both lies in the form of greater FDI and FII inflows letting a Capital
Current Account and Capital Account is always balanced. Account surplus tackle any BOP crisis.

INDIA’S BALANCE OF PAYMENT


The international Balance of Payments (BOP) of a country was food scarcity too. Large amounts of food grains had to be
reflects its economic strengths and weaknesses. A typical imported to feed the huge population.
problem of the developing countries is that of a chronic BOP India faced a major BOP crisis in the early 1990s. We had to
deficit, India being no exception. Our country has been facing borrow from IMF to be able to sustain the economy. This loan
BOP disequilibrium right since independence, culminating into came with a number of conditions. The following are some of
a disaster in 1990-91, the year of the acute BOP crisis. India the conditions that came with the medium term loan given by
then had foreign exchange reserve of mere 1 billion dollar, IMF to India for restructuring of the economy:
barely sufficient to finance a month’s import bill. The country
•• Government expenditure to be cut by 10% per annum,
was on the verge of defaulting. This crisis led to the massive
changes in the country’s economic policy, popularly known as •• Devaluation of rupee by 22%,
the Structural Adjustment Program or New Economic Policy •• Excise duty to be increased by 20%,
(NEP) regime, focusing on liberalization and globalization of •• Custom duty to be cut drastically from the peak of 130%
the economy. to 30%.
We opted for a very cautious approach and today after having Even though there was strong opposition within India to
overcome the initial hiccups of a newly liberalized economy, such a loan which came with so many conditions, the Indian
we have a fairly comfortable BOP situation. Even though we government went ahead with its plan of restructuring Indian
have attained a comfortable BOP position reflecting a strong economy guided by the pre conditions of the IMF. It led to
emerging economy, BOP management still remains a tightrope liberalisation of the Indian economy.
walk for policy makers, as now we are exposed to each and
every change in the global economic scenario. Protectionist Policies
Trends & Problems Of India’s The main objective of the Second Five Year Plan (1956-57 to
1960-61) was to attain self reliance through industrialization.
Bop – 1949-50 To 1999-2000. Self reliance was to be achieved through import substitution.
The disequilibrium in India’s BOP has been caused by both For this basic industries had to be set up which required import
internal as well as external factors. The need for development of capital goods. Exports were expected to automatically take-
of such a big nation with a huge population is one of the main off with industrialization. The import substitution strategy
causes for the recurring BOP problem. The BOP is always was based on non-price, physical- interventionist policies like
under pressure and had huge deficits due to high imports of food licensing, quotas and other physical restrictions on imports.
grains and capital goods, the heavy external borrowings and Heavy capital goods were imported but other imports were
its payment and poor exports. After achieving independence, severely restricted to shut off competition in order to promote
the foremost challenge before India was of attaining economic domestic industries. All focus was on import substitution, with
growth with social justice. India’s aim after attaining gross neglect of exports. Such inward looking protectionist
independence was to attain economic self- reliance. For this, policies did result in some self-reliance in the consumer goods
the country had to tap both the internal as well as the external industries, but the capital goods industries remained mostly
resources. Not only was our technology backward then, there import intensive.
E-126 Foreign Trade and Investment in India

The high degree of protection to Indian industries led India followed a strongly inward looking policy, laying stress
to inefficiency and poor quality products due to lack of on import substitution. Ideally, imports should be financed
competition. The high cost of production further eroded our by export earnings. But because there was export pessimism,
competitive strength. the deficit was financed either by the invisible earnings or by
Rising petroleum products demand, the two oil shocks, harvest foreign aid or depletion of valuable foreign exchange reserve.
failure, all put severe strain on the economy. The BOP situation Much import constraint to check trade deficit was also not
possible because India’s imports were mainly ‘maintenance
remained weak throughout the 1980s, till it reached the crisis
imports’. On one hand import reduction was not possible and
situation in 1990-91, when India was on the verge of defaulting
on the other exports suffered due to the recession in the 1980s.
due to heavy debt burden and constantly widening trade deficit.
India’s BOP was thus beset with several problems. The process
of liberalization began from the mid 1980s. Restriction on
External Debt certain imports were removed, particularly those which were
India had to resort to large scale foreign borrowings for used as inputs for export production. But by then the situation
its developmental efforts in the field of basic social and was already bad and all the mismanagement ultimately led to
industrial infrastructure. The country’s resources were very the 1990-91 BOP crisis.
much limited due to low per capita income and savings. The
situation worsened because Government of India resorted to Trade Policy
heavy foreign borrowings to correct the BOP situation in the India’s trade policy since Independence had been guided by
short run out of panicky. By the Seventh Five Year Plan, the what has been described as “export pessimism” and “economic
debt service obligations rose sharply because of harder average nationalism. The former assumed that India could never hope
terms of external debt, involving commercial borrowing, to compete with developed countries due to considerations of
repayments to the IMF and a fall in concessional aid flow. quality and cost. The latter assumed that self-sufficiency and
domestic production was somehow superior to trade. This
Export Promotion concept, which can be traced to the Swadeshi movement, was
further buttressed by the Mahalanobis strategy adopted during
Although by the Sixth Five Year Plan we had done away with the Second Plan and continued with modifications till the early
the need of food grain imports and some crude oil was being 1980s. It was only from 1985-86 that a genuine attempt was
produced domestically, BOP position was still not comfortable made towards trade liberalisation. The result was spectacular
due to low exports. The need for export promotion was felt as export increased at an average rate of 17 % per annum in
during the 1960s. The Third Five Year Plan introduced certain dollar terms between 1986 and 1990. A combination of factors
export promotion policies like cash compensatory schemes, tax such as bad policy, weak government and external factors led
exemptions, duty drawbacks, Rupee devaluation, etc. However to the decline of this performance to 9% in 1990-91 and 4% in
our exports remained discouraging. Indian exports depended the subsequent years.
largely on world trade situation. We were mainly primary As part of economic liberalisation, government introduced a
product exporters, the price of which fluctuated heavily with series of trade reforms on the fourth of 1991. The major thrust
fluctuations in world market demand. was to make exports competitive, unshackle foreign trade
from the clutches of a control regime and allow import of most
•• Primary products exporting countries have an unfavorable
goods using only tariff as a restraint.
term of trade. The earnings from primary product exports
were low and unstable. Foreign Trade Policy 2015-20
•• Secondly, the quality of Indian products was not up to Aiming to nearly double India’s exports of goods and services
the world standards due to which we could not sustain to $900 billion by 2020, the government has announced several
markets. incentives in the five-year Foreign Trade Policy for exporters
•• Third, only residue products were mainly exported. The and units in the Special Economic Zones (SEZ). Unveiling
fact that export earnings also contribute to economic the first trade policy of the NDA government, Commerce
development was overlooked. Cumbersome procedures Minister Nirmala Sitharaman said the FTP (2015-20) will
for license, etc. served as disincentives for exporters. introduce Merchandise Exports from India Scheme (MEIS) and
Domestic inflation further reduced the competitiveness of Services Exports from India Scheme (SEIS) to boost outward
India’s export. shipments.The new policy aims at boosting India’s exports and
it is believed that PM Narendra Modi’s pet projects, ‘Make
in India’ and ‘Digital India’ will be integrated with the new
Exchange Rate Foreign Trade Policy.
The instability of the exchange value of the rupee was another
problem. The constant devaluations (to promote exports) Trade Policy (2015-20) Key Features
raised the amount of external debt. The value of rupee was •• India to be made a significant participant in world trade
managed by the central bank (fixed exchange rate). The gap by 2020.
between official and market exchange rate created problems •• Merchandize exports from India (MEIS) to promote
for the exporters and importers. The strict foreign exchange specific services for specific Markets Foreign Trade
controls also encouraged hawala trade. Policy.
Foreign Trade and Investment in India E-127

•• FTP would reduce export obligations by 25% and give exports, with Asia’s first EPZ set up in Kandla in 1965. With a
boost to domestic manufacturing. view to attract larger foreign investments in India, the Special
•• FTP 2015-20 introduces two new schemes, namely Economic Zones (SEZs) Policy was announced in April 2000.
“Merchandise Exports from India Scheme (MEIS)” Today, there are approximately 3,000 SEZs operating in 120
and “Services Exports from India Scheme (SEIS)”. countries which account for over US $ 600 billion in exports
The ‘Services Exports from India Scheme’ (SEIS) is for and about 50 million jobs.
increasing exports of notified services. These schemes As a major step forward meant to invoke confidence in
(MEIS and SEIS) replace multiple schemes earlier in investors and signal the government’s commitment to a stable
place, each with different conditions for eligibility and SEZ policy regime, a comprehensive Special Economic Zones
usage. Incentives (MEIS and SEIS) to be available for Act, 2005 was passed by the Parliament in May 2005. It
SEZs also e-Commerce of handicrafts, handlooms, books, received Presidential assent on the 23rd of June 2005. This Act
etc. eligible for benefits of MEIS. FTP benefits from both came into force w.e.f. February 10, 2006.
MEIS and SEIS will be extended to units located in SEZs.
•• Agricultural and village industry products to be supported The main objectives of the SEZ Act are:
across the globe at rates of 3% and 5% under MEIS. (a) Generation of additional economic activity.
•• Higher level of support to be provided to processed and (b) Promotion of exports of goods and services.
packaged agricultural and food items under MEIS. (c) Promotion of investment from domestic and foreign
•• Industrial products to be supported in major markets at sources.
rates ranging from 2% to 3%. (d) Creation of employment opportunities.
•• Served from India Scheme (SFIS) will be replaced with
(e) Development of infrastructure facilities.
Service Export from India Scheme (SEIS).
•• Branding campaigns planned to promote exports in sectors The salient features of SEZs Act are:
where India has traditional strength. •• Exemption from customs duty, excise duty, etc. on import/
•• SEIS shall apply to ‘Service Providers located in India’ domestic procurement of goods for the development,
instead of ‘Indian Service Providers’. operation and maintenance of SEZs and the units therein.
•• The criteria for export performance for recognition of •• 100% income tax exemption for 5 years, 50% for the next
status holder have been changed from Rupees to US 5 years and 50% of ploughed back export profits for 5
dollar earnings. years thereafter for SEZs units.
•• Manufacturers who are also status holders will be enabled •• Exemption from capital gains on transfer of an undertaking
to self-certify their manufactured goods as originating from an urban area of SEZs.
from India.
•• 100% income tax exemption to SEZ developers for a
•• Reduced Export Obligation (EO) to (75%) for domestic block of 10 years in 15 years.
procurement under EPCG scheme.
•• Exemption from dividend distribution tax to SEZ
•• Online procedure to upload digitally signed document developers.
by Chartered Accountant/Company Secretary/Cost
•• 100% income tax exemption for 5 years and 50% for next
Accountant to be developed.
five years for off shore Banking units located in SEZ.
•• Inter-ministerial consultations to be held online for issue
•• Exemption to SEZ developer and units from Minimum
of various licences.
Alternate Tax.
•• No need to repeatedly submit physical copies of documents
available on Exporter Importer Profile. •• CST exemption to SEZ developer and units on inter-state
purchase of goods.
•• Export obligation period for export items related to
defence, military store, aerospace and nuclear energy to •• Constitution of an authority for each SEZ with a view to
be 24 months instead of 18 months. providing greater administrative, financial and functional
autonomy to these zones.
•• Calicut Airport, Kerala and Arakonam ICDS, Tamil
Nadu notified as registered ports for import and export; •• Establishment of designated courts and a single enforcement
Vishakhapatnam and Bhimavarm added as Towns of agency to ensure speedy trial and investigation of offences
Export Excellence. committed in SEZs.
•• Certificate from independent chartered engineer for •• Encouragement to State Governments to liberalise State
redemption of EPCG authorisation no longer required. laws and delegate their powers to the Development
Commissioners to the SEZs to facilitate single window
clearance.
Special Economic Zone (SEZ)
Special Economic Zone (SEZ) is a specifically delineated duty- Export Oriented Units, Export processing Zone
free enclave that has economic laws different from a country’s and Special Economic Zone Schemes
typical economic laws, usually the goal is to increase foreign The Government has liberalised the scheme for export-oriented
investments. units and export processing zones. Agriculture, horticulture,
India was one of the first in Asia to recognise the effectiveness poultry, fisheries and dairying have been included in export-
of the Export processing Zone (EPZ) model in promoting oriented units. Export processing zone units have also been
E-128 Foreign Trade and Investment in India

allowed to export through trading and star trading houses and Odisha, Meghalaya, Manipur, Nagaland, Mizoram and
can have equipments on lease. These units have been allowed Tripura. While the EPIP in Rajasthan at Sitapura (Distt.
cent per cent participation in foreign equities. Jaipur), Bangalore (Karnataka), Ambarnath (Distt.
1. Export Processing Zones Thane, Maharashtra), Kakkinad (Distt. Ernakulam,
Before getting converted into Special Economic Zones Kerala), Surajpur (Distt. Gautambudh Nagar, U.P.),
(SEZs), these Export Processing Zones (EPZs) were Gummidipoondi (Distt. Chengalpattu, Tamil Nadu),
playing important role in promoting exports of the country. Pashamylaram (Distt. Mendak, Andhra Pradesh) and
These zones were created to develop such an environment Amingaon (Near Guwahati, Assam) have been completed.
in the economy which may provide capability of facing Exports have already commenced from Karnataka, Kerala
international competition. The Export Processing Zone and Rajasthan EPIPs. Other parks are at various stages of
(EPZs) set up as enclaves, separated from the Domestic implementation.
Tariff Area by fiscal barriers, were intended to provide a At present, the number of formally approved SEZs is 413, 327
competitive duty free environment for export production. SEZs have been notified, 202 SEZs are operational and the total
All the 8 EPZs, located at Kandla and Surat (Gujarat), number of units approved in SEZs is 4,102. A total investment
Santa Cruz (Maharshtra), Cochin (Kerala), Chennai of Rs.3,48,983.22 crores has been done till 2015. Moreover,
(Tamil Nadu), Vishakhapatnam (Andhra Pradesh), Faeta till now15,04,597 persons have received employment through
(West Bengal) and Noida (U.P) have been converted into SEZs.
Special Economic Zones.
2. Export-Oriented Units Foreign Direct Investment (FDI)
Since 1981, the Government introduced a complementary Foreign direct investment (FDI) is an investment in a business by
plan of EPZ (Now converted into SEZ) scheme for an investor from another country for which the foreign investor
promoting export units (making export of their cent per cent has control over the company purchased. The Organisation
production. Under this scheme the Government provides of Economic Co-operation and Development (OECD) defines
various incentives to increase the production capacity of control as owning 10% or more of the business. Businesses that
these units so as to increase exports of the country. This make foreign direct investment are after called Multinational
scheme offers a wider source of raw materials, hinterland Corporations (MNCs) or Multinational Enterprises (MNEs).
facilities, availability of technological skills, existence of • A MNE may create a new foreign enterprise by making a
an industrial base and the need for a larger area of land direct investment, which is called a greenfield investment.
for the project.
• A MNE may make a direct investment by the acquisition of
3. Export Houses, Trading Houses and Star Trading a foreign firm, which is called an acquisition or prownfield
Houses investment .
To increase the marketable efficiency of exporters, the
Government introduced the concept of export houses, Advantages of foreign Direct Investment
trading houses and star trading houses. Those registered 1. Economic Development Stimulation.
exporters who have shown good export performances 2. Easy International Trade.
over past few years have been given the status of 3. Employment and Economic Boost.
export houses, and trading houses. Units having such
4. Development of human capital Resources.
classification are required to achieve the prescribed
5. Tax incentives.
average export performance level and earning of foreign
exchange. These units are provided some special facilities 6. Resource Transfer.
and benefits by the Government. 7. Reduced disparity between revenues and costs.
Since April 1, 1994 a new category named Golden Super 8. Increased productivity.
Star Trading Houses was added by the Government 9. Increment in income.
which has the highest average annual foreign exchange Disadvantages of Foreign Direct Investment
earnings. On March 31, 2003 there were 4 Golden Super
Star Trading House working in the country. 1. Hindrances to domestic Investment.
4. Export Promotion Industrial Parks (EPIP) 2. Risk from political changes.
3. Negative influence on exchange rates.
A Centrally-sponsored ‘Export Promotion Industrial Park
(EPIP)’ scheme was introduced in August 1994 with a 4. Higher costs.
view to involving the state governments in the creation 5. Economic non-viability.
of infrastructure facilities for export oriented production. 6. Expropriation.
It provides for 75% (limited to 10 crore) grant to state 7. Modern-day Economic colonialism.
government towards creation of such facilities. The FDI, being a non-debt capital flow, is a leading source of external
Central Government has so far approved 25 proposals financing, especially for the developing economies. It not only
for establishments of EPIPs in the states of Punjab, brings in capital and technical know-how but also increases
Haryana, Himachal Pradesh, Rajasthan, Karnataka, the competitiveness of the economy. Overall it supplements
Kerala, Maharashtra, Tamil Nadu, Andhra Pradesh, domestic investment, much required for sustaining the high
U.P., Gujarat, Bihar, J&K, Assam, M.P., West Bengal, growth rate of the country. Since 2000, significant changes
Foreign Trade and Investment in India E-129

have been made in the FDI policy regime by the government The following are some of the sectors with
to ensure that India becomes an increasingly attractive and 100% FDI
investor-friendly destination.
Advertising, agriculture, air transport services (domestic
The current phase of FDI policy is characterized by negative
airlines), courier services, drugs and pharmaceuticals,
listing, permitting FDI freely except in a few sectors indicated
electricity, power, films and studios, hotel and tourism,
through a negative list. Under the current policy regime, there
housing and real estate, construction, mass rapid transport
are three broad entry options for foreign direct investors.
system, mining (gold and silver), NBFC, marketing, pipelines
1. In some sectors, FDI is not permitted (negative list); and refining of petroleum products, tourism, transport
2. In another small category of sectors, foreign investment infrastructure, townships, SEZs, railways, single brand retail
is permitted only till a specified level of foreign equity (upto 49% automatic and from 49 to 100% has to be approved
participation, and by FIPB), telecommunications (upto 49% automatic, 49-100%
3. The third category, comprising all the other sectors, by FIPB), and asset reconstruction companies ( upto 49%
is where foreign investment up to 100 % of equity automatic, 49-100% by FIPB).
participation is allowed. The third category has two
subsets – 74% FDI
a. one consisting of sectors where automatic approval Airports, broadcasting, coal and lignite, credit information
is granted for FDI (often foreign equity participation companies, direct to home (DTH), mining ( diamonds &
less than 100 %), and precious stones), satellites, and private sector banking are the
b. the other consisting of sectors where prior approval sectors with FDI limit of 74%.
from the Foreign Investment Approval Board (FIPB)
26-49% FDI
is required.
FDI policy changes increasingly reflect the requirements of Airlines/aviation, defence, insurance and pension are the
industry and are based on stakeholder’s consultation. Upfront sectors which have 49% FDI limit. Sectors with 26% FDI
listing of negative sectors has helped focus on reform areas, limits print media (newspaper – 26%, scientific & periodicals
which are reflected in buoyant FDI inflows. – 100% ) and FM radio. Public sector Banks have the lowest
FDI limit of 20%.
E-130 Foreign Trade and Investment in India

1. Which of the following organisation provides guarantee 9. Which of the following is not one of the features of
to exporters? the Special Economic Zones (SEZ) being set up for
(a) EXIM Bank promoting exports ?
(b) Export Loan Guarantee Corporation (a) The SEZ area will be treated as foreign territory for
(c) RBI trade operations, duties and tariff.
(d) Commerce Ministry (b) No licence is required for import into the zone.
2. Which of the following does not form part of current (c) Foreign workers will be allowed free entry without
account of Balance of Payments? visa restrictions.
(a) Export and import of goods (d) There will be no routine examination by customs
(b) Export and import of services authorities of import/export cargo.
(c) Income receipts and payments 10. Which of the following is considered lending for
(d) Capital receipts and payments promotion of exports?
3. Which institution is known as ‘soft loan window’ of (a) Packing Credit
World Bank? (b) Overdraft
(a) IFC (c) Cash Credit Account
(b) IDA (d) Bill Discounting
(c) IMF 11. For National Manufacturing and Investment Zones
(d) Indian Development Forum (NMIZ), Special Economic Zone (SEZ) and EOUs
4. Global capital-flows to developing countries increased (Exports-Oriented Units), which of the following
significantly during the nineties. In view of the East statement is true?
Asian financial crisis and Latin American experience, (a) NMIZs and EOUs will be located within SEZs
which type of inflow is good for the host country? (b) SEZs and EOUs will be located within NMIZs.
(a) Commerical loans (c) NMIZs are independent of SEZs and EOUs.
(b) Foreign Direct Investment (d) NMIZs and SEZs will be competitors in nature
(c) Foreign Portfolio Investment 12. Both Foreign Direct Investment (FDI) and Foreign
(d) External Commercial Borrowings Institutional Investor (FII) are related to investment in
5. The earnings of India from diamond export is quite high. a country. Which one of the following statements best
Which one of the following factors has contributed to it? represents an important difference between the two ?
(a) pre-independene stock-piling of diamonds in the (a) FII helps bring better management skills and
country which are now exported technology, while FDI only brings in capital.
(b) large production of industrial diamonds in the (b) FII helps in increasing capital availability in general,
country while FDI only targets specific sectors.
(c) expertise available for cutting and polishing of (c) FDI flows only into the secondary market while FII
imported diamonds which are then exported
targets primary market
(d) as in the past, India produces huge quantity of gem
(d) FII is considered to be more stable than FDI.
diamonds which are exported
13. A great deal of Foreign Direct Investment (FDI) to India
6. Which one of the following modes of privatization is the
comes from Mauritius than from many major and mature
most comprehensive and complete?
economies like UK and France. Why?
(a) introduction of private capital in public sector
(a) India has preference for certain countries as regards
(b) contracting out management of public enterprises to
receiving FDI
the private sector
(c) transferring ownership and management to the (b) India has double taxation avoidance agreement with
workers Mauritius
(d) transferring ownership and management to the (c) Most citizens of Mauritius have ethnic identity with
private sector India and so they feel secure to invest in India
7. Which one of the following types of borrowings from the (d) Impending dangers of global climatic change prompt
IMF has the softest servicing conditions? Mauritius to make huge investments in India
(a) Second tranche loan 14. TRIPS (Trade Related aspects of Intellectual Property
(b) SAF Rights) agreements is administered by
(c) ESAF (a) United Nations Conference on Trade and
(d) Oil facility Development (UNCTAD)
8. Which unit of valuation is known as ‘paper gold’? (b) United Nations Organization (UNO)
(a) Eurodollar (b) Petrodollar (c) World Trade Organization (WTO)
(c) SDR (d) GDR (d) World Bank (WB)
Foreign Trade and Investment in India E-131

15. Which is the role of the International Monetary Fund (c) The foreign capital earns profit in the country which
(IMF)? is repatriated
(a) To implement and advance global trade agreements (d) The Balance of Trade of the country is badly affected
(b) To settle industrial and trade disputes between by Foreign Capital Inflows
members 23. TANKAN is revision of the Industry Classification of
(c) To help poorer countries with their economic the Short-term Economic Survey of Enterprises (usually
development quarterly ) is used in following country ?
(d) To maintain international financial stability in global (a) Japan (b) South Korea
financial markets (c) China (d) All of them
16. Which of the following is not an argument for 24. Which among the following is the most important source
protectionism? region of NRI remittances to India ?
(a) To protect infant industries (a) North America (b) Europe
(b) To increase the level of imports (c) Middle East (d) Asia Pacific
(c) To protect strategic industries (e) South America
(d) To improve the balance of payments 25. In context with the two way trade of India with different
17. Dumping in the context of international trade refers to : regions, which among the following region is India’s
(a) Exporting goods at prices below the cost of largest trade partner?
(a) EU Region (b) Gulf Region
production
(c) North America (d) Latin America
(b) Exporting goods of inferior quality
26. Which among the following will be a debit entry in India’s
(c) Exporting goods only to re-import them at cheaper
balance of payments?
rates
(a) Imports of goods by India.
(d) Exporting goods without paying appropriate taxes in (b) Income of Indian investments abroad.
the receiving country (c) Receipts of transfer payments.
18. Foreign Direct Investment involves: (d) Exports of services by India.
(a) A speculator trying to make a profit by buying 27. Which among the following countries is currently the
company shares on a foreign stock exchange. biggest supplier of crude oil to India?
(b) A UK energy company buying territory abroad (a) Iran (b) Saudi Arabia
where it expects to find oil reserves. (c) UAE (d) Nigeria
(c) A tourist purchasing foreign currency to spend on a 28. IMF can grant loan to __________?
holiday abroad. (a) Any sovereign country of the World
(d) A company signing an agreement with a wholesaler (b) Any sovereign country of the World and Public
to distribute its products in foreign markets. Sector companies backed by Sovereign guarantee
19. Many a times we read about Hot Money in newspapers. (c) Any Member country of IMF
Which among the following options rightly describes hot (d) Any Member country of IMF and Public Sector
money? Companies backed by Sovereign guarantee of
(a) Hot money is useful and generally durable and is Member Country ‘
good for the country in all weathers 29. In which among the following forms, the Special Drawing
(b) Hot money is dangerous and volatile and leaves the Rights (SDR) are kept as currency of International
country in bad weather conditions Monetary Fund?
(c) Hot money is good and adds to the development of (a) Paper Currency
the country & it comes from exports of services (b) Gold
(d) Hot money is bad & useless as its arises from (c) Book Keeping Entry
unusual activities like casinos, gambling, horse (d) A combination of all of three
races, speculations etc. 30. A new term Lourdes Treatment and Resuscitation Option
20. A systematic record of all economic transactions (LTRO) was making news in context with which among
completed between residents of a country and the rest of the following?
the world in a year is known as..? (a) World Bank
(a) Net Capital Flow (b) Balance of Payment (b) International Monetary Fund
(c) Balance of Trade (d) Absolute Flow (c) European Central Bank
21. Asian Financial Crisis of 1997 started from which of the (d) Federal Reserve Bank of America
following countries? 31. Christine Lagarde is the head of which among the
(a) Myanmar (b) Thailand following international agencies / bodies?
(c) Cambodia (d) Malaysia (a) Asian Development Bank
22. Many a times we read in the newspapers that when (b) Non-alignment Movement
Foreign Capital is allowed to enter the country freely, (c) International Atomic Energy Agency
it can affect the economy adversely. Which among the (d) International Monetary Fund
following is a correct reason for the above assumption? 32. AoA in context with World Trade Organization is ___?
(a) It affects the balance of payments of the country by (a) Article of Association
adversely affecting the Current Account (b) Agreement on Agriculture
(b) It poses risks to the value of the country’s currency (c) Agreement on Association
as well as management of local liquidity (d) Administration of Agriculture
E-132 Foreign Trade and Investment in India

6. The New Economic Policy (1991) was launched in the


Statement Based MCQ background of the following economic indicators:
1. Consider the following statements. 1. India’s foreign exchange reserves had fallen to
Most internationals agencies which fund development US$1 billion.
programmes in Indian on inter-government bilateral 2. The fall of the Soviet Union had deprived India of
agreements, mainly provide. almost a quarter of its export market.
1. Technical assistance 3. There was negative growth in real GDP.
2. Soft loans which are required to be paid back with 4. Indian rupee had to be devalued by 45 per cent.
interest (a) 1, 2, 3 and 4 (b) 1 only
3. Grants, not required to be paid back (c) 1 and 2 (d) 1, 2 and 3
4. Food assistance to alleviate poverty 7. Which of the following pairs are correctly matched?
(a) 2 and 4 are correct (b) 1, 2 and 3 are correct 1. Increase in—Monetary expansion
(c) 1, 2 and 4 are correct (d) 3 and 4 are correct 2. Low import growth rate in India-Recession in Indian
2. FDI in Multi-Brand Retail Trade (MBRT) in all products
industry
is now permitted in India subject to
3. Euro-issues—Shares held by Indian companies in
1. a ceiling of 51%
2. minimum amount to be brought in as FDI by the European countries
foreign investor is US $ 100 million. 4. Portfolio investment—Foreign institutional investors.
3. atleast 50% of the procurement of manufactured/ (a) 1, 2 and 4 (b) 3 and 4
processed products should be sourced from ‘small (c) 1, 2 and 3 (d) 1, 2, 3 and 4
industries’. 8. Which of the following would include Foreign Direct
4. retail sales locations set-up only in cities with a Investment in India?
population of more than 10 lakh. 1. Subsidiaries of foreign companies in India
Select the correct answer using the codes given below 2. Majority foreign equity holding in Indian companies
(a) 1, 2, 3 and 4 (b) 1 and 4 3. Companies exclusively financed by foreign
(c) Only 2 (d) 1, 2 and 4 companies
3. Which of the following were the aims behind the setting 4. Portfolio investment
up of the World Trade Organization (WTO)? Select the correct answer using the codes given below
1. promotion of free trade and resource flows across (a) 1, 2, 3 and 4 (b) 2 and 4
countries
(c) 1 and 3 (d) 1, 2 and 3
2. protection of intellectual property rights
9. Which of the following is correct about ‘Foreign direct
3. managing balanced trade between different countries
4. promotion of trade betwen the former East Bloc investment’ (FDI)?
countries and the Western World 1 Through FDI a company buys a company in the
(a) 1, 2, 3 and 4 (b) 1 and 2 target country.
(c) 2 and 3 (d) 1 and 4 2. Through FDI a company expand operations of an
4. Consider the following statements in respect of the existing business in the target country.
functions of World Trade Organisation (WTO): 3. FDI is an investment in the securities of another
1. It seeks to make international trade free by establishing country such as stocks and bonds.
predictable and transparent ground rules. 4. FDI is an investment directly into production in a
2. It promotes capital flow from developed to country by a company located in another country.
developing countries. (a) 1 and 2 (b) 1, 2 and 4
3. It provides a forum for trade negotiations. (c) 1, 2 and 3 (d) 1, 2, 3 and 4
4. It works as a neutral agency for settlement of trade 10. International Monetary Fund (IMF) argues that developing
disputes. economies hold a massive stockpile of foreign exchange.
Identify the correct statements from the above:
Which could be the consequences of such stockpiling?
(a) 1, 3 and 4 (b) 2, 3 and 4
1. A trade imbalance or major difference in value of
(c) 1, 2, 3 and 4 (d) 1, 2 and 3
5. Which of the following statement is not correct in relation imports/exports is generated between the developing
to International Monetary Fund? and emerging economies and the rich countries.
1. India is a founder member of the IMF. Such an imbalance is not a healthy sign.
2. IMF conducts regular review of India’s economic 2. These imbalances have caused the current global
status under Article IV. economic crisis.
3. India’s quota in the IMF is more than 2 per cent 3. Such stockpiling is responsible not only for problems
4. Finance Minister is ex-officio Governor of the IMF. in trade balances but leads to money laundering.
(a) 1 and 4 (b) 3 only (a) 1 only (b) 2 only
(c) 1 only (d) 2 and 4 (c) 3 only (d) 1, 2 and 3
Foreign Trade and Investment in India E-133

11. Consider the following statements relating to the World 17. We read in the newspapers that the Foreign Direct
Trade Organization (WTO): Investments are preferred over the Capital Inflow. In this
1. The WTO deals with the global rules of trade context, please consider the following :
between nations. 1. FDI brings in latest technology
2. The goal of the WTO is to help producers of goods 2. FDI does not involve large outflow
and services, exporters, and importers conduct their 3. FDI Improves efficiency of the economy
business. Which among the above is / are most suitable reasons for
3. The WTO, which is a successor body of the General the given statement?
Agreement on Tariffs and Trade, came into being (a) Only 1 (b) Only 1 & 3
following the Uruguay Round of Negotiations. (c) Only 1 & 2 (d) All 1, 2 & 3
4. The WTO distances itself in framing of rules on trad 18. Which of the following constitute the Capital Account ?
in intellectual property rights. 1. Foreign Loan.
Which of the statements given above are correct? 2. Foreign Direct Investment.
(a) l, 2 and 3 (b) 2, 3 and 4 3. Private Remittances.
(c) 1, 2 and 4 (d) 1 and 3 only 4. Portfolio Investment.
12. Foreign Direct Investments are preferred over Capital Select the correct answer using the codes given below
Inflow. In this context, please consider the following. (a) 2 & 4 (b) 1 & 3
1. FDI brings in latest technology (c) 1 & 2 (d) 1, 2 & 4
2. FDI does not involve large outflow 19. Consider the following:
3. FDI improves working efficiency 1. International Bank for Reconstruction and
Which is/are most suitable reason/reasons for the given Development (IBRD)
statement? 2. International Development Association (IDA)
(a) 1 only (b) 1 and 3 3. International Finance Corporation (IFC)
(c) 1 and 2 (d) All 1, 2 and 3 4. Multilateral Investment Guarantee Agency (MIGA)
13. Consider the following: 5. International Centre for Settlement of Investment
1. Balance of trade 2. Net factor income Disputes (ICSID)
3. Net transfer payments Which among the above constitute the World Bank
Which among the above make the part of the “Capital group?
Account”? (a) 1 2 (b) 1 2 3
(a) Only 1 (b) 1 & 2
(c) 1 2 3 4 (d) 1 2 3 4 5
(c) 2 & 3 (d) 1, 2 & 3
20. Consider the following statements:
14. Consider the following statements:
1. The Foreign Exchange Management Act FEMA
1. Indian Rupee has faced 2 devaluations till now
became an act on the 1st day of June, 2002.
2. Indian Rupee faced highest devaluation in 1966
Which among the above statements is / are correct? 2. It replaced FERA.
(a) Only 1 is correct 3. It made all offenses regarding foreign exchange civil
(b) Only 2 is correct offenses, as opposed to criminal offenses as dictated
(c) Both 1 and 2 are correct by FERA.
(d) Neither 1 nor 2 is correct Which of the statements given above is/are correct?
15. Consider the following statements regarding the Indian (a) 1 and 2 (b) 2 and 3
Depository Receipts? (c) 1 and 3 (d) None
1. An IDR declares ownership of shares of a Foreign 21. Consider the following statements:
Company 1. Kelkar Committee is associated to oil and gas.
2. The Shares in an IDR are held by an Indian Custodian 2. Kelkar Committee has to prepare a roadmap for
3. Using IDR, the Indian Residents get a chance to enhancing import of oil and gas.
invest in a listed foreign entity Which of the statements given above is/are correct?
Which among the above statements is / are correct? (a) 1 only (b) 2 only
(a) Only 1 (b) Only 2 (c) 1 and 2 (d) None
(c) Only 1 & 2 (d) Only 1 & 3 22. Consider the following statements:
16. Market Economy: 1. The Petrapole-Benapole border checkpoint controls
1. The role of market forces and competition is the foreign trade between India and Bangladesh.
eliminated by law 2. Petrapole is on Bangladesh side and Benapole is on
2. There is a high degree of competition in both Indian side.
commodity and factor markets Which of the statements given above is/are correct?
3. Private gains are the main motivating and guiding (a) 1 only (b) 2 only
force for carrying out economic activities (c) 1 and 2 (d) None
Which among the above is/ are correct statements? 23. Which of the following would include Foreign Direct
(a) Only 1 & 2 are correct Investment in India?
(b) Only 2 & 3 are correct 1. By incorporating a wholly owned subsidiary or
(c) All are correct company anywhere
(d) None of them is correct 2. By acquiring shares in an associated enterprise
E-134 Foreign Trade and Investment in India

3. Through a merger or an acquisition of an unrelated II. Under a fixed system, a currency can rise or fall due
enterprise to changes in demand or supply of currencies on the
4. Participating in an equity joint venture with another foreign exchange market.
investor or enterprise III. Under this imports and exports can readjust to move
Select the answer using the codes given below. the balance of payments back towards a desirable
(a) 1 and 2 (b) 2, 3 and 4 equilibrium.
(c) 1, 3 and 4 (d) 1, 2, 3 and 4 (a) I & II (b) I & III
24. Consider the following statements: (c) II & III (d) Only I
1. International Monetary Fund (IMF) was initiated in 30. Why did the fixed exchange rate system collapsed?
1944 at the Bretton Woods Conference and formally I. The build up of US debts abroad as a result of the
created in 1945. need to fund the war in Vietnam
2. IMF grants loan to member country and other II. Inflation in the USA
developing countries. III. Growing doubts about the stability of the US$.
Which of the statements given above is/are correct? (a) Only I (b) I & II
(a) 1 only (b) 2 only (c) II & III (d) All the above
(c) 1 and 2 (d) None 31. Match the following percentage of world trade in different
25. Consider the following statements regarding the years of India:
Federation of Indian Export Organisation (FIEO): Year Percentage
1. FIEO was set up in 1965 under the aegis of Ministry I. 1950’s a. 0.59%
of Commerce. II. 1990’s b. 2%
2. It is an apex body of Export Promotion Organisations III. 2015 c. 1.78%
3. FIEO renders an integrated package of services (a) I –b, II – c, III – a (b) I – c, II – a, III – b
to various organizations connected with export (c) I - a, II- b, III – c
promotion. 32. What are the categories in which India export?
Which of the statements given above is/are correct? I. Agricultural and Allied Products
(a) 1 only (b) 2 only II. Mineral Fuel and Lubricants
(c) 1 and 2 (d) 1, 2 and 3 III. Coal and petroleum
26. Match the following countries with its currencies: (a) Only II (b) I & II
Country Currency (c) Only III (d) None of the above
I. US a. Yen 33. What are the facts regarding structural change in India’s
export since 1991?
II. China b. Dollar
I. There are indications that during 1990s, some
III. Japan c. Pound
of Indian exports have moved upwards in value
IV. Britain d. Yuan
addition chain whereby instead of exporting raw
(a) I – b, II – d, III – a, IV – c
materials, the country has switched over to export
(b) I – a, II – d, III – b, IV – c
of processed goods.
(c) I – b, II – c, III – a, IV – d
II. There were significant compositional shift within
(d) I – a, II – c, III – b, IV – d
the major manufactured product groups such as
27. Which sentence is correct regarding foreign exchange?
engineering goods, chemicals and allied products,
I. Foreign exchange reserves in India comprises of 6
etc.
elements.
(a) Only II (b) Only I
II. Foreign exchange management can be done in three (c) Both I & II (d) None of the above
possible ways: Fixed currency regime, floating 34. Match the following export products of India with the
currency regime and managed exchange rates. Percentage they constitute:
III. Silver is an element of foreign exchange reserve Products Percentage
(a) I & II (b) Only II I. Petroleum a. 4%
(c) Only III (d) None of the above II. Gems & Jewellery b. 7.8%
28. Which sentences are correct regarding special drawing III. Transport & c. 20.4%
rights? Equipments
I. It was created in 1980 IV. Machinery d. 16.3%
II. It is also known as “paper gold”. (a) I – c, II – d, III – b, IV – a
III. Its value is based on a basket of five key international (b) I – a, II – c, III – b, IV – d
key currencies and SDRs can be exchanged for (c) I – b, II – a, III – d, IV – c
freely usable currencies (d) I – d, II – c, III – a, IV – b\
(a) Only II (b) I & III 35. Which statements are true regarding India’s Import?
(c) II & III (d) All the above I. In 1947-48 the main items of India’s imports were
29. Which of the statement is correct according to Fixed machineries, oil, grains, cotton, cutlery, hardware
Exchange rates? implements, chemicals, etc.
I. Fixed rates are currency values which are tied to a II. They constituted 70% of India’s imports
precious metal such as gold, or anchored to another (a) Only II (b) Only I
currency, like the US Dollar. (c) Both I & II (d) None of the above
Foreign Trade and Investment in India E-135

36. What is meant by term Balance of Payment? (a) Only II (b) Only I
I. Those transactions arising out of exports and imports (c) Both I & II (d) None of the above
(the visible items) 42. Which of the below statements are correct?
II. It is astatistical statement of all transactions made I. Reverse account balance makes an adjustment
between one particular country and all other between current account balance and capital account
countries during a specified period of time balance.
III. This account is the summary of all international II. If surplus in the Capital Account is more than deficit
trade transactions of the domestic country in one in the Current Account, there is net increase in the
year Forex Reserves of the country at the end of the year.
(a) I & III (b) Only I III. If deficit in the current account is more than surplus
(c) Only II (d) None of the above in the Capital Account then there is net decrease in
37. What is meant by term Balance of Trade?
Foreign Reserves of the country at the end of the
I. Those transactions arising out of exports and imports
year.
(the visible items)
(a) I & II (b) I & III
II. It is a statistical statement of all transactions made
between one particular country and all other (c) Only II (d) All the above
countries during a specified period of time 43. Which statement is correct regarding balance of payment
III. This account is the summary of all international crises?
trade transactions of the domestic country in one I. If international reserves of a country are not enough
year to balance a combined deficit in current and capital
(a) I & III (b) Only I account on a sustained basis, then the phenomenon
(c) Only II (d) None of the above is called a BoP crisis.
38. Which statement is correct regarding Current account? II. It can be tackled by exporting more or by limiting
I. Those transactions arising out of exports and imports imports through tariffs, quotas, etc. Contractionary
(the visible items) fiscal and monetary policies can also tackle the crisis
II. It is a statistical statement of all transactions made through lower import demand with fall in average
between one particular country and all other income levels.
countries during a specified period of time III. Another short term solution is currency devaluation
III. This account is the summary of all international which encourages exports and discourages costlier
trade transactions of the domestic country in one imports.
year (a) Both I & II (b) Only I
(a) I & II (b) Only I (c) II & III (d) All the above
(c) II & III (d) Only III 44. Which statement is incorrect regarding India’s balance of
39. Which statement is incorrect regarding balance of payment?
payment? I. 1994-95 was the year of the acute BOP crisis in India.
I. BoP compares the dollar difference of the amount of II. India then had foreign exchange reserve of mere 3
exports and imports, including all financial exports billion dollar
and imports.
III. This crisis led to the massive changes in the
II. A negative balance of payments means that more
country’s economic policy, popularly known as the
money is flowing out of the country than coming in
Structural Adjustment Program or New Economic
and vice versa.
III. The balance of payment is maintained by State Bank Policy (NEP) regime, focusing on liberalization and
of India globalization of the economy.
(a) I & III (b) Only III (a) I & III (b) I & II
(c) II & III (d) Only I (c) Only III (d) Only I
40. Match the following current accounts with their examples: 45. What conditions are correct that came with the medium
CA term loan given by IMF to India for restructuring of the
I. Visible Trade a. Gifts economy?
II. Invisible Trade b. Tea I. Government expenditure to be cut by 20% per annum
III. Unilateral Transfer c. Insurance II. Devaluation of rupee by 22%.
(a) I – b, II – c, III – a III. Excise duty to be increased by 40% .
(b) I – a, II – c, III – b IV. Custom duty to be cut drastically from the peak of
(c) I – c, II – a, III – b 130% to 30%
41. Which statement is correct regarding capital account? (a) II & IV (b) I & II
I. On the credit side of this account receipt of foreign (c) III & IV (d) Only II
exchange due to Foreign Direct Investment (FDI), 46. What are the factors on which import substitution strategy
Foreign Capital Investment (FCI) and Foreign was based on?
Borrowing (FB) is recorded. I. Non-price
II. On the debit side of capital account payment of II. Physical- interventionist policies like licensing,
foreign exchange due to Direct Investment Abroad quotas and other physical restrictions on imports
(DIA), Portfolio Investment Abroad (PIA) and (a) only II (b) Only I
Foreign Lending (FL) is recorded. (c) Both I & II (d) None of the above
E-136 Foreign Trade and Investment in India

47. Why there was the need for export promotion in India? IV. FTP benefits from both MEIS & SEIS will be
I. Primary products exporting countries have an extended to units located in SEZs.
unfavourable term of trade. The earnings from (a) I & II (b) II & IV
primary product exports were low and unstable (c) Only III (d) All the above
II. Secondly, the quality of Indian products was not up 53. Which statement is not correct according to MEIS?
to the world standards due to which we could not I. Agricultural and village industry products to be
sustain markets. supported across the globe at rates of 3% and 5%
III. Third, only residue products were mainly exported.
under MEIS.
The fact that export earnings also contribute to
II. Higher level of support to be provided to processed
economic development was overlooked.
(a) I & III (b) II & III and packaged agricultural and food items under
(c) Only III (d) All the above MEIS.
48. Which of the below statements are correct regarding (a) Only I (b) Only II
exchange trade? (c) Both I & II (d) None of the above
I. The value of rupee was managed by the state bank 54. What are the objectives of SEZ act?
II. The strict foreign exchange controls also encouraged I. Generation of additional economic activity.
hawala trade II. Promotion of exports of goods & services.
III. India followed a strongly inward looking policy, III. Promotion of investment from domestic and foreign
laying stress on import substitution sources.
(a) Only I (b) Both II & III IV. Creation of employment opportunities
(c) Only III (d) All the above (a) I & II (b) II & IV
49. Choose the correct sentence regarding the trade policy: (c) Only III (d) All the above
I. Mahalanobis strategy adopted during the First plan 55. Which points are correct regarding SEZ act?
II. Export increased at an average rate of 29 per cent I. Exemption to SEZ developer and units from
per annum in dollar terms between 1986 and 1990 Minimum Alternate Tax.
III. A combination of factors such as bad policy, weak
II. Constitution of an authority for each SEZ with a
government and external factors led to the decline of
view to providing greater administrative, financial
this performance to nine per cent in 1990-91 and 4
and functional autonomy to these zones.
per cent in the subsequent years.
(a) Only III (b) I & II III. Establishment of designated courts and a single
(c) II & III (d) All the above enforcement agency to ensure speedy trial and
50. What was the main reason of introducing trade reforms investigation of offences committed in SEZs.
in 1991? (a) I & II (b) Only II
I. Make exports competitive (c) Only III (d) All the above
II. Unshackle foreign trade from the clutches of a 56. Consider the following statements:
control regime I. Agriculture, horticulture, poultry, fisheries and
III. Allow import of most goods using only tariff as a dairying have been included in export-oriented
restraint units.
(a) I & II (b) Only II II. Export processing zone units have also been allowed
(c) Only III (d) All the above to export through trading and star trading houses
51. What are the key features of trade policy? and can have equipment on lease.
I. India to be made a significant participant in world Which of the following statements are true regarding
trade by 2025 Export Units:
II. Merchandize exports from India (MEIS) to promote (a) Only II (b) Only I
specific services for specific Markets Foreign Trade (c) Both I & II (d) None of the above
Policy 57. Consider the following statements:
III. FTP would reduce export obligations by 50% and
I. These zones were created to develop such an
give boost to domestic manufacturing
environment in the economy which may provide
(a) I & II (b) Only II
(c) II & III (d) All the above capability of facing international competition.
52. Which is the correct statement according to FTP 15 – 20? II. The Export Processing Zone (EPZs) set up as
I. FTP 2015-20 introduces two new schemes, namely enclaves, separated from the Domestic Tariff
“Merchandise Exports from India Scheme (MEIS)” Area by fiscal barriers, were intended to provide
and “Services Exports from India Scheme (SEIS)” a competitive duty free environment for export
II. These schemes (MEIS and SEIS) replace multiple production
schemes earlier in place, each with different III. There are total 10 EPZs in our country
conditions for eligibility and usage. Which statement is/are correct regarding export
III. Incentives (MEIS & SEIS) to be available for SEZs processing zone
also e-Commerce of handicrafts, handlooms, books (a) Only III (b) I & II
etc., eligible for benefits of MEIS. (c) II & III (d) All the above
Foreign Trade and Investment in India E-137

58. Match the States with their cities where there are EPZs in that India becomes an increasingly attractive and
India: investor-friendly destination.
States City (a) I & II (b) II & III
I. Gujarat a. Noida (c) Only IV (d) All the above
II. West Bengal b. Kandla 64. Are the statements given below the subset of the third
III. U.P c. Faeta (it is a placein West category of FDI?
Bengal which is mentioned I. It consists of sectors where automatic approval is
in the chapter) granted for FDI (often foreign equity participation
(a) I-b, II-c, III-a (b) I-a, II-c, III-b less than 100 per cent)
(c) I-b, II-a, III-c (d) I-c, II-b, III-a II. It consists of sectors where prior approval from
59. Which statement is correct regarding export oriented the Foreign Investment Approval Board (FIPB) is
units? required
I. Since 1981, the Government introduced a (a) Only I (b) Only II
complementary plan of EPZ (Now converted into (c) Both I & II
SEZ) scheme for promoting export units. 65. What are the sectors with 100% FDI?
II. Under this scheme the Government provides various I. Agriculture
incentives to increases the production capacity of II. Advertising
these units so as to increase exports of the country. III. Air transport service
III. This scheme offers a wider source of raw materials, IV. Drugs and pharmaceuticals
hinterland facilities, availability of technological (a) I & II (b) II & IV
skills, existence of an industrial base and the need (c) Only III (d) All the above
for a larger area of land for the project. 66. Match the following sectors with the percentage of FDI:
(a) Only III (b) I & II Sectors Percentage
(c) II & III (d) All the above I. Private sector a. 49%
60. Which statement is correct regarding Export houses? banking
I. To increases the marketable efficiency of exporters, II. Defence b. 20%
the Government introduced the concept of export III. Print media c. 74%
houses, trading houses and star trading houses. IV. Public sector banks d. 26%
II. Those registered exporters who have shown good (a) I-c, II-a, III-d, IV-b (b) I-d, II-a, III-c, IV-b
export performances over past few years have been (c) I-a, II-a, III-c, IV-b (d) I-c, II-d, III-a, IV-b
given the status of export houses, and trading houses. 67. Foreign exchange reserves of India comprise of which
III. Units having such classification are required to elements?
achieve the prescribed average export performance I. Gold
level and earning of foreign exchange. II. Special Drawing Rights (SDR)
III. Foreign currency assets
(a) Only III (b) I & II
IV. Reserve Tranche Position (RTP) in the IMF
(c) II & III (d) All the above
(a) I & II (b) II & III
61. Which statement is correct regarding EPIP?
(c) Only III (d) All the above
I. EPIP was introduced in 1996
68. Which statement is correct regarding FDI?
II. It is sponsored by centre
I. The success of foreign exchange management by RBI
III. It provides 80% grant to the state
can be seen in the fact that from US$ 5.8 billion in
(a) II & III (b) Only I March 1991 (enough to meet the import requirements
(c) I & II (d) All the above for three weeks) the country’s reserves have grown
62. Match the area with the state where there are EPIPs in to around US $ 352 billion by December 2015.
India: II. It is pertinent to note here that some of the increase
Column I Column II in the reserve is attributed to the weakness of the
I. Rajasthan a. Amingaon dollar in the post-Iraq war scenario.
II. Maharashtra b. Sitapura (a) Only I (b) Only II
III. Kerala c. Ambarnath (c) Both I & II
IV. Assam d. Kakkinad 69. Imports of India are classified into:
(a) I-b, II-c, III-d, IV-a (b) I-a, II-c, III-d, IV-b I. Petroleum Products
(c) I-d, II-c, III-b, IV-a (d) I-c, II-b, III-a, IV-d II. Pearls and Precious Stones
63. Which statement is correct regarding FDI? III. Iron and Steel
I. FDI is a non-debt capital flow, is a leading source IV. Fertilizers
of external financing, especially for the developing (a) I & II (b) Only II
economies. (c) Only IV (d) All the above
II. It not only brings in capital and technical know- 70. Match the following imports of products with the
how but also increases the competitiveness of the percentage:
economy. Column I Percentage
III. Overall it supplements domestic investment, much I. Gold a. 7.3%
required for sustaining the high growth rate of the II. Petroleum b. 12.7%
country. III. Electronic goods c. 30.7%
IV. Since 2000, significant changes have been made in (a) I-b, II-c, III-a (b) I-a, II-c, III-b
the FDI policy regime by the government to ensure (c) I-c, II-b, III-a (d) I-b, II-a, III-c
E-138 Foreign Trade and Investment in India

EXERCISE-1 The reach of retail sales outlets of foreign multi


1. (b) brand retail trader will be limited to only those
2. (d) Capital receipts and payments do not form part of cities with a population of 1 million (including an
current account of Balance of Payment. area of 10 kilometres around the municipal/urban
3. (b) 4. (b) 5. (c) 6. (d) 7 (c) agglomeration limits of such cities).
8. (c) 9. (c) 10. (a) 11. (b) 12. (b) 3. (b) 4. (a) 5. (b) 6. (c)
13. (b) India has comprehensive Double Taxation 7. (a) Euro issue includes issue of ADR (American
Avoidance Agreements (DTAA ) with 23 countries. Depositary Receipts) and GDR (Global Depositary
This means that there are agreed rates of tax and Receipts). A scheme has been initiated during 1992
jurisdiction on specified types of income arising under which Indian companies are permitted to raise
in a country to a tax resident of another country. foreign currency resources through issue of Foreign
Under the Income Tax Act 1961 of India, there are Currency Convertible Bonds (FCCBs) and/or issue
two specific provisions, Section 90 and Section 91, of ordinary equity shares through Global Depositary
which provide specific relief to taxpayers to save Receipts (GDRs)/American Depositary Receipts
them from DTAA. Section 90 is for taxpayers who (ADRs) to foreign investors i.e. institutional
have paid the tax in a country with which India has investors or individuals (including NRIs) residing
signed DTAA, while Section 91 provides relief to abroad.
taxpayers who have paid tax to a country with which 8. (d) All statements except 4th regarding F.D.I in India
India has not signed a DTAA. Thus, India gives are correct.
relief to both kind of taxpayers Mauritius by itself is 9. (b) 10. (d)
a low tax counting. 11. (a) The World Trade Organization (WTO) is the only
14. (c) 15. (d) global international organization dealing with the
16. (b) Protectionism would reduce the level of imports into rules of trade between nations. At its heart are the
an economy. WTO agreements, negotiated and signed by the
17. (a) bulk of the world’s trading nations and ratified in
18. (b) The energy company will own and control the
their parliaments. The goal is to help producers
territory and the oil reserves it contains.
of goods and services, exporters and importers
19. (b) 20. (b) 21. (b) 22. (b) 23. (a) 24. (a)
conduct their business. The Uruguay Round led to
25. (b) 26. (b) 27. (b) 28. (c) 29. (c)
the creation of the World Trade Organization, with
30. (c) European Central Bank European Central Bank’s
GATT remaining as an integral part of the WTO
(ECB’s) LTRO — Long Term Refinancing Operation
agreements. The agreements fall into a simple
is more appropriately termed the Lourdes Treatment
structure with six main parts, intellectual property
and Resuscitation Option.
31. (d) (Agreement on Trade-Related Aspects of Intellectual
32. (b) Different agreements of WTO are: Property Rights (TRIPS)) was one of them.
1. Multi-Fiber agreement (MFA). 12. (d) 13. (d) 14. (b) 15. (d)
2. Agreement on agriculture (AOA). 16. (b) 17. (a) 18. (d) 19. (d)
3. Trade related investment measures (TRIMS). 20. (b) The Foreign Exchange Management Act (FEMA)
4. Trade related intellectual property right (TRIPS). has been introduced as a replacement for earlier
5. General agreement on trade and services (GATS) Foreign Exchange Regulation Act (FERA). FEMA
became an act on the 1st day of June, 2000. FEMA
EXERCISE-2 made all offenses regarding foreign exchange civil
1. (b) offenses, as opposed to criminal offenses as dictated
2. (d) In respect to multi-brand retail trading, changes by FERA.
made in 2012 permitted up to 51 % FDI with prior 21. (a) Vijay Kelkar Committee is prepared to prepare
government approval. The foreign investor has a road map for enhancing domestic production
to bring in a minimum investment of USD 100 of oil and gas so as to reduce the nation’s import
million in an entity engaged in multi brand retail dependency by 2030.
trading. Similar to the requirement of mandatory 22. (a) The Petrapole-Benapole border checkpoint controls
local sourcing as applicable in single brand product the foreign trade between India and Bangladesh. It
trading (prior to Cabinet meeting) at least 30% is in North 24 Parganas district of West Bengal. Out
of the procurement of manufactured/ processed of them, Petrapole is on Indian side and Benapole is
products shall be sourced from `small industries`. on Bangladesh side.
Foreign Trade and Investment in India E-139

23. (d) Foreign direct investment (FDI) is a direct 39. (b) BoP compares the dollar difference of the amount of
investment into production or business in a country exports and imports, including all financial exports
by an individual or company of another country, and imports. A negative balance of payments means
either by buying a company in the target country or that more money is flowing out of the country than
by expanding operations of an existing business in coming in and vice versa. The balance of payment
that country. Foreign direct investment is in contrast is maintained by Central Bank of India i.e. Reserve
to portfolio investment. Bank of India.
24. (a) International Monetary Fund (IMF) was initiated in 40. (a)
1944 at the Bretton Woods Conference and formally 41. (c) On the credit side of this account receipt of foreign
created in 1945 to foster global growth and economic exchange due to Foreign Direct Investment (FDI),
stability. IMF grants loan to member country only. Foreign Capital Investment (FCI) and Foreign
25. (d) FIEO an apex body of Export Promotion Borrowing (FB) is recorded. On the debit side of
Organisations was set up in 1965 to renders
capital account payment of foreign exchange due
an integrated package of services to various
to Direct Investment Abroad (DIA), Portfolio
organizations connected with export promoting
Investment Abroad (PIA) and Foreign Lending (FL)
undertaken to stimulate and diversify the country’s
export trade. is recorded.
26. (a) The basket of five international currencies includes 42. (d) If surplus in the Capital Account is more than deficit
US dollar, euro, Chinese yuan, Japanese yen and in the Current Account, there is net increase in the
British pound. Forex Reserves of the country at the end of the year.
27. (b) Foreign exchange management can be done in three On the other hand if deficit in the current account is
possible ways: Fixed currency regime, floating more than surplus in the Capital Account then there
currency regima and managed exchange rates.The is net decrease in Foreign Reserves of the country at
foreign exchange reserves of India comprise of four the end of the year.
elements 43. (d) If international reserves of a country are not enough
28. (c) SDR is an international monetary reserve currency, to balance a combined deficit in current and capital
created by International Monetary Fund (IMF) in account on a sustained basis, then the phenomenon
1969. It operates as a supplement to the existing is called a BoP crisis. It can be tackled by exporting
reserves of member countries. It is also known as more or by limiting imports through tariffs, quotas,
“paper gold”. etc. Contractionary fiscal and monetary policies
29. (d) Fixed rates are currency values which are tied to a can also tackle the crisis through lower import
precious metal such as gold, or anchored to another demand with fall in average income levels. Another
currency, like the US Dollar. This method was short term solution is currency devaluation which
brought by the International Monetary Fund (IMF). encourages exports and discourages costlier imports.
30. (d) The system collapsed in 1971 for a variety of 44. (b) Our country has been facing BOP disequilibrium
reasons, including the build up of US debts abroad right since independence, culminating into a disaster
as a result of the need to fund the war in Vietnam, in 1990-91, the year of the acute BOP crisis. India
inflation in the USA and growing doubts about the then had foreign exchange reserve of mere 1 billion
stability of the US$. dollar, barely sufficient to finance a month’s import
31. (c) 32. (b)
bill. The country was on the verge of defaulting.
33. (c) There are indications that during 1990s, some of
This crisis led to the massive changes in the
Indian exports have moved upwards in value addition
country’s economic policy, popularly known as the
chain whereby instead of exporting raw materials,
the country has switched over to export of processed Structural Adjustment Program or New Economic
goods. There were significant compositional shift Policy (NEP) regime, focusing on liberalization and
within the major manufactured product groups such globalization of the economy.
as engineering goods, chemicals and allied products, 45. (a) Government expenditure to be cut by 10% per
etc. annum, Devaluation of rupee by 22%, Excise duty
34. (a) 35. (c) to be increased by 20% so as to neutralize the effect
36. (c) Balance of payments (BoP) is astatistical statement of lodd of revenue dur to custom cut, Custom duty
of all transactions made between one particular to be cut drastically from the peak of 130% to 30%
country and all other countries during a specified 46. (c) The import substitution strategy was based on
period of time. It also include export-import of non-price, physical- interventionist policies like
visible and invisible items. licensing, quotas and other physical restrictions on
37. (b) Balance of trade takes into account only those imports.
transactions arising out of exports and imports of 47. (d) 48. (b)
only (the visible items). 49. (a) Mahalanobis strategy adopted during the Second
38. (d) This account is the summary of all international Plan and continued with modifications till the early
trade transactions of the domestic country in one 1980s. It was only from 1985-86 that a genuine
year attempt was made towards trade liberalisation. The
E-140 Foreign Trade and Investment in India

result was spectacular as export increased at an as to increase exports of the country. This scheme
average rate of 17 per cent per annum in dollar terms offers a wider source of raw materials, hinterland
between 1986 and 1990. A combination of factors facilities, availability of technological skills,
such as bad policy, weak government and external existence of an industrial base and the need for a
factors led to the decline of this performance to nine larger area of land for the project.
per cent in 1990-91 and 4 per cent in the subsequent 60. (d) To increases the marketable efficiency of exporters,
years. the Government introduced the concept of export
50. (d) The major thrust was to make exports competitive, houses, trading houses and star trading houses.
unshackle foreign trade from the clutches of a control Those registered exporters who have shown good
regime and allow import of most goods using only export performances over past few years have been
tariff as a restraint. given the status of export houses, and trading houses.
51. (b) Units having such classification are required to
52. (d) These schemes (MEIS and SEIS) replace multiple achieve the prescribed average export performance
schemes earlier in place, each with different level and earning of foreign exchange. These units
conditions for eligibility and usage. Incentives are provided some special facilities and benefits by
(MEIS & SEIS) to be available for SEZs also the Government.
e-Commerce of handicrafts, handlooms, books etc., 61. (a) 62. (a)
eligible for benefits of MEIS. FTP benefits from 63. (d) a non-debt capital flow, is a leading source of
both MEIS & SEIS will be extended to units located external financing, especially for the developing
in SEZs. economies. It not only brings in capital and technical
53. (c) Agricultural and village industry products to be know-how but also increases the competitiveness
supported across the globe at rates of 3% and 5%
of the economy. Overall it supplements domestic
under MEIS, Higher level of support to be provided
investment, much required for sustaining the high
to processed and packaged agricultural and food
growth rate of the country. Since 2000, significant
items under MEIS.
changes have been made in the FDI policy regime
54. (d) 55. (d)
by the government to ensure that India becomes
56. (c) Agriculture, horticulture, poultry, fisheries and
an increasingly attractive and investor-friendly
dairying have been included in export-oriented
destination.
units. Export processing zone units have also been
allowed to export through trading and star trading 64. (c) The third category has two subsets – one consisting
houses and can have equipments on lease. of sectors where automatic approval is granted for
57. (b) These zones were created to develop such an FDI (often foreign equity participation less than
environment in the economy which may provide 100 per cent) and the other consisting of sectors
capability of facing international competition. The where prior approval from the Foreign Investment
Export Processing Zone (EPZs) set up as enclaves, Approval Board (FIPB) is required.
separated from the Domestic Tariff Area by fiscal 65. (d) 66. (a) 67. (d)
barriers, were intended to provide a competitive 68. (c) The success of foreign exchange management
duty free environment for export production. by RBI can be seen in the fact that from US$ 5.8
58. (a) billion in March 1991 (enough to meet the import
59. (d) Since 1981, the Government introduced a requirements for three weeks) the country’s reserves
complementary plan of EPZ (Now converted into have grown to around US $ 352 billion by December
SEZ) scheme for promoting export units (making 2015. It is pertinent to note here that some of the
export of their cent percent production. Under this increase in the reserve is attributed to the weakness
scheme the Government provides various incentives of the dollar in the post-Iraq war scenario.
to increases the production capacity of these units so 69. (d) 70. (a)
INDUSTRY AND
7
INFRASTRUCTURE

Introduction
Growth of industies has been always focus area of the government. The reason behind this is the important role played by
the industries in Gross Domestic Product (GDP) of India. The government, therefore, has many schemes and incentives to
facililate growth and development of industries.
It has been generally noticed in the most of developing countries shift from agriculture to industries in the overall economic
growth of the country.
Indian economy, however portrays a contradictory trend. Here, the economic has by passed industries growth to rapid growth
of services sector. The rapid growth of services may be Largely attributed to reform and liberalisation of the 1990s. Indian
industries contribute 18% of India’s GDP and employ about 19% of work force. Major industries of our country are textiles,
Chemicals, food processing, steel, cement, petroleum, pharmaceuticals etc.

Indian Industries

Introduction Status of Micro, Small and Major Public Sector Small Scale
Indian Industries Medium Enterprise Industries Units Industries

Before Acts and Industrial


Cottage, Small
Independence Regulations and Village Structure Sickness
After Industries of PSES Disinvestment
Independence Agro and Rural Role in Policy
Industrial Policy
Industries economy Approach for
Resolution, 1948
Cotton and Textfile Disinvestment
Industrial Industry
Development and Energy
Sugar Industry
Regulation Act 1951 Crude oil
Industrial Policy 1956 Iron and Steel Industry
Coal
MRTP Act 1969 Petroleum Industry Power
FERA 1973 Railway
Oil and Gas Production
Industrial Policy 1973
Gems and Jewellery Road
Industrialization post 1990 Transport
Industrial Policy 1991 Chemical Industry Inland waterways
Competition Act 2002 Infrastructure Shipping
Competition Civil Aviation
(Ammendment) Tele communication
Bill, 2007 Information Technology
E-142 Industry and Infrastructure

STATUS OF INDIAN INDUSTRIES


Before Independence •• A group of 18 specified industries (of medium category)
Status of Indian Industries : India was a manufacturing were in control of the state governments in liaison with
economy before industrial revolution of England in 19th the Central government.
century. Its exports of cotton, silk fabrics and handicrafts were •• The remaining industrial options which were not
popular in many areas. But, due to import of cheap machine- covered by either the centre or state lists, were left open
made goods during the colonial period, Indian manufacturing to the private sector.
declined. •• This policy was to be reviewed after 10 years.
The development thinking in the 1940s and 1950s was influenced
by the economic prospects of newly-independent countries Industrial Development and
like India, the experience of Latin America and writings of
influential economists of ‘the import substitution’ school. The Regulation Act 1951
argument was that political freedom could be sustained only The act gave complete authority to the government. This
by achieving economic independence from foreign elements. resulted in the bureaucracy extending complete control over
At Independence, foreigners controlled 65% of India’s tea the industrialization of the country.
plantation, more than 90% of jute manufacturing, 70% of
•• They controlled the authorization of capability,
coal production and 73% of minerals. Colonial policy was
export-oriented, as only exports provided a means of transfer whereabouts and growth of any request for manufacture
of wealth from the colonies. Therefore, after independence, of new products.
many of the former colonies, including India, chose an anti- •• They controlled the authorization of foreign exchange
export and inward looking industrial regime in the 1950s. expenditure on the import of plant and machinery.
But the government later modified its policies with the •• They controlled the authorization for the terms of
following points: international joint ventures.
•• Defence and strategic industries were to be the exclusive
domain of the government.
Industrial Policy In 1956
•• Existing units in basic and key industries could continue In 1956 a new policy for industrialization was initiated.
in the private sector but no fresh private investment in •• All basic and sensitive industries in India were under the
these sectors was to be allowed. purview of public sector enterprises (PSUs) and were
•• Twenty important industries were allowed in private called as category A type of industries. The Centre had
sector but under strict supervision of the state. complete monopoly over these industries. The then PM
•• All industries not covered by the above three categories Pandit Jawaharlal Nehru termed the PSUs the “temples
were allowed in the private sector under general of modern India” in this industrial policy.
supervision of the state. •• In category B, industries were a joint venture of both
public and private enterprises. 12 industrial areas were
Industrial Growth AfterIndependence put under this category. This category also carried the
Prior to independence the ownership or control of much of the provision of compulsory licensing. This provision led
large private industries were in the hands of managing agencies, to the establishment of the so called ‘Licence- Quota-
which grew under the British system and had access to London Permit raj’ in the economy.
money markets. Thus the owners of these managing agencies •• The remaining industries came under category C, to be
controlled a major portion of the economy, prior to independence. under the control of private initiative.
But things changed after independence. Parliament enacted a •• The policy of 1956, for the first time, recognized the
legislation to curb the powers of managing agencies. By 1971 contribution of small scale industries in the growth of the
the government had banned the managing agencies. Indian economy. It laid stress on rational distribution of
national income and effective utilization of resources.
The Industrial Policy Resolution,1948, clearly put forward
This policy is considered one of the most important
the goal of the Government’s policy with respect to
industrial policies of India as it decided the nature and
industrialization. This was the first economic policy of our
scope of the Indian economy till the reforms of 1991.
country. It declared that India would be a mixed economy.
Following are the major highlights of this resolution. MRTP Act -1969
•• Those industries completely owned by the Government The Government of India appointed a Monopolies Inquiry
e.g. ordinance, atomic energy, railways and any industry Commission, under Justice K. C. Dasgupta, to study the
of national importance were to be the exclusive domain of presence and outcomes of concentration of economic power
the Central government. Certain important industries like in private sector. The Commission observed the presence of
coal, iron and steel, aircraft manufacture, ship building, monopolistic and restrictive practices in certain key sectors of
telephone, telegraphs and communications, were given the economy. The Commission recommended the setting up of
the permission to operate for ten years, at the end of the Monopolies and Restrictive Trade Practices Commission
which the government would nationalize them. and this eventually resulted in the MRTP Act in 1969.
Industry and Infrastructure E-143

(ii) Introduction of Industrial Entrepreneur’s Memorandum


FERA 1973 (i.e. no industrial approval is required for industries
The Foreign Exchange and Regulation Act (FERA) was not requiring compulsory licensing).
passed in 1973. This resulted in a tremendous shift in the (iii) Liberalisation of Locational Policy.
foreign investment policy of the Government of India.
(iv) Liberalised policy for Small Scale Sectors.
Foreign Investment was allowed in only those industries that
were directly into exports. (v) Non-Resident Indians Scheme (NRIs are allowed to
Restrictions were placed on foreign investments. International invest upto 100% equity on non-repatriation basis in
companies could hold a maximum of 40% equity. But some all activities except for a small negative list).
industries in the field of advanced technology were given (vi) Electronic Hardware Technology Park (EHTP),
permission for 51% foreign capital. This has often been Software Technology Park (STP) Scheme for building
called a draconian act which hampered the modernisation up strong electronic industry to enhance exports.
and growth of Indian industries. (vii) Liberalised policy for Foreign Direct Investment
(FDI).
Industrial Policy 1973 (viii) Abolition of the MRTP limit.
The policy listed out the various Appendix 1 industries (ix) FERA was replaced by highly liberal FEMA.
that could be started by large business houses so that small
industries were not driven out of business. The establishment Competition Act, 2002
of small and medium industries was encouraged. In the present era of LPG (Liberalisation, Privatisation and
Private industries were encouraged to set up production units Globalisation), it was felt that the existing Monopolies and
in rural areas and in backward areas with a vision to give thrust Restrictive Trade Practices Act, 1969 has become hurdle in
for the economic development of those areas. certain respects and there is a need to shift our focus from
Industrialization Post 1990 curbing monopolies to promoting competition. Hence a new
law, the Competition Act has been enacted and published in the
•• Exemption from licensing was allowed for all start ups
gazette of India on January 14,2003 for bringing competition
and for those with an investment worth Rs 2.5 crores in
in the Indian market. The main objectives of the Act are to
fixed assets and a right to import up to 30% of the total
establish a commission to prevent practices having adverse
value of plant and machinery.
effect on competition, to promote and sustain competition in
•• Foreign equity investment was allowed up to 40%. markets in India, to protect the interests of consumers and to
•• Geographical restrictions and investment cap for small ensure freedom of trade carried on by participants in market in
industries were Removed. India and for related matters.
At the time of liberalization the Indian industries were not
competitive in the global scenario. They could not face the The Act mainly covers the following aspects:
stiff competition from the foreign industries; hence many (i) Prohibition of anti competitive agreements;
industries sold their companies to multinational corporations (ii) Prohibition of abuse of dominance;
or entered into joint ventures with foreign companies or shut (iii) Regulation of combination (acquisitions, mergers and
down the business. amalgamations of certain size);
At the same time a new wave of service industries emerged, (iv) Establishment of Competition Commission of India (CCI);
which positioned itself in the outsourcing segment. IT and and
ITE’s industries flourished providing employment to millions
(v) Functions and powers of CCI.
of graduates.
The Act is expected curb those practices, which would have an
Industrial Policy 1991 appreciable adverse effect on competition.
(A) Objectives Competition (Amendment) Bill, 2007
• to maintain a sustained growth in productivity. On September 10, 2007 Parliament finally passed the
• to enhance gainful employment. long pending Competition (Amendment) Bill, 2007 that
• to achieve optimum utilisation of human resources. empowers the Competition Commission of India (CCI) to
• to attain international competitiveness. act as the competition regulator and to deal with a host of
• to transform India into a major partner and players in contemporary economic issues including monopolies and
the global arena. take-overs of corporate firms.
(B) Main Focus on According to the Bill’s provisions, the CCA will replace
Monopolies and Restrictive Trade Practice Commission
• deregulating Indian industry.
(MRTPC). The CCA was established in 2003. Under new
• allowing the industry freedom and flexibility in
provisions, the MRTPC will continue till two years after the
responding to market forces, and
Constitution of CCA for dealing with pending cases but after
• providing a policy regime which facilitates and
two years MRTPC will be dissolved.
fosters growth of Indian industry.
However, MRTPC would not entertain any new cases after
(C) Policy Measures the CCI is constituted. Cases pending with MRTPC after two
(i) Liberalisation of Industrial Licensing Policy. years of setting up of CCI will be transferred to the latter.
E-144 Industry and Infrastructure

Micro, Small and Medium Enterprise The following are the investment requirements under the
Service Enterprises category:
Enterprises have been categorized broadly into those
Micro Enterprises investment includes companies with
engaged in (i) manufacturing and (ii) providing/ rendering
investment upto ` 10 lakh. Small Enterprises need an
of services. Both categories have been further classified
investment above 10 lakh and upto 2 crore. Medium
into micro, small and medium enterprises, based on their
Enterprises are those enterprises which have an investment
investment in plant and machinery (for manufacturing
above 2 crore and upto 5 crore.
enterprises) or in equipment (in case of enterprises providing
Small and Medium Enterprises Development Bill 2005
or rendering services) as under:
(which was introduced in the Parliament on May 12, 2005)
The following are the investment requirements under the
was approved by the President and became an Act. This
Manufacturing Enterprises category:
Act, named as ‘Small and Medium Enterprise Development
Micro Enterprises are those enterprises which have
Act, 2006’ became effective from October 2, 2006. This
investment upto ` 25 lakh. Small Enterprises are those
Act makes a different category for medium level enterprises.
enterprises with investment above 25 lakh and upto 5 crore.
Medium Enterprises are those enterprises which have
investment above ` 5 crore and upto `10 crore..

MAJOR INDUSTRIES
Major Cottage, Small and Village Cotton and Textiles Industry
Industries Cotton Industry in India is the most organised industry which
In a broad sense cottage, small and village industries are contributes to about 4% of GDP, 14% of manufacturing
treated similar but they fundamentally differ from each other. value and 13.5% to total export earnings. It provides
employment to millions of skilled and semi-skilled workers.
Cottage industry is run by family members on full or part
The industry is characterised by wide diversity ranging from
time basis. It possesses negligible capital investment. The the organised mill sector to the decentralised power-loom
production is done by hands and no wage earning person is sector and employs millions of artisans and weavers.
employed in cottage industry. •• The first Indian modernised cotton mill, located in Kolkata
Small industrial units employ wage earning labour and in 1818, was not successful. The second mill named
production is done by the use of modern techniques. Capital ‘Bombay Spinning and Weaving Co’ was established in
investment is also present in small industries. A few cottage 1854 at Bombay by KGN Daber. Truly speaking, this
industries which are export-oriented have been included in the mill only laid the foundation stone of modern cotton
category of small sector so that facilities provided to small industry in India.
units may also be given to export-oriented cottage industries. •• Since 1854, the number of cotton mills has been rapidly
increasing.
The industries established in rural areas having population
•• Development of cotton industry had a great role in
below 10,000 and having less than ` 15,000 as fixed capital
Indian freedom struggle. Various movements like Non-
investment per worker will be termed as village industries. Cooperation Movement, Quit India Movement, etc.
KVIC and state village Industries Board provide economic created a wave of boycotting foreign goods and accepting
and technical assistance in establishing and operating these swadeshi goods, which helped a lot in developing
industrial units. indigenous industries.
•• The partition of India adversely affected the Indian
Agro and Rural Industries cotton industry. Most of the muslim weavers migrated
to Pakistan and this industry got divided into two parts.
Development of agro and rural industries based on local •• There were 394 cotton mills in India before the partition.
raw materials, skills and technology has been identified as Out of these, 14 mills went to Pakistan and the remaining
one of the most important activities for gainful employment 380 cotton mills continued to operate in India.
in the rural non-farm sector and for overall growth of the •• On the other hand, 40% of cotton producing area became
national economy. Various policies and programmes are the part of Pakistan and only 60% area was transferred
being implemented by the Government through Khadi and to India.
Village Industries Commission (KVIC) and Coir Board and •• So after independence India had to import raw material to
Prime Minister’s Rozgar Yojana (PMRY) with the active co- meet the input requirements of 380 cotton mills.
operation and participation by RBI, other banks and the state •• India’s Five Year Plans proved a boon to cotton industry,
governments for the promotion of agro and rural industries in as this industry not only made remarkable development but
rural areas and small towns. also established itself in international markets.
•• The Government by its Textile Development and Regulation
For the development of Khadi and Village Industries (KVI)
Order 1993 has made this industry licence-free.
Sector, the Government is implementing various programmes/
•• The textile industry is concentrated mainly in the states of
schemes through KVIC. Maharashtra, Tamil Nadu and Gujarat.
Industry and Infrastructure E-145

Sugar Industry Pradesh, Tamil Nadu, Andhra Pradesh, Rajasthan, Coastal


area of Kerala and Andman & Nicobar Islands.
Sugar industry occupies an important place among agriculture India has 21 refineries - 17 in the public sector, 3 in the
based industries. Sugar industry is the second largest industry private sector and one in joint venture. Out of 17 public sector
after cotton textile industry among agro based industries of refineries, 8 are owned by Indian Oil Corporation Ltd. (IOCL),
the country. This industry provides not only employment to a
2 each by Chennai Petroleum Corporation Ltd. (a subsidiary
substantial number of persons but also holds the potentialities
of IOCL), Hindustan Petroleum Corporation Ltd. (HPCL),
of developing other industries related to its by-products.
Bharat Petroleum Corporation Ltd. (BPCL) and Oil & Natural
India is the largest consumer of sugar and the second largest Gas Corporation Ltd. (ONGC). Numaligarh Refinery Limited
producer of sugar with a share of over 15% of world sugar (a subsidiary of BPCL) and Manglore Refinery and Petrol
production. The importance of sugar industry in India can be Chemicals Ltd. have one refinery each. The Joint Venture
estimated from the fact that about 45 million sugarcane growers Refinery belongs to Bharat Petroleum Corporation Ltd.
and a large number of rural labourers depend on sugarcane and
sugar industry for their livelihood. Sugar cultivation accounts
for 3% of total cultivated area and contributes 7.5% of the
Oil and Gas Production
gross value of agricultural production. Efficient and reliable energy supply is a prerequisite for
Approximately `1,250 crore is invested in this industry and it accelerated growth of the Indian economy. While the energy
provides livelihood for nearly 2.86 lakh workers. The industry needs of the country, especially oil and gas, are going to
also benefits the nearly 2.5 crore people who grow sugarcane increase at a rapid rate in the coming decades, the indigenous
in India. In India, the major sugar producing states are energy resources are limited. Oil and gas occupy a major
Maharashtra, Gujarat, Uttar Pradesh, Haryana, Tamil Nadu, position of total energy consumption at 45%. At the same
Punjab, Karnataka, Bihar and Andhra Pradesh. time, the dependence on imports of petroleum and petroleum
The Central Government fixes the Statutory Minimum Price products continues to be around 80% of total oil consumption
(SMP) of sugarcane for each sugar season. The SMP is fixed in the country.
on the basis of the recommendations of the Commission for
Agricultural Cost and Prices (CACP) and after consulting the Gems and Jewellery Industry
State Governments and associations of sugar industry and cane
Gems and Jewellery industry emerged as leading export
growers.
product in India due to traditional strength in craftsmanship
Iron and Steel Industry and its reasonable share in global business. The gems and
jewellery sector, contributing about 12% of India’s total
The advent of Iron and Steel Industry took place in India in merchandise exports continued to maintain India as the
the year 1870 when Bengal Iron Works Company established largest cutting and polishing centre of diamonds in the world
its plant at Kulti near Jharia, West Bengal. This plant both in terms of quantity and value. In terms of carat, India’s
produced only cast iron. Large scale iron and steel production share in this sector is about 80% of the world market. Gold
was started in 1907 by TISCO established at Jamshedpur. jewellery and coloured gem segments accounts for about 12%
In 1919, Indian Iron and Steel Company (IISCO) was and 5%, respectively of India’s gem and jewellery export in
established at Burnpur. Both TISCO and IISCO are private value terms.
sector companies. The first public sector company was
‘Vishwashwaraiya Iron and Steel Works at Bhadravati’. Chemical Industry
After independence, though a thought was given to develop
Chemical industry is one of the oldest industries. It includes
Iron and Steel Industry in First Five Year Plan, but this
basic chemicals and its products, petrochemicals, fertilisers,
materialised in Second Five Year Plan. The Second Plan
paints and varnishes, gases, soap, perfumes, toiletries and
established three steel plants in the public sector-Bhilai
pharmaceuticals. This industry is one of the most diversified of
(with assistance of USSR), Durgapur (with assistance of
all industrial sectors covering more than 70,000 commercial
U.K.) and Rourkela (with assistance of West Germany).
products. It not only plays a crucial role in meeting the daily
All the three public sector plants started production between
needs of the common man, but also contributes significantly
1956 and 1962.
towards industrial and economic growth of the nation. It is
All the three public sector steel plants were expanded during an important constituent of the Indian economy. Its turnover
Third Five Year Plan and attempts were made to establish size is estimated at around US $ 160 billion approx., which
one more public sector steel plant at Bokaro with the is equivalent to about 3% of India’s GDP. Major chemicals
assistance of USSR. undergo several stages of processing to be converted into
downstream chemicals. These processed chemicals are
Petroleum Industry used in agriculture and industry as auxiliary materials such
Initially, Digboi (Assam) was the only oil producing area of the as adhesives, unprocessed plastics, dyes and fertilizers.
country. But now, a number of regions having oil reserves have Chemicals are also directly used by consumers in the form of
been identified and the oil is being extracted in these regions. pharmaceuticals, cosmetics, household products, paints, etc.
Oil regions in India are Assam, Tripura, Manipur, West Alkali chemicals, inorganic chemicals and organic chemicals
Bengal, Mumbai, Gujarat, Jammu and Kashmir, Himachal constitute the major segments of the chemicals industry.
E-146 Industry and Infrastructure

Infrastructure increasing dependence on oil in the energy sector and immediate


steps should be taken to tackle this increasing dependence.
Production of goods requires supporting services such as
transporting of raw materials and finished goods, institutions Crude Oil
for offering credit, energy sources such as coal, oil and electric Our production of crude oil declined over the years. The decline
power, communication facilities and social overheads like is attributed to unexpected reservoir behaviour and lack of new
health, education and housing. All these supporting structures major discoveries. It is estimated that at the current level of
are collectively known as infrastructure. consumption, the recoverable reserves of oil in India (approx.
Infrastructure is broadly of two types: 550 million tonnes) would last for only 20-25 years. India’s
•• Economic infrastructure is located within the system crude oil production increased at highest pace in last four years
of production and distribution. Some of these are the of 5.6% to 3.193 million tonnes in August 2015.
transport system, banking system and power. The oil ministry has estimated India will import 188.23 million
•• Social infrastructure contributes to the production tonne of crude oil in FY16 at a cost of ` 4,72,932 crore,
process from outside the system. Some examples are compared to 189.43 million tonnes crude worth ` 6,87,416
education, training, hospitals and housing. crore imported in FY15, saving ` 2,14,484 crore on fuel bill.
•• Communication system is treated as both social and
economic infrastructure as it has a role both within and Coal
outside the production process. Coal is the most important indigenous source of energy. The
Although there has been massive investment in infrastructure energy derived from coal in India is twice that of energy
in successive five-year plans, India is even now far behind derived from oil, whereas on the other hand, energy derived
global standards in the matter of infrastructure development. from coal is about 30% less than energy derived from oil the
According to the Global Competitiveness Report, 2015 India’s world over.
ranking in terms of Global Competitiveness Index was 55 out India is today the third largest producer of coal in the world.
of 140 countries. Million tonnes
Different methods needed to attract private investment in
State Proved Indicated Inferred Total
infrastructure activities must include simplification and
transparency in clearing procedures; creating an independent Total 1259029 142506 33149 301564
regulatory framework and unbundling infrastructure projects West Bengal 13403 13022 4893 31318
so that private sectors may be able to chose sub- segments in Jharkhand 41377 32780 6559 80716
a project.
Bihar 0 0 160 160
Some ongoing government schemes to help in development of
Madhya Pradesh 10411 12382 2879 25673
infrastructure are: Jawarharlal Nehru National Urban Renewal
Mission (JNNURM), National Urban Transport Policy (2006), Chhattisgarh 16052 33253 3228 52533
National Highways Development Project (NHDP), Special Uttar pradesh 884 178 0 1062
Accelerated Road development Programme for North East Maharashtra 5667 3186 2110 10964
Region (SARDP-NE), Pradhan Mantri Gram Sadak Yojna
Odisha 27791 37873 9408 75073
(PMGSY), etc.
The Budget 2015 shows a significant increase of ` 70000 crore Andhra Pradesh 9729 9670 3068 22468
in investment in infrastructure in 2015-16 over the current Assam 465 47 3 515
year, with a focus on railways and roads. The money will come Sikkim 0 58 43 101
from an additional public investment outlay of ` 1.25 trillion
Arunachal 31 40 19 90
over that of 2014-15. The allocation for roads and railways is `
Pradesh
40,000 crore each, an increase of ` 14,031 crore and ` 10,050
crore respectively over the current fiscal year. Meghalaya 89 17 471 576
Nagaland 9 0 307 315
Energy (Source: Geological Survey of India)
Energy is the most fundamental input into any production
process. Advanced countries of North America and Europe Categorization of Resources
had developed their economies at a time when the cost of The Coal resources of India are available in older Gondwana
energy was very low. Coal was the most important source of Formations of peninsular India and younger Tertiary formations of
energy during the Industrial Revolution. In the 20 century, north-eastern region. Based on the results of Regional/ Promotional
it was replaced by oil, which was an even cheaper source. Exploration, where the boreholes are normally placed 1-2 Km
When the oil producing countries formed a cartel (OPEC) and apart, the resources are classified into ‘Indicated‘ or ‘Inferred‘
increased the price of crude oil from $2.1 per barrel in 1973 category. Subsequent Detailed Exploration in selected blocks,
to as much as $28 per barrel in 1980, it hit the developing where boreholes are less than 400 meter apart, upgrades the
countries like India the most. resources into more reliable ‘Proved‘ category. The Formation-
Although India has abundant reserves of coal, our development wise and Category-wise Coal resources of India as on 1.4.2014
process has not utilised this indigenous source. There is an are given in the table below:
Industry and Infrastructure E-147

Million tonnes railway has been contributing to the industrial and economic
Formation Proved Indicated Inferred Total landscape for over 150 years. Of the two main segments
Total 125909 142506 33149 301564 (freight and passenger) of the Indian railways, the freight
segment accounts for roughly two thirds of revenues.
Gondwana Coals 125315 142407 32350 300072
Out of the freight and passenger traffic, the freight segment
Tertiary Coals 594 99 799 1493 accounts for about 70% of revenue. Within the freight
State of Coal Resources in India Last 5 years segment, bulk traffic accounts for nearly 84% of revenue-
earning freight traffic (in physical terms), of which about
As a result of Regional, Promotional and detailed Exploration
44% is coal.
by GSI, CMPDI, SCCL, MECL, State Govts. etc., the
estimation of Coal resources in India reached to 301.56 billion The railway budget 2015-16 announced four major goals
tones. The increase/up gradation of Coal resources in the of attaining sustained improvement in customer facilities,
country during last 5 years is furnished in table below: making railways safer means of travel, expanding
substantially and making Indian Railways self- sustainable.
Million tonnes
In order to increase transportation capacity, reduce unit
As on Geological Resources of Coal costs of transportation, and improve service quality, the
Proved Indicated Inferred Total mega rail transport projects, namely the Eastern and western
1.4.2009 105820 123470 37920 267210 Dedicated Freight Corridors, are being undertaken.
1.4.2010 109798 130654 36358 276810 Road Transport
1.4.2011 114002 137471 34390 285862 India has one of the largest road networks in the world
1.4.2012 118145 142169 33183 293497 aggregating to about 3.62 million km at present. A series of
1.4.2013 123182 142632 33101 298914 initiatives have been undertaken in recent years, to set stage
for a quantum leap in India’s road system. These initiatives
1.4.2014 125909 142506 33149 301564
combine new institutional arrangements, highway engineering
Power of international standards, founded on a self-financing revenue
Power has been the most crucial problem in India’s economic model comprising tolls and a cess on fuel. Three initiatives in
development. The share of hydro power is 26%, thermal 66%, the road sector were begun in recent years:
renewable energy sources 5% and nuclear 3%. The National The National Highway Development Project (NHDP),
Electricity Policy, 2005 recognises electricity as a “basic human Pradhan Mantri Bharat Jodo Pariyojana (PMBJP), and
need” and lays down per capita availability of power from the
Pradhan Mantri Gram Sadak Yogana (PMGSY).
present 631 units to 1000 units per annum by the end of 2012. In
budget 2015, emphasising the need to generate more electricity NHDP dealt with building high quality highways. The PNBJP
from clean energy sources, the government announced a massive dealt with linking up major cities to the NHDP Highways.
renewable power production target of 1,75,000 MW in the next The PMGSY addressed rural roads.
seven years. Of the total 1,75,000 MW proposed to be tapped While the Central Sector Programme pertains mainly to
by 2022, solar power will have the major share of 1,00,000
National Highways, the responsibility for development of
MW followed by 60,000 MW from wind energy, 10,000 MW
biomass energy and 5,000 MW of small hydro projects. other categories of roads vests with the State/Union territories.
A major initiative to expedite power generation has been the The Indian road network, the second largest in the world
development of coal-based-Ultra Mega Power Projects (UMPP) consists of 96,260 km of National Highways, 1,31,899 km of
each with a capacity of 4000 MW or above. Nine sites have been State Highways and about 27,17,763 km of Other District and
identified including four pithead sites (Chhattisgarh, Jharkhand, Rural Roads. The National Highways account for about 2% of
Madhya Pradesh and Odisha) and five coastal sites (Andhra the total road network but carry as much as 40% of the total
Pradesh, Gujarat, Karnataka, Maharashtra and Tamil Nadu). road traffic in the country. Out of the total length of National
The pithead units will get coal from captive mines while the Highways, 24% is single lane/intermediate lane, 52% is 2-lane
coastal plants will import coal.
standard and balance of 24% is four-lane standard or more.
Railway Indian Road Network
The first train in India was started on a small rail route of 34 Indian road network of 33 lakh Km. is second largest in
kilometers between Bombay and Thane on April 16, 1853. the world and consists of:
At present, the Indian Railway consists of an extensive
Length (In Km)
network spread over 63974 km comprising Broad Gauge
(54257 km) , Meter Gauge (7180 km) and Narrow Gauge Expressways 200
(2537 km).With such a large rail route, the Indian Railway National Highways 96,260.72
network has become the biggest railway of Asia and the State Highways 1,31,899
third in the world. Out of this, about 30% of the route
Major District Roads 4,67,763
kilometre, 41% of running track kilometre and 43% of total
track kilometre has been electrified. Railways absorb about Rural and Other Roads 26,50,000
41% of the total central government employees. Indian Total Length 33 Lakhs Kms (Approx)
E-148 Industry and Infrastructure

Modal Shift National Waterway 6 (NW6)


The last in the list, NW6, is under construction. The waterway
• About 65% of freight and 80% passenger traffic is
carried by the roads. is proposed to cover the distance between Lakhipur and Bhanga
• National Highways constitute only about 1.7% of the near the Bangladesh border in Assam, on the Barak River.
road network but carry about 40% of the total road The first phase of the two-phase project is scheduled to be
traffic. completed by 2016-17. The waterway aims to benefit people
• Number of vehicles has been growing at an average from the northeastern states.
pace of 10.16% per annum over the last five years. Plans for 106 inland waterways in India
The Union Cabinet, chaired by Prime Minister Narendra
Inland Waterways Modi, has approved to pass the official amendment of the
The inland waterways are another mode of transport which was National Waterways Bill, 2015.
once important but came to be neglected after the development Here are some key points you need to know:
of railways. It is still an important mode in the North-East • The bill seeks to add 106 inland waterways to the existing
where almost half of the cargo traffic between Calcutta and six National Waterways on the recommendations of
the Parliamentary Standing Committee on Transport,
Assam is handled by this mode. Total navigable waterways
Tourism and Culture and comments of several state
extend to around 14,500 kms of which more than 3700 kms are governments.
suited for mechanical crafts. The inland Waterways Authority of • The bill will also look after the renovation and
India, established in 1986, is entrusted with the responsibility maintenance of the existing waterways.
of developing and maintaining national waterways. • Out of the 106 new waterways, 18 have already been
identified. These include five waterways each from
The six existing National Waterways in India Karnataka and Meghalaya, three each from Maharashtra
National Waterway 1 (NW1) and Kerala, one each from Tamil Nadu and Rajasthan.
• The bill also aims to help the Inland Waterways Authority
The National Waterway No. 1 uses a 1,620-kilometre stretch of India (IWAI) to develop the feasible stretches for
of the Ganges River. It was declared a national waterway inthe Shipping and Navigation.
year 1986 and runs from Allahabad in Uttar Pradesh to Haldia
in West Bengal Shipping
National Waterway 2 (NW2) Shipping is another important infrastructure in the country’s
The National Waterway No. 2 consists of an 891-kilometre trade and industry. Indian overseas fleet ranks the 17 in
stretch on the Brahmaputra River. The waterway was declared the world in terms of tonnage, with 659 ships totalling 77
on September 1, 1988, and uses the stretch from Dhubri near lakh gross registered tonnage (GRT). 102 Indian shipping
companies are in operation of which 65 are engaged only in
the Assam-Bangladesh border and Sadiya in North-East Assam.
coastal shipping. The public sector Shipping Corporation of
National Waterway 3 (NW3) India (SCI) has a fleet of 112 vessels aggregating three million
Popularly known as the West Coast Canal, the National GRT. In spite of this, Indian ships carry only 29% of India’s
Waterway No. 3 is a 168-kilometre stretch that runs from total sea borne trade. More than half of this tonnage is crude
Kollam to Kottapuram in Kerala. The waterway does not oil and its products.
follow a specific river. It consists of several canals that form There are twelve major ports under Central Government and
the Kerala Backwaters. 139 minor ports under state control along the Indian coastline
National Waterway 4 (NW4) of about 5600 Kms. The major ports together handle a cargo
The most complex inland waterway, the National Waterway of 251 million tonnes.
No. 4, was declared on November 24 in 2008. Civil Aviation
The waterway consists of the Kakinada-Pondicherry stretch of
The rapid expansion of this sector during the last five years
canals, the Kaluvelly tank, has necessitated urgent investment in infrastructure. There
Bhadrachalam-Rajahmundry stretch of River Godavari and the are 14 scheduled airline operators using 334 aircraft. It
Wazirabad-Vijayawada stretch of River Krishna. is expected that another 250 aircraft will be added during
It traverses around 1,095 kilometre from Kakinada in Andhra the next five years. In addition, there are 65 non-scheduled
Pradesh to the Union Territory of Pondicherry. airlines operators who have 201 aircraft. The existing
National Waterway 5 (NW5) airports are unable to manage this increased volume.
Another waterway with multiple riverlines and rivulets is the The numbers also demand better regulation of the operators
National Waterway No. 5. The waterway was declared in in order to ensure satisfactory service. An Airport Economic
November 2008. The NW5 consists of stretches from Talcher Regulatory Authority (AERA) was set up to ensure healthy
competition among operators and settlement of disputes
to Dhamra on River Brahmani, the Geonkhali-Charbatia
among operators as well as between them and passengers.
stretch of the East Coast Canal, the Charbatia-Dhamra stretch
Another major reform in this sector has been the merger of
of River Matai and the Mangalgadi-Paradip stretch of River
the Indian Airlines and Air India into a new entity called the
Mahanadi Delta. The waterway also includes a 91 kilometre National Aviation Company of India Limited (NACIL).
stretch between Geonkhali and Nasirabad in West Bengal.
Industry and Infrastructure E-149

Telecommunication •• This notification is a following step of the Union Cabinet’s


Telecommunication has witnessed the most dramatic approval of the 74% FDI in telecom in March 2007.
growth since the ending of the monopoly of Department of •• As per new norms, FDI upto 49% will continue to be
Telecommunication, as part of the New Telecom Policy of on the automatic route.
2012 (NTP 2012) •• If the FDI limit were to cross 49% then it would require
the approval of the Foreign Investment Promotion
New Telecom Policy: Highlights
Board (FIPB).
1. Rural Tele-density – To improve rural teledensity from •• By this notification, FIPB is also empowered to note
the current level of around 39 to 70 by the year 2017 and that the investment is not coming from countries of
100 by the year 2020. concern or unfriendly entities.
2. Broadband – “Broadband For All” at a minimum •• The guidelines allow foreigners to hold key positions
download speed of 2 Mbps. (like Chairman, MD, CEO, etc.) of telecom companies,
3. Domestic Manufacturing – Making India a global hub. subject to clearance from the Telecom Ministry on a
4. Convergence of Network, Services and Devices. yearly basis.
5. Liberalization of Spectrum – any Service in any
Technology. Information Technology
6. Simplification of Licensing regime – Unified Licensing, The enactment of the Information Technology Act (2000)
delinking of Spectrum from License, Online real time was a milestone in the development of India’s IT industry.
submission and processing. It provided a legal framework for e-commerce and electronic
transactions. It also addresses the security needs of privacy,
7. Consumer Focus – Achieve One Nation – Full Mobile
authenticity,integrity and non-repudiation over the Internet.
Number Portability and work towards One Nation – Free
A study by NASSCOM said that software exports for 2015-
Roaming.
16 would grow between 12-14% to about $110-112 billion,
8. Resale of Services. compared with the 13-15% growth estimate for the fiscal year
9. VOIP – Voice over Internet Protocol. 2014-15. The Indian domestic market is expected to grow by
10. Cloud Computing – Next Generation Network including 14% to $48 billion in the current fiscal year, mainly because of
IPV6. e-commerce growth. Domestic growth is expected to be led by
e-commerce, government initiatives and technology adoption
•• The number of telephone subscribers in India increased
by industries. Investment of $26 billion by the government in
to 1002.05 million in May, 2015. The overall tele-
2014-15 also helped domestic revenue growth.
density in India reached 79.67%.
The IT industry is the biggest private sector employer in India,
•• Central government in its decision made on April and it added 230,000 employees in 2014-15, thus making the
19, 2007 notified the enhancement of foreign direct total number of jobs in the industry close to 3.5 million.The IT
investment (FDI) in telecom from 49% to 74%. sector also accounted for 9.5% of the gross domestic product
•• As per the notification, all companies have been given (GDP). The IT industry holds the largest share of total services
three months time to comply with the revised norms. exports at 38%.

PUBLIC SECTOR UNITS


Most countries, including India, have adopted a mixed economy shift to a more open economy with greater reliance on
model that encourages the role of the public as well as private market forces and a larger role of the private sector,
sector enterprises. Since Independence, the Indian government including foreign investment.
made sustained efforts to bring about balanced development •• The PSEs were exposed to competition from domestic
by setting up public sector enterprises (PSEs). The role of the private sector companies as well as foreign MNCs.
PSEs was earlier limited to basic, heavy and core industries, •• To sustain in a growing competitive scenario, PSEs have
which were considered of strategic importance and vital for undertaken several steps to perform and operate on par
mass consumption. Furthermore, the government had to step with their private peers such as adopting state-of-the-art
in and set up these huge PSEs because the nation was in the technology, focusing on improving productivity, giving
initial stages of development and private enterprises could not performance-related pay, offering additional welfare
afford massive investments which the core sector called for. In facilities and other benefits to employees, establishing
recent years, PSEs have penetrated into production of essential brands and increasing marketing efforts.
consumer goods and have begun to spread into wide areas of the
economy including non-infrastructure and other non-core areas. Structure of PSEs in India
•• The public sector banks (PSBs), which have played an
important role in shaping up the Indian economy since the The PSEs in India are basically categorised under four broad
pre-independence period until now, continue to dominate types based on their ownership structure. These include:
the Indian banking industry, accounting for more than (i) departmental undertakings,
70% share of total banking business in India. (ii) statutory corporations,
•• With the introduction of economic reforms and (iii) government-owned companies and
liberalisation in 1991, the Government initiated a systemic (iv) autonomous bodies set up as registered societies.
E-150 Industry and Infrastructure

(iv) Autonomous bodies


Autonomous bodies are set up whenever it is felt that certain
functions need to be discharged outside the governmental set
up with some amount of independence and flexibility without
day-to-day interference from the governmental machinery.
These bodies are set up by the concerned ministries or
their departments and are funded through grants-in-aid,
either fully or partially, depending on the extent which such
institutes generate internal resources of their own. These
grants are regulated by the Ministry of Finance (MOF)
through their instructions. They are mostly registered as
societies under the ‘Societies Registration Act’ and in certain
cases they have been set up as statutory institutions under
(i) Departmental undertakings the provisions contained in various Acts.
Departmental undertakings are primarily meant to provide CPSE’s Role in Economy
essential services such as railways. They function under
the control of the respective ministries of Government •• CPSE investments have a multiplier effect on the
of India (GOI). A departmental undertaking structure is economy
considered suitable for activities the government aims to During the first five-year plan (1950-51 to 1955-56) there
keep in its control in view of the public interest. were only 5 CPSEs with a total financial investment (Including
equity plus long-term loans) of Rs. 290 million, whereas at
(ii) Statutory corporations present, in 2015, there are as many as 290 CPSEs, wherein,
Statutory corporations are public enterprises that came 169 are Holding CPSEs and 121 are the subsidiaries.As of
into existence by a Special Act of the Parliament. The Mar 31, 2012, the total financial investment of CPSEs was
Act defines the powers and functions, rules and regulations 7,292.3 billion, showing a CAGR of around 12% during the
governing the employees and the relationship of the same period.
corporation with government departments. •• CPSEs continue to dominate domestic output of key
sectors
(iii) Government-owned companies CPSEs continue to hold control across several industries, despite
Government-owned or controlled companies refer to opening up of several sectors for private investment. CPSEs
companies in which 51% or more of the paid up capital continue to have complete monopoly in nuclear power generation.
is held by the central or any state government (partly Other leading areas of dominance are coal (over 80%), crude
or wholly by both). It is registered under the Indian oil (over 70%), refineries (over 55%) and wired lines (over
Companies Act and is fully governed by the provisions of 80%). However, their share has decreased considerably, with
this Act. the exception of coking coal and power generation.

SMALL SCALE INDUSTRIES

Small scale Industries (SSI) Employment


Small scale industries (SSI) are production units with a capital •• SSI Sector in India creates largest employment
investment of up to ` one crore. opportunities for the Indian populace, next only to
•• It plays a vital role in the growth of India. It contributes Agriculture. It has been estimated that 100,000 rupees
almost 40% of the gross industrial value added in the of investment in fixed assets in the small-scale sector
Indian economy, 6% of GDP and 35% of exports. generates employment for four persons.
•• It has been estimated that a million rupees of investment in •• Food products industry has ranked first in generating
fixed assets in the small scale sector produces 4.62 million employment, providing employment to 0.48 million
worth of goods or services. The small scale sector in India persons (13.1%). The next two industry groups were
is very diverse producing 8000 products from traditional Non-metallic mineral products with employment of
handicraft to high end technical instruments. 0.45 million persons (12.2%) and Metal products with
•• The small-scale sector has grown rapidly over the years. 0.37 million persons (10.2%).
The growth rates during the various plan periods have •• In chemicals and chemical products, machinery parts except
been very impressive. electrical parts, wood products, basic metal industries,
•• When the performance of this sector is viewed against the paper products and printing, hosiery and garments, repair
growth in the manufacturing and the industry sector as a services and rubber & plastic products, the contribution
whole, it instils confidence in the resilience of the small- ranged from 9% to 5%, the total contribution by these
scale sector. eight industry groups being 49%.
Industry and Infrastructure E-151

As per the recommendations Rangarajan Committee on


•• In all other industries the contribution was less than
Disinvestment of PSEs, government launched a programme of
5%.
disinvestment. However, the first instalment of disinvestment
•• SSI Sector plays a major role in India’s present export of shares valued at 6480 crore fetched only 3038 crore in
performance. 45%-50% of the Indian Exports is 1991-92. The process attracted a lot of criticism. Moreover,
contributed by SSI Sector. Direct exports from the SSI instead of raising resources either to strengthen the healthier
Sector account for nearly 35% of total exports. Besides PSUs or at least to retire the national debt, the government has
direct exports, it is estimated that small-scale industrial been looking at extra budgetary support to bridge the deficit.
units contribute around 15% to exports indirectly. This
In order to speed up the process of disinvestment, Government
takes place through merchant exporters, trading houses
of India set up a separate Department of Disinvestment in
and export houses. They may also be in the form of
March 2000. A decision was also taken to reduce Government’s
export orders from large units or the production of parts
equity in non-strategic PSUs to 26% or less through strategic
and components for use for finished exportable goods.
sale of such shares. Only three areas-defence, atomic energy
and railway-were identified as strategic and all other sectors
INDUSTRIAL SICKNESS were open to disinvestment up to 26%.
Industrial Sickness refers to the situation where an industrial The then NDA Government pursued this policy of disinvestment
firm performs poorly, incurs losses for many years and defaults and several profit-making PSUs, including IPCL, VSNL and
in its repayment obligations. IBP were partially divested. Many state governments were also
RBI defines a sick unit as “One which has incurred a cash following similar disinvestment policy in respect of state PSUs.
loss for one year and likely to continue incurring losses in the
following year. The unit which has an inbalance in its financial
Disinvestment Policy of UPA (2009-14)
structure such as current ratio is less than 1:1 and there is The current disinvestment policy was initially decided by the
worse trend in debt equity ratio. UPA government (2009-2014), under its Common Minimum
Industrial sickness can be caused by internal and external factors. Programme. The main ideology, as listed on the site of
•• Internal factors could be mismanagement, lack of finance Ministry of Finance, says that:
wrong dividend policy, not providing for reserves or (i) Citizens have every right to own part of the shares of
deliberate ‘milking’ of the unit by the owners. Public Sector Undertakings.
•• External factors could be wrong government policy, (ii) Public Sector Undertakings are the wealth of the Nation
power shortage, non-availability of raw materials, and this wealth should rest in the hands of the people.
transport problem, lack of technological advancement and (iii) While pursuing disinvestment, Government has to retain
labour disputes. majority shareholding, i.e. at least 51% and management
Since 1987, the entire responsibility to tackle sickness was control of the Public Sector Undertakings.
entrusted to the quasi judicial body set up for the purpose, i.e.
the Board for Industrial and Financial Reconstruction (BIFR).
Approach for Disinvestment
The BIFR approves and sanctions rehabilitation schemes for On 5th November 2009, Government approved the following
sick companies that are referred to it. The functioning of action plan for disinvestment in profit making government
BIFR, however, has been criticised over the years because companies:
of the adoption of time consuming procedures which lead to (i) Already listed profitable CPSEs (not meeting mandatory
delays in rehabilitation. shareholding of 10%) are to be made compliant by ‘Offer
The government set up the Justice Eradi committee to for Sale’ by Government or by the CPSEs through issue
recommend a mechanism for revival as well as closure of a sick of fresh shares or a combination of both.
unit. On the recommendations of the panel, the government has (ii) Unlisted CPSEs with no accumulated losses and having
decided to set up a National Company law Tribunal (NCLT) earned net profit in three preceding consecutive years
whose main purpose will be to make it easier to liquidate sick are to be listed.
companies within a short timeframe of two years. The NCLT (iii) Follow-on public offers would be considered taking
will act as the single forum for sick companies, replacing the into consideration the needs for capital investment of
existing three forums- BIFR, Company Law Board (which CPSE, on a case by case basis, and Government could
works under the Companies Act, 1956 and handles some cases simultaneously or independently offer a portion of its
of dispute resolution and compliance) and High Courts (which equity shareholding.
deal with winding up of companies).
(iv) In all cases of disinvestment, the Government would
retain at least 51% equity and the management control.
DISINVESTMENT POLICY (v) All cases of disinvestment are to be decided on a case
The process of selling of government equities in public sector by case basis.
enterprises is known as disinvestment. Disinvestment in India (vi) The Department of Disinvestment is to identify CPSEs
is mainly a tool for public sector reforms and was, at the in consultation with respective administrative Ministries
beginning, motivated by the need for gaining resources for and submit proposal to Government in cases requiring
allocations made in the yearly budgets. Offer for Sale of Government equity.
E-152 Industry and Infrastructure

1. What is a ‘limited company’? (c) Indian large scale producers cannot compete in
(a) In which shareholders possess the ownership limited outside markets
to their paid up capital (d) Indian SSI units are inherently inefficient
(b) In which shares are issued 9. The Committee on Competition Policy and Law
(c) A company of Government ownership (Raghavan Committee) has recommended the replacement
(d) A Registered Company of which of the following institutions?
2. In India the public sector is the most dominant in (a) Securities and Exchanges Board
(a) steel production (b) Disinvestment Commission
(b) organized term-ending financial institution (c) Monopolies and Restrictive Trade Practices
(c) transport Commission
(d) commercial banking (d) Company Law Board
3. Tourism industry in India is quite small compared to 10. Which of the following can be termed an infrastructural
many other countries in terms of India’s potential and
bottleneck in the development of India’s economy?
size. Which one of the following statements is correct in
(a) Existence of a large variety of financial institutions
this regard?
(b) The federal nature of Indian policy
(a) Distances in India are too far apart and its luxury
(c) The volatility of the Indian rupee
hotels are too expensive for Western tourists.
(b) For most of the months India is too hot for Western (d) Delay in the administration of justice
tourists to feel comfortable. 11. With reference to India, which one of the following
(c) Most of the picturesque resorts in India such as statements is NOT correct?
in the northeast and Kashmir are, for all practical (a) IPCL is India’s largest petrochemical company
purposes, out of bounds. (b) RIL is the largest private sector company in India
(d) In India, the infrastructure required for attracting (c) MTNL is listed on NYSE
tourists is inadequate. (d) BSNL is the first telecom service organization in
4. ‘Level playing field’ argument of industries requires India to launch a nationwide cellular service at one
(a) MNCs to be stopped from investing in India. time
(b) licence for MNCs to be given only in environment- 12. Which one of the following statements is not correct?
friendly technology. (a) Rourkela Steel Plant, the first integrated steel plant
(c) domestic industry to be given preference. in the Public Sector of India was set up with the
(d) domestic industry to be treated on par with MNCs. Soviet Union collaboration.
5. Which of the following is not an internal factor for (b) Salem Steel Plant is a premier producer of stainless
industrial sickness? steel in India.
(a) Mismanagement (c) Maharashtra Elektrosmelt Ltd. is a subsidiary of the
(b) Power shortage Steel Authority of India Ltd.
(c) Wrong dividend policy (d) Visakhapatnam Steel Plant is a unit of the Rashtriya
(d) Diversion of funds
Ispat Nigam Ltd.
6. The Eastern Dedicated Freight Corridor being built by
13. Explain ‘joint sector’ in India?
Indian Railway will connect
(a) Where the government has more than 50% share
(a) Panipat and Allahabad
(b) Product produced together by the public and private
(b) Rajkot and Bhubaneswar
(c) Jawaharlal Nehru Port and Tuglakabad sectors is called the joint sector
(d) Ludhiana and Sonnagar (c) It is an enterprise owned jointly by both the sectors
7. British colonial authorities did not oppose the setting up (d) It is an enterprise owned jointly by an Indian and a
of cotton mills in India by local capitalists during the 19th foreign enterprise
century because 14. Which sector has maximum weightage in the index of
(a) the coast of producing mill cloth in India was higher industrial production assigned?
(b) the first mills set up in India were spinning mills (a) Mining and quarrying
which competed with indigenous hard spinning (b) Manufacturing
(c) it was too expensive to transport textiles to U.K. (c) Electricity
(d) the quality of Indian mill-made cloth was inferior (d) All equal
8. It is argued that continuation of reservation for Small 15. The moral case for CSR, i.e. that it is the right thing to
Scale Industries after import liberalisation is anachronistic do, is justified by which of the following arguments?
because (a) That the corporation is a creation of society and
(a) such reservation does not exist outside India should therefore serve its needs.
(b) Indian small scale manufacturers have to compete (b) That the corporation is a legal creation and therefore
with large scale manufacturers from outside cannot be a moral agent.
Industry and Infrastructure E-153

(c) Large corporations do not have the power or 27. In an oligopolistic or monopolistically competitive
resources to address society’s problems. market, firms do not raise their prices because even a
(d) Business decisions will have social and small price increase will lose many customers.
environmental consequences which will be Which among the following is the most suitable terms
addressed by governments. used for this concept ?
16. In India ‘Report on Currency and Finance’ is the annual (a) Supracompetitive pricing (b) Swing Demand
publication of (c) Kinked Demand (d) Imperfect
(a) SEBI (b) RBI 28. Which among the following was the first deepwater
(c) Finance Commission (d) Finance Ministry block in India to start production?
17. Which of the following is related to the EPCG Scheme (a) Krishna Godavari D-6
of Central Government ? (b) Mahanadi deepwater block NECDWN-2002/1
(a) Foreign Investment (b) Foreign Trade (c) SB-11
(c) Communication (d) Banking (d) KG-DWN- 2005/2
18. Which of the following commissions / companies of 29. Who among the following heads the Genetic Engineering
Indian Origin signed a pact for obtaining oil-blocks in Approvals Committee (GEAC) in India?
Libya? (a) Minister of Environment & Forests
(a) Reliance petroleum (b) Oil India Ltd. (b) Minister of Science and Technology
(c) ONGC (d) BPCL (c) Minister of Agriculture
19. NASSCOM is the organisation of companies of which (d) None of them
sector? 30. Bring out the incorrect statement regarding India’s textile
(a) Media sector:
(b) Mobile phone service (a) India’s textile and clothing sector currently employs
(c) Information technology 35 million people
(d) Automobiles (b) After agriculture it is second largest provider of
20. Over how many years periodic review of the “Miniratna” employment
& “Navratna” status of public enterprises on the basis of (c) Textile sector has a tendency to shrink as the GDP
their performance? and Economy grows
(a) every year (b) 2 years (d) Textile sector is a major absorber of low-skilled
(c) 3 years (d) 4 years
labor
21. Which of the following funds was established in India in
31. Who among the following are the beneficiaries of the
1992 to provide assistance to cover the costs of retraining
MAFALDA?
and redeployment of employees arising as a result of
modernization, technology up gradation and industrial (a) Landless Laborers
restructuring? (b) Jobless Youth
(a) National Renewal Fund (c) Small Industries
(b) National Insurance Fund (d) Small and Marginal Farmers
(c) National Social Security Fund 32. Tourism industries India is quite a small compared to
(d) National Up-gradation Fund many other countries in term of India potentials and size.
22. Which among the following is the apex organization of Which one of the following statement is correct in this
Industrial Finance in India? regard?
(a) IDBI (b) ICICI (a) Distances in India are too far apart and its luxury
(c) IFCI (d) RIDF hotels are too expensive for western tourists
23. Who among the following leaders announced the (b) For most of the months India is too hot for western
Industrial Policy of 1956? tourist to feel comfortable
(a) Sardar Patel (b) Jawaharlal Nehru (c) Most of the picturesque resort in India such as
(c) Balwant Rai mehta (d) J B Kriplani in North- East and Kashmir are, for all practical
24. Which among the following is the correct full form of purposes, out of bounds
SIDO? (d) In India the infrastructure required for attracting
(a) Small Industries Development Organization tourists is inadequate
(b) Sick Industries Development Organization 33. Which one of the following is the objective of National
(c) Small Industries Development Office Renewal Fund ?
(d) State Industrial Development Organization (a) To safeguard the interest of workers who may be
25. Which among the following is the most important bulk affected by technology upgradation of industry or
import of India?
closer of sick units.
(a) Nonelectrical machinery (b) Electric Goods
(b) To develop a core sector of economy.
(c) Petroleum (d) Food Grains
(c) For the development of the infrastructure such as
26. In which among the following years, a new “ Liberalized
Industrial Policy “ in India was announced for the first energy, transport, communication and irrigation.
time? (d) For human resources development such as full
(a) 1986 (b) 1991 literacy, employment, population control, housing
(c) 1992 (d) 1993 and drinking water.
E-154 Industry and Infrastructure

34. The Council of Scientific and Industrial Research (CSIR) 38. The balance of payments of a country is a systematic
links 37 constituent laboratories with diverse research record of
portfolios as varied as drug discovery to oceanography. (a) all import and export transactions of a country
Recently we read in the newspapers that CSIR’s latest during a given period of time, normally a year
imitative is launching of an Open Source Drug Discovery (b) good exported from a country during a year
(OSDD) Programme. (c) economic transaction between the government of
What is the main purpose of Open Source Drug one country to another
(d) capital movements from one country to another.
Discovery (OSDD) Programme?
39. An increase in the Bank Rate generally indicates that the
(a) To make India a hub of drug research and
(a) market rate of interest is likely to fall
development (b) Central Bank is no longer making loans to
(b) To bring the scientists of the world under one roof commercial banks
(c) To bring down the costs of the new drugs and make (c) Central Bank is following an easy money policy
them affordable to all (d) Central Bank is following a tight money policy
(d) To make India self reliant in Drug production 40. The sales tax you pay while purchasing a toothpaste
35. Which among the following sectors contributes maximum is a
excise revenue to Indian exchequer? (a) tax imposed by the Central Government
(a) Agriculture (b) Steel industry (b) tax imposed by the Central Government but
(c) Oil and gas (d) Aviation collected by the State Government
36. What does PKI stands for? (c) tax imposed by the State Government but
(a) Private Key Infrastructure collected by the Central Government
(d) tax imposed and collected by the State Government
(b) Primary Key Infrastructure
41. What does venture capital mean?
(c) Public Key Infrastructure
(a) A short-term capital provided to industries
(d) Public Key Institute (b) A long-term start-up capital provided to new
37. Disguised unemployment generally means               entrepreneurs
(a) large number of people remain unemployed (c) Funds provided to industries at times of incurring
(b) alternative employment is not available losses
(c) marginal productivity of labour is zero (d) Funds provided for replacement and renovation of
(d) productivity of workers is low industries

1. Consider the following statements: control and compulsory pre-shipment inspection of


Small-scale industries are in most cases, not as various exportable commodities.
efficient and competitive as the large-scale ones. Yet Which of these statements is / are correct?
the government provides preferential treatment and (a) 1 only (b) 1 and 2
reservations in a range of products to the small firms (c) 2 and 3 (d) 3 only
because small-scale industries. 3. With reference to the National Investment Fund to which
1. provide higher employment on a per unit capital the disinvestment proceeds are routed, consider the
development basis.
following statements ?
2. promote a regional dispersion of industries and
1. The assets in the National Investment Fund are
economic activities.
managed by the Union Ministry of Finance.
3. have performed better in export on manufactured
products than the large scale ones. 2. The National Investment Fund is to be maintained
4. provide jobs to low-skill workers, who otherwise within the Consolidated Fund of India.
may not find employment revenues elsewhere. 3. Certain Asset Management companies are appointed
Which of the above statements are correct? as the fund managers.
(a) 1 and 4 (b) 1 and 2 4. A certain proportion of annual income is used for
(c) 2 and 3 (d) 3 and 4 financing select social sectors.
2. With reference to the Public Sector Undertakings in Which of the statements given above is/are correct ?
India, consider the following statement (a) 1 and 2 (b) 2 only
1. Minerals and Metals Trading corporation of India (c) 3 and 4 (d) 3 only
Limited is the largest non-oil importer of the 4. Despite having large reserves of coal, why does India
country. import millions of tonnes of coal?
2. Project and Equipment Corporation of India Limited 1. It is the policy of India to save its own coal reserves
is under the Ministry of Industry. for future, and import it from other countries for the
3. One of the objectives of Export Credit Guarantee present use.
Corporation of India Limited is to enforce quality
Industry and Infrastructure E-155

2. Most of the power plants in India are coal-based and 2. It is the first company outside Japan to get the
they are not able to get sufficient supplies of coal deming application prize in 2008 for excellence in
from within the country. total quality management
3. Steel companies need large quantity of coking coal 3. Immediately after the enactment of the Provident
which has to be imported. Fund Law in India, Tata steel introduced provident
Which of the statements given above is/are correct? fund for its employees.
(a) 1 only (b) 2 and 3 4. It is the first company in the world to get social
(c) 1 and 3 (d) 1, 2 and 3 accountability 8000 certification from the social
5. With reference to the Public Sector Undertakings in accountability international from the USA.
India, consider the following statements Which of the statements given above are correct?
1. Minerals and Metals Trading Corporation of India (a) 2 and 3 (b) 3 and 4
Limited is the largest non-oil importer of the (c) 1, 2 and 3 (d) 1, 2, 3 and 4
country. 10. Consider the following statements :
2. Project and Equipment Corporation of India Limited 1. MMTC limited is India’s largest international
is under the Ministry of Industry. trading organisation.
2. Neelanchal Ispat Nigam Limited has been set up by
3. One of the objectives of Export Credit Guarantee
MMTC jointly with the government of Odisha.
Corporation of India Limited is to enforce quality
Which of the statements given above is/are correct?
control and compulsory pre-shipment inspection of
(a) 1 only (b) 2 only
various exportable commodities.
(c) Both 1 and 2 (d) Neither 1 nor 2
Which of these statements is/are correct? 11. Consider the following statements about the Navaratna
(a) 1 only (b) 1 and 2 status industries.
(c) 2 and 3 (d) 3 only 1. Navaratna was originally assigned to nine public
6. What is/are the recent policy initiative(s) of Government sector enterprises in 1997.
of India to promote the growth of manufacturing sector? 2. The number of PSEs (Public Sector Enterprises)
1. Setting up of National Investment and Manufacturing having Navaratna status is now more than 15
Zones. industries.
2. Providing the benefit of ‘single window clearance’. Which of the statements given above is/are correct?
3. Establishing the Technology Acquisition and (a) 1 only (b) 2 only
Development Fund. (c) Both 1 and 2 (d) Neither 1 nor 2
Select the correct answer using the codes given below: 12. Consider the following statements :
(a) 1 only (b) 2 and 3 1. The first petrochemical industry in India was started
(c) 1 and 3 (d) 1, 2 and 3 with the establishment of National organic chemical
7. SEZs were established with the objective of ____ . industry limited in sixties.
1. attracting foreign investment directly. 2. The real thrust of petrochemical industry in India
2. protect domestic market from competition from came with the establishment of Indian Petrochemical
multinationals. Corporation Limited (IPCL) at Baroda.
3. providing more capital to agricultural and allied Which of the statements given above is/are
activities. (a) 1 only (b) 2 only
(a) 1 only (b) 2 only (c) Both 1 and 2 (d) Neither 1 nor 2
(c) 3 only (d) All of the above 13. Consider the following statements :
8. Consider the following statements: 1. Miniratna public sector enterprises can be a joint
Small-scale industries are, in most cases, not as venture, set subsidiary company or overseas office
efficient and competitive as the large-scale ones. Yet but with certain conditions.
the Government provides preferential treatment and 2. Till now there are less than 50 public sector
reservations in a range of products to the small firms enterprises which have been given Miniratna status
because small-scale industries by the Indian government.
Which of the statements given above is/are correct?
1. provide higher employment on a per unit capital
(a) 1 only (b) 2 only
deployment basis.
(c) Both 1 and 2 (d) Neither 1 nor 2
2. promote a regional dispersion of industries and
14. Consider the following statements :
economical activities.
1. India has attained self-sufficiency in almost all
3. have performed better in export of manufactured consumer goods.
products than the large scale ones. 2. In terms of capital goods, India still depends on the
4. provide jobs to low-skill workers, who otherwise import.
may not find employment avenues elsewhere. Which of the statements given above is/are correct?
Which of the above statements are correct? (a) 1 only (b) 2 only
(a) 1 and 4 (b) 1 and 2 (c) Both 1 and 2 (d) Neither 1 nor 2
(c) 2 and 3 (d) 3 and 4 15. Consider the following statements about Competition
9. Consider the following statements about Tata steel: Commission of India.
1. It is the Asia’s first privately own integrated iron 1. Competition Commission of India (CCI) was
and steel plant. established with effect from 14th october 2003.
E-156 Industry and Infrastructure

2. The goal is to combat the monopoly tendencies government provides preferential treatment and
and faster the competition among public sector reservation in a range of product to the small firms
enterprises. because small industries
3. CCI (Competition Commission of India) consists 1. Provide higher employment on a par unit capital
of a chairperson and 6 members appointed by the development basis.
central government. 2. Promote a regional dispersion of industries and
Which of the statements given above is/are correct? economic activities.
(a) 1 only (b) 2 only 3. Have promoted better in export of manufacture
(c) 1 and 2 (d) All of the above products than the large scales one.
16. Arrange the following Acts in order in which they were 4. Provides job to low skills workers, who otherwise
enacted. Use the codes given below. may not find employment avenues elsewhere.
1. The Industrial Employment (Standing orders) Act. Which of the above statement(s) are correct ?
2. The Indian Trade Unions Act (a) 1 and 4 (b) 1 and 2
3. The Industrial Disputes Act (c) 2 and 3 (d) 3 and 4
4. The Employment Exchange (Compulsory 22. Consider the following factors regarding an industry:
Notification) Act 1. Capital investments
Codes: 2. Business turnover
(a) 2, 1, 3, 4 (b) 3, 4, 1, 2 3. Labor forces
(c) 1, 2, 4, 3 (d) 4, 3, 2, 1 4. Power consumption
17. In context of the present ceiling on investment for Which of the following factor determine the nature and
categorization of various enterprises as Micro, Small and size of the industry
Medium Manufacturing Enterprises (MSMEs), which of (a) 1, 3 and 4 (b) 1, 2 and 4
the following statements are correct: (c) 2, 3 and 4 (d) 2 and 3
1. A micro enterprise is an enterprise where the 23. With the reference to the public sector undertaking in
investment in equipment does not exceed `25 lakh. India, consider the following statements :
2. A small enterprise is an enterprise where the 1. Minerals and Metals Trading Corporation of
investment in equipment is more than `25 lakh but India limited is the largest non-oil importer of the
does not exceed `5 crore. country.
3. A medium enterprise is an enterprise where the 2. Project and Equipment Corporation of India Limited
investment in equipment is more than `5 crore but is under the ministry of industry.
does not exceed `10 crore. 3. One of the objectives of Export Credit Guarantee
(a) Only 1 (b) Only 2
Corporation of India Limited is to enforce quality
(c) Only 3 (d) All of them
control and compulsory pre-shipment inspection of
18. Consider the following factors:
1. High Investment in industries various exportable commodities.
2. Expanding market for product Which of the above statement(s) are correct ?
3. Dynamic entrepreneurship (a) 1 only (b) 1 and 2
Which of the above are required for accelerated growth (c) 2 and 3 (d) 3 only
of an economy ? 24. The Reserve Bank of India regulates the commercial
(a) Only 1 (b) 1 & 2 banks in matters of
(c) 1, 2 & 3 (d) 2 & 3 1. liquidity of assets
19. India has a high export earning from Diamonds. 2. branch expansion
In this context, consider the following: 3. merger of banks
1. High Diamonds Stocks 4. winding-up of banks
2. Large Production of Industrial Diamonds Select the correct answer using the codes given below.
3. Availability of experts on cutting and polishing (a) 1 and 4 only (b) 2, 3 and 4 only
Which among the above is/ are most appropriate reason/ (c) 1, 2 and 3 only (d) 1, 2, 3 and 4
reasons behind the above fact? 25. Which of the following grants / grant direct credit
(a) Only 1 (b) Only 1 & 3 assistance to rural households?
(c) Only 3 (d) 1, 2 & 3 1. Regional Rural Banks
20. Consider the following statements: 2. National Bank for Agriculture and Rural
1. Lack of adequate entrepreneurship and leadership in Development
business. 3. Land Development Banks
2. Lack of saving to invest. Select the correct answer using the codes given below.
3. Lack of technology, skills and infrastructure. (a) 1 and 2 only (b) 2 only
4. Limited purchasing power among the larger masses. (c) 1 and 3 only (d) 1, 2 and 3
Which of the above statement(s) are correct ? 26. Consider the following statements:
(a) 1, 2 and 3 (b) 1, 3 and 4 1. Inflation benefits the debtors.
(c) 2, 3 and 4 (d) 1, 2 and 4 2. Inflation benefits the bondholders.
21. Consider the following statements: Which of the statements given above is/are correct?
Small-Scale industries are in most cases not as efficient (a) 1 only (b) 2 only
as and competitive as the large scales one. Yet the (c) Both 1 and 2 (d) Neither 1 nor 2
Industry and Infrastructure E-157

27. Consider the following liquid assets: II. Existing units in basic and key industries could
1. Demand deposits with the banks continue in the private sector but no fresh private
2. Time deposits with the banks investment in these sectors was to be allowed.
3. Saving deposits with the banks III. Twenty important industries were allowed in private
4. Currency sector but under strict supervision of the state.
The correct sequence of these assets in the decreasing IV. All industries not covered by the above three
order of liquidity is categories were allowed in the private sector under
(a) 1-4-3-2 (b) 4-3-2-1 general supervision of the state.
(c) 2-3-1-4 (d) 4-1-3-2 (a) Only I (b) I & II
28. What is/are the facility/facilities the beneficiaries can (c) II & III (d) All the above
get from the services of Business Correspondent (Bank 33. What are the major points when India was declared as
Saathi) in branchless areas? mixed economy?
1. It enables the beneficiaries to draw their subsidies I. Those industries completely owned by the
and social security benefits in their villages. Government e.g. ordinance, atomic energy,
2. It enables the beneficiaries in the rural areas to railways and any industry of national importance
make deposits and withdrawals. were to be the exclusive domain of the Central
Select the correct answer using the code given below. government.
(a) 1 only (b) 2 only II. A group of 18 specified industries (of medium
(c) Both 1 and 2 (d) Neither 1 nor 2 category) were in control of the state governments
29. In the context of Indian economy, which of the following in liaison with the central government.
is/are the purpose/purposes of ‘Statutory Reserve III. The remaining industrial options which were not
Requirements’? covered by either the centre or state lists, were left
1. To enable the Central Bank to control the amount open to the private sector.
of advances the banks can create IV. This policy was to be reviewed after 5 years.
2. To make the people’s deposits with banks safe and (a) I & II (b) I, II & III
liquid (c) Only III (d) All the above
3. To prevent the commercial banks from making 34. Which sentence is correct regarding Industrial
excessive profits Development and Regulation Act 1951?
4. To force the banks to have sufficient vault cash to I. They controlled the authorization of capability,
meet their day-to-day requirements whereabouts and growth of any request for
Select the correct answer using the code given below. manufacture of new products.
(a) 1 only (b) 1 and 2 only II. They controlled the authorization of foreign
(c) 2 and 3 only (d) 1, 2, 3 and 4 exchange expenditure on the import of plant and
30. List-I List-II machinery.
(Industries) (Industrial Centres) III. They controlled the authorization for the terms of
(A) Pearl fishing (1) Pune international joint ventures.
(a) I & II (b) Only III
(B) Automobiles (2) Tuticorin
(c) II & III (d) All the above
(C) Ship building (3) Pinjore 35. Match the following:
(D) Engineering (4) Marmagao Column I Column II
goods I. PSU’s a. Category C
(a) A – 2; B – 1; C – 4; D – 3 II. Joint Venture b. Category A
(b) A – 2; B – 1; C – 3; D – 4 III. Private Sector c. Category B
(c) A – 1; B – 2; C – 4; D – 3 (a) I – b, II – c, III – a
(d) A – 1; B – 2; C – 3; D – 4 (b) I – a, II – c, III - b
31. Match the following products with their percentage (c) I – c, II – b, III - a
part that was controlled by the foreigners at the time of (d) I – b, II – a, III – c
independence: 36. Which statement is correct regarding joint venture?
Product Percentage I. 15 industrial areas were put under this category.
II. This category also carried the provision of
I. Tea a. 70%
compulsory licensing.
II. Jute b. 73% III. This provision led to the establishment of the so-
III. Coal c. 65% called ‘Licence- Quota- Permit raj’ in the economy.
IV. Minerals d. 90% (a) I & II (b) II & III
(a) I – c, II – d, III – a, IV – b (c) Only I (d) All the above
(b) I – b, II – d, III – a, IV - c 37. Which statement is correct regarding Industrial Policy
(c) I – c, II – b, III – a, IV - d 1956?
(d) I – a, II – d, III – c, IV - b I. The policy of 1956, for the first time, recognized
32. What were the modified policies of government after the contribution of small scale industries in the
1950’s? growth of the Indian economy.
I. Defence and strategic industries were to be the II. It laid stress on rational distribution of national
income and effective utilization of resources.
exclusive domain of the government.
E-158 Industry and Infrastructure

III. This policy is considered one of the most important II. Electronic Hardware Technology Park (EHTP),
industrial policies of India as it decided the nature Software Technology Park (STP) Scheme for
and scope of the Indian economy till the reforms of building up strong electronic industry to enhance
1991. exports.
(a) I & II (b) II & III III. Liberalised policy for Foreign Direct Investment
(c) All the above (d) Only I (FDI).
38. Which statement is correct regarding FERA 1973? IV. Abolition of the MRTP limit.
I. The Foreign Exchange and Regulation Act (FERA) (a) I & III (b) III & IV
was passed in 1980. (c) Only II (d) All the above
II. This resulted in a tremendous shift in the foreign 44. Match the following investment requirements under the
investment policy of the Government of India. Manufacturing Enterprises category:
III. Foreign Investment was allowed in only those Column I Column II
industries that were directly into exports. I. Micro Enterprise a. 5 crores to upto 10 crores
(a) I & II (b) Only III
II. Small Enterprise b. 25 lakh to upto 5 crores
(c) II & III (d) All the above
39. Consider the following statements: III. Medium Enterprise c. Upto 25 lakh
I. The policy listed out the various Appendix 1 (a) I – c, II – b, III – a
industries that could be started by large business (b) I – b, II – c, III - a
houses so that small industries were not driven out (c) I – c, II – a, III - b
of business. (d) I – a, II – b, III - c
II. The establishment of small and medium industries 45. Match the following investment requirements under the
was encouraged. Service Enterprises category:
III. Private industries were encouraged to set up Column I Column II
production units in rural areas and in backward I. Micro Enterprise a. 2 crores to upto 5 crores
areas with a vision to give thrust for the economic II. Small Enterprise b. 10 lakh to upto 2 crores
development of those areas. III. Medium Enterprise c. Upto 10 lakh
Which statement is /are correct regarding Industrial (a) I – c, II – b, III – a
Policy 1973: (b) I – b, II – c, III – a
(a) I & II (b) II & III (c) I – c, II – a, III – b
(c) Only III (d) All the above (d) I – a, II – b, III – c
40. Which statements are correct regarding Industrialization 46. Which statement is correct regarding cottage industries?
post 1990? I. Cottage industry is run by family members on full
I. Exemption from licensing was allowed for all start
or part time basis.
ups and for those with an investment worth `2.5
crores in fixed assets and a right to import up to II. It possesses negligible capital investment.
30% of the total value of plant and machinery. III. The production is done by hands and no wage
II. Foreign equity investment was allowed up to 70%. earning person is employed in cottage industry.
III. Geographical restrictions and investment cap for (a) I & II (b) II & III
small industries were removed. (c) Only II (d) All the above
(a) I & II (b) Only II 47. Consider the statements:
(c) I & III (d) All the above I. Small industrial units employ wage earning labour
41. Which among the below statements are correct regarding and production is done by the use of modern
the Features of Industrial Policy 1991? techniques.
I. To maintain a sustained growth in productivity. II. Capital investment is also present in small industries
II. To enhance gainful employment. Which among the above statement is correct regarding
III. Liberalisation of Industrial Licensing Policy. small industries?
IV. Introduction of Industrial Entrepreneur’s Memorandum (a) Only I (b) Only II
(i.e., no industrial approval is required for industries (c) Both I & II (d) None of the above
not requiring compulsory licensing). 48. Which statement is correct regarding village industries?
(a) I & II (b) I & III I. The industries established in rural areas having
(c) Only IV (d) All the above population below 10,000 and having less than
42. Industrial Policy 1991 mainly focuses on which points? ` 15,000 as fixed capital investment per worker will
I. Deregulating Indian industry. be termed as village industries.
II. Allowing the industry freedom & flexibility in II. KVIC and state village Industries Board provide
responding to market forces economic and technical assistance in establishing
III. Providing a policy regime which facilitates and and operating these industrial units.
fosters growth of Indian industry. (a) I & II (b) Only I
(a) I & II (b) II & III (c) Only II (d) None of the above
(c) Only III (d) All the above 49. Match the following according to the contribution of
43. What were the policy measures taken under Industrial cotton industries in different sectors:
Policy 1991? Column I Column II
I. Non-Resident Indians Scheme (NRIs are allowed to I. GDP a. 13.5%
invest upto 100% equity on non-repatriation basis in II. Manufacturing Value b. 4%
all activities except for a small negative list). III. Export earning c.14%
Industry and Infrastructure E-159

(a) I – b, II – c, III - a 55. What is government owned companies?


(b) I – a, II – c, III - b I. They are primarily meant to provide essential
(c) I – b, II – a, III - c services such as railways.
(d) I – c, II – b, III - a II. They are public enterprises that came into existence
50. Which statement is correct regarding cotton industry?
by a Special Act of the Parliament.
I. The first Indian modernised cotton mill, located in
Kolkata in 1818 III. They are companies in which 51% or more of the
II. The second mill named ‘Bombay Spinning and paid up capital is held by the central or any state
Weaving Co.’ was established in 1854 at Bombay government (partly or wholly by both).
by KGN Daber IV. They are set up whenever it is felt that certain functions
III. There were 394 cotton mills in India before the need to be discharged outside the governmental
partition set up with some amount of independence and
(a) I & II (b) II & III flexibility without day-to-day interference from the
(c) Only II (d) All the above
governmental machinery
51. Which statement is correct regarding iron and steel
industry? (a) Only III (b) I & III
I. Large scale iron and steel production was started in (c) Only II (d) None of the above
1907 by TISCO established at Jamshedpur. 56. What are autonomous bodies?
II. In 1919, Indian Iron and Steel Company (IISCO) I. They are primarily meant to provide essential
was established at Burnpur. Both TISCO and IISCO services such as railways.
are private sector companies. II. They are public enterprises that came into existence
III. The first public sector company was ‘Vishwashwaraiya by a Special Act of the Parliament.
Iron and Steel Works at Bhadravati’. III. They are companies in which 51% or more of the
(a) I & II (b) Only II paid up capital is held by the central or any state
(c) II & III (d) All the above government (partly or wholly by both).
52. What are the categories of PSEs? IV. They are set up whenever it is felt that certain functions
I. Departmental undertakings need to be discharged outside the governmental
II. Statutory corporations
set up with some amount of independence and
III. Government-owned companies
IV. Autonomous bodies flexibility without day-to-day interference from the
(a) II & III (b) II & IV governmental machinery
(c) All the above (d) Only I (a) Only IV (b) I & III
53. What is departmental undertaking? (c) Only II (d) None of the above
I. They are primarily meant to provide essential 57. Which sentence is correct regarding CPSE?
services such as railways. I. During the first five-year plan (1950-51 to 1955-56)
II. They are public enterprises that came into existence there were only five CPSEs with a total financial
by a Special Act of the Parliament. investment (Including equity plus long-term loans)
III. They are companies in which 51% or more of the of ` 290 million
paid up capital is held by the central or any state II. In 2015, there are as many as 290 CPSEs, wherein,
government (partly or wholly by both). 169 are Holding CPSEs and 121 are the subsidiaries.
IV. They are set up whenever it is felt that certain III. As of Mar 31, 2012, the total financial investment
functions need to be discharged outside the of CPSEs was 7,292.3 billion, showing a CAGR of
governmental set up with some amount of around 12% during the same period.
independence and flexibility without day-to-day (a) I & II (b) Only I
interference from the governmental machinery (c) Only III (d) All the above
(a) Only I (b) I & III 58. Match the following products with the percentage of
(c) Only III (d) None of the above CPSE control over them:
54. What is Statutory Corporation? Column I Column II
I. They are primarily meant to provide essential I. Coal a. 70%
services such as railways. II. Crude Oil b. 55%
II. They are public enterprises that came into existence III. Refineries c. 80%
by a Special Act of the Parliament. (a) I – c, II – a, III – b
III. They are companies in which 51% or more of the (b) I – a, II – c, III – b
paid up capital is held by the central or any state (c) I – c, II – b, III – a
government (partly or wholly by both). (d) I – b, II – a, III – c
IV. They are set up whenever it is felt that certain functions 59. Which statement is true regarding employment in India?
need to be discharged outside the governmental I. Food products industry has ranked first in generating
employment, providing employment to 0.48 million
set up with some amount of independence and
persons (13.1%).
flexibility without day-to-day interference from the II. The next two industry groups were Non-metallic
governmental machinery mineral products with employment of 0.45 million
(a) Only I (b) I & III persons (12.2%) and Metal products with 0.37
(c) Only II (d) None of the above million persons (10.2%).
E-160 Industry and Infrastructure

III. In chemicals & chemical products, machinery parts 65. Consider the following statements:
except electrical parts, wood products, basic metal I. Economic infrastructure is located within the system
industries, paper products & printing, hosiery of production and distribution. Some of these are
& garments, repair services and rubber & plastic the transport system, banking system and power.
products, the contribution ranged from 9% to 5% II. Social infrastructure contributes to the production
(a) I & II (b) II & III process from outside the system. Some examples
(c) Only III (d) All the above are education, training, hospitals and housing.
60. Which sentences are correct regarding export in India? Which statement is correct regarding Infrastructure?
I. 45%-50% of the Indian Exports is contributed by (a) Only I (b) Only II
SSI Sector. (c) Both I & II (d) None of these
II. Direct exports from the SSI Sector account for 66. Which statement is correct regarding energy?
nearly 45% of total exports. I. Coal was the most important source of energy during
III. Besides direct exports, it is estimated that small- the Industrial Revolution. In the 20th century, it was
scale industrial units contribute around 15% to replaced by oil, which was an even cheaper source.
exports indirectly. II. When the oil producing countries formed a cartel
(a) I & II (b) I & III (OPEC) and increased the price of crude oil from
(c) Only II (d) All the above $2.1 per barrel in 1973 to as much as $28 per barrel
61. Consider the below statements: in 1980, it hit the developing countries like India
I. Internal factors could be mismanagement, lack of the most.
finance wrong dividend policy, not providing for (a) Only I (b) Only II
reserves or deliberate ‘milking’ of the unit by the (c) I & II (d) None of these
owners. 67. Match the following power with their share of
II. External factors could be wrong government
percentage:
policy, power shortage, non-availability of raw
Column I Column II
materials, transport problem, lack of technological
I. Thermal a. 3%
advancement and labour disputes.
II. Hydro b. 5%
Which statement is correct regarding industrial sickness?
(a) Only I (b) Only II III. Renewable energy c. 66%
(c) Both I & II (d) None of the above IV. Nuclear d. 26%
62. What are the aspects of Competition Act 2002? (a) I – a, II – d, III – b, IV – c
I. Prohibition of anti competitive agreements (b) I – c, II – d, III – b, IV – a
II. Prohibition of abuse of dominance (c) I – c, II – d, III – a, IV – b
III. Regulation of combination (acquisitions, mergers (d) I – b, II – c, III – d, IV – a
and amalgamations of certain size 68. Which sentence is correct regarding Power?
IV. Establishment of Competition Commission of India I. A major initiative to expedite power generation has
(CCI) been the development of coal-based-Ultra Mega
(a) I & II (b) II & III Power Projects (UMPP) each with a capacity of
(c) Only I (d) All the above 4000 MW or above.
63. Which statement is correct regarding disinvestment II. Nine sites have been identified including four pithead
policy? sites (Chhattisgarh, Jharkhand, Madhya Pradesh
I. Citizens have every right to own part of the shares and Odisha) and five coastal sites (Andhra Pradesh,
of Public Sector Undertakings. Gujarat, Karnataka, Maharashtra and Tamil Nadu).
II. Public Sector Undertakings are the wealth of the III. The pithead units will get coal from captive mines
Nation and this wealth should rest in the hands of while the coastal plants will import coal.
the people. (a) I & II (b) II & III
III. While pursuing disinvestment, Government has (c) Only III (d) All the above
to retain majority shareholding, i.e. at least 51% 69. Which statement is correct regarding railways?
and management control of the Public Sector I. The first train in India was started on a small rail
Undertakings. route of 34 kilometers between Bombay and Thane
(a) I & II (b) II & III on April 16, 1853.
(c) Only I (d) All the above
II. The Indian Railway network has become the biggest
64. What are the approaches of disinvestment?
I. In all cases of disinvestment, the Government would railway of Asia and the third in the world
retain at least 31% equity and the management III. Within the freight segment, bulk traffic accounts for
control. nearly 84% of revenue-earning freight traffic (in
II. All cases of disinvestment are to be decided on a physical terms), of which about 44% is coal.
case by case basis (a) I & II (b) Only III
III. The Department of Disinvestment is to identify (c) II & III (d) All the above
CPSEs in consultation with respective administrative 70. Match the following length of various railway track:
Ministries and submit proposal to Government
in cases requiring Offer for Sale of Government Column I Column II
equity. I. Broad Gauge a. 2537 km
(a) I & II (b) Only I II. Meter Gauge b. 54257 km
(c) II & III (d) All the above III. Narrow Gauge c. 7180 km
Industry and Infrastructure E-161

(a) I – b, II – c, III – a II. It is expected that another 350 aircraft will be added
(b) I – a, II – c, III - b during the next five years.
(c) I – b, II – a, III – c III. An Airport Economic Regulatory Authority (AERA)
(d) I – c, II – b, III – a is being set up to ensure healthy competition
71. Which statement is correct regarding road transport? among operators and settlement of disputes among
I. India has one of the largest road networks in the operators as well as between them and passengers.
world aggregating to about 3.62 million km at (a) I & II (b) Only ii
present.
(c) II & III (d) All the above
II. Three initiatives in the road sector were begun in
74. What are the points covered in new telecom policy?
recent years: The National Highway Development
I. Rural Tele-density – To improve rural teledensity
Project (NHDP), Pradhan Mantri Bharat Jodo
Pariyojana (PMBJP) and Pradhan Mantri Gram from the current level of around 39 to 70 by the
Sadak Yogana (PMGSY) year 2017 and 100 by the year 2020.
III. NHDP dealt with building high quality highways. II. Broadband – “Broadband For All” at a minimum
The PNBJP dealt with linking up major cities to the download speed of 2 Mbps
NHDP Highways. The PMGSY addressed rural III. Domestic Manufacturing – Making India a global
roads. hub
(a) I & II (b) II & III IV. Convergence of Network, Services and Devices
(c) Only I (d) All the above (a) I & II (b) Only II
72. Which statement is correct regarding shipping? (c) II & III (d) All the above
I. Indian overseas fleet ranks the 17th the world in 75. Which statement is correct regarding information
terms of tonnage, with 659 ships totalling 77 lakh technology?
gross registered tonnage (GRT). I. The IT industry is the biggest private sector
II. 102 Indian shipping companies are in operation of employer in India, and it added 230,000 employees
which 65 are engaged only in coastal shipping. in 2014-15, thus making the total number of jobs in
III. The public sector Shipping Corporation of India
the industry close to 3.5 million.
(SCI) has a fleet of 112 vessels aggregating three
II. The IT sector also accounted for 9.5% of the gross
million GRT.
(a) I & II (b) Only II domestic product. The IT industry holds the largest
(c) II & III (d) All the above share of total services exports at 38%.
73. Which statement is incorrect regarding civil aviation? (a) Only I (b) Only II
I. There are 20 scheduled airline operators using 334 (c) Both I & II (d) None of these
aircraft.

EXERCISE-1 initiatives launched by the government in the early


1. (a) 2. (c) 3. (d) 4. (d) 5. (b) ‘90s. The basic purpose of the scheme was to allow
6. (d) 7. (b) 8. (b) 9. (c) 10. (d) exporters to import machinery and equipment at
11. (a) ONGC is India’s largest petrochemical company. affordable prices so that they can produce quality
IPCL is the second largest petrochemical company. products for the export market.
12. (a) Rourkela Steel Plant (RSP) is the first intergrated 18. (b) Oil sector majors, ONGC Videsh (OVL) and the
steel plant in the public sector in India. It was set up Oil India-IndianOil (OIL-IOC) combine have
with German collaboration with an installed capacity bagged an oil block each in Libya. This will enhance
of 1 million tonnes. exploration possibilities for the Indian companies in
13. (c) 14. (b) 15. (a) northern Africa.
16. (b) In India, ‘Report on Currency and Finance’ is the 19. (c) NASSCOM is the organisation of companies of
annual publication of Reserve Bank of India.The Information Technology.
report highlights the evaluation or devaluation of 20. (c) 21. (a) 22. (a) 23. (b) 24. (d) 25. (c)
rupee as a currency along with it other aspects and 26. (b) 27. (c) 28. (a) 29. (d) 30. (c) 31. (d)
projects the financial condition of the economy. 32. (d) 33. (a) 34. (c)
17. (b) The Export Promotion Capital Goods (EPCG) 35. (c) Oil and gas industry is the largest contributor to
scheme was one of the several export-promotion excise revenues in India followed by steel industry.
E-162 Industry and Infrastructure

36. (c) PKI stands for Public Key Infrastructure. The Nigam Limited (NINL), is a company promoted by
Reserve Bank of India released a draft report on MMTC Ltd, Industrial Promotion and Investment
making payment system effective to secure a safe, Corporation of Orissa limited (IPICOL).
secure, efficient, robust and sound payment system 11. (c) Navratna was the title given originally to 9
in the country. Public Sector Enterprises (PSEs) identified by the
37. (c) Disguised unemployment is a situation when people government in 1997 as public sector companies
do not have productive full-time employment, that have comparative advantages enjoying greater
but are not counted in the official unemployment autonomy to compete in the global market so as
statistics. to support in their drive to become global giants.
38. (c) The Balance of Payments(BoPs) accounts are an However, the number of PSEs (Public Sector
accounting record of all monetary transactions Enterprises) having Navaratna status is now more
between country and the rest of the world. These than 15 industries.
transactions include payment for the country’s 12. (c) The first petrochemical industry in India was started
exports and imports of goods, services, financial with the establishment of National Organic Chemical
capitals and financial transfers. Industry Limited in 1961. The company manufacture
39. (d) A tight monetary policy is a course of action various organic and inorganic chemicals. The real
undertaken by Central bank to constrict spending in thrust of Petrochemical industry in India came
an economy, or to curb inflation when it is rising too with the establishment of Indian Petrochemical
fast. The increased bank rate increases the cost of Corporation Limited (IPCL) at Baroda on March
borrowing and effectively reduces its attractiveness. 22, 1969, with a view to promote and encourage
40. (d) Taxes on tooth paste come under GST which is the use of plastics in India.
administered by State government.Sales tax is paid 13. (c) Miniratna public sector enterprises can be a joint
to sales tax authority in the state from where the venture, set subsidiary company or overseas office
goods are moved. but with certain conditions i.e Miniratna-I is up to
41. (b) Venture capital (VC) is a long term financial capital ` 500 crore or equal to their net worth, whichever
provided to early-stage, high-potential, growth start is lower. Miniratna-II: is up to ` 300 crore or up
up companies or new companies. to 50% of their net worth, whichever is lower. Till
now less than 50 public sector enterprises have been
EXERCISE-2
given Miniratna status by the Indian government.
1. (d) 2. (a) 14. (b) In terms of capital goods, India still depends on the
3. (c) On 27 January 2005, the Government had decided to import has not attained self sufficiency in almost all
constitute a ‘National Investment Fund’ (NIF) into consumer goods.
which realization from sale of minority shareholding 15. (d) Competition Commission of India is a body of
of the Government in CPSEs would be channelised. the government of India responsible for enforcing
This fund is managed by professionals and a part of the Competition Act, 2002 throughout India and
it is used for use in social sector – like education, to prevent activities that have an adverse effect
health care and employment. on competition in India. It was established on 14
4. (b) October 2003.The goal is to combat the monopoly
5. (a) Minerals and Metals Trading Corporation Limited tendencies and faster the competition among public
(MMTC) is one of the two biggest foreign exchange sector enterprises. CCI (Competition Commission
earner for India. It is a leading international trading of India) consists of a chairperson and 6 members
company with a turnover of over 68,885 crore appointed by the central government.
rupees in 2010–11. It is the largest international 16. (a) 17. (d) 18. (c) 19. (c)
trading company of India and the first Public Sector 20. (d) 21. (a) 22. (a) 23. (a)
Enterprise to be accorded the status of FIVE STAR 24. (d) The Reserve Bank of India is the main monetary
EXPORT HOUSE by Govt. of India for long authority of the country and beside that, in its
standing contribution to exports. MMTC is the capacity as the central bank, acts as the bank of
largest non-oil importer in India. the national and state governments. Some times it
6. (c) 7. (d) 8. (b)
happens that some of the banks close down due to
9. (c) Tata steel is the asia’s first privately owned
non recovery of loans or such other issues. In such
integrated iron and steel plant. It is the first company
conditions people have to suffer as their money
outside Japan to get the Doming Application Prize
is with the bank then. For this reason there is
in 2008 for excellence in total quality management.
provision for winding up of the banking company
Tata Steel started provident fund for its employees
under the Banking Regulation Act, 1949. The power
as early as in 1920, however, it became a law for
of winding up of Bank lies in the hand of Reserve
all employers under the Provident Fund Act only in
Bank of India.
1952.
25. (c) Land development bank started financing long term
10. (c) MMTC Ltd., Metals and Minerals Trading
Corporation of India, is one of the two highest loan for more significant rural development activities
earners of foreign exchange for India and India’s like rural and cottage industries, rural artisans etc.
largest public sector trading body. Neelanchal Ispat The main purpose of RRB’s is to mobilize financial
Industry and Infrastructure E-163

resources from rural / semi-urban areas and grant 39. (d) The policy listed out the various Appendix 1
loans and advances mostly to small and marginal industries that could be started by large business
farmers, agricultural laborers and rural artisans. houses so that small industries were not driven out
26. ( a) Those who benefit from higher inflation are debtors of business. The establishment of small and medium
and those who suffer from it are creditors. If one has industries was encouraged. Private industries were
substantial debt, each rupee one has to repay would encouraged to set up production units in rural areas
be worth less than when it was borrowed. In this and in backward areas with a vision to give thrust
way, one pays back less in real terms. for the economic development of those areas.
27. (d) Currency/cash is the most liquid ,then the demand 40. (c) Exemption from licensing was allowed for all start
deposits (current accounts), then the saving deposits ups and for those with an investment worth ` 2.5
with bank and finally the least liquid is the time crores in fixed assets and a right to import up to
deposits with the bank (fixed deposits). 30% of the total value of plant and machinery.
28. (c) Besides giving access to banking, it also enables Foreign equity investment was allowed up to 40%.
government subsidies and social security benefits Geographical restrictions and investment cap for
to be directly credited to the accounts of the small industries were removed.
beneficiaries, enabling them to draw the money 41. (a)
from the bank saathi or business correspondents in 42. (d) Deregulating Indian industry. Allowing the industry
their village itself. freedom & flexibility in responding to market forces
29. (b) Because Reserve requirements are designed as and providing a policy regime which facilitates and
“precautionary measures” and not to stop banks fosters growth of Indian industry.
from “excessive” profit. This eliminates (c) and 43. (d)
(d). 44. (a) Micro Enterprises are those enterprises which have
30. (a) Industries – Industrial centres investment up to ` 25 lakh. Small Enterprises are
(A) Pearl fishing – Tuticorin those enterprises with investment above 25 lakh
(B) Automobile industry– Pune and up to 5 crores. Medium Enterprises are those
(C) Ship building – Marmagao enterprises which have investment above ` 5 crores
(D) Engineering goods– Pinjore and up to ` 10 crores.
31. (a) 45. (a) Micro Enterprises investment includes companies
32. (d) Defense and strategic industries were to be the with investment up to ` 10 lakh. Small Enterprises
exclusive domain of the government. Existing units need an investment above 10 lakh and up to 2
in basic and key industries could continue in the crore. Medium Enterprises are those enterprises
private sector but no fresh private investment in which have an investment above 2 crore and up to 5
these sectors was to be allowed. Twenty important crore.
industries were allowed in private sector but under 46. (d) Cottage industry is run by family members on full
strict supervision of the state. All industries not or part time basis. It possesses negligible capital
covered by the above three categories were allowed investment. The production is done by hands and
in the private sector under general supervision of the no wage earning person is employed in cottage
state. industry.
33. (d) 34. (d) 35. (a) 47. (c) Small industrial units employ wage earning labour
36. (b) 12 industrial areas were put under this category. This and production is done by the use of modern
category also carried the provision of compulsory techniques. Capital investment is also present in
licensing. This provision led to the establishment of small industries.
the so- called ‘Licence- Quota- Permit raj’ in the 48. (a) The industries established in rural areas having
economy. population below 10,000 and having less than Rs.
37. (c) The policy of 1956, for the first time, recognized 15,000 as fixed capital investment per worker will
the contribution of small scale industries in the be termed as village industries. KVIC and state
growth of the Indian economy. It laid stress on village Industries Board provide economic and
rational distribution of national income and effective technical assistance in establishing and operating
utilization of resources. This policy is considered these industrial units.
one of the most important industrial policies of 49. (a) Cotton Industry in India is the most organized
India as it decided the nature and scope of the Indian industry which contributes to about 4% of GDP,
economy till the reforms of 1991. 14% of manufacturing value and 13.5% to total
38. (c) The Foreign Exchange and Regulation Act (FERA) export earnings.
was passed in 1973. This resulted in a tremendous 50. (d)
shift in the foreign investment policy of the 51. (d) Large scale iron and steel production was started
Government of India. Foreign Investment was in 1907 by TISCO established at Jamshedpur. In
allowed in only those industries that were directly 1919, Indian Iron and Steel Company (IISCO) was
into exports. established at Burnpur. Both TISCO and IISCO are
E-164 Industry and Infrastructure

private sector companies. The first public sector 63. (d)


company was ‘Vishwashwaraiya Iron and Steel 64. (d) In all cases of disinvestment, the Government would
Works at Bhadravati’. retain at least 51% equity and the management
52. (c) The PSEs in India are basically categorized under control. All cases of disinvestment are to be
four broad types based on their ownership structure. decided on a case by case basis. The Department of
These include: departmental undertakings, statutory Disinvestment is to identify CPSEs in consultation
corporations, government-owned companies and with respective administrative Ministries and submit
autonomous bodies set up as registered societies. proposal to Government in cases requiring Offer for
53. (a) Departmental undertakings are primarily meant Sale of Government equity.
to provide essential services such as railways. 65. (c)
They function under the control of the respective 66. (c) Coal was the most important source of energy during
ministries of Government of India (GoI). the Industrial Revolution. In the 20th century, it was
54. (c) Statutory corporations are public enterprises replaced by oil, which was an even cheaper source.
that came into existence by a Special Act of the When the oil producing countries formed a cartel
Parliament. The Act defines the powers and
(OPEC) and increased the price of crude oil from
functions, rules and regulations governing the
$2.1 per barrel in 1973 to as much as $28 per barrel
employees and the relationship of the corporation
in 1980, it hit the developing countries like India the
with government departments.
most.
55. (a) Government-owned or controlled companies refer
67. (b) the share of hydro power is 26 per cent, thermal 66
to companies in which 51% or more of the paid up
capital is held by the central or any state government per cent, renewable energy sources 5 per cent and
(partly or wholly by both). It is registered under the nuclear 3 per cent.
Indian Companies Act and is fully governed by the 68. (d) A major initiative to expedite power generation has
provisions of this Act. been the development of coal-based-Ultra Mega
56. (a) Autonomous bodies are set up whenever it is felt Power Projects (UMPP) each with a capacity
that certain functions need to be discharged outside of 4000 MW or above. Nine sites have been
the governmental set up with some amount of identified including four pithead sites (Chhattisgarh,
independence and flexibility without day-to-day Jharkhand, Madhya Pradesh and Odisha) and five
interference from the governmental machinery. coastal sites (Andhra Pradesh, Gujarat, Karnataka,
57. (d) During the first five-year plan (1950-51 to 1955-56) Maharashtra and Tamil Nadu). The pithead units
there were only five CPSEs with a total financial will get coal from captive mines while the coastal
investment (Including equity plus long-term loans) plants will import coal.
of ` 290 million, whereas at present, in 2015, 69. (d)
there are as many as 290 CPSEs, wherein, 169 are 70. (a) At present, the Indian Railway consists of an
Holding CPSEs and 121 are the subsidiaries. As extensive network spread over 63974 km comprising
of Mar 31, 2012, the total financial investment of Broad Gauge (54257 km) Meter Gauge (7180 km)
CPSEs was 7,292.3 billion, showing a CAGR of and Narrow Gauge (2537 km).
around 12% during the same period. 71. (d)
58. (a) 59. (d) 72. (d) Indian overseas fleet ranks the 17th the world in
60. (b) 45%-50% of the Indian Exports is contributed by SSI terms of tonnage, with 659 ships totalling 77 lakh
Sector. Direct exports from the SSI Sector account gross registered tonnage (GRT). 102 Indian shipping
for nearly 35% of total exports. Besides direct companies are in operation of which 65 are engaged
exports, it is estimated that small-scale industrial only in coastal shipping. The public sector Shipping
units contribute around 15% to exports indirectly. Corporation of India (SCI) has a fleet of 112 vessels
61. (c) Internal factors could be mismanagement, lack of aggregating three million GRT.
finance wrong dividend policy, not providing for 73. (a) An Airport Economic Regulatory Authority (AERA)
reserves or deliberate ‘milking’ of the unit by the is being set up to ensure healthy competition among
owners. External factors could be wrong government operators and settlement of disputes among operators
policy, power shortage, non-availability of raw
as well as between them and passengers.
materials, transport problem, lack of technological
74. (d)
advancement and labour disputes.
75. (c) The IT industry is the biggest private sector
62. (d) The Act mainly covers the following aspects: (i)
employer in India, and it added 230,000 employees
Prohibition of anti competitive agreements; (ii)
in 2014-15, thus making the total number of jobs in
Prohibition of abuse of dominance; (iii) Regulation
the industry close to 3.5 million. The IT sector also
of combination (acquisitions, mergers and
accounted for 9.5% of the gross domestic product.
amalgamations of certain size); (iv) Establishment
The IT industry holds the largest share of total
of Competition Commission of India (CCI); and (v)
services exports at 38%.
Functions and powers of CCI.

You might also like